Está en la página 1de 326

Variable Compleja

y Cálculo Operacional
Teoría y Práctica con MATLAB

William La Cruz
Variable Compleja
y Cálculo Operacional
Teorı́a y Práctica con MATLAB

William La Cruz
Universidad Central de Venezuela
Facultad de Ingenierı́a
Escuela de Ingenierı́a Eléctrica
Departamento de Electrónica, Computación y Control

Versión preliminar para el uso en el curso


Variable Compleja y Cálculo Operacional

ADVERTENCIA: Esta guı́a se distribuye para los estudiantes


de mis cursos. Por favor, contácteme para otros usos.
william.lacruz@ucv.ve

c 2018 W. La Cruz
Contenido

Prefacio v

I Variable Compleja 1

1 Números Complejos 2
1.1 Definición . . . . . . . . . . . . . . . . . . . . . . . . . . . . . . . . . . . . . 2
1.2 Operaciones Algebraicas . . . . . . . . . . . . . . . . . . . . . . . . . . . . . 3
1.3 Representación Geométrica . . . . . . . . . . . . . . . . . . . . . . . . . . . 5
1.4 Valor Absoluto y Conjugado . . . . . . . . . . . . . . . . . . . . . . . . . . . 6
1.5 Coordenadas Polares . . . . . . . . . . . . . . . . . . . . . . . . . . . . . . . 9
1.5.1 Fórmula de Euler . . . . . . . . . . . . . . . . . . . . . . . . . . . . . 12
1.6 Potencias y Raı́ces . . . . . . . . . . . . . . . . . . . . . . . . . . . . . . . . 13
1.7 Regiones en el Plano Complejo . . . . . . . . . . . . . . . . . . . . . . . . . 13
1.8 Aritmética Compleja con M ATLAB . . . . . . . . . . . . . . . . . . . . . . . . 17
1.9 Problemas Resueltos . . . . . . . . . . . . . . . . . . . . . . . . . . . . . . . 20
1.10 Problemas Propuestos . . . . . . . . . . . . . . . . . . . . . . . . . . . . . . 22

2 Funciones de Variable Compleja 24


2.1 Definición . . . . . . . . . . . . . . . . . . . . . . . . . . . . . . . . . . . . . 24
2.2 Lı́mite y Continuidad . . . . . . . . . . . . . . . . . . . . . . . . . . . . . . . 25
2.2.1 Funciones Componentes . . . . . . . . . . . . . . . . . . . . . . . . . 26
2.2.2 Lı́mite . . . . . . . . . . . . . . . . . . . . . . . . . . . . . . . . . . . 26
2.2.3 Continuidad . . . . . . . . . . . . . . . . . . . . . . . . . . . . . . . . 28
2.3 Diferenciación . . . . . . . . . . . . . . . . . . . . . . . . . . . . . . . . . . . 31
2.3.1 Fórmulas o Reglas de Diferenciación . . . . . . . . . . . . . . . . . . 31
2.3.2 Ecuaciones de Cauchy-Riemann . . . . . . . . . . . . . . . . . . . . . 32
2.4 Funciones Analı́ticas . . . . . . . . . . . . . . . . . . . . . . . . . . . . . . . 34
2.5 Funciones Armónicas . . . . . . . . . . . . . . . . . . . . . . . . . . . . . . . 36
2.6 Funciones Elementales . . . . . . . . . . . . . . . . . . . . . . . . . . . . . . 39
2.6.1 Función Exponencial . . . . . . . . . . . . . . . . . . . . . . . . . . . 40
2.6.2 Funciones Trigonométricas . . . . . . . . . . . . . . . . . . . . . . . 41
2.6.3 Funciones Hiperbólicas . . . . . . . . . . . . . . . . . . . . . . . . . . 43
2.6.4 Función Logaritmo . . . . . . . . . . . . . . . . . . . . . . . . . . . . 44
2.6.5 Función Exponente Complejo . . . . . . . . . . . . . . . . . . . . . . 46
2.6.6 Funciones Trigonométricas Inversas . . . . . . . . . . . . . . . . . . . 48
2.6.7 Funciones Hiperbólicas Inversas . . . . . . . . . . . . . . . . . . . . . 49
2.7 Mapeos . . . . . . . . . . . . . . . . . . . . . . . . . . . . . . . . . . . . . . 50
2.7.1 Mapeo w = z + c . . . . . . . . . . . . . . . . . . . . . . . . . . . . . 51
2.7.2 Mapeo w = bz . . . . . . . . . . . . . . . . . . . . . . . . . . . . . . 53
2.7.3 Mapeo w = bz + c . . . . . . . . . . . . . . . . . . . . . . . . . . . . 55
2.7.4 Mapeo Inversión . . . . . . . . . . . . . . . . . . . . . . . . . . . . . 56

i
ii

2.7.5 Mapeo Bilineal . . . . . . . . . . . . . . . . . . . . . . . . . . . . . . 59


2.8 Funciones, Lı́mite, Derivada y Mapeos con M ATLAB . . . . . . . . . . . . . . 61
2.9 Problemas Resueltos . . . . . . . . . . . . . . . . . . . . . . . . . . . . . . . 66
2.10 Problemas Propuestos . . . . . . . . . . . . . . . . . . . . . . . . . . . . . . 71

3 Series de Potencias y Singularidades Aisladas 77


3.1 Serie de Números Complejos . . . . . . . . . . . . . . . . . . . . . . . . . . 77
3.1.1 Serie de Potencias . . . . . . . . . . . . . . . . . . . . . . . . . . . . 79
3.1.2 Serie de Taylor . . . . . . . . . . . . . . . . . . . . . . . . . . . . . . 81
3.1.3 Serie de Laurent . . . . . . . . . . . . . . . . . . . . . . . . . . . . . 85
3.1.4 Propiedades Adicionales de las Series . . . . . . . . . . . . . . . . . . 87
3.2 Singularidades Aisladas . . . . . . . . . . . . . . . . . . . . . . . . . . . . . 88
3.2.1 Polo de Orden m . . . . . . . . . . . . . . . . . . . . . . . . . . . . . 90
3.2.2 Punto Singular Esencial . . . . . . . . . . . . . . . . . . . . . . . . . 93
3.2.3 Punto Singular Removible . . . . . . . . . . . . . . . . . . . . . . . . 94
3.3 Series de Potencias y Singularidades con M ATLAB . . . . . . . . . . . . . . . 94
3.4 Problemas Resueltos . . . . . . . . . . . . . . . . . . . . . . . . . . . . . . . 98
3.5 Problemas Propuestos . . . . . . . . . . . . . . . . . . . . . . . . . . . . . . 104

4 Integración Compleja 105


4.1 Integral Definida . . . . . . . . . . . . . . . . . . . . . . . . . . . . . . . . . 105
4.2 Integración de Lı́nea . . . . . . . . . . . . . . . . . . . . . . . . . . . . . . . 106
4.2.1 Contornos . . . . . . . . . . . . . . . . . . . . . . . . . . . . . . . . . 106
4.2.2 Integral de Lı́nea . . . . . . . . . . . . . . . . . . . . . . . . . . . . . 108
4.3 Teorema de Cauchy-Goursat . . . . . . . . . . . . . . . . . . . . . . . . . . . 110
4.3.1 Extensión del Teorema de Cauchy-Goursat . . . . . . . . . . . . . . . 111
4.4 Integral Indefinida . . . . . . . . . . . . . . . . . . . . . . . . . . . . . . . . 113
4.5 Fórmula Integral de Cauchy . . . . . . . . . . . . . . . . . . . . . . . . . . . 115
4.6 Residuo . . . . . . . . . . . . . . . . . . . . . . . . . . . . . . . . . . . . . . 117
4.6.1 Cálculo del Residuo . . . . . . . . . . . . . . . . . . . . . . . . . . . 118
4.6.2 Teorema de los Residuos . . . . . . . . . . . . . . . . . . . . . . . . . 120
4.6.3 Expansión en Fracciones Parciales . . . . . . . . . . . . . . . . . . . . 121
4.7 Integración y Residuos con M ATLAB . . . . . . . . . . . . . . . . . . . . . . 124
4.8 Problemas Resueltos . . . . . . . . . . . . . . . . . . . . . . . . . . . . . . . 128
4.9 Problemas Propuestos . . . . . . . . . . . . . . . . . . . . . . . . . . . . . . 135

II Cálculo Operacional 139

5 Funciones de Dominios Continuo y Discreto 140


5.1 Funciones de Dominio Continuo . . . . . . . . . . . . . . . . . . . . . . . . . 140
5.1.1 Impulso Unitario . . . . . . . . . . . . . . . . . . . . . . . . . . . . . 141
5.1.2 Escalón Unitario . . . . . . . . . . . . . . . . . . . . . . . . . . . . . 143
5.1.3 Pulso Rectangular . . . . . . . . . . . . . . . . . . . . . . . . . . . . 144
5.1.4 Pulso Triangular . . . . . . . . . . . . . . . . . . . . . . . . . . . . . 145
5.1.5 Función Signo . . . . . . . . . . . . . . . . . . . . . . . . . . . . . . 145
5.1.6 Pulso Exponencial . . . . . . . . . . . . . . . . . . . . . . . . . . . . 146
5.1.7 Función Rampa . . . . . . . . . . . . . . . . . . . . . . . . . . . . . . 147
iii

5.1.8 Funciones Sa y sinc . . . . . . . . . . . . . . . . . . . . . . . . . . . 147


5.1.9 Relación entre u(t) y δ(t) . . . . . . . . . . . . . . . . . . . . . . . . 148
5.1.10 Derivada Generalizada . . . . . . . . . . . . . . . . . . . . . . . . . . 149
5.1.11 Convolución en el Dominio Continuo . . . . . . . . . . . . . . . . . . 151
5.2 Funciones de Dominio Discreto . . . . . . . . . . . . . . . . . . . . . . . . . 155
5.2.1 Impulso Unitario Discreto . . . . . . . . . . . . . . . . . . . . . . . . 155
5.2.2 Escalón Unitario Discreto . . . . . . . . . . . . . . . . . . . . . . . . 156
5.2.3 Función Rampa Discreta . . . . . . . . . . . . . . . . . . . . . . . . . 156
5.2.4 Función Signo Discreto . . . . . . . . . . . . . . . . . . . . . . . . . . 157
5.2.5 Relación entre u(n) y δ(n) . . . . . . . . . . . . . . . . . . . . . . . . 158
5.2.6 Convolución en el Dominio Discreto . . . . . . . . . . . . . . . . . . 159
5.3 Introducción al Cálculo Operacional con M ATLAB . . . . . . . . . . . . . . . 162
5.4 Problemas Resueltos . . . . . . . . . . . . . . . . . . . . . . . . . . . . . . . 170
5.5 Problemas Propuestos . . . . . . . . . . . . . . . . . . . . . . . . . . . . . . 182

6 Transformada de Fourier 185


6.1 Definición . . . . . . . . . . . . . . . . . . . . . . . . . . . . . . . . . . . . . 185
6.1.1 Transformada Inversa de Fourier . . . . . . . . . . . . . . . . . . . . 186
6.2 Propiedades de la Transformada de Fourier . . . . . . . . . . . . . . . . . . 187
6.3 Algunos Pares de Transformadas . . . . . . . . . . . . . . . . . . . . . . . . 194
6.4 Magnitud y Fase . . . . . . . . . . . . . . . . . . . . . . . . . . . . . . . . . 200
6.5 Transformada de Fourier con M ATLAB . . . . . . . . . . . . . . . . . . . . . 205
6.6 Problemas Resueltos . . . . . . . . . . . . . . . . . . . . . . . . . . . . . . . 207
6.7 Problemas Propuestos . . . . . . . . . . . . . . . . . . . . . . . . . . . . . . 216

7 Transformada de Laplace 218


7.1 Definición . . . . . . . . . . . . . . . . . . . . . . . . . . . . . . . . . . . . . 218
7.1.1 Región de Convergencia . . . . . . . . . . . . . . . . . . . . . . . . . 219
7.1.2 Transformada Inversa de Laplace . . . . . . . . . . . . . . . . . . . . 220
7.2 Propiedades de la Transformada de Laplace . . . . . . . . . . . . . . . . . . 220
7.3 Algunos Pares de Transformadas . . . . . . . . . . . . . . . . . . . . . . . . 228
7.4 Cálculo de la Transformada Inversa de Laplace . . . . . . . . . . . . . . . . 233
7.4.1 Integración de Contornos . . . . . . . . . . . . . . . . . . . . . . . . 233
7.4.2 Inversión por Tablas . . . . . . . . . . . . . . . . . . . . . . . . . . . 236
7.5 Transformada de Laplace con M ATLAB . . . . . . . . . . . . . . . . . . . . . 237
7.6 Problemas Resueltos . . . . . . . . . . . . . . . . . . . . . . . . . . . . . . . 239
7.7 Problemas Propuestos . . . . . . . . . . . . . . . . . . . . . . . . . . . . . . 246

8 Transformada z 249
8.1 Definición . . . . . . . . . . . . . . . . . . . . . . . . . . . . . . . . . . . . . 249
8.1.1 Región de Convergencia . . . . . . . . . . . . . . . . . . . . . . . . . 250
8.1.2 Transformada z Inversa . . . . . . . . . . . . . . . . . . . . . . . . . 251
8.2 Propiedades de la Transformada z . . . . . . . . . . . . . . . . . . . . . . . . 252
8.3 Algunos Pares de Transformadas . . . . . . . . . . . . . . . . . . . . . . . . 258
8.4 Cálculo de la Transformada z Inversa . . . . . . . . . . . . . . . . . . . . . . 263
8.4.1 Integración Compleja . . . . . . . . . . . . . . . . . . . . . . . . . . 263
8.4.2 Expansión en Serie de Potencias . . . . . . . . . . . . . . . . . . . . . 264
8.4.3 Inversión por Tablas . . . . . . . . . . . . . . . . . . . . . . . . . . . 265
iv

8.5 Transformada z con M ATLAB . . . . . . . . . . . . . . . . . . . . . . . . . . 266


8.6 Problemas Resueltos . . . . . . . . . . . . . . . . . . . . . . . . . . . . . . . 268
8.7 Problemas Propuestos . . . . . . . . . . . . . . . . . . . . . . . . . . . . . . 273

Respuesta a los Problemas Propuestos 275

A Introducción a M ATLAB 282


A.1 Escritorio de M ATLAB . . . . . . . . . . . . . . . . . . . . . . . . . . . . . . 282
A.2 Objetos y Sintaxis . . . . . . . . . . . . . . . . . . . . . . . . . . . . . . . . . 284
A.3 Matrices . . . . . . . . . . . . . . . . . . . . . . . . . . . . . . . . . . . . . . 288
A.4 Elementos de Programación . . . . . . . . . . . . . . . . . . . . . . . . . . . 295
A.5 Gráficos . . . . . . . . . . . . . . . . . . . . . . . . . . . . . . . . . . . . . . 305

Bibliografı́a 314

Índice 315
Prefacio
“La persona que nunca se equivoca
nunca prueba algo nuevo”
Albert Einstein (1879-1955)

Actualmente existe la creencia bastante común que los problemas matemáticos en su


mayorı́a se resuelven usando una computadora sin otro conocimiento previo o que las so-
luciones de los mismos se encuentran en Iternet, simplemente buscando en Google un link
que nos dirija mágicamente a la respuesta. Esta visión inmediatista, sin mediación alguna,
sin esfuerzo, es consecuencia del gran desarrollo de la computación y la consolidación de
las redes sociales, pero tal creencia no es cierta. Sólo basta con pensar y hacerse las si-
guientes preguntas: ¿Cómo se diseñan las computadoras? o ¿Cómo funciona Internet? Las
respuestas a estas preguntas encierran un gran cúmulo de conceptos, métodos y técnicas,
todos ellos relacionados con la matemática y la ingenierı́a. Por ejemplo, para diseñar un
procesador (CPU) de una computadora, una red de comunicaciones, un puente colgante o
un tornillo, es necesario manejar ciertos conceptos tales como: microprocesadores, grafos,
curva catenaria, constante de elasticidad y muchos otros más, que a su vez requieren para
su comprensión el manejo de la suma, la multiplicación, la derivación, la integración, entre
otros conceptos y operaciones matemáticas. Por tanto, uno de los objetivos de esta guı́a es
lograr que el estudiante se familiarice profundamente con los conceptos matemáticos.
El motivo para la elaboración de esta guı́a, que a su vez es otro objetivo de la misma,
es brindar al estudiante el libre acceso al conocimiento, por su puesto, presentando los
conceptos de una manera sencilla pero formal, usando para ello solo herramientas ma-
temáticas correspondientes al cuarto semestre de Ingenierı́a. Especı́ficamente, el escrito
presenta la teorı́a básica de Variable Compleja y el Cálculo Operacional.
Para la comprensión de los conceptos expuestos en el escrito, es necesario que el lector
posea un nivel aceptable de conocimientos de Cálculo (particularmente, del Cálculo Infini-
tesimal Real en una y dos variables); también debe manejar con destreza las operaciones
algebraicas en los conjuntos reales, racionales y enteros. En otras palabras, el material está
dirigido a estudiantes cursantes del cuarto semestre de Ingenierı́a.
La guı́a está estructurada en dos partes: I. Variable Compleja y II. Cálculo Operacional,
ya que la misma, en principio, está diseñada según el contenido temático del curso Variable
Compleja y Cálculo Operacional, ubicado en el cuarto semestre del Plan de Estudios de
Ingenierı́a Eléctrica. En los capı́tulos 1, 2, 3 y 4, se dan las propiedades del conjunto
de los números complejos; se presentan los fundamentos matemáticos de la continuidad
y diferenciación de una función de variable compleja; se estudian las series de números
complejos haciendo énfasis en los desarrollos de Taylor y de Laurent; y se describen los
conceptos básicos de la integración compleja: integral definida, integral de lı́nea, primitiva
y teorı́a de residuos.
En el capı́tulo 5 se da una introducción al cálculo operacional, comenzando con la
caracterización de las funciones de dominio continuo, pasando luego por la convolución

v
vi PREFACIO

en el dominio continuo. En esta parte se estudia con interés la función impulso unitario.
Aquı́ también se describen las funciones de dominio discreto y se define la convolución de
funciones de dominio discreto. En los capı́tulos 6, 7 y 8, se presentan, respectivamente, la
definición matemática y se prueban todos los teoremas y propiedades, de la transforma de
Fourier, la transformada de Laplace y la transformada z.
Es de destacar que estas notas han evolucionado a medida que han transcurrido los
semestres en los que se ha dictado la asignatura Variable Compleja y Cálculo Operacional,
agregando un problema resuelto o una imagen que permita la mejor visualización de un
concepto o un método. En todo caso, siempre en mejora del aprendizaje. Con esto en
mente, cada capı́tulo posee secciones de problemas resueltos y problemas propuestos. Los
problemas resueltos fueron seleccionados de exámenes parciales, además, en el escrito
se presenta en forma rigurosa el proceso de resolución de los mismos. Los problemas
propuestos son una colección de problemas orientados al fortalecimiento y consolidación
de los conceptos y métodos estudiados. Todos los problemas propuestos tienen respuesta,
que se muestra al final de la guı́a.
Con el propósito de visualizar de manera más aplicada la signatura Variable Compleja y
Cálculo Operacional, se agregó a la guı́a un enfoque computacional del análisis complejo,
es decir, en el escrito también se presenta una visión computacional de los conceptos,
usando para ello el software M ATLAB (R2015a), que se emplea en numerosas universidades
y posee muchas facilidades gráficas de calidad, flexibilidad e interactividad. Al final de
cada capı́tulo se presentan ejemplos prácticos codificados en M ATLAB . Tal iniciativa tiene
el propósito de introducir al estudiante en el uso de una herramienta computacional, pero
dejándoles ver que tal herramienta por sı́ sola no resuelve los problemas, sino que es
necesario que el usuario tenga la destreza de entender y definir correctamente el problema
para luego resolverlo con la computadora. Para que el estudiante tuviera a la mano un
pequeño manual de M ATLAB , se agregó como apéndice una breve introducción a M ATLAB .
Finalmente, a los lectores les pido el gran favor de apuntar mis errores, ya que citando
nuevamente a Albert Einstein, “la persona que nunca se equivoca nunca prueba algo nuevo”,
o como yo lo interpreto: al momento de crear se está sujeto a errar, pero ello no debe ser
un obstáculo para continuar y mejorar.

William La Cruz
Universidad Central de Venezuela, Caracas
Junio de 2018
Parte I

Variable Compleja

1
1
Números Complejos
La primera referencia escrita de la raı́z cuadrada de un número negativo la encon-
tramos en la obra Stereometrı́a de Herón de Alejandrı́a alrededor de la mitad del siglo I.
La siguiente referencia sobre la raı́z cuadrada de un número negativo data del año 275
en la obra de Diophantus, Arithmetica. En su intento del cálculo de los lados de un
triángulo rectángulo de perı́metro 12 y área 7, Diophantus planteó resolver la ecuación
336x2 + 24 = 172x, ecuación cuyas soluciones no son números reales. En 1545, Jerome
Cardano (Italia, 1501-1576), un matemático, fı́sico y filósofo italiano, publica “Ars Magn”
(El Gran Arte) en el cual describe un método para resolver ecuaciones algebraicas de grado
tres y cuatro. En esta obra Cardano plantea el siguiente problema: Si alguien te pide dividir
10 en dos partes cuyo producto sea 40, es evidente que esta cuestión es imposible. Aparente-

mente √ este problema parece imposible, pero Cardano da las siguientes soluciones: 5+ −5
y 5 − −5. Cardano obtiene tales soluciones resolviendo las ecuaciones: x + y = 10,
Trate de resolver xy = 40. Dos siglos y medio cubrieron las dudas sobre el significado y la autenticidad
las ecuaciones de
Cardano.
de los números complejos. No obstante, fueron estudiados por un gran número de ma-
temáticos. Cabe√mencionar que fue Leonhard Euler (Suiza, 1707-1783) el primero en usar
la notación i = −1. El matemático alemán Carl Friedrich Gauss (Alemania, 1777-1855),
fue quien les dio nombre y los definió rigurosamente.
En este capı́tulo se introducen algunas propiedades del conjunto de los números com-
plejos. Tal conjunto de números es ampliamente utilizado en el desarrollo de las ideas
teóricas y prácticas de Ingenierı́a, en especial, de Ingenierı́a Eléctrica.

1.1 Definición
Se dice que z es un número complejo si se expresa como

z = x + iy

o, de manera equivalente,
z = x + y i,
donde x e y son números reales (recuerde que R denota el conjunto de los números reales).
El sı́mbolo i se conoce como unidad imaginaria. El conjunto de los números complejos se
denota como C, y z ∈ C indica que z pertenece al conjunto de los números complejos.
Por otra parte, un número complejo z = x + i y también se puede definir como el par
ordenado z = (x, y). Esta definición equivalente de número complejo permite definir a i
como el número complejo dado por

i = (0, 1).

Más adelante se demostrará que i2 = −1.

2
CAPÍTULO 1. NÚMEROS COMPLEJOS 3

Se denota con x = Re z la parte real del número complejo z, y con y = Im z la parte


imaginaria de z. Los números complejos de la forma x + i 0, se denominan reales puros o,
simplemente, reales. Además, los números complejos de la forma 0 + i y, se denominan
imaginarios puros.
Los números reales 0 y 1, también se pueden definir como números complejos. El cero
de los números complejos, denotado 0, se define como 0 = 0 + i 0. El 1 de los números
complejos, denotado 1, se define por 1 = 1 + i 0.

Definición 1.1. Se dice que dos números complejos z y w son iguales si, y sólo si sus
partes reales son iguales y sus partes imaginarias son iguales. En otras palabras, si
z = x + i y y w = u + i v, entonces z = w, si y sólo si

x=u e y = v.

En particular,
z = x + iy = 0 ⇔ x = y = 0.

Observación 1.1. No existe relación de orden en los números complejos. En los números
reales, por ejemplo, se tiene que 5 > 3, pero no tiene sentido afirmar que 1 + i < 2 + i 3. Recuerde que los
números reales
se pueden repre-
sentar como un
punto en una
recta, que se
1.2 Operaciones Algebraicas recorre de iz-
quierda a dere-
cha, con lo cual
se impone un
Seguidamente se definen las operaciones algebraicas de números complejos, a saber: orden.
suma, resta, multiplicación y división.

Definición 1.2. Sean z1 = x1 + i y1 , y z2 = x2 + i y2 números complejos. Se define la


suma, resta, multiplicación y división de z1 y z2 como sigue.
Suma:
z1 + z2 = (x1 + x2 ) + i (y1 + y2 ).
Resta:
z1 − z2 = (x1 − x2 ) + i (y1 − y2 ).
Multiplicación:

z1 · z2 = z1 z2 = (x1 x2 − y1 y2 ) + i (x1 y2 + x2 y1 ).

División:    
z1 x1 x2 + y 1 y 2 x2 y 1 − x1 y 2
z1 ÷ z2 = = +i ,
z2 x22 + y22 x22 + y22
siempre que x2 6= 0 y y2 6= 0.

En la siguiente proposición se describen algunas propiedades de las operaciones al-


gebraicas de los números complejos. Esta proposición permite asegurar que el conjunto
de números complejos conforma un cuerpo, esto es, un conjunto algebraico con leyes de
composición interna como la suma y la multiplicación.
4 1.2. OPERACIONES ALGEBRAICAS

Proposición 1.1. Para todo z, w, s ∈ C se cumplen las siguientes propiedades:

1. Conmutativa
• z+w =w+z
• zw = wz
2. Asociativa
• z + (w + s) = (z + w) + s
• z(ws) = (zw)s
3. Elemento Neutro
• z+0=z
• 1·z =z
4. Elemento Inverso
• Para todo z ∈ C, existe −z ∈ C, llamado inverso aditivo, tal que z + (−z) = 0.
• Para todo z 6= 0, existe z −1 ∈ C, denominado inverso multiplicativo, tal que

z z −1 = 1.

5. Distributiva
• z(w + s) = zw + zs.

Demostración. Es una consecuencia inmediata de las propiedades de los números reales,


la definición de número complejo y las definiciones de suma y multiplicación de números
complejos.
Note que las operaciones suma, resta y multiplicación, son leyes de composición interna
sobre el conjunto de los números complejos, es decir, son operaciones que asocian a cada
par de números complejos otro número complejo. La división también es una ley de com-
posición interna sobre el conjunto de los números complejos distintos de cero. Además,
estas operaciones pueden verse como una extensión de las operaciones algebraicas de los
números reales.
Ejercicio 1.1. Sean z1 = 1 − i, z2 = i − 3, y z3 = i/2. Realizar las siguientes operaciones
algebraicas:

a) z1 + z2 + z3 d) z2 /z3

b) z1 · z2 · z3 e) z1 /(z2 · z1 )

c) z1 (z2 + z3 ) f) z3 /(z1 /z2 )

La siguiente proposición relaciona la división de números complejos con el inverso mul-


tiplicativo. También proporciona una manera equivalente de definir división de números
complejos.

Proposición 1.2. Sean z, w ∈ C. Si w 6= 0, entonces


z
= z w−1 .
w
CAPÍTULO 1. NÚMEROS COMPLEJOS 5

z
Demostración. Demostremos que las partes real e imaginaria de los números complejos
w
y z w−1 son iguales. Tomemos z = x + i y, w = u + i v y asumamos que w =
6 0. Por
definición, se tiene que
z xu + yv yu − xv
= 2 2
+i 2 . (1.1)
w u +v u + v2

Ahora, veamos la forma que tiene w−1 . Asumamos que w−1 = a+i b. Por la Proposición 1.1
se tiene que w w−1 = 1. Por tanto, podemos escribir:

1 = w w−1 = (u + i v)(a + i b) = (ua − vb) + i (ub + va),

de donde se obtiene el siguiente sistema de ecuaciones lineales con incógnitas a y b.


ua − vb = 1
va + ub = 0

Resolviendo este último sistema de ecuaciones lineales obtenemos:

u −v
a= y b= ,
u2 + v2 u2
+ v2

luego,
u −v
w−1 = +i 2 .
u2 +v 2 u + v2

Empleando es última ecuación se tiene que

xu + yv yu − xv
z w−1 = 2 2
+i 2 . (1.2)
u +v u + v2

z
Por (1.1) y (1.2) podemos concluir que = z w−1 .
w

1.3 Representación Geométrica

Un número complejo z = x + i y también puede definirse como un único par ordenado


(x, y), es decir, la parte real de z es la primera coordenada de (x, y) y la parte imaginaria es
la segunda coordenada. Ası́, el número complejo z puede representarse geométricamente
como un punto en el plano cartesiano xy, como ejemplo de ello observe la gráfica que se
muestra en la Figura 1.1.
Cuando se utiliza el plano cartesiano para representar un número complejo, éste se
denomina plano complejo o plano z. Además, el eje x, o eje de las abscisas, se denomina
eje real, mientras que el eje y, o eje de las ordenadas, se conoce como eje imaginario (ver
Figura 1.1).
6 1.4. VALOR ABSOLUTO Y CONJUGADO

Eje imaginario

z = (x, y)
y b

x Eje real

Figura 1.1. Representación de un número complejo como un par ordenado

Otra representación posible de z = x + i y en el plano cartesiano es en forma de vector


(ver Figura 1.2), es decir, z se puede representar como un vector orientado cuyo extremo
inicial es el origen (0, 0) y su extremo final es el punto (x, y). De esta forma, a un número
complejo lo podemos representar como un par ordenado o como un vector en el plano xy.
¿Por qué se toma
a z como el vec-
tor orientado Eje imaginario
con origen (0, 0)
y extremo final
(x, y), y no otro z = x + iy
vector? y

x Eje real

Figura 1.2. Representación de un número complejo como un vector

1.4 Valor Absoluto y Conjugado


En la sección anterior notamos que un número complejo z se puede representar como
un vector orientado. Este hecho permite que z herede algunas propiedades de los vectores,
entre ellas la magnitud o valor absoluto.

Definición 1.3 (Valor absoluto). El valor absoluto del número complejo z = x + i y,


denotado por |z|, se define como la longitud del vector z, la cual se calcula a través de
la fórmula p
|z| = x2 + y 2 .
Geométricamente, el valor absoluto |z| representa la distancia del origen (0, 0) al
punto (x, y).

Definición 1.4 (Conjugado). El conjugado del número complejo z = x + i y, denotado


por z, se define como
z = x + i (−y).
Geométricamente, el conjugado z es la reflexión del punto (x, y) con respecto al eje real.
En la Figura 1.3 se muestra un ejemplo gráfico del conjugado de un número complejo.
CAPÍTULO 1. NÚMEROS COMPLEJOS 7

z = x + iy
y b

−y b

z = x − iy

Figura 1.3. Conjugado de un número complejo

La Proposición 1.3 presenta algunas propiedades del valor absoluto y el conjugado.

Proposición 1.3. Sean z1 y z2 , números complejos. Las siguientes propiedades se cumplen:

1) z1 = z1 .

2) z1 ± z2 = z1 ± z2 .

3) z1 z2 = z1 z2 .
 
z1 z1
4) = , siempre y cuando z2 6= 0.
z2 z2
5) |z1 | = |z1 |.

6) z1 z1 = |z1 |2 .
z1
7) z1−1 = , siempre y cuando z1 6= 0.
|z1 |2
8) |z1 | ≥ |Re z1 | ≥ Re z1 .

9) |z1 | ≥ |Im z1 | ≥ Im z1 .

10) |z1 z2 | = |z1 | |z2 |.


z1 |z1 |
11) = , siempre y cuando z2 6= 0.
z2 |z2 |

Demostración. Se deja como ejercicio para el lector.

La suma de los números complejos z1 = x1 + i y1 y z2 = x2 + i y2 , tiene una interpre-


tación simple en términos vectoriales. El vector que representa la suma de los números
z1 y z2 se obtiene sumando vectorialmente los vectores de z1 y z2 , es decir, empleando
la regla del paralelogramo (ver Figura 1.4). Este esquema geométrico se puede utilizar
para obtener la desigualdad triangular: la longitud de un lado cualquiera de un triángulo
es menor o igual que la suma de las longitudes de los otros lados. Esto es, la longitud corre-
spondiente a z1 + z2 , |z1 + z2 |, es menor o igual que la suma de las longitudes, |z1 | y |z2 |.
En la Proposición 1.4 damos la expresión matemática de la desigualdad triangular.
8 1.4. VALOR ABSOLUTO Y CONJUGADO

z1 z2
+
z1
z2

Figura 1.4. Suma vectorial de números complejos.

Proposición 1.4 (Desigualdad triangular). Sean z1 y z2 números complejos. Entonces, la


siguiente desigualdad se cumple

|z1 + z2 | ≤ |z1 | + |z2 |.

Demostración. Usando Proposición 1.3 6), podemos escribir:

|z1 + z2 |2 = (z1 + z2 ) (z1 + z2 ) = (z1 + z2 ) (z1 + z2 ) = z1 z1 + z1 z2 + z2 z1 + z2 z2


= |z1 |2 + z1 z2 + z2 z1 + |z2 |2 = |z1 |2 + z1 z2 + z1 z2 + |z2 |2 ,

pero
z1 z2 + z1 z2 = 2 Re (z1 z2 ) ≤ 2 |z1 z2 | = 2 |z1 | |z2 | = 2 |z1 | |z2 |,
luego
|z1 + z2 |2 ≤ |z1 |2 + 2 |z1 | |z2 | + |z2 |2 = (|z1 | + |z2 |)2 ,
de donde se deduce que
|z1 + z2 | ≤ |z1 | + |z2 |,
con lo cual queda establecida la desigualdad triangular.

La desigualdad triangular se puede extender a más de dos vectores, es decir, la longitud


de la suma de un número finito de vectores es menor o igual que la suma de las longitudes
de tales vectores. En la siguiente proposición se muestra la expresión matemática de este
resultado, el cual se denomina desigualdad triangular generalizada.

Proposición 1.5 (Desigualdad triangular generalizada). Sean z1 , z2 , . . . , zn números


complejos, donde n es un entero mayor o igual que 2. Entonces, la siguiente desigualdad se
cumple
|z1 + z2 + · · · + zn | ≤ |z1 | + |z2 | + · · · + |zn |.

Demostración. Realicemos la demostración por inducción. Por la Proposición 1.4, la desi-


gualdad se cumple para n = 2. Supongamos, como hipótesis inductiva, que la desigualdad
se cumple para n = h, es decir, se tiene que

|z1 + z2 + · · · + zh | ≤ |z1 | + |z2 | + · · · + |zh |.


CAPÍTULO 1. NÚMEROS COMPLEJOS 9

Ahora demostremos que esta desigualdad se cumple para n = h + 1. Por la hipótesis


inductiva y la Proposición 1.4, podemos escribir:

|z1 + z2 + · · · + zh + zh+1 | ≤ |z1 + z2 + · · · + zh | + |zh+1 |


≤ |z1 | + |z2 | + · · · + |zh | + |zh+1 |,

para todo entero h ≥ 1. Con esto queda demostrada la desigualdad.

1.5 Coordenadas Polares


Del hecho que un número complejo z = x+i y puede ser representado geométricamente
como un vector, se puede derivar su representación en coordenadas polares (r, θ), donde
r = |z| y θ es el argumento de z.

Definición 1.5 (Argumento de z). El argumento del número complejo z 6= 0, denotado


por arg z, es cualquiera de los ángulos orientados comprendidos entre la parte positiva
del eje real y el número complejo z.
|

z = x + iy
|z
=
r

Figura 1.5. Coordenadas polares de z

En la Figura 1.5 se muestra un ejemplo de la representación en coordenadas polares de


un número complejo z = x + i y ubicado en el primer cuadrante. Se tiene que x = r cos θ
e y = r sen θ. Ası́ pues, la forma polar de z se define como

z = r(cos θ + i sen θ).

Los valores de r y θ = arg z definen de manera única a z, es decir, para cada par
(r, θ) existe un único número complejo z que tiene como coordenadas polares a (r, θ). En
cambio, el número complejo z caracteriza de manera única a r, pero no a θ = arg z, esto
es, dado un número complejo z, entonces existe un único r > 0 tal que r = |z|, pero existen
infinitos valores de θ = arg z. Observe el siguiente ejemplo.

Ejemplo 1.1. Para el número complejo z = 1, se tiene que los valores de θ están dados
por: θ = arg z ∈ {0, ±2π, ±4π, . . .}. 

La existencia de infinitos valores de θ es una consecuencia de la periodicidad de las


funciones trigonométricas. En efecto, si la forma polar de un número complejo z1 es

z1 = r1 (cos θ1 + i sen θ1 ),
10 1.5. COORDENADAS POLARES

entonces se tiene que

r1 (cos(θ1 + 2nπ) + i sen (θ1 + 2nπ)) = r1 (cos θ1 + i sen θ1 ) = z1 , para n ∈ Z,

en otras palabras, (r, θ1 + 2nπ) también son coordenadas polares de z1 .


Ahora bien, para obtener una única representación en coordenadas polares de un
número complejo z, el valor de θ se debe tomar en una determinación, es decir, se debe
escoger un valor de θ en un intervalo de longitud 2π, por ejemplo, θ ∈ [0, 2π), ó θ ∈ (−π, π],
etc. El valor de θ que generalmente se emplea es el valor del argumento de z que pertenece
al intervalo (−π, π], el cual se denomina argumento principal.

Definición 1.6 (Argumento principal). El argumento principal de z 6= 0 o valor principal


de arg z, denotado por Arg z, se define como el único valor de arg z tal que

−π < Arg z ≤ π.

Los valores de θ = arg z, y en especial el valor de Arg z, se pueden encontrar empleando


la ecuación:
y
tan θ = .
x
En sı́, para calcular el argumento principal de un número complejo z = x + i y se emplea
Note que la ecuación θ = arctan(y/x). La manera de emplear esta última ecuación en el cálculo del
arctan(y/x) es
un valor entre
argumento principal de z 6= 0, se muestra en la siguiente expresión:
−π/2 y π/2. 


 0, x > 0, y = 0,




 arctan(y/x), x > 0, y > 0,




 π/2, x = 0, y > 0,

arctan(y/x) + π, x < 0, y > 0,
Arg z =
π,
 x < 0, y = 0,





 arctan(y/x) − π, x < 0, y < 0,




 −π/2, x = 0, y < 0,

arctan(y/x), x > 0, y < 0.

Note que la fórmula anterior considera la posición z en el plano complejo para calcular
el valor de Arg z, esto es, dependiendo del cuadrante donde se encuentre z, su argumento
principal se calcula de una forma muy particular, por supuesto, usando para ello el valor
de arctan(y/x). Observe el siguiente ejemplo.

Ejemplo 1.2. Utilizando el argumento principal, represente en forma polar los siguientes
números complejos: z1 = 1 + i , z2 = −1 + i , z3 = −1 − i, y z4 = 1 − i.
√ √
Solución. Se tiene que el valor absoluto de z1 es r = |z1 | = 12 + 12 = 2. Como z1 = 1+i
está en el primer cuadrante, su argumento principal es
π
Arg z1 = arctan(1) = .
4
Por lo tanto, la fórmula polar de z1 = 1 + i empleando el argumento principal es:

z1 = 2(cos(π/4) + i sen (π/4)).
CAPÍTULO 1. NÚMEROS COMPLEJOS 11

Por un razonamiento similar, se puede verificar que la forma polar de los números comple-
jos z2 = −1 + i , z3 = −1 − i y z4 = 1 − i, es respectivamente:

z2 = 2(cos(3π/4) + i sen (3π/4)),

z3 = 2(cos(3π/4) − i sen (3π/4)),

z4 = 2(cos(π/4) − i sen (π/4)).

Se deja como ejercicio para el lector verificar cada una de estas formas polares. 

En el ejemplo anterior calculamos el argumento principal de un número complejo. Es


natural preguntarse: ¿a partir del valor del argumento principal se pueden obtener todos
los valores de arg z? La respuesta a esta pregunta es afirmativa, más aún, conocido un
valor cualquiera del argumento (no necesariamente el principal) se pueden hallar todos
los valores del argumento. Con la siguiente expresión se encuentran todos los valores del
argumento de un número complejo z, utilizando el valor de Arg z:

arg z = Arg z + 2nπ, para n ∈ Z.

Por otra parte, pasemos a describir algunas propiedades del argumento, las cuales se
muestran en la siguiente proposición.

Proposición 1.6. Si z1 y z2 son números complejos distintos de cero, entonces las siguientes
identidades se cumplen:

1. arg (z1 z2 ) = arg z1 + arg z2 .

2. arg (1/z2 ) = −arg (z2 ).

3. arg (z1 /z2 ) = arg z1 − arg z2 .

Demostración. Sean z1 y z2 números complejos cuya representación en forma polar es:


z1 = r1 (cos θ1 + i sen θ1 ) y z2 = r2 (cos θ2 + i sen θ2 ). Utilizando identidades trigonométricas
podemos escribir:

z1 z2 = (r1 (cos θ1 + i sen θ1 )) (r2 (cos θ2 + i sen θ2 ))


= (r1 r2 ) [(cos θ1 cos θ2 − sen θ1 sen θ2 ) + i (cos θ1 sen θ2 + sen θ1 cos θ2 )]
= (r1 r2 ) [cos(θ1 + θ2 ) + i sen (θ1 + θ2 )] ,

que es la forma polar de z1 z2 y θ1 + θ2 = arg z1 + arg z2 es un valor de arg (z1 z2 ). Por lo


tanto, se cumple la identidad 1.
Empleando la forma polar de z2 podemos escribir:

1 1 1 cos θ2 − i sen θ2
= = · = r2−1 (cos(−θ2 ) + i sen (−θ2 )),
z2 r2 (cos θ2 + i sen θ2 ) r2 cos2 θ2 + sen 2 θ2

que es la forma polar de 1/z2 y arg (1/z2 ) = −θ2 ; luego, se cumple la identidad 2.
Como
z1 1
= z1 · ,
z2 z2
entonces, por las identidades 1 y 2, se prueba que la identidad 3 se cumple.
12 1.5. COORDENADAS POLARES

Ejemplo 1.3. La Proposición 1.6 se puede utilizar para calcular un valor del argumento de
= −1 + i. La forma polar de estos números
z1 z2 o z1 /z2 . Por ejemplo, tomemos z1 = i y z2 √
complejos es z1 = cos(π/2) + i sen (π/2) y z2 = 2(cos(3π/4) + i sen (3π/4)), por supuesto,
usando el argumento principal. Ası́, un valor del argumento de z1 z2 es
π 3π 5π
+ =
arg (z1 z1 ) = ,
2 4 4
lo cual efectivamente es un valor del argumento de z1 z2 = −1 − i (verifique esta afir-
mación). Pero, este valor no es el argumento principal de z1 z2 = −1 − i, aunque π/2 y
3π/4 sean los argumentos principales de z1 y z2 , respectivamente. 

Un ejercicio in- El Ejemplo 1.3 nos permite asegurar que, en general,


teresante es
encontrar z1 Arg (z1 z2 ) 6= Arg z1 + Arg z2 y Arg (z1 /z2 ) 6= Arg z1 − Arg z2 .
y z2 tales que
Arg (z1 z2 ) =
Arg z1 + Arg z2 1.5.1 Fórmula de Euler
La fórmula de Euler es una notación de la forma polar de un número complejo que per-
mite ahorrar escritura e introduce la función exponencial compleja, la cual estudiaremos
en los capı́tulos subsiguientes. El nombre de Euler es en honor a Leonhard Euler (1707-
1783), matemático y fı́sico suizo. Tal fórmula consiste en lo siguiente. Sea z = x + i y un
número complejo con módulo |z| = 1 y θ = arg z. Escribiendo
eiθ = cos θ + i sen θ
se obtiene
z = eiθ ,
que se conoce como fórmula de Euler. Ahora, para cualquier número complejo
z = r(cos θ + i sen θ),
con r > 0, se puede utilizar la fórmula de Euler para reescribir a z como:
z = reiθ .
Además, si z1 = r1 eiθ1 y z2 = r2 eiθ2 , entonces las siguientes identidades son ciertas:
1. z1 z2 = (r1 r2 )ei(θ1 +θ2 ) .
1 −iθ
2. 1/z1 = (1/r1 ) ei(−θ1 ) = e .
r1
3. z1 /z2 = (r1 /r2 )ei(θ1 −θ2 ) .
Considerando esta notación, la forma polar de z utilizando la fórmula de Euler es
z = reiθ , donde r y θ son sus coordenadas polares (θ no necesariamente es el argumento
principal de z). En el siguiente ejemplo se emplea la fórmula de Euler para expresar
algunos números complejos en su forma polar.
Ejemplo 1.4. La forma polar, utilizando la fórmula de Euler, de los números complejos
z1 = 1 + i , z2 = −1 + i , z3 = −1 − i, y z4 = 1 − i, es
√ √ √ √
z1 = 2 eiθπ/4 , z2 = 2 eiθ3π/4 , z3 = 2 e−iθ3π/4 , z4 = 2 e−iθπ/4 .
Observe que en la forma polar de los números dados se utilizó el argumento principal,
pero pudiera también emplearse cualquier valor del argumento. 
CAPÍTULO 1. NÚMEROS COMPLEJOS 13

1.6 Potencias y Raı́ces

Definición 1.7 (Potencia n-ésima). Sean z = r eiθ y n un entero positivo. La potencia


n-ésima de z, denotada por z n , se define como

z n = r n einθ = r n (cos nθ + i sen nθ).

Definición 1.8 (Raı́ces n-ésimas). Sean z = r eiθ y n un entero positivo. Las raı́ces
n-ésimas de z, denotadas por z 1/n , se definen como
    
1/n √n i θ+2kπ √
n
θ + 2kπ θ + 2kπ
z = r e n = r cos + i sen ,
n n

para k = 0, 1, . . . , n − 1, donde n r denota la raı́z n-ésima positiva del número real r.

Observación 1.2. En las definiciones de potencia n-ésima y raı́z n-ésima, se utiliza cuales-
quiera valor de θ, no es necesario que θ sea el argumento principal de z.

Ejemplo 1.5. Se deja como ejercicio para el lector verificar que



2 3π √ π +2kπ
4
(1 + i)3 = 23 ei 4 (1 + i)1/5 = 2 ei
10
y 5 , k = 0, 1, . . . , 4.

1.7 Regiones en el Plano Complejo


En esta sección se describen conjuntos especiales de números complejos. Comencemos
con dos conjuntos que utilizaremos con mucha regularidad.

Definición 1.9. Los conjuntos de números complejos

{z ∈ C : |z| = 1} y {z ∈ C : |z| < 1}

se denominan circunferencia unitaria y disco unitario, respectivamente.

En la Figura 1.6 se muestra la representación gráfica de la circunferencia unitaria y el


disco unitario. Note que la definición dada de circunferencia unitaria es equivalente a la
siguiente definición: el conjunto de puntos (x, y) ∈ R2 tales que x2 + y 2 = 1, que es una
circunferencia de centro 0 y radio 1.Ahora, una circunferencia de centro z0 ∈ C y radio Si z = x + iy,
entonces la ecua-
r > 0, se define como el conjunto {z ∈ C : |z − z0 | = r} o, simplemente, |z − z0 | = r. Este ción |z| = 1
último conjunto nos permite introducir nuevos conjuntos en el plano complejo. es equivalente
a la ecuación
x2 + y 2 = 1
Definición 1.10 (Vecindad). Dados z0 ∈ C y ε > 0, el conjunto de puntos denotado por
B(z0 , ε) = {z ∈ C : |z − z0 | < ε}, se denomina vecindad de z0 . En otras palabras, una
vecindad es un disco de centro z0 y radio ε > 0 o, equivalentemente, el interior de la
circunferencia de centro z0 y radio ε > 0.

La representación geométrica de una vecindad B(z0 , ε) se aprecia en la Figura 1.7.


14 1.7. REGIONES EN EL PLANO COMPLEJO

Circunferencia
unitaria

1
b

Disco
unitario

Figura 1.6. Representación gráfica de la circunferencia unitaria y el disco unitario

ε
b
z
b

z0

Figura 1.7. Vecindad de un número complejo

Definición 1.11 (Conjunto abierto). Se dice que un conjunto de números complejos S


es abierto, si para cada z ∈ S existe una vecindad de z, B(z, ε), tal que B(z, ε) ⊂ S.

Ejemplo 1.6. Los conjuntos S1 , S2 y S3 que se muestran en la Figura 1.8 son abiertos.

1 1 S2 1
S1
S3
b

−1 1 1 −1 1

−1
a) b) c)

Figura 1.8. Ejemplos de conjuntos abiertos

La lı́nea de puntos indica que los puntos de la misma no pertenecen al conjunto. Por
¿Todo conjunto ello, ninguno de los conjuntos S1 , S2 y S3 contiene su borde.Los conjuntos S1 , S2 y S3 se
del plano que
no contiene su
definen de la siguiente manera.
borde es abierto?
a) S1 = {z ∈ C : |z| < 1}

b) S2 es la región del plano complejo formada por los puntos interiores del cuadrado
cuyos vértices son los puntos z1 = 0, z2 = 1, z3 = 1 + i, y z4 = i. Analı́ticamente, S2
CAPÍTULO 1. NÚMEROS COMPLEJOS 15

es el conjunto puntos z = x+i y que satisfacen el siguiente sistema de inecuaciones:



0≤x≤1
0≤y≤1

c) S3 es la región del plano complejo formada por los puntos z = x + i y que satisfacen
el siguiente sistema de inecuaciones:

 x − y > −1
x+y < 1

y > 0

Definición 1.12 (Frontera de un conjunto). Sea S un conjunto de números comple-


jos. La frontera de S es el conjunto formado por los puntos z ∈ C tales que todas las
vecindades de z contienen puntos de S y puntos que no están en S.

Ejemplo 1.7. En la Figura 1.9 observamos un conjunto S y tres puntos z1 , z2 y z3 . En este


caso, z1 es un punto de la frontera de S, en cambio z2 y z3 no pertenecen a la frontera de
S (se deja al lector justificar esta afirmación).

S z1
b

z2
b

z3
b

Figura 1.9. Frontera de un conjunto

Ejercicio 1.2. Determine la expresión matemática de la frontera de los conjuntos S1 , S2 y


S3 definidos en el Ejemplo 1.6.

Definición 1.13 (Punto de acumulación). Sea S un conjunto de números complejos. Se


dice que z0 es un punto de acumulación si toda vecindad de z0 contiene por lo menos un
punto de S diferente de z0 .

Ejemplo 1.8. Considere el conjunto S de la Figura 1.9. Aquı́ observamos que z1 y z2 son
puntos de acumulación de S, pero z3 no es un punto de acumulación de S, ya que existe
ε > 0 tal que B(z3 , ε) ∩ S = ∅. 

Definición 1.14 (Conjunto cerrado). Se dice que un conjunto de números complejos S


es cerrado, si todos sus puntos de acumulación pertenecen a él. De forma equivalente,
S es cerrado si contiene a su frontera.
¿Todo conjunto
del plano que
contiene su
borde es ce-
rrado?
16 1.7. REGIONES EN EL PLANO COMPLEJO

Ejemplo 1.9. Considere todos los conjuntos mostrados en las Figuras 1.7, 1.8 y 1.9. Su-
ponga que con estos conjuntos se forman nuevos conjuntos que contienen los puntos de la
frontera. Estos últimos conjuntos son todos cerrados. 

Definición 1.15 (Conjunto conexo). Se dice que un conjunto de números complejos S


es conexo, si dados dos puntos cualesquiera de S, existe una trayectoria formada por
segmentos de recta que los une y cuyos puntos pertenecen todos a S.

Ejemplo 1.10. En la Figura 1.10 observamos dos conjuntos S1 y S2 . El conjunto S1 es


conexo, en cambio S2 no lo es. (¿Por qué?)

S1 S2

Figura 1.10. Conjuntos conexo y no conexo




Definición 1.16 (Dominio). Se dice que un conjunto de números complejos S es un


dominio, si S es abierto y conexo.

Ejemplo 1.11. Todos los conjuntos mostrados en las Figuras 1.7, 1.8 y 1.9 son dominios.
Asimismo, el conjunto S1 de la Figura 1.10 también es un dominio. (¿Por qué?) 

Definición 1.17 (Conjuntos acotado y no acotado). Se dice que un conjunto de números


complejos S es acotado, si existe un número real R > 0 tal que todo punto de S queda
dentro de la circunferencia |z| = R. Por el contrario, si |z| > R para todo R > 0 y algún
z ∈ S, se dice que S es no acotado.
¿Todo conjunto
cerrado es aco-
tado? Ejemplo 1.12. En la Figura 1.11 observamos dos conjuntos S1 y S2 . El conjunto S1 es
acotado y S2 es no acotado.

S2
S1

Figura 1.11. Conjuntos acotado y no acotado



CAPÍTULO 1. NÚMEROS COMPLEJOS 17

1.8 Aritmética Compleja con M ATLAB


Las operaciones con números complejos en M ATLAB son muy fáciles de hacer, ya que
este software trabaja con aritmética compleja. Ver Capı́tulo A para una introducción a
M ATLAB o el libro de Higham y Higham [4].
Un número complejo z = x + iy se representa en M ATLAB usando en esencia su misma
estructura, por ejemplo, los números complejos z1 = 1 + 3i y z2 = −1 − 2i se definen en
M ATLAB como:

>> z1 = 1+3i , z2 = −1−2i
z1 =
1.0000 + 3.0000 i
z2 =
−1.0000 − 2.0000 i
✂ ✁
Para M ATLAB , i o j representan la unidad imaginaria. Ahora, para las operaciones
aritméticas de suma, resta, división y multiplicación, se utilizan respectivamente los si-
guientes sı́mbolos: +, -, / y *. Por ejemplo, considerando los números complejos z1 = 1+3i
y z2 = −1 − 2i, las operaciones z1 + z2 , z1 − z2 , z1 z2 y z1 /z2 , se hacen en M ATLAB como:

>> z1+z2
ans =
0 + 1.0000 i
>> z1−z2
ans =
2.0000 + 5.0000 i
>> z1 ∗ z2
ans =
5.0000 − 5.0000 i
>> z1 / z2
ans =
−1.4000 − 0.2000 i
✂ ✁

Los comandos: real(z), imag(z), conj(z), abs(z) y angle(z), se utilizan para cal-
cular la parte real, la parte imaginaria, el conjugado, el módulo y el argumento de z,
respectivamente. Observe el siguiente ejemplo. ¿Qué resul-
tado arrojarı́a

el comando
>> z = 1+i /2
angle(i)?
z =
1.0000 + 0.5000 i
>> r e a l ( z )
ans =
1
>> imag ( z )
ans =
0.5000
>> c o n j ( z )
ans =
1.0000 − 0.5000 i
>> abs ( z )
ans =
1.1180
>> a n g l e ( z )
ans =
0.4636
✂ ✁
18 1.8. ARITMÉTICA COMPLEJA CON MATLAB

Note que el valor angle(z) está expresado en radianes, además, es el argumento principal
de z. Observe el siguiente ejemplo.

>> z= 1+i ; a n g l e ( z )
ans =
0.7854
>> z= −1+i ; a n g l e ( z )
ans =
2.3562
>> z= −1−i ; a n g l e ( z )
ans =
−2.3562
>> z= 1− i ; a n g l e ( z )
ans =
−0.7854
✂ ✁

Para más detalles El comando plotpuede usarse para ver la representación gráfica de un número com-
de plot, teclee
help plot en
plejo como un par ordenado. Por ejemplo, con las instrucciones
el escritorio de z = 1/3 + s q r t (2)∗ i ;
M ATLAB p l o t ( z , ’ ob ’ , ’ MarkerEdgeColor ’ , ’ b ’ , ’ MarkerFaceColor ’ , ’ b ’ , ’ MarkerSize ’ , 5 )
t e x t ( 0 . 4 , 1 . 6 , ’ $z = \ f r a c {1}{3} + \ s q r t {2}\ , i $ ’ , ’ F o n t S i z e ’ , 1 4 , . . .
’ Backgrou ndColor ’ , ’w ’ , ’ I n t e r p r e t e r ’ , ’ l a t e x ’ )
xlabel ( ’ x ’ )
ylabel ( ’ y ’ )
g r i d on
a x i s square

se genera la gráfica de la Figura 1.12, donde se representa al número complejo z = 31 + 2i
como un par ordenado.
2.5

1

z= 3 + 2i
1.5
y

0.5

0
−1 −0.5 0 0.5 1 1.5
x

1

Figura 1.12. Representación como un par ordenado del número z = 3 + 2i

El comando compass(z) dibuja a z = x + i y como un vector con extremo inicia en el


origen y extremo final en el par ordenado (x, y). Las instrucciones
zz = [1+i , −1−i , −(1/2)+ i , 1− i ] ;
compass ( zz )

se utilizaron para representar como vectores a los números complejos z1 = 1 + i, z2 =


Para más deta- −1 − i, z3 = − 12 + i, z4 = 1 − i, cuya gráfica se aprecia en la Figura 1.13.
lles de compass,
teclee help
compass en el
escritorio de M A -
TLAB
CAPÍTULO 1. NÚMEROS COMPLEJOS 19

90
1.5
120 60

150 30

0.5

180 0

210 330

240 300

270

Figura 1.13. Representación como vectores de los números complejos z1 = 1 + i, z2 =


−1 − i, z3 = − 12 + i, z4 = 1 − i

Conocidas las coordenadas polares, (r, θ), de un número complejo z, se puede utilizar
la función exp para hallar la forma cartesiana de z. Por ejemplo, la forma cartesiana,
z = x + iy, del número complejo z = 2 eiπ/3 , que está representado en forma polar usando
la fórmula de Euler, se halla de la siguiente manera:

>> z = 2∗ exp ( i ∗ p i /3)
z =
1.0000 + 1.7321 i
✂ ✁
El comando z^n permite calcular la potencia enésima del número complejo z. Observe
el siguiente ejemplo donde se calcula (1 + i)5 .

>> (1+ i )ˆ5
ans =
−4.0000 − 4.0000 i
✂ ✁
En cambio, con el comando z^(1/n) se puede calcular un valor de la raı́z enésima de z.
Observe el siguiente ejemplo donde se calcula un valor de (1 + i)1/5 .

>> (1+ i ) ˆ ( 1 / 5 )
ans =
1.0586 + 0.1677 i
✂ ✁
Ahora, el Programa 1.1 muestra la función raizn que calcula las n raı́ces enésimas de z.

Programa 1.1. Función raizn.m

f u n c t i o n szn = r a i z n ( z , n )
%r a i z n H a l l a l a s n r aı́ c e s de z .
% szn e s un a r r e g o que c o n t i e n e l a s n r aı́ c e s de z

t h e t a = angle ( z ) ;
r = abs ( z ) ;
k = 0 : 1 : n−1;
szn = ( r ˆ(1/n ) ) ∗ exp ( ( t h e t a + 2∗k∗ p i )∗ i /n ) ;
end
20 1.9. PROBLEMAS RESUELTOS

A continuación se utiliza la función raizn para calcular las raı́ces quintas de z = 1 + i.



>> r a i z n (1+ i , 5 )
ans =
1.0586 + 0.1677 i 0.1677 + 1.0586 i −0.9550 + 0.4866 i −0.7579 − 0.7579 i
0.4866 − 0.9550 i
✂ ✁

Ejercicio 1.3. Emplee convenientemente las funciones √ compass y raizn para obtener la
gráfica de las raı́ces séptimas de la unidad, esto es, 7 1.

1.9 Problemas Resueltos


Problema 1.1. Sean z1 y z2 números complejos tales que z1 + z2 es un número real dife-
rente de cero y Re (z1 − z2 ) = 0. Pruebe que z12 − z22 es un número complejo puro.

Solución. Sean z1 = x1 + iy1 y z2 = x2 + iy2 . Como z1 + z2 = (x1 + x2 ) + i(y1 + y2 ) es


un número real diferente de cero y Re (z1 − z2 ) = x1 − x2 = 0, se tiene que y1 = −y2 y
x1 = x2 . Ası́,
z12 − z22 = (z1 + z2 )(z1 − z2 ) = (2x1 )(2y1 i) = 4x1 y1 i,
en otras palabras, z12 − z22 es un número complejo puro. 

√ √
Problema 1.2. Demuestre que |(2 z + 5)( 2 − i)| = 3|2z + 5|.

Solución. Utilizando la definición y las propiedades del módulo podemos escribir:


√ √ √ √
|(2 z + 5)( 2 − i)| = |2 z + 5| | 2 − i| = 2z + 5 2 + 1 = 3|2z + 5|.

Problema 1.3. Sean z1 y z2 números complejos. Si |z1 | =


6 |z2 |, entonces demuestre que

|z1 + z2 | ≥ ||z1 | − |z2 ||.

Solución. Por la Proposición 1.3, podemos escribir:

|z1 + z2 |2 = (z1 + z2 )(z1 + z2 ) = |z1 |2 + z1 z 2 + z2 z 1 + |z2 |2



= |z1 |2 + z1 z 2 + z1 z 2 + |z2 |2 = |z1 |2 + 2Re (z1 z 2 ) + |z2 |2
≥ |z1 |2 − 2|z1 z 2 | + |z2 |2 = |z1 |2 − 2|z1 | |z2 | + |z2 |2
= (|z1 | − |z2 |)2 ,

de donde se deduce que |z1 + z2 | ≥ ||z1 | − |z2 ||. 

Problema 1.4. Sean z1 , z2 y z3 números complejos. Si |z2 | 6= |z3 |, entonces demuestre que

z1 |z1 |
≤ .
z2 + z3 ||z2 | − |z3 ||
CAPÍTULO 1. NÚMEROS COMPLEJOS 21

Solución. Tomando z1 = z2 y z2 = z3 en el Problema 1.3, obtenemos:

|z2 + z3 | ≥ ||z2 | − |z3 ||,

de donde se deduce que


1 1
≤ ,
|z2 + z3 | ||z2 | − |z3 ||
ahora, multiplicando por |z1 | en ambos lados de la desigualdad anterior se tiene
|z1 | |z1 |
≤ ,
|z2 + z3 | ||z2 | − |z3 ||
lo que implica
z1 |z1 |
≤ ,
z2 + z3 ||z2 | − |z3 ||
que era lo que deseábamos demostrar. 

Problema 1.5. Si z1 z2 6= 0, aplicar la forma polar para demostrar que Re (z1 z2 ) = |z1 | |z2 |
si, y sólo si θ1 + θ2 = 2nπ, (n = 0, ±1, ±2, . . .), donde θ1 = arg z1 y θ2 = arg z2 .

Solución. En la demostración utilizamos el sı́mbolo “⇔” que denota equivalencia. Sean


z1 = r1 (cos θ1 + i sen θ1 ) y z2 = r2 (cos θ2 + i sen θ2 ). Ası́, tenemos que r1 = |z1 |, r2 = |z2 | y

Re (z1 z2 ) = r1 r2 (cos θ1 cos θ2 − sen θ1 sen θ2 ) .

Ahora, usando la ecuación anterior podemos escribir:

Re (z1 z2 ) = |z1 | |z2 | ⇔ r1 r2 (cos θ1 cos θ2 − sen θ1 sen θ2 ) = r1 r2


⇔ cos θ1 cos θ2 − sen θ1 sen θ2 = 1
⇔ cos(θ1 + θ2 ) = 1
⇔ θ1 + θ2 = 2nπ, para n = 0, ±1, ±2, . . .

Problema 1.6. Demostrar que la identidad

1 − z n+1
1 + z + z2 + · · · + zn = ,
1−z
se satisface para todo entero n ≥ 1 y todo z 6= 1.

Solución. Tenemos que

(1 + z + z 2 + · · · + z n )(1 − z) = 1 − z n+1 , para todo z ∈ C.

Como z 6= 1, entonces dividiendo por (1 − z) en ambos lados de la ecuación anterior


obtenemos:
1 − z n+1
1 + z + z2 + · · · + zn = ,
1−z
para todo entero n ≥ 1 y todo z 6= 1, que era lo que deseábamos demostrar. 
22 1.10. PROBLEMAS PROPUESTOS

1.10 Problemas Propuestos


1.1. Exprese los siguientes números complejos en la forma cartesiana x + iy:

a) (2 + 3i) + (4 + i) e) (8 + 6i)3
 2
2 + 3i 3
b) f) 1 +
4+i 1+i
1 3 1 + 2i 2 − i
c) + g) +
i 1+i 3 − 4i 5i
d) (2 + 3i)(4 + i) h) (1 − i)4

1.2. Encuentre las soluciones de las siguientes ecuaciones:

a) z 2 = 3 − 4i b) (z + 1)2 = 3 + 4i

1.3. Simplifique las siguientes expresiones:



a) (−i)−1 d) 1 − 2i
b) (1 − i)−1
1+i 1
c) e) √
1−i 1+ −2i

1.4. Resuelva las siguientes ecuaciones:

a) z 5 − 2 = 0 c) z 6 + 8 = 0
b) z 4 + i = 0 d) z 3 − 4 = 0

1.5. Determine el conjugado de los siguientes números complejos:

(3 + 8i)4 i(2 + 3i)(5 − 2i)


a) c)
(1 + i)10 (−2 − i)
(1 − 2i)10 (2 − 3i)2
b) d)
(2 + 2i)5 (8 + 6i)2

1.6. Calcule el módulo de los siguientes números complejos:

(3 + 8i)4 i(2 + 3i)(5 − 2i)


a) c)
(1 + i)10 (−2 − i)
(1 − 2i)10 (2 − 3i)2
b) d)
(2 + 2i)5 (8 + 6i)2

1.7. Sea z ∈ C tal que |z| = 1. Entonces, calcular

|1 + z|2 + |1 − z|2 .
CAPÍTULO 1. NÚMEROS COMPLEJOS 23

1.8. Encontrar todos los valores del argumento de z cuando el número complejo z está
dado por:
√ √
i 3 + 3 + (3 − 3)i
a) z = c) z =
i−1 3 + 3i
 √ √ 
(2 3 + 1) + ( 3 − 2) i (3 + i)
b) z =
5 − 5i

1.9. Exprese las caracterı́sticas de los siguientes conjuntos de puntos, en cuanto a:


cerrado o abierto, conexo o no, acotado o no.

a) {z ∈ C : Re z = −2} f) {z ∈ C : 0 < |z − 2 + 1| < 2}


b) {z ∈ C : Re z ≤ Im z} g) {z ∈ C : −1 < Re z < 1}
c) {z ∈ C : zz > 2Re z} h) {z ∈ C : |arg z| < π/2}
d) {z ∈ C : |z − 3 + 4i| ≤ 6} i) {z ∈ C : Re (z − i) = 2}
e) {z ∈ C : |z − 2 + 5i| ≤ 0} j) {z ∈ C : |z + 2i| + |z − 2i| ≤ 10}
2
Funciones de Variable Compleja
Las funciones de variable compleja poseen muchas aplicaciones en distintas áreas de la
Ingenierı́a, por ejemplo, en la teorı́a de corrientes alternas, el movimiento de fluidos o el
procesamiento de señales. En este capı́tulo se presentan los fundamentos matemáticos de
las funciones de variable compleja. Se presta mucha atención a la definición y a las pro-
piedades de continuidad y diferenciación de una función de variable compleja. Asimismo,
se describe un procedimiento para transformar regiones del plano complejo a través de
funciones lineales y el cociente de funciones lineales.

2.1 Definición
Una función f de variable compleja es una regla de asignación que le hace corresponder
a un número complejo z = x + i y uno o varios números complejos w = u + i v. El
número w se llama valor o imagen de f en z y se designa por f (z), es decir, w = f (z) o,
equivalentemente, u + i v = f (x + i y).

Definición 2.1 (Funciones monovaluadas y multivaluadas). Sea f (z) una función de


variable compleja. Si a cada número complejo z la función f (z) le hace corresponder
una y sólo una imagen w, se dice que f (z) es monovaluada. Ahora, si la función f
le hace corresponder a z más de una imagen, digamos w1 , w2 , . . ., se dice que f (z) es
multivaluada.

En la Figura 2.1 se muestra una representación gráfica, a través de diagramas, de


¿Todas las fun- funciones monovaluadas y multivaluadas.
ciones vistas
en los cursos
de Cálculo son
monovaluadas? w = f (z) w = f (z) b w1
b b b
b w2
z w z ..
.
b

wn

Función monovaluada Función multivaluada

Figura 2.1. Funciones monovaluada y multivaluada

Definición 2.2 (Dominio y rango). El conjunto de números complejos z en donde la


función f (z) está bien definida (no hay división por cero) se denomina dominio de f (z).
El conjunto de números complejos conformado con todas las imágenes w = f (z) es
llamado rango de f (z).

24
CAPÍTULO 2. FUNCIONES DE VARIABLE COMPLEJA 25

Definición 2.3 (Polinomio complejo). Sean n ≥ 0 un entero y a0 , a1 , . . . , an constantes


complejas. La función

p(z) = a0 + a1 z + a2 z 2 + · · · + an z n , (an 6= 0)

se denomina polinomio complejo o, simplemente, polinomio de grado n.

Ejercicio 2.1. Demuestra que tanto el dominio como el rango de un polinomio de grado
n, es todo el plano complejo. Recuerde el teo-
rema fundamen-
tal del Álgebra:
Definición 2.4 (Función racional). Sean p(z) y q(z) polinomios. La función r(z) dada todo polinomio
por de una variable
no constante con
p(z) coeficientes com-
r(z) =
q(z) plejos tiene una
raı́z compleja
se denomina función racional y está definida en todo número complejo z, excepto donde
q(z) = 0, esto es, el conjunto {z ∈ C : q(z) 6= 0} que no contiene ninguna de las raı́ces
del polinomio q(z).

Ejemplo 2.1. Determinar el dominio y el rango de la función

z+1
r(z) = .
z−i

Solución. Se tiene que el dominio de r(z) es el conjunto de número complejos z tales que
r(z) está bien definida, es decir, el conjunto de números complejos que no produzcan una
división por 0, que en este caso es {z ∈ C : z 6= i}. Ahora, para determinar el rango de r(z)
se utiliza el siguiente procedimiento. Se toma w = r(z) y luego se despeja a z en función
de w. A la función obtenida (la inversa de r(z)), que depende de w, se le determina el
dominio. Este último conjunto de números complejos constituye el rango de r(z). Ası́, se
tiene
z+1
w= ,
z−i
despejando z en función de w, se obtiene

1 − iw
z= ,
w−1

de lo cual se deduce que el rango de r(z) es {w ∈ C : w 6= 1}. 

2.2 Lı́mite y Continuidad


Pasemos ahora a estudiar las propiedades de las funciones de variable compleja relacio-
nadas con el análisis infinitesimal, esto es, lı́mite, continuidad y derivabilidad. El objetivo
de esta parte es utilizar los resultados del análisis infinitesimal de variable real para de-
sarrollar los conceptos de lı́mite, continuidad y diferenciación de funciones de variable
compleja. Para ello, inicialmente se introduce el concepto de funciones componentes.
26 2.2. LÍMITE Y CONTINUIDAD

2.2.1 Funciones Componentes


Toda función de variable compleja se puede expresar en términos de una par de fun-
ciones de variable real monovaluadas. Esto es, si f (z) es una función de variable compleja
y z = x + i y, entonces f (z) se puede expresar como

f (z) = u(x, y) + i v(x, y),

donde u : R2 → R y v : R2 → R, se denominan funciones componentes de f (z).

Ejemplo 2.2. Las funciones componentes, u(x, y) y v(x, y), de f (z) = z 2 + a, donde a =
a1 + i a2 , están dadas por: u(x, y) = x2 − y 2 + a1 y v(x, y) = 2xy + a2 . Verifique esta
afirmación. 

1
Ejemplo 2.3. Las funciones componentes, u(x, y) y v(x, y), de f (z) = z + están dadas
z
por
x(x2 + y 2 + 1) y(x2 + y 2 − 1)
u(x, y) = , v(x, y) = .
x2 + y 2 x2 + y 2
Verifique esta afirmación. 

2.2.2 Lı́mite
Definición 2.5 (Lı́mite). Sea f (z) una función definida en todos los puntos de cierta
vecindad de z0 excepto, posiblemente en el mismo z0 . Se dice que w0 es un lı́mite
de w = f (z) cuando z tiende a z0 , si para cada número positivo ε, existe un número
positivo δ tal que

|f (z) − w0 | < ε, siempre que 0 < |z − z0 | < δ.

Denotamos con
lı́m f (z) = w0
z→z0

para indicar que w0 es un lı́mite de f (z), cuando z tiende a z0 .


Note que la defi-
nición de lı́mite
de una función
Algunas de las fórmulas de lı́mites que se estudiaron en el cálculo elemental tienen sus
de variable com- homólogas en el caso de funciones de variable compleja. Las fórmulas equivalentes, que
pleja es una ex-
se presentan aquı́, se demuestran usando la definición de lı́mite.
tensión del lı́mite
de una función
de variable real. Teorema 2.1. Sean f (z) y g(z) funciones de variable complejas tales que lı́m f (z) = w0
z→z0
y lı́m g(z) = W0 . Entonces,
z→z0

1. lı́m [f (z) + g(z)] = w0 + W0 ;


z→z0

2. lı́m [f (z) · g(z)] = w0 · W0 ;


z→z0

w0
3. lı́m [f (z)/g(z)] = , W0 6= 0.
z→z0 W0
CAPÍTULO 2. FUNCIONES DE VARIABLE COMPLEJA 27

Demostración. Sea ε > 0. Como w0 y W0 son respectivamente los lı́mites de f (z) y g(z)
cuando z tiende a z0 , podemos escribir:
ε
|f (z) − w0 | < , siempre que 0 < |z − z0 | < δ1 ,
2
y
ε
|g(z) − W0 | < , siempre que 0 < |z − z0 | < δ2 ,
2
donde δ1 > 0 y δ2 > 0 dependen de ε. Ası́, por la desigualdad triangular tenemos que

|[f (z) + g(z)] − [w0 + W0 ]| ≤ |f (z) − w0 | + |g(z) − W0 | < ε,

siempre que 0 < |z − z0 | < δ, donde δ ≤ mı́n(δ1 , δ2 ). Por lo tanto, w0 + W0 es el lı́mite de


f (z) + g(z) cuando z tiende a z0 .
Supongamos que
r
ε
|f (z) − w0 | < , siempre que 0 < |z − z0 | < δ3 ,
2
y r
ε
|g(z) − W0 | < , siempre que 0 < |z − z0 | < δ4 ,
2
donde δ3 > 0 y δ4 > 0 dependen de ε. Como W0 (f (z) − w0 ) = W0 f (z) − W0 w0 y w0 (g(z) −
W0 ) = w0 g(z) − w0 W0 , entonces es claro que lı́m W0 f (z) = W0 w0 y lı́m w0 g(z) = w0 W0 ;
z→z0 z→z0
luego, por 1., lı́m (W0 f (z) − w0 g(z)) = 0. Ahora, tenemos que
z→z0

f (z)g(z) − w0 W0 = (f (z) − w0 )(g(z) − W0 ) − (W0 f (z) − w0 g(z)).

Ası́, por todo lo anterior podemos escribir

|f (z)g(z) − w0 W0 | = |(f (z) − w0 )(g(z) − W0 ) − (W0 f (z) − w0 g(z))|


≤ |(f (z) − w0 )| |(g(z) − W0 )| + |(W0 f (z) − w0 g(z))|
r r
ε ε ε
< · + = ε,
2 2 2

siempre que 0 < |z − z0 | < δ, donde δ ≤ mı́n(δ3 , δ4 ). En otras palabras, w0 W0 es el lı́mite


de f (z)g(z) cuando z tiende a z0 .
La demostración de la identidad 3., se deja como ejercicio para el lector.

El siguiente teorema muestra la relación entre el lı́mite de una función de variable


compleja y el lı́mite de sus funciones componentes que, por supuesto, son funciones reales.

Teorema 2.2. Sean f (z) = u(x, y) + i v(x, y) una función de variable compleja y los
números complejos z0 = x0 + i y0 y w0 = u0 + i v0 . Entonces,

lı́m f (z) = w0
z→z0

si, y sólo si
lı́m u(x, y) = u0 y lı́m v(x, y) = v0 .
(x,y)→(x0 ,y0 ) (x,y)→(x0 ,y0 )
28 2.2. LÍMITE Y CONTINUIDAD

Demostración. Sean f1 (z) y f2 (z) funciones de variable compleja definidas respectivamente


por
f1 (z) = u(x, y) y f2 (z) = v(x, y).
Es claro que f (z) = f1 (z) + i f2 (z), además, si lı́m f (z) = w0 , entonces por el Teorema 2.1
z→z0
tenemos que

lı́m f1 (z) = u0 = lı́m u(x, y) y lı́m f2 (z) = v0 = lı́m v(x, y).


z→z0 (x,y)→(x0 ,y0 ) z→z0 (x,y)→(x0 ,y0 )

Ahora, si

lı́m u(x, y) = lı́m f1 (z) = u0 y lı́m v(x, y) = lı́m f2 (z) = v0 ,


(x,y)→(x0 ,y0 ) z→z0 (x,y)→(x0 ,y0 ) z→z0

entonces, por el Teorema 2.1 obtenemos lı́m f (z) = w0 .


z→z0

2.2.3 Continuidad
La continuidad es uno de los conceptos más importantes del análisis. A continuación
veremos que la continuidad de una función de variable compleja se puede ver como una
Observe que la extensión del concepto de continuidad de una función de variable real.
definición con-
tinuidad de una
función com- Definición 2.6. Se dice que una función f (z) es continua en z0 , si satisface las dos
pleja, es una condiciones siguientes:
extensión de la
definición de
continuidad de (i) el valor f (z0 ) está bien definido;
un función real.
(ii) lı́mz→z0 f (z) existe y
lı́m f (z) = f (z0 ).
z→z0

Se dice que f (z) es continua en un dominio D, si es continua en todo z ∈ D.

En el siguiente ejemplo se utiliza la definición de continuidad combinada con los teo-


remas de lı́mite, para identificar el conjunto de números complejos donde una función es
continua.
Ejemplo 2.4. Estudiemos la continuidad de la función
 2
 z − 1 , z 6= 1,
f (z) = z − 1

1, z = 1.

Tomemos z = x + i y. Para z 6= 1, la función f (z) se puede expresar como


(z − 1)(z + 1)
f (z) = = z + 1 = (x + 1) + i y,
(z − 1)
luego, las funciones componentes de f (z) son u(x, y) = x + 1 y v(x, y) = y. Como el valor
de f (z0 ) está bien definido para todo z0 = x0 + i y0 6= 1, entonces por el Teorema 2.2
podemos escribir:

lı́m f (z) = lı́m u(x, y) + i lı́m v(x, y) = x0 + 1 + i y0 = z0 + 1 = f (z0 ),


z→z0 (x,y)→(x0 ,y0 ) (x,y)→(x0 ,y0 )
CAPÍTULO 2. FUNCIONES DE VARIABLE COMPLEJA 29

en otras palabras, f (z) es continua en todo z0 6= 1.


Para z = 1, se tiene que f (1) está bien definido, pero lı́m f (z) = 2 6= f (1) = 1. Por lo
z→1
tanto, f (z) no es continua en z = 1. 

Pasemos ahora a ver algunos teoremas de continuidad. El siguiente teorema dice que
las funciones definidas como la suma, multiplicación o división de funciones continuas,
son también funciones continuas en su dominio de definición. La demostración de este
teorema se obtiene utilizando el Teorema 2.1.
Teorema 2.3. Sean f (z) y g(z) dos funciones de variable compleja continuas en un domi-
nio D. Entonces:

1. la función f (z) + g(z) es continua en D;

2. la función f (z)g(z) es continua en D;

3. la función f (z)/g(z) es continua en D \ {z ∈ D : g(z) = 0}.

Observación 2.1. El Teorema 2.3 nos permite asegurar que un polinomio es una función
continua en todo el plano complejo; también nos permite aseverar que una función racio-
nal es continua en todo el plano complejo excepto donde su denominador se anule.
En el análisis real se tiene que la composición de funciones de variable real continuas
produce una nueva función continua. En el caso de las funciones de variable compleja
también se satisface este hecho, es decir, la composición de funciones de variable compleja
continuas es continua. Esta afirmación se plantea en el siguiente teorema, cuya demos-
tración se obtiene directamente de la definición de continuidad. En general, los
resultados de
Teorema 2.4. Sean f (z) y g(z) dos funciones de variable compleja definidas respectiva- continuidad para
funciones com-
mente en los dominios D y E, tales que f (D) ⊆ E. Si f es continua en D y g es continua plejas son ex-
en f (D), entonces la función h(z) = g(f (z)) es continua en D. tensiones de los
resultados de
continuidad para
Por otra parte, dado que el lı́mite de una función de variable compleja se puede calcular funciones reales.
a través del lı́mite de sus funciones componentes (Teorema 2.2), es natural inferir que la
continuidad de una función de variable compleja se corresponde con la continuidad de
sus funciones componentes. Seguidamente se muestra un teorema que trata este aspecto,
cuya demostración es inmediata utilizando la definición de continuidad de funciones reales
conjuntamente con el Teorema 2.2.

Teorema 2.5. Sea f (z) = u(x, y) + i v(x, y) una función de variable compleja. Entonces,
f (z) es continua en un punto z0 = x0 + i y0 si, y sólo si sus funciones componentes, u(x, y)
y v(x, y), son continuas en el punto (x0 , y0 ).

A continuación damos un ejemplo que muestra la aplicación de los Teoremas 2.3 y 2.5,
en el estudio de la continuidad de una función de variable compleja.
Ejemplo 2.5. Estudiemos la continuidad en z = 1 de la función
x − iy
f (z) = 2x + iy + .
x2 + y 2
Verifiquemos, a través de dos procedimientos, la continuidad de f (z). Primero, utilizando
el Teorema 2.3; después, empleando el Teorema 2.5. Escribamos a f (z) en función de z,
30 2.2. LÍMITE Y CONTINUIDAD

para ello consideremos las siguientes expresiones de x e y,

z+z z−z
x= , y= .
2 2i

Sustituyendo convenientemente estas expresiones en la fórmula de definición de f (z),


obtenemos:

   z+z
z+z z−z 2 + i z−z
2i
f (z) = 2 +i +
2 2i zz
z−z z
= z+z+ +
2 zz
3 1 1
= z+ z+ .
2 2 z

Definamos tres funciones f1 (z), f2 (z) y f3 (z) como

3 1 1
f1 (z) = z, f2 (z) = z, f3 (z) = .
2 2 z

Las funciones f1 (z) y f3 (z) son, respectivamente, un polinomio de grado uno y una función
racional. Ası́, f1 (z) es continua en todo el plano, particularmente en z = 1, y f3 (z) es
continua en todo el plano excepto en z = 0, por lo tanto f3 (z) es continua en z = 1. En
cuanto a la función f2 (z), se deja como ejercicio para el lector verificar que es continua
en z = 1 (ayuda: utilice el Teorema 2.5). En consecuencia, como f1 (z), f2 (z) y f3 (z) son
continuas en z = 1 y f (z) = f1 (z) + f2 (z) + f3 (z), entonces por el Teorema 2.3 se tiene
que f (z) es continua en z = 1.
Ahora empleemos el Teorema 2.5 para estudiar la continuidad de f (z) en z = 1. Para
ello es necesario encontrar las funciones componentes, u(x, y) y v(x, y), de f (z). Operando
obtenemos:
   
x y
f (z) = 2x + 2 +i y− 2 ,
x + y2 x + y2

de donde se deduce que

x y
u(x, y) = 2x + y v(x, y) = y − .
x2 + y2 x2 + y2

Como u y v están bien definidas en el punto (1, 0) y

lı́m u(x, y) = 3 = u(1, 0), lı́m v(x, y) = 0 = v(1, 0),


(x,y)→(0,1) (x,y)→(0,1)

podemos concluir que u y v son continuas en (1, 0). Entonces, por el Teorema 2.5 la función
¿Qué procedi- f (z) es continua en z = 1. 
miento le pa-
reció más efi-
ciente, usar el
Teorema 2.3 o
el Teorema 2.5?
¿Por qué?
CAPÍTULO 2. FUNCIONES DE VARIABLE COMPLEJA 31

2.3 Diferenciación
.
Inmediatamente damos la definición de derivada de una función de variable compleja. ¿Por qué en la
definición de de-
rivada es necesa-
Definición 2.7 (Derivada). Sea f (z) una función de variable compleja definida en un rio que z0 sea un
dominio D. Sea z0 un punto de acumulación de D. La derivada de f (z) en z0 , denotada punto de acumu-
lación?
d
por f (z0 ) o f ′ (z0 ), es una función de variable compleja que se define mediante la
dz
ecuación
d f (z0 + h) − f (z0 )
f (z0 ) = f ′ (z0 ) = lı́m , (2.1)
dz h→0 h
donde h es un número complejo. Se dice que f (z) es derivable en un dominio D ⊂ C, si
f ′ (z) existe en cada punto z ∈ D.

Teorema 2.6. Si la derivada de f (z) en z0 existe, entonces f (z) es continua en z0 .

Demostración. Como la derivada de f (z) existe en z0 , entonces |f ′ (z0 )| < ∞ y f (z0 ) está
bien definido. Por tanto,

lı́m |f (z0 + h) − f (z0 )| = lı́m |h| |f ′ (z0 )| = 0,


h→0 h→0

de donde se deduce que f (z) es continua en z0 .

Observación 2.2. El Teorema 2.6 nos garantiza que toda función derivable es continua. Lo
que no es cierto es que toda función continua es derivable; por ejemplo, f (z) = |z|2 es
continua en todo el plano complejo, pero es derivable solamente en z = 0 (se deja al lector
verificar este hecho).

2.3.1 Fórmulas o Reglas de Diferenciación


A continuación se describen las mismas reglas de diferenciación dadas en el Cálculo
Elemental, pero ahora se plantean para funciones de variable compleja. Supongamos que
las funciones f (z), f1 (z), . . . , fn (z) son derivables en z ∈ C. Las siguientes propiedades
son ciertas y su demostración se obtiene fácilmente utilizando la definición de derivada y
los Teoremas 2.1 y 2.2. Tenga presente
que para usar
las reglas de
1. Si f (z) = c, entonces f ′ (z) = 0, donde c ∈ C. derivación, las
funciones involu-
cradas deben ser
2. Si h(z) = c f (z), entonces h′ (z) = c f ′ (z), donde c ∈ C. derivables.

3. Si f (z) = z, entonces f ′ (z) = 1.

4. [f1 (z) + f2 (z) + · · · + fn (z)]′ = f1′ (z) + f2′ (z) + · · · + fn′ (z).

5. [f1 (z)f2 (z)]′ = f1′ (z)f2 (z) + f2′ (z)f1 (z).

6. [(f (z))m ]′ = m (f (z))m−1 f ′ (z), donde m es un entero.

7. Si f (z) = z m , entonces f ′ (z) = m z m−1 , donde m es un entero.


32 2.3. DIFERENCIACIÓN

8. Si f (z) = a0 + a1 z + a2 z 2 + · · · + am z m , entonces

f ′ (z) = a1 + 2a2 z + 3a3 z 2 + · · · + m am z m−1 .


 ′
f1 (z) f1′ (z)f2 (z) − f2′ (z)f1 (z)
9. = , siempre y cuando f2 (z) 6= 0.
f2 (z) (f2 (z))2
10. Regla de la Cadena. Sean f (z) derivable en z0 y g(w) derivable en f (z0 ). Entonces la
función h(z) = g(f (z)) es derivable en z0 , y

h′ (z0 ) = g′ (f (z0 )) f ′ (z0 ).

Note que todas las reglas de diferenciación se cumplen, si las funciones involucradas
son derivables. De esta forma, para aplicar alguna de estas reglas, es necesario primero
verificar que las funciones consideradas sean derivables. En el Cálculo Elemental, por lo
general, era relativamente sencillo verificar si la derivada de una función existı́a o no.
Por ejemplo, observando la gráfica de una función f : R → R se podı́a identificar los
puntos donde existı́a la derivada. Este procedimiento, que por demás es muy sencillo,
no se puede usar para verificar que una función de variable compleja es derivable. En la
siguiente sección damos un procedimiento para estudiar la derivabilidad de las funciones
de variable compleja.

2.3.2 Ecuaciones de Cauchy-Riemann


Si una función de variable compleja es derivable en un punto, sus funciones componen-
tes satisfacen las llamadas ecuaciones de Cauchy-Riemann en ese punto. A continuación
damos una breve deducción de estas ecuaciones, que son las condiciones necesarias para
Las ecuacio- la existencia de la derivada de una función de variable compleja.
nes de Cauchy-
Riemann
también se pue-
Teorema 2.7. Sea f (z) = u(x, y) + i v(x, y) derivable en z0 = x0 + i y0 . Entonces, las
den expresar en funciones componentes, u(x, y) y v(x, y), satisfacen las ecuaciones de Cauchy-Riemann en
la siguiente no-
el punto (x0 , y0 ), a saber:
tación más com-
pacta: ux = vy ,  ∂u
uy = −vx ∂v
 ∂x (x0 , y0 ) = (x0 , y0 )


∂y
(2.2)
 ∂u (x0 , y0 ) = − ∂v (x0 , y0 )


∂y ∂x

Demostración. Como f (z) = u(x, y) + i v(x, y) es derivable en z0 = x0 + i y0 , se tiene

f (z0 + h) − f (z0 )
f ′ (z0 ) = lı́m .
h→0 h
Ahora bien, utilizando las funciones componentes de f (z) y tomando h = h1 + i h2 , la
ecuación anterior se expresa como:

[u(x0 + h1 , y0 + h2 ) + i v(x0 + h1 , y0 + h2 )] − [u(x0 , y0 ) + i v(x0 , y0 )]


f ′ (z0 ) = lı́m
h→0 h
[u(x0 + h1 , y0 + h2 ) − u(x0 , y0 )] + i [v(x0 + h1 , y0 + h2 ) − v(x0 , y0 )]
= lı́m .
h→0 h
CAPÍTULO 2. FUNCIONES DE VARIABLE COMPLEJA 33

Como la derivada de f (z) existe en z0 , podemos tender a 0 tomando h un real puro o


un imaginario puro y el lı́mite, que define la derivada de f (z) en z0 , siempre es igual.
Supongamos primero que h = h1 > 0, es decir, h2 = 0, entonces obtenemos:

[u(x0 + h1 , y0 ) − u(x0 , y0 )] + i [v(x0 + h1 , y0 ) − v(x0 , y0 )]


f ′ (z0 ) = lı́m
h1 →0 h1
u(x0 + h1 , y0 ) − u(x0 , y0 ) v(x0 + h1 , y0 ) − v(x0 , y0 )
= lı́m + i lı́m
h1 →0 h1 h1 →0 h1
∂u ∂v
= (x0 , y0 ) + i (x0 , y0 ). (2.3)
∂x ∂x
Ahora, si tomamos h = i h2 con h2 > 0, es decir, h1 = 0, entonces obtenemos:

[u(x0 , y0 + h2 ) − u(x0 , y0 )] + i [v(x0 , y0 + h2 ) − v(x0 , y0 )]


f ′ (z0 ) = lı́m
h2 →0 i h2
u(x0 , y0 + h2 ) − u(x0 , y0 ) v(x0 , y0 + h2 ) − v(x0 , y0 )
= lı́m + i lı́m
h2 →0 i h2 h2 →0 i h2
∂u ∂v
= −i (x0 , y0 ) + (x0 , y0 ). (2.4)
∂y ∂y

De las ecuaciones (2.3) y (2.4) se deduce que

∂u ∂v ∂v ∂u
(x0 , y0 ) + i (x0 , y0 ) = (x0 , y0 ) − i (x0 , y0 ),
∂x ∂x ∂y ∂y

es decir, f (z) satisface las ecuaciones de Cauchy-Riemann (2.2) en (x0 , y0 ).

El hecho que una función satisfaga las ecuaciones de Cauchy-Riemann en un punto, no


es suficiente para garantizar que f (z) sea derivable en tal punto. Por ejemplo, la función
 2
z
 , z 6= 0,

f (z) = z


0, z = 0,

satisface las ecuaciones de Cauchy-Riemann en z = 0, pero no es derivable allı́ (se deja al


lector verificar este hecho).
Ası́ pues, la validez de las ecuaciones de Cauchy-Riemman en un punto es una con-
dición necesaria para que exista la derivada en dicho punto. El solo hecho que las ecuacio-
nes de Cauchy-Riemman sean válidas para una función, no significa que todas las trayecto-
rias por las que z0 + h se aproxime a z0 den lugar a un mismo valor lı́mite de la definición
de derivada. El siguiente teorema muestra las condiciones necesarias y suficientes para
que una función sea derivable en un punto.

Teorema 2.8. Sean f (z) = u(x, y) + i v(x, y) y z0 = x0 + i y0 . Si u(x, y) y v(x, y) tienen


primeras derivadas parciales continuas, con respecto a x e y, en (x0 , y0 ), y satisfacen las
ecuaciones de Cauchy-Riemann (2.2) en (x0 , y0 ), entonces f ′ (z0 ) existe y está dada por:

∂u ∂v ∂v ∂u
f ′ (z0 ) = (x0 , y0 ) + i (x0 , y0 ) o f ′ (z0 ) = (x0 , y0 ) − i (x0 , y0 ).
∂x ∂x ∂y ∂y
34 2.4. FUNCIONES ANALÍTICAS

Demostración. La demostración es inmediata utilizando el Teorema 2.7 y el hecho que


u(x, y) y v(x, y) tienen primeras derivadas parciales continuas, con respecto a x e y, en
(x0 , y0 ).

Ejemplo 2.6. Comprobemos que la función f (z) = ex cos y + i ex sen y es derivable en todo
punto z del plano complejo. Las funciones componentes de f (z) están dadas por:

u(x, y) = ex cos y, v(x, y) = ex sen y.

Ası́, las derivadas parciales de u y v, con respecto a x e y, son:


∂u ∂u
= ex cos y, = −ex sen y,
∂x ∂y
∂v ∂v
= ex sen y, = ex cos y,
∂x ∂y
las cuales son continuas en todo R2 y, además, satisfacen las ecuaciones de Cauchy-
Riemann en todo z = x + i y. Entonces, por el Teorema 2.8 la función f (z) = ex cos y +
La función f (z) i ex sen y es derivable en todo el plano complejo. 
del Ejemplo 2.6
satisface que
f ′ (z) = f (z).
¿Será f (z) la
función exponen-
cial compleja?
2.4 Funciones Analı́ticas
En esta sección se introduce un conjunto de funciones de variable compleja denomina-
das analı́ticas, que poseen una propiedad muy particular: si f (z) es analı́tica en un punto
z0 , entonces es derivable en todo punto z muy cercano a z0 . Esta cualidad de las fun-
ciones analı́ticas las hace muy importantes para el desarrollo teórico de aplicaciones en
Ingenierı́a.

Definición 2.8 (Función analı́tica). Se dice que una función f (z) es analı́tica en z0 ∈ C,
si la derivada de f (z) existe en z0 y en todo punto z de alguna vecindad de z0 . Si f (z)
es analı́tica en todos los puntos de un dominio D ⊂ C, se dice que f (z) es analı́tica en
D.

Ejemplo 2.7. Determinemos el conjunto de números complejos z = x+i y donde la función


f (z) = (x3 − 3xy 2 ) + i (3x2 y − y 3 ) es analı́tica. Se tiene que las funciones componentes de
f (z) están dadas por:

u(x, y) = x3 − 3xy 2 y v(x, y) = 3x2 y − y 3 .

Ahora, las derivadas parciales de u y v, con respecto a x e y, son


∂u ∂u
= 3x2 − 3y 2 , = −6xy,
∂x ∂y
∂v ∂v
= 6xy, = 3x2 − 3y 2 .
∂x ∂y
Como las derivadas parciales de u y v son continuas y satisfacen las ecuaciones de Cauchy-
Riemann en todos los puntos (x, y) del plano, entonces f (z) es derivable en todo el plano
complejo. Por lo tanto, f (z) es analı́tica en todo el plano complejo. 
CAPÍTULO 2. FUNCIONES DE VARIABLE COMPLEJA 35

Ejemplo 2.8. Determinemos el conjunto de números complejos z = x + i y donde la


función f (z) = x2 + i y 2 es analı́tica. Las funciones componentes de f (z) son: u(x, y) = x2
y v(x, y) = y 2 . Al forzar que se satisfagan las ecuaciones de Cauchy-Riemann obtenemos:

∂u ∂v ∂u ∂v
= 2x = 2y = y =0=− ,
∂x ∂y ∂y ∂x

de donde se deduce que f (z) es derivable únicamente en los puntos z = x + i y que


pertenecen a la recta y = x. Si z0 es un punto de esta recta, toda vecindad de z0 contiene
puntos en los que f ′ (z0 ) no existe. Ası́ pues, f (z) no es analı́tica en ningún punto del plano
complejo. 

Los ejemplos anteriores muestran que la propiedad de analiticidad es muy fuerte, dado
que al verificar que una función es analı́tica en un punto, se está comprobando no solo
que la función es derivable en dicho punto, sino que también es derivable en todo punto
de una vecindad del punto en cuestión. Por ello, las funciones analı́ticas, gracias a sus
maravillosas propiedades, juegan un papel muy importante en la teorı́a de las funciones
de variable compleja.
El siguiente teorema afirma que la suma, diferencia, multiplicación y división de funcio-
nes analı́ticas es otra función analı́tica. También establece que la composición de funciones
analı́ticas es una función analı́tica. La demostración de este teorema se deja al lector.

Teorema 2.9. Sean f (z) y g(z) dos funciones analı́ticas en un dominio D ⊂ C. Entonces,
f (z) + g(z), f (z) − g(z) y f (z)g(z), son funciones analı́ticas en D. La función f (z)/g(z) es
analı́tica en el conjunto {z ∈ D : g(z) 6= 0}. Si la función g(z) es analı́tica en el conjunto
f (D), entonces la función h(z) = g(f (z)) es analı́tica en D.

Definición 2.9 (Función entera). Se dice que una función f (z) es entera si es analı́tica
en todo punto z del plano complejo.

Ejemplo 2.9. En el Ejemplo 2.6 vimos que la función f (z) = ex cos y+i ex sen y es derivable
en todo punto z del plano complejo. Por lo tanto, f (z) es analı́tica en todo el plano, de lo
cual se deduce que f (z) es entera. 

Ahora bien, para ciertas funciones que son analı́ticas en determinados conjuntos de
números complejos, existen puntos donde ellas no son analı́ticas. Tales puntos se denomi-
nan puntos singulares y es de mucha importancia identificarlos. Seguidamente damos la
definición formal de punto singular.

Definición 2.10 (Punto singular). Sea f (z) una función de variable compleja. Se dice
que un punto z0 ∈ C es un punto singular de f (z), si f (z) no es analı́tica en z0 y para
toda vecindad de z0 f (z) es analı́tica en al menos un punto z diferente de z0 .
36 2.5. FUNCIONES ARMÓNICAS

Otros puntos muy importantes que también se deben identificar son los ceros de una
función.

Definición 2.11 (Cero de una función). Sea f una función de variable compleja. Se
dice z0 ∈ C es un cero de f (z) si

lı́m f (z) = 0.
z→z0

Observación 2.3. La función racional


a0 + a1 z + · · · + a n z n
r(z) =
b0 + b1 z + · · · + bm z m

es analı́tica en todo el plano complejo, excepto en las raı́ces del polinomio del denomina-
dor, en otras palabras, salvo en los puntos z tales que b0 + b1 z + · · · + bm z m = 0. Estas raı́ces
son los puntos singulares de r(z). Además, las raı́ces del polinomio a0 + a1 z + · · · + an z n
son los ceros de r(z).

Ejemplo 2.10. Los puntos z0 = 0 y z1 = 1 son, respectivamente, el único punto singular y


el único cero de la función f (z) = (z − 1)/z. 

2.5 Funciones Armónicas


Pasemos ahora a estudiar la relación de las funciones analı́ticas con una familia de
funciones de variable real muy importante, denominadas armónicas.

Definición 2.12 (Función armónica). Se dice que una función h : R2 → R es armónica


en un dominio D ⊂ R2 , si en todo punto (x, y) ∈ D tiene derivadas parciales, primera y
segunda, continuas y satisface la ecuación en derivadas parciales

∂2h ∂2h
∇2 h(x, y) = 2
(x, y) + 2 (x, y) = 0,
∂x ∂y

conocida como ecuación de Laplace.

Definición 2.13 (Armónica conjugada). Sean u : R2 → R y v : R2 → R dos funciones.


Si u y v son armónicas en un dominio D ⊂ R2 y sus primeras derivadas parciales
satisfacen las ecuaciones de Cauchy-Riemann (2.2) para todo (x, y) ∈ D, se dice que v
es armónica conjugada de u.

El siguiente teorema establece una relación entre las funciones armónicas y las funcio-
nes analı́ticas.
Teorema 2.10. Una función f (z) = u(x, y) + i v(x, y) es analı́tica en un dominio D ⊂ C
si, y sólo si v es armónica conjugada de u.

Demostración. (⇒) Supongamos que f (z) = u(x, y) + i v(x, y) es analı́tica en un dominio


D ⊂ C y demostremos que v es armónica conjugada de u. Como f (z) es analı́tica, sus
funciones componentes, u y v, satisfacen las ecuaciones de Cauchy-Riemann, además, sus
CAPÍTULO 2. FUNCIONES DE VARIABLE COMPLEJA 37

primeras derivadas parciales son continuas en D. Por ello, basta demostrar que u y v son
armónicas para garantizar que v es armónica conjugada de u. Ası́, derivando con respecto
a x la ecuación de Cauhy-Riemann ∂u ∂v
∂x = ∂y , obtenemos:

∂2u ∂2v
= . (2.5)
∂x2 ∂x∂y
∂u ∂v
Ahora, derivando con respecto a y la otra ecuación de Cauchy-Riemann ∂y = − ∂x , obte-
nemos:
∂2u ∂2v
= − . (2.6)
∂y 2 ∂y∂x
Como las primeras derivadas parciales de u y v son continuas en D, se tiene que

∂2v ∂2v
= . (2.7)
∂x∂y ∂y∂x

Por (2.5), (2.6) y (2.7) podemos escribir:

∂2u ∂2u
∇2 u(x, y) = (x, y) + (x, y) = 0,
∂x2 ∂y 2

con lo cual se demuestra que u es armónica en D. Utilizando un razonamiento similar se


demuestra que v también es armónica. En consecuencia, u y v son funciones armónicas
que satisfacen las ecuaciones de Cauchy-Riemann en D, es decir, v es armónica conjugada
de u.
(⇐) Supongamos que v es armónica conjugada de u en D y demostremos que la función
f (z) = u(x, y) + i v(x, y) es analı́tica en D. Como v es armónica conjugada de u en
D, entonces u y v son armónicas en D y satisfacen las ecuaciones de Cauchy-Riemann;
además, sus primeras derivadas parciales son continuas en D. Ası́, por el Teorema 2.8, la
función f (z) = u(x, y) + i v(x, y) es analı́tica en D.

Observación 2.4. El Teorema 2.10 nos garantiza que una función f (z) = u(x, y) + i v(x, y)
es analı́tica, si v es armónica conjugada de u. Pero, no es cierto que si u y v son armónicas,
entonces f (z) = u(x, y) + i v(x, y) es analı́tica; por ejemplo, si tomamos u(x, y) = x + y y
v(x, y) = x, observamos que u y v son armónicas en todo el plano, pero f (z) = (x + y) + i x
no es analı́tica en ningún punto del plano (se deja al lector demostrar esta afirmación).
Es bueno aclarar que si v es armónica conjugada de u en cierto dominio D, no es en ge-
neral cierto que u sea armónica conjugada de v en ese dominio. Por ejemplo, consideremos
las funciones u(x, y) = x2 − y 2 y v(x, y) = 2xy. Vemos que u y v son las partes real e imagi-
naria, respectivamente, de f (z) = z 2 , que es una función entera. Por el Teorema 2.10, v es
armónica conjugada de u en todo el plano, pero u no puede ser una armónica conjugada
de v, ya que la función g(z) = 2xy + i (x2 − y 2 ) no es analı́tica en ningún punto del plano,
porque sus funciones componentes no satisfacen las ecuaciones de Cauchy-Riemann. Note que g(z) =
2xy + i(x2 − y 2 )
Por otra parte, es cierto que una armónica conjugada, cuando existe, es única, excepto solo es derivable
por una constante aditiva. Expliquemos con un ejemplo un método para obtener una en z = 0.
armónica conjugada de una función armónica dada.

Ejemplo 2.11. Determine v(x, y), la armónica conjugada de u(x, y), si u(x, y) = x + y.
Solución. Para que v sea armónica conjugada de u, la función v debe ser armónica y,
38 2.5. FUNCIONES ARMÓNICAS

además, se deben satisfacer las ecuaciones de Cauchy-Riemann (2.2). El método para


generar v, una armónica conjugada de u, consiste en forzar que se cumplan las ecuacio-
nes de Cauchy-Riemann y luego resolver ciertas ecuaciones diferenciales. Apliquemos tal
metodologı́a al problema planteado.
Como u(x, y) = x + y, se tiene que u es armónica y

∂u ∂u
= 1, y = 1.
∂x ∂y

Ahora, utilizando una de las ecuaciones de Cauchy-Riemann, digamos

∂u ∂v
= ,
∂x ∂y

podemos escribir:
∂v
= 1.
∂y
Integrando esta última ecuación con respecto a y, obtenemos:

v(x, y) = y + φ(x), (2.8)

donde φ : R → R. Ahora, utilizando la otra ecuación de Cauchy-Riemann,

∂u ∂v
=− ,
∂y ∂x

podemos escribir:
∂v ∂u
φ′ (x) = =− = −1,
∂x ∂y
es decir,
φ′ (x) = −1.
Integrando la ecuación anterior con respecto a x, obtenemos:

φ(x) = −x + c,

donde c es una constante real. Sustituyendo la expresión de φ(x) en (2.8), la función v


armónica conjugada de u está dada por

v(x, y) = y − x + c,

que es armónica (se deja al lector verificar esta afirmación). Es decir, hemos construido
una familia de funciones armónicas conjugadas de u que se diferencian entre sı́ por una
constante. Para obtener una de ellas es necesario un valor de v en algún punto del plano.
Por ejemplo, si v(0, 0) = 1, entonces la constante c correspondiente es c = 1 y la función v
es v(x, y) = y − x + 1. 

El procedimiento anterior para calcular funciones v armónicas conjugadas de u, también


se puede utilizar, según el Teorema 2.10, para construir funciones analı́ticas a partir de una
de sus funciones componentes, como se muestra en el siguiente ejemplo.
CAPÍTULO 2. FUNCIONES DE VARIABLE COMPLEJA 39

Ejemplo 2.12. Sea v(x, y) = x. Determine una función analı́tica f (z) = u(x, y) + i v(x, y)
tal que f (0) = −1.
Solución. Como v(x, y) = x, se tiene que v es armónica y

∂v ∂v
=1 y = 0.
∂x ∂y
∂u ∂v
Es indiferente Ahora, utilizando la ecuación de Cauchy-Riemann ∂x = ∂y , podemos escribir:
la ecuación de
Cauchy-Riemann
con que se co- ∂u
mience el pro- = 0.
cedimiento. Un
∂x
ejercicio intere-
sante es deter- Integrando esta última ecuación con respecto a x, obtenemos:
minar la fun-
ción u(x, y),
comenzando u(x, y) = φ(y),
con la ecuación
uy = −vx ∂u ∂v
y luego utili-
donde φ : R → R. Ahora, utilizando la otra ecuación de Cauchy-Riemann, ∂y = − ∂x ,
zar la ecuación podemos escribir:
ux = vy . ∂u ∂v
φ′ (y) = =− = −1,
∂y ∂x
es decir,
φ′ (y) = −1.

Integrando la ecuación anterior con respecto a y, obtenemos:

φ(y) = −y + c,

donde c es una constante real. Por lo tanto, considerando la expresión de φ(y), la función
u está dada por
u(x, y) = −y + c.

Por construcción, v es armónica conjugada de u, luego, por el Teorema 2.10 las funciones
f (z) están dadas por
f (z) = c − y + i x,

las cuales constituyen una familia de funciones enteras que se diferencian entre sı́ por una
constante real c. Ahora, la función deseada debe satisfacer f (0) = −1. Forzando esta
última condición y despejando c, se tiene que c = −1 y la función pedida es

f (z) = −1 − y + i x = iz − 1.

2.6 Funciones Elementales


A continuación damos una breve descripción de las funciones elementales de variable
compleja.
40 2.6. FUNCIONES ELEMENTALES

2.6.1 Función Exponencial


La función exponencial, denotada por ez , se define como

ez = ex cos y + i ex sen y, (2.9)

para todo número complejo z = x + i y. Las funciones componentes de ez son

u(x, y) = ex cos y y v(x, y) = ex sen y,

las cuales satisfacen las ecuaciones de Cauchy-Riemann y sus derivadas parciales son con-
tinuas en todo el plano complejo. Luego, la función exponencial es analı́tica en todo el
plano complejo (ver Ejemplo 2.6). Por lo tanto, ez es una función entera, cuya derivada
está dada por:
d z ∂u ∂v
e = +i = ex cos y + i ex sen y = ez ,
dz ∂x ∂x
que coincide con la propiedad deseada de la función exponencial real.

Otras propiedades de la función exponencial

• Si z = a ∈ R, entonces ez = ea cos 0+i ea sen 0 = ea , es decir, ez se reduce a la función


ex , si z toma valores reales.

• El rango de la función exponencial es todo el plano complejo excepto z = 0. En


efecto, como |ez | = ex > 0, para todo x ∈ R, entonces |ez | > 0 para todo z ∈ C. Por
lo tanto, ez 6= 0 para todo z ∈ C.

• La función exponencial es periódica con un periodo imaginario puro de 2πi. En


efecto, por la periodicidad de las funciones trigonométricas sen y cos, podemos escri-
bir:

e(z+2πi) = ex+i (y+2π)


= ex (cos(y + 2π) + i sen (y + 2π))
= ex (cos y + i sen y)
= ez ,

es decir, ez es periódica con periodo 2πi.

• Propiedades algebraicas. Sean z1 = x1 + i y1 y z2 = x2 + i y2 . Las siguientes propie-


dades son ciertas:

i) ez1 ez2 = ez1 +z2 ;


ez1
ii) z2 = ez1 −z2 ;
e
iii) e0 = 1;
1
iv) z1 = e−z1 ;
e
v) (ez1 )n = en z1 , (n = 0, ±1, ±2, . . .).
CAPÍTULO 2. FUNCIONES DE VARIABLE COMPLEJA 41

2.6.2 Funciones Trigonométricas


Se definen, para todo z ∈ C, la función seno complejo como

eiz − e−iz
sen z = (2.10)
2i
y la función coseno complejo como

eiz + e−iz
cos z = . (2.11)
2

Las funciones sen z y cos z son combinaciones lineales de las funciones enteras eiz y e−iz ,
por lo tanto, sen z y cos z son funciones enteras; además, sus derivadas son:

d i(eiz + e−iz )
sen z = = cos z
dz 2i
y
d i(eiz − e−iz )
cos z = = −sen z.
dz 2i

Otras propiedades de las funciones seno y coseno

• Cuando z es un número real, sen z y cos z coinciden con las funciones reales sen x y
cos x. En efecto, si z = x ∈ R, entonces por la definición de sen z, cos z y ez , podemos
escribir:

eix − e−ix (cos x + i sen x) − (cos(−x) + i sen (−x)) 2i sen x


sen z = = = = sen x,
2i 2i 2i
eix + e−ix (cos x + i sen x) + (cos(−x) + i sen (−x)) 2 cos x
cos z = = = = cos x.
2 2 2i

• Las identidades trigonométricas que satisfacen el seno y el coseno real también son
válidas en el caso complejo; por ejemplo:

sen 2 z + cos2 z = 1,
sen (z1 ± z2 ) = sen z1 cos z2 ± cos z1 sen z2 ,
cos(z1 ± z2 ) = cos z1 cos z2 ∓ sen z1 sen z2 ,

entre otras. (Se deja como ejercicio para el lector la comprobación de cada una de
las identidades).

• A partir de la definición de sen z dada en (2.10) podemos escribir, para z = x + i y:

ei(x+i y) − e−i(x+i y)
sen z =
2i
e−y ey
= (cos x + i sen x) − (cos x − i sen x)
2i   2i  
ey + e−y ey − e−y
= sen x + i cos x .
2 2
42 2.6. FUNCIONES ELEMENTALES

En otras palabras, la función sen z se puede expresar equivalentemente como:


sen z = sen x cosh y + i cos x senh y. (2.12)
De la misma manera, al utilizar la definición de cos z dada en (2.11) obtenemos:
cos z = cos x cosh y − i sen x senh y. (2.13)

• Las funciones seno y coseno son periódicas con periodo 2π. (Se deja como ejercicio
para el lector la demostración).
• Los ceros de sen z son todos los números complejos z = nπ, para n = 0, ±1, ±2, . . ..
En efecto, para que z ∈ C sea un cero de sen z, se debe cumplir que sen z = 0, luego,
utilizando la ecuación (2.12) podemos escribir:
sen x cosh y = 0, cos x senh y = 0.
Ahora, utilizando la ecuación sen x cosh y = 0, se deduce que y = 0. Sustituyendo
este valor de y en la ecuación sen x cosh y = 0, obtenemos sen x = 0, de lo cual se
tiene que x = nπ, para n = 0, ±1, ±2, . . ... En consecuencia, los ceros de sen z son
todos los números complejos z = nπ, para n = 0, ±1, ±2, . . ..
• Los ceros de cos z son todos los números complejos z = (n + 21 )π, para n = 0, ±1, . . ..
(Se deja al lector verificar este hecho).

Otras funciones trigonométricas


Las demás funciones trigonométricas de argumento complejo se definen fácilmente por
analogı́a con las funciones trigonométricas de argumento real, esto es,
Tangente
sen z
tan z =
cos z
Cotangente
cos z
cot z =
sen z
Secante
1
sec z =
cos z
Cosecante
1
csc z =
sen z
Las funciones tan z, cot z, sec z y csc z, son analı́ticas en todos los números complejos z
donde no se anule el denominador de la expresión que las define, además, la derivada de
cada una de ellas es, respectivamente,
d
tan z = sec2 z,
dz
d
cot z = − csc2 z,
dz
d
sec z = tan z sec z,
dz
d
csc z = − cot z csc z.
dz
CAPÍTULO 2. FUNCIONES DE VARIABLE COMPLEJA 43

2.6.3 Funciones Hiperbólicas


Se definen, para todo z ∈ C, la función seno hiperbólico complejo como

ez − e−z
senh z = (2.14)
2

y la función coseno hiperbólico complejo como

ez + e−z
cosh z = . (2.15)
2

Las funciones senh z y cosh z son combinaciones lineales de las funciones enteras ez y e−z ,
por lo tanto, senh z y cosh z son funciones enteras; además, sus derivadas son:

d
senh z = cosh z
dz
y
d
cosh z = senh z.
dz

Otras propiedades de las funciones seno y coseno hiperbólicos

• Cuando z es un número real, senh z y cosh z coinciden con las funciones reales senh x
y cosh x.

• Las identidades hiperbólicas que satisfacen el seno y coseno hiperbólicos reales también
son válidas en el caso complejo; por ejemplo:

cosh2 z − senh2 z = 1,
senh(z1 + z2 ) = senh z1 cosh z2 + cosh z1 senh z2 ,
cosh(z1 + z2 ) = cosh z1 cosh z2 + senh z1 senh z2 ,

entre otras. (Se deja como ejercicio para el lector la comprobación de cada una de
las identidades).

• A partir de las definiciones de senh z y cosh z se deduce, para z = x + i y:

senh z = senh x cos y + i cosh x sen y (2.16)

y
cosh z = cosh x cos y + i senh x sen y. (2.17)

• Las funciones seno y coseno hiperbólicos son periódicas con periodo 2πi.

• Los ceros de senh z son todos los números complejos z = nπi, n = 0, ±1, ±2, . . .

• Los ceros de cosh z son todos los números complejos z = (n + 12 )πi, n = 0, ±1, ±2, . . .
44 2.6. FUNCIONES ELEMENTALES

Otras funciones hiperbólicas


Las demás funciones hiperbólicas de argumento complejo se definen fácilmente por
analogı́a con las funciones hiperbólicas de argumento real, esto es,

Tangente hiperbólica
senh z
tanh z =
cosh z
Cotangente hiperbólica
cosh z
coth z =
senh z
Secante hiperbólica
1
sech z =
cosh z
Cosecante hiperbólica
1
csch z =
senh z
Las funciones tanh z, coth z, sech z y csch z, son analı́ticas en todos los números complejos
z donde no se anule el denominador de la expresión que las define, además, la derivada
de cada una de ellas es, respectivamente,
d
tanh z = sech 2 z,
dz
d
coth z = −csch 2 z,
dz
d
sech z = − tanh z sech z,
dz
d
csch z = − coth z csch z.
dz

2.6.4 Función Logaritmo


Se define la función logaritmo, para todo z 6= 0, como

log z = ln |z| + i arg z, (2.18)

En la ecuación donde ln · denota el logaritmo natural. Veamos que log z es una función multivaluada. Se
(2.18) se puede
utilizar un loga-
tiene que, para todo z 6= 0, el argumento de z se puede escribir como:
ritmo en cual-
quier base, pero arg z = Arg z + 2nπ (n = 0, ±1, ±2, . . .).
lo más reco-
mendable en la
práctica es utili- De esta forma, la ecuación (2.18) se puede escribir equivalentemente como
zar la base e.
log z = ln |z| + i (Arg z + 2nπ) (n = 0, ±1, ±2, . . .).

Observe que para cualquier z 6= 0, los valores de log z tienen la misma parte real y sus
partes imaginarias difieren en múltiplos enteros de 2π. Por lo tanto, log z es una función
multivaluada. El siguiente ejemplo ilustra este hecho.
CAPÍTULO 2. FUNCIONES DE VARIABLE COMPLEJA 45

π
Ejemplo 2.13. Calculemos todos los valores de log(1 + i). Se tiene que Arg (1 + i) = y
√ 4
|1 + i| = 2. Ası́,
√ π 
log(1 + i) = ln 2 + i + 2nπ (n = 0, ±1, ±2, . . .).
4


Definición 2.14 (Valor principal del logaritmo). El valor principal de log z es el valor
que se obtiene de la fórmula (2.18) cuando se utiliza el argumento principal de z. Ese
valor se denota como Log z y se da por medio de la ecuación

Log z = ln |z| + i Arg z, para z 6= 0. (2.19)

Algunas propiedades del valor principal del logaritmo


En general,
se tiene que:
Log (z a ) 6=
• La función w = Log z es monovaluada y su dominio de definición es el conjunto de aLog z,
todos los números complejos diferentes de cero; su rango es la franja −π < Im w ≤ π. Log (a/b) 6=
Log (a) − Log (b)
¿Por qué?
• El valor principal del logaritmo se reduce al logaritmo natural si z es un número real
positivo, en otras palabras, si z = r > 0, entonces Log z = ln r.

• La función inversa de Log z es ez ; en otras palabras, si w es un número complejo tal


que −π < Im w ≤ π, entonces w = Log z si, y sólo si z = ew .

• La función Log z es continua en el dominio (|z| > 0, −π < arg z < π). (Se deja al
lector la prueba).

• La función Log z es analı́tica en el dominio (|z| > 0, −π < arg z < π), y su derivada
está dada por
d 1
Log z = .
dz z
(Se deja al lector la verificación).

Ramas del logaritmo

Definición 2.15 (Rama de una función multivaluada). Una rama de una función mul-
tivaluada f (z), es una función monovaluada F (z) que es analı́tica en cierto dominio
D ⊂ C y que coincide con f (z) en D, es decir, F (z) = f (z) para todo z ∈ D.

Definición 2.16 (Corte y punto ramal). Un corte ramal es una lı́nea o curva de pun-
tos singulares que se introducen al definir una rama de una función multivaluada. El
punto común a todos los cortes ramales de una función multivaluada se denomina punto
ramal.

Observe que Log z es una rama de la función multivaluada log z, además, se tiene que
Log z = log z, para todo z 6= 0 tal que |z| > 0 y −π < arg z < π. La función Log z se
denomina rama principal del logaritmo. El corte ramal de Log z es el eje real negativo con
el origen (ver Figura 2.2).
46 2.6. FUNCIONES ELEMENTALES

corte ramal punto ramal

Figura 2.2. Corte ramal de Log z

Otras ramas del logaritmo complejo se definen como


log z = ln |z| + i arg z (|z| > 0, α < arg z < α + 2π),
donde α es un número real fijo. El corte ramal de esta rama es el rayo arg z = α (ver
Figura 2.3).

al
punto ramal r am
t e
cor
α
b

Figura 2.3. Corte ramal de una rama de log z

Ejemplo 2.14. Determinemos el valor de log(1 + i), si log z está definido por
log z = ln |z| + i arg z (|z| > 0, π/2 < arg z < 5π/2).
√ iπ/4
Se tiene que 1 + i = 2e , pero π/4 6∈ (π/2, 5π/2). Luego, el argumento principal de
1+i no es el valor indicado para calcular log(1+i). Para encontrar arg (1+i) ∈ (π/2, 5π/2),
se utiliza el argumento principal de 1 + i. Se tiene que el valor del argumento de 1 + i
buscado está dado por:
π 9π
+ 2π = ∈ (π/2, 5π/2);
4 4
por lo tanto, arg (1 + i) = 9π/4 es el valor indicado para calcular log(1 + i). Ası́,
√ 9π
log(1 + i) = ln 2+i .
4


2.6.5 Función Exponente Complejo


Sea c un número complejo. Se define la función exponente complejo como
z c = ec log z , (2.20)
para todo z 6= 0. Esta función generalmente es multivaluada. Los siguientes ejemplos
ilustran este hecho.
CAPÍTULO 2. FUNCIONES DE VARIABLE COMPLEJA 47

Ejemplo 2.15. Calculemos todos los valores de ii . Se tiene que


1 1
ii = ei log i = ei(ln 1+i(2n+ 2 )π) = e−(2n+ 2 )π ,

para n = 0, ±1, ±2, . . .. 

Ejemplo 2.16. Calculemos el valor de i2 . Se tiene que


1
i2 = e2 log i = e2(ln 1+i(2n+ 2 )π) = e(4n+1)πi ,

para n = 0, ±1, ±2, . . .; pero,

e(4n+1)πi = e4nπi eπi = −1,

para todo n = 0, ±1, ±2, . . .; por lo tanto, i2 = −1. 

El Ejemplo 2.15 muestra que la función exponente complejo es, generalmente, una
función multivaluada. Por ello, se pueden definir ramas de esta función. Las ramas de la
función exponente complejo se definen como

z c = ec(ln |z|+i arg z) ,

donde |z| > 0 y α < arg z < α + 2π, con α un número real fijo. En otras palabras, las ramas
de la función exponente complejo se definen según la rama del logaritmo que se esté
utilizando. De esta forma, cuando α = −π, estaremos empleando la rama principal del
logaritmo para definir la rama principal de la función exponente complejo. Para obtener
la derivada de la función exponente complejo, se emplea la regla de la cadena. Ası́, la
derivada de cada una de las ramas de la función exponente complejo está dada por

d c
z = c z c−1 .
dz

Función exponencial de base c

La función exponencial de base c, donde c es un número complejo distinto de cero, se


define como
cz = ez log c , (2.21)

para todo z ∈ C. De igual manera que la función exponente complejo, la función expo-
nencial de base c es generalmente multivaluada, cuyas ramas se definen según la rama
del logaritmo que se esté empleando; por ejemplo, si utilizamos el valor principal de log c,
entonces la rama que se obtiene es la rama principal de la función exponencial de base c.
Para cada rama de la función exponencial de base c, su derivada está dada por:

d z
c = cz log c.
dz
48 2.6. FUNCIONES ELEMENTALES

2.6.6 Funciones Trigonométricas Inversas


Comencemos con la deducción de una expresión cerrada para la función inversa del
seno. Un procedimiento similar se puede utilizar para la deducción de las expresiones
de las demás funciones trigonométricas inversas. Para definir la función inversa de sen z,
denotada por sen −1 z, se escribe w = sen −1 z, siempre que z = sen w. Ahora, utilizando la
definición de sen w podemos escribir:
eiw − e−iw
z=
2i
o, equivalentemente,
e2iw − 2iz eiw − 1 = 0.
Para expresar w en términos de z, primero se despeja eiw al resolver la ecuación anterior,
que es una ecuación cuadrática en eiw . Se tiene que
eiw = iz + (1 − z 2 )1/2 ,
donde (1−z 2 )1/2 es una función bivaluada de z. Si tomamos logaritmo en ambos miembros
y recordamos que w = sen −1 z, se llega a la siguiente fórmula cerrada de la función sen −1 z
 
sen −1 z = −i log iz + (1 − z 2 )1/2 . (2.22)

Las funciones inversa del coseno e inversa de la tangente se pueden obtener fácilmente
realizando un procedimiento similar al antes descrito. Las expresiones de cos−1 z y tan−1 z
son, respectivamente:  
cos−1 z = −i log z + i(1 − z 2 )1/2 , (2.23)
 
i i+z
tan−1 z = log . (2.24)
2 i−z
Busque en la Ejercicio 2.2. Deducir las expresiones cerradas de sec−1 z, csc−1 z y cot−1 z.
red a través de
Google las ex- En general, las funciones trigonométricas inversas son multivaluadas. Por ejemplo,
presiones de
sec−1 z, csc−1 z
para la función sen −1 z podemos elegir dos valores igualmente válidos para la raı́z cua-
y cot−1 z drada que aparece en su definición. Una vez que hemos elegido una valor, existe un
número infinito de valores posibles para el logaritmo de iz + (1 − z 2 )1/2 . En resumen,
vemos que, debido a la raı́z cuadrada y al logaritmo, hay dos conjuntos distintos de valo-
res de sen −1 z, cada uno de los cuales posee un número infinito de elementos. Este hecho
sucede de forma semejante para las restantes funciones trigonométricas inversas.
Las derivadas de las funciones trigonométricas inversas se obtienen directamente de
sus expresiones cerradas. Las derivadas de las funciones inversa del seno e inversa del
coseno dependen de los valores escogidos de la raı́ces cuadradas, a saber:
d 1
sen −1 z = ,
dz (1 − z 2 )1/2
d −1
cos−1 z = .
dz (1 − z 2 )1/2
En cambio, la derivada de la inversa de la tangente,
d 1
tan−1 z = ,
dz 1 + z2
no depende de la manera en que la función se haga monovaluada.
CAPÍTULO 2. FUNCIONES DE VARIABLE COMPLEJA 49

Ejercicio 2.3. Determinar las expresiones cerradas de las derivadas de sec−1 z, csc−1 z y
cot−1 z.

El siguiente ejemplo ilustra la aplicación de las expresiones de las funciones inversas


en la resolución de ecuaciones que involucran funciones trigonométricas.

Ejemplo 2.17. Resolver la ecuación

sen 2 z + 2i sen z − 1 = 0.

Solución. La ecuación dada se puede escribir equivalentemente como

(sen z + i)2 = 0,

la cual es cierta si, y sólo si la siguiente ecuación se cumple

sen z = −i.

Al tomar la inversa del seno en ambos miembros de la ecuación anterior, podemos escribir:
 
z = sen −1 (−i) = −i log i(−i) + (1 − (−i)2 )1/2
   √ 
= −i log 1 + 21/2 = −i log 1 ± 2 .

Según el valor de la raı́z cuadrada empleado, obtenemos dos conjuntos de valores de z que
resuelven la ecuación dada. Ası́, uno de estos conjuntos de números complejos está dado
por:  √   √ 
zk = −i log 1 + 2 = 2kπ − i ln 1 + 2 , para k = 0, ±1, ±2, . . .

El otro conjunto es:


 √  √
zn = −i log 1 − 2 = (2n + 1)π − i ln 1 − 2 , para n = 0, ±1, ±2, . . .

2.6.7 Funciones Hiperbólicas Inversas


Las expresiones cerradas de las funciones hiperbólicas inversas se pueden obtener
fácilmente a través de un procedimiento similar al empleado en la deducción de la función
inversa del seno. Las expresiones cerradas de las funciones inversas del senh z, cosh z y
tanh z, son, respectivamente:
 
senh−1 z = log z + (z 2 + 1)1/2 ,
 
cosh−1 z = log z + (z 2 − 1)1/2 ,
 
−1 1 1+z
tanh z = log .
2 1−z

Ejercicio 2.4. Deducir las expresiones cerradas de sech −1 z, csch −1 z y coth−1 z.


50 2.7. MAPEOS

Las derivadas de las funciones senh−1 z y cosh−1 z, dependen de los valores escogidos
de las raı́ces cuadradas, a saber:
d 1
senh−1 z = ,
dz (z 2 + 1)1/2
d −1
cosh−1 z = .
dz (z − 1)1/2
2

La derivada de la función tanh−1 z,


d 1
tanh−1 z = ,
dz 1 − z2
no depende de la manera en que la función se haga monovaluada.

2.7 Mapeos
En esta sección se estudia la transformación de regiones del plano complejo a través de
funciones de variable compleja, tales como polinomios de grado 1 y funciones racionales
obtenidas como el cociente de dos polinomios de grado 1. Recordemos que en el cálculo
elemental se podı́a obtener la gráfica de una función real y = f (x) y con ello estudiar
su comportamiento; en cambio para una función w = f (z) de una variable compleja, no
es tan fácil elaborar su gráfica. Para ello se requieren dos números x e y para definir un
valor z cualquiera y otros dos números para los valores de u y v correspondientes. Por lo
tanto, se requiere un espacio de cuatro dimensiones para representar w = f (z) en forma
gráfica. Evidentemente una gráfica de cuatro dimensiones no es un medio conveniente
para estudiar el efecto gráfico de una función de variable compleja. Es preciso recurrir a
otros medios para visualizar w = f (z).
y v
S w = f (z) f (S)

z Mapeo o w
b b

Transformación

x u

Plano z Plano w

Figura 2.4. Representación gráfica de un mapeo

Aquı́, visualizamos la relación w = f (z) como el efecto geométrico que tiene la función
f (z) sobre un conjunto de puntos complejos. A este proceso lo denominamos mapear o
transformar y a la función f (z) de variable compleja la denominamos transformación o
mapeo (ver Figura 2.4). El objetivo principal de esta sección es determinar analı́tica y
gráficamente el efecto que tiene un mapeo sobre un conjunto de puntos del plano com-
plejo; en particular, estudiamos el efecto de los mapeos lineales, inversión y bilineales,
sobre rectas y circunferencias. Para esta parte necesitamos la siguiente notación:
CAPÍTULO 2. FUNCIONES DE VARIABLE COMPLEJA 51

• Denotamos con w = f (z) la imagen de z bajo f , donde z = x + i y y w = u + i v.

• Para todo conjunto S de números complejos, denotamos con f (S) el transformado o


la imagen de S bajo f ;

• La imagen inversa de un punto w del rango de f es el conjunto de todos los puntos z,


en el dominio de f , que tienen a w como su imagen.

Definición 2.17 (Mapeo inyectivo). Sean z1 , z2 ∈ C. Se dice que el mapeo w = f (z) es


inyectivo, si z1 6= z2 implica que f (z1 ) 6= f (z2 ).

Inicialmente describimos los mapeos lineales: w = z + c, w = bz y w = bz + c.


Seguidamente mostramos el mapeo inversión w = 1/z. Por último, describimos el mapeo
bilineal w = (az + b)/(cz + d).

2.7.1 Mapeo w = z + c
El mapeo del plano z en el plano w definido por la ecuación

w = z + c, (2.25)

donde c es una constante compleja, es una traslación en la dirección del vector c. En la Fi-
gura 2.5 se aprecia el movimiento geométrico que tiene un punto z0 cuando se transforma
con una traslación.
y z0 v
b
w =z+c z0
b

b
Traslación
w0 = z0 + c

x u
b
c

Figura 2.5. Movimiento geométrico de la traslación

El mapeo (2.25) es inyectivo; por tanto, posee mapeo inverso definido como

f −1 (w) = w − c. (2.26)

A continuación demostramos que la traslación transforma rectas a rectas y circunfe-


rencias a circunferencias. Sean z = x + i y, w = u + i v y c = c1 + i c2 . Para obtener
el transformado de una recta o una circunferencia bajo el mapeo w = z + c, utilizamos
un procedimiento general que permite determinar, a través del mapeo inverso, el transfor-
mado de un conjunto S descrito por un conjunto de ecuaciones. El procedimiento es el
siguiente: primero, se emplea la ecuación

x + i y = f −1 (u + i v),
52 2.7. MAPEOS

para expresar a x e y en función de u y v; luego se sustituyen convenientemente estos


valores en las ecuaciones que describen el conjunto S, con lo cual se obtiene el conjunto
de ecuaciones (en función de u y v) que describen el transformado de S.
En el caso de las rectas y circunferencias, se tiene que la ecuación general de una recta
o una circunferencia está dada por
α(x2 + y 2 ) + βx + δy + γ = 0, (2.27)
donde α, β, δ, γ ∈ R. Es decir, todo punto z = x + i y tal que x e y satisfacen la ecuación
(2.27) pertenece a una recta o a una circunferencia. Ahora, utilizando el mapeo inverso
(2.26) podemos escribir:
x + i y = f −1 (u + i v) = u + i v − c1 + i c2 = (u − c1 ) + i (v − c2 ),
de donde se deduce que
x = u − c1 e y = v − c2 .
Sustituyendo convenientemente estos últimos valores en la ecuación (2.27), obtenemos la
siguiente ecuación
α((u − c1 )2 + (v − c2 )2 ) + β(u − c1 ) + δ(v − c2 ) + γ = 0,
que describe analı́ticamente el conjunto transformado de una recta o una circunferencia,
bajo el mapeo w = z + c. Entonces, si α = 0, la ecuación (2.27) describe una recta en el
plano z, y la ecuación del transformado de esta recta bajo el mapeo w = z + c está dada
por
βu + δv + (δ − βc1 − δc2 ) = 0,
que describe una recta en el plano w. En cambio, si α 6= 0, la ecuación (2.27) describe una
circunferencia en el plano z, y la ecuación del transformado de esta circunferencia bajo el
mapeo w = z + c está dada por
α(u2 + v 2 ) + (β − 2c1 α)u + (δ − 2c2 α)v + (αc21 + αc22 − βc1 − δc2 + γ) = 0,
que también describe una circunferencia en el plano w.
A continuación damos un ejemplo donde se emplea la ecuación z = f −1 (w) para ob-
tener el transformado, bajo el mapeo traslación, de un conjunto de números complejos
definido por inecuaciones.
Ejemplo 2.18. Sea S el conjunto de números complejos que se muestra en la Figura 2.6.
Determinar el transformado de S bajo el mapeo w = f (z) = z − i.
Solución. Se tiene que todo z = x + i y ∈ S satisface el siguiente conjunto de inecuaciones

 2x − y ≥ 1
S: 2x + y ≤ 7

y ≥ 1

Además, la función inversa de f (z) = z −i es f −1 (w) = w+i. Ahora, utilizando la ecuación


x + i y = f −1 (u + i v) obtenemos: x = u, e y = v + 1. Sustituyendo convenientemente estas
últimas expresiones en el conjunto de inecuaciones que describe a S, se tiene el siguiente
sistema de inecuaciones 
 2u − v ≥ 2
f (S) : 2u + v ≤ 6

v ≥ 0
CAPÍTULO 2. FUNCIONES DE VARIABLE COMPLEJA 53

que describe los números complejos w = u + i v del transformado de S bajo el mapeo


w = z − i, cuya gráfica se aprecia en la Figura 2.5. 

3
S
2

x
1 2 3

Figura 2.6. Conjunto S del Ejemplo 2.18

2 f (S)

u
1 2 3

Figura 2.7. Conjunto f (S) del Ejemplo 2.18

2.7.2 Mapeo w = bz

El mapeo del plano z en el plano w definido por la ecuación

w = bz, (2.28)

donde b es un número complejo distinto de cero, es una rotación en el ángulo Arg b y una
expansión ó contracción según sea el valor de |b|. Si |b| < 1, se dice que el mapeo (2.28)
es una rotación y contracción; en cambio, si |b| ≥ 1, el mapeo (2.28) es una rotación
y expansión. En la Figura 2.8 se aprecia el movimiento geométrico que tiene un punto
z = reiθ cuando se transforma con el mapeo (2.28). Si tomamos b = leiβ y z = reiθ ,
entonces el transformado de z bajo el mapeo w = bz está dado, en su forma polar, por
w = (lr)ei(β+θ) , de lo cual se infiere que (2.28) es una rotación en el ángulo β y una
expansión o contracción, según sea el valor de l.
54 2.7. MAPEOS

y v
w = (lr)ei(θ+β)
b
w = bz

z = reiθ
b

θ
b
b = leiβ θ+β
β
x u

Figura 2.8. Movimiento geométrico de la rotación y expansión o contracción

El mapeo (2.28) es inyectivo; por tanto, posee mapeo inverso definido como

w
f −1 (w) = . (2.29)
b

Además, el mapeo (2.28) transforma rectas a rectas y circunferencias a circunferencias.

Ejercicio 2.5. Demuestre que el mapeo lineal (2.28) transforma rectas a rectas y circunfe-
rencias a circunferencias. Ayuda: utilice la ecuación general de una recta o una circunfe-
rencia (2.27) conjuntamente con la expresión del mapeo inverso (2.29).

Ejemplo 2.19. Sea S el conjunto de números complejos que se muestra en la Figura 2.9.
Determinar el transformado de S bajo el mapeo w = f (z) = (1 − i)z.

y
2 S

1 b

x
1 2

Figura 2.9. Conjunto S del Ejemplo 2.19

Solución. Se tiene que todo z = x + i y ∈ S satisface que |z − (1 + i)| ≤ 1; además,


f −1 (w) = w/(1 − i). De esta forma, utilizando la ecuación z = f −1 (w) = w/(1 − i)
convenientemente con |z − (1 + i)| ≤ 1, obtenemos:

w |w − 2|
|z − (1 + i)| = − (1 + i) = ≤ 1,
1−i |1 − i|

es decir, todo punto w que pertenece a f (S) satisface que |w − 2| ≤ 2. En la Figura 2.8
se observa el transformado de S bajo el mapeo w = (1 − i)z. 
CAPÍTULO 2. FUNCIONES DE VARIABLE COMPLEJA 55

v
2 f (S)

1 √
2
b

u
1 2 3
−1

Figura 2.10. Conjunto f (S) del Ejemplo 2.19

2.7.3 Mapeo w = bz + c
El mapeo del plano z en el plano w definido por la ecuación

w = bz + c, (2.30)

donde b y c son números complejos, es una rotación en el ángulo Arg b y una expansión ó
contracción según sea el valor de |b|, seguida de una traslación en la dirección del vector c.
En efecto, el mapeo (2.30) se puede expresar a través de los siguientes mapeos sucesivos:

z / Z = bz /w =Z +c
Rotación y Traslación
Expansión ó
Contracción

El mapeo (2.30) es inyectivo ya que es la composición de mapeos inyectivos; por tanto,


posee mapeo inverso definido como
w−c
f −1 (w) = , b 6= 0. (2.31)
b
Además, como (2.30) es la composición de una rotación y una traslación, entonces el
mapeo w = bz + c transforma rectas a rectas y circunferencias a circunferencias.

Ejemplo 2.20. Sea S el conjunto de números complejos que se muestra en la Figura 2.11.
Determinar el transformado de S bajo el mapeo w = f (z) = iz + 3 − i.
y

3
S
2

1 b

x
1 2 3

Figura 2.11. Conjunto S del Ejemplo 2.20


56 2.7. MAPEOS

Solución. Se tiene que el mapeo inverso es f −1 (w) = −iw +1+3i, además, el conjunto S se
puede escribir como S = D1 ∪ D2 , donde D1 y D2 son los conjuntos de números complejos
definidos respectivamente por:

D1 = {z = x + i y : 2x − y ≥ 1, 2x + y ≤ 7, y ≥ 1}

y
D2 = {z = x + i y : (x − 2)2 + (y − 1)2 ≤ 1, y ≤ 1}.
De esta forma, como el mapeo es inyectivo, se tiene que el transformado de S está dado
por f (S) = f (D1 ) ∪ f (D2 ). Ahora, utilizando la ecuación x + i y = f −1 (u + i y), tenemos:
x = v + 1, y = 3 − u. Sustituyendo los valores de x e y en las inecuaciones que describen
a los conjuntos D1 y D2 , obtenemos que los conjuntos transformados de D1 y D2 , bajo el
mapeo w = iz + 3 − i, están dados por

f (D1 ) = {w = u + i v : u + 2v ≥ 2, −u + 2v ≤ 2, u ≤ 2}

y
f (D2 ) = {w = u + i v : (u − 2)2 + (v − 1)2 ≤ 1, u ≥ 2}.
Ası́, la unión de f (D1 ) y f (D2 ) conforma el transformado de S bajo el mapeo w = iz +3−i,
cuya representación gráfica se aprecia en la Figura 2.12.

f (S)
2

1 b

u
1 2 3

Figura 2.12. Conjunto f (S) del Ejemplo 2.20




2.7.4 Mapeo Inversión


El mapeo del plano z en el plano w definido por la ecuación
1
w= , (2.32)
z
para todo z 6= 0, se denomina mapeo inversión, y establece una correspondencia uno a uno
entre los puntos de los planos z y w distintos de cero. Por lo tanto, el mapeo inverso de
f (z) = 1/z es f −1 (w) = 1/w, es decir, el mismo mapeo inversión.
Por otra parte, el mapeo (2.32) también puede escribirse como
z
w= , para todo z 6= 0.
|z|2
CAPÍTULO 2. FUNCIONES DE VARIABLE COMPLEJA 57

Por ello, el mapeo inversión se puede expresar a través de los siguientes mapeos sucesivos:

z /Z= 1 z /w=Z
2|z|

El primero de estos mapeos se denomina inversión con respecto a la circunferencia unitaria


|z| = 1. Es decir, la imagen de un punto z 6= 0 es el punto Z = (1/|z|2 )z con las siguientes
propiedades:
1
|Z| = y arg Z = arg z.
|z|
En otras palabras, los puntos exteriores a la circunferencia |z| = 1 se transforman en los
puntos diferentes de cero interiores a la misma y recı́procamente. Cualquier punto sobre
la circunferencia unitaria se transforma en sı́ mismo, bajo el mapeo Z = (1/|z|2 )z. El
segundo mapeo w = Z es, simplemente, una reflexión con respecto al eje real.

Transformación de Rectas y Circunferencias


Pasemos a ver el efecto que tiene el mapeo inversión sobre las rectas y las circunfe-
rencias. Recordemos que la ecuación general de una recta o una circunferencia está dada
por
α(x2 + y 2 ) + βx + δy + γ = 0, (2.33)
donde α, β, δ, γ ∈ R. También recordemos que si α = 0, la ecuación anterior describe una
recta en el plano z; en cambio, si α 6= 0, describe una circunferencia.
Si z = x + i y es un punto que satisface la ecuación general de una recta o una cir-
cunferencia para ciertos valores de α, β, δ, γ, entonces su transformado w = u + i v = 1/z
satisface la siguiente ecuación (se deja al lector verificar esta relación):

γ(u2 + v 2 ) + βu − δv + α = 0. (2.34)

A continuación consideramos ciertos valores de α, β, δ, γ, que describen algunas rectas


y circunferencias muy particulares, que nos servirán como ejemplo para mostrar la utilidad
de las ecuaciones (2.33) y (2.34), para transformar rectas y circunferencias bajo el mapeo
inversión.

• Circunferencia que no pasa por el origen.


La ecuación (2.33) con α 6= 0 y δ 6= 0, describe una circunferencia que no pasa por el
origen en el plano z. Por la ecuación (2.34), esta circunferencia se transforma, bajo la
inversión, en una circunferencia que no pasa por el origen en el plano w.
y 1 v
w=
z

x u

Ejemplo 2.21. La circunferencia |z − (1 + i)| = 1 que no pasa por el origen en el plano


z, se transforma, bajo la inversión, en la circunferencia |w − (1 − i)| = 1 que no pasa por
el origen en el plano w. 
58 2.7. MAPEOS

• Circunferencia que pasa por el origen.


La ecuación (2.33) con α 6= 0 y δ = 0, describe una circunferencia que pasa por el
origen en el plano z. Por la ecuación (2.34), esta circunferencia se transforma, bajo la
inversión, en una recta que no pasa por el origen en el plano w.

y 1 v
w=
z

x u

Ejemplo 2.22. La circunferencia (x − 1)2 + (y + 1)2 = 2 que pasa por el origen en el


plano z, se transforma, bajo la inversión, en la recta 2u + 2v = 1 que no pasa por el
origen en el plano w. 

• Recta que no pasa por el origen.


La ecuación (2.33) con α = 0 y δ 6= 0, describe una recta que no pasa por el origen en
el plano z. Por la ecuación (2.34), esta recta se transforma, bajo la inversión, en una
circunferencia que pasa por el origen en el plano w.

y 1 v
w=
z

x u

Ejemplo 2.23. La recta x−y = −1 que no pasa por el origen en el plano z, se transforma,
bajo la inversión, en la circunferencia (u + 1)2 + (v + 1)2 = 2 que pasa por el origen en
el plano w. 

• Recta que pasa por el origen.


La ecuación (2.33) con α = 0 y δ = 0, describe una recta que pasa por el origen en el
plano z. Por la ecuación (2.34), esta recta se transforma, bajo la inversión, en una recta
que pasa por el origen en el plano w.

y 1 v
w=
z

x u

Ejemplo 2.24. La recta x − y = 0 que pasa por el origen en el plano z, se transforma,


bajo la inversión, en la recta u + v = 0 que pasa por el origen en el plano w. 
CAPÍTULO 2. FUNCIONES DE VARIABLE COMPLEJA 59

Ejercicio 2.6. Compruebe cada uno de los conjuntos transformados de los Ejemplos 2.21-
2.24.
Ejemplo 2.25. Sea S el conjunto de números complejos dado en el Ejemplo 2.18 y que se
muestra en la Figura 2.6. Comprobemos que el transformado de S bajo el mapeo inversión
es el conjunto de números complejos que se muestra en la siguiente figura.

1
b

u
b
1 2
−1
f (S)

Figura 2.13. Conjunto f (S) del Ejemplo 2.25

Se tiene que
S = {z = x + i y : 2x − y ≥ 1, 2x + y ≤ 7, y ≥ 1},
además, la función inversa de f (z) = 1/z es f −1 (w) = 1/w. Ahora, utilizando la ecuación
x + i y = f −1 (u + i v) obtenemos: x = u/(u2 + v 2 ), e y = −v/(u2 + v 2 ). Sustituyendo
convenientemente estas últimas expresiones en el conjunto de inecuaciones que describe
a S, se tiene que los puntos w = u + i v del transformado de S bajo el mapeo inversión
satisfacen las siguientes inecuaciones:
       
2 1 2 5 1 2 1 2 5 2 1 2 1
(u − 1) + v − ≤ , u− + v+ ≥ , u + v+ ≤ .
2 4 7 14 196 2 4
Por lo tanto, el transformado de S bajo el mapeo inversión es el conjunto de números
complejos que se muestra en la Figura 2.13. 

2.7.5 Mapeo Bilineal


El mapeo del plano z en el plano w definido por la ecuación
az + b
w= , (2.35)
cz + d
donde a, b, c y d son números complejos, se denomina mapeo bilineal o transformación de
Möbius. Este mapeo es inyectivo para todo z ∈ C tal que cz + d 6= 0; por tanto, en ese
conjunto de puntos posee mapeo inverso definido como
−dw + b
f −1 (w) = . (2.36)
cw − a
Cuando c = 0, el mapeo (2.35) adquiere la forma
a b
w= z+ ,
d d
60 2.7. MAPEOS

lo cual indica que (2.35) es un mapeo lineal, estudiado en las secciones anteriores. Ahora,
cuando c 6= 0, el mapeo (2.35) se puede escribir equivalentemente como
 
a bc − ad 1
w= + ,
c c cz + d

donde el número ad − bc se denomina determinante del mapeo. Esto nos permite afirmar
que el mapeo (2.35) se puede expresar a través de los siguientes mapeos sucesivos:
 
/W = 1 /w= a+ bc − ad
z / Z = cz + d W
Rotación con Inversión Z Rotación con c c
Expansión ó Expansión ó
Contracción, y Contracción, y
Traslación Traslación

En otras palabras, el mapeo (2.35) es la composición de mapeos lineales, por lo cual él
transforma circunferencias o rectas en el plano z a circunferencias o rectas en el plano w.
Por otra parte, cuando el determinante del mapeo es cero, ad − bc = 0, el mapeo (2.35)
adquiere la forma
a
w= ,
c
es decir, el mapeo (2.35) transforma todo el plano complejo en el punto w = a/c.

Ejemplo 2.26. Sea S el conjunto de números complejos dado en el Ejemplo 2.18 y que se
muestra en la Figura 2.6. Determinar el transformado de S bajo el mapeo bilineal

iz + 1 + i
w = f (z) = .
iz + i
Solución. Se tiene que

S = {z = x + i y : 2x − y ≥ 1, 2x + y ≤ 7, y ≥ 1}.

Además, la función inversa de f (z) = (iz + 1 + i)/(iz + i) es

i
f −1 (w) = − 1.
1−w

Ahora, utilizando la ecuación x + i y = f −1 (u + i v) obtenemos:

−v − ((u − 1)2 + v 2 ) 1−u


x= , y= .
(u − 1)2 + v 2 (u − 1)2 + v 2

Sustituyendo convenientemente estas últimas expresiones en el conjunto de inecuaciones


que describe a S, se tiene que los puntos w = u + i v del transformado de S, bajo el mapeo
bilineal dado, satisfacen las siguientes inecuaciones
 2    2    2
7 1 2 5 17 2 2 5 1 1
u− + v+ ≤ , u− + v+ ≥ 2, u− + v2 ≤ .
6 3 36 18 18 18 2 4

La gráfica del transformado de S se aprecia en la Figura 2.14. 


CAPÍTULO 2. FUNCIONES DE VARIABLE COMPLEJA 61

b
b

b u
1 2
−1
f (S)

Figura 2.14. Conjunto f (S) del Ejemplo 2.26

2.8 Funciones, Lı́mite, Derivada y Mapeos con M ATLAB


En esta sección se utilizan las herramientas de matemática simbólica (Symbolic Math
Toolbox, Matlab R2015a) de M ATLAB . Para usar esta herramienta es necesario definir
las variables como objetos simbólicos. Por ejemplo, para declarar x e y como objetos
simbólicos se usa el comando syms:

>> syms x y
✂ ✁
Los objetos simbólicos se pueden manipular siguiendo las reglas de las operaciones ma-
temáticas, por ejemplo:

>> x + y− 5∗x + yˆ2
ans =
yˆ2 + y − 4∗x
✂ ✁
Una función de variable compleja f (z) también se puede crear como una función
simbólica. Primero se crea el objeto simbólico z, luego se define la función simbólica f(z)
según su forma algebraica. En el siguiente ejemplo se crea la función f (z) = z 2 + iz − 1.

>> syms z
>> f ( z ) = z ˆ2 + i ∗ z − 1
f (z) =
z ˆ2 + z ∗ i − 1
✂ ✁
Después de crear una función simbólica, se puede diferenciar, integrar o simplificar,
evaluarla en valores y realizar otras operaciones matemáticas. Considerando la función
f (z) = z 2 + iz − 1 creada previamente, seguidamente se evalúa en z = i.

>> f ( i )
ans =
−3
✂ ✁
También se pueden hacer operaciones entre funciones simbólicas: sumar, multiplicar,
dividir funciones, entre otras. En el siguiente ejemplo se crean dos funciones, f1 (z) = z 2 −i
y f2 (z) = (z − 1)(z − 2), con las que se crean otras tres funciones: g1 (z) = f1 (z) + f2 (z),
g2 (z) = f1 (z)f2 (z) y g3 (z) = f1 (z)/f2 (z).

>> syms z
>> f 1 ( z ) = zˆ2−i , f 2 ( z ) = ( z −1)∗(z−2)
f1 ( z ) =
z ˆ2 − i
f2 ( z ) =
( z − 1)∗( z − 2)
62 2.8. FUNCIONES, LÍMITE, DERIVADA Y MAPEOS CON MATLAB

>> g1 ( z)=f 1 ( z)+f 2 ( z ) , g2 ( z)=f 1 ( z )∗ f 2 ( z ) , g3 ( z)=f 1 ( z ) / f 2 ( z )


g1 ( z ) =
( z − 1)∗( z − 2) + z ˆ2 − i
g2 ( z ) =
( z ˆ2 − i ) ∗ ( z − 1)∗( z − 2)
g3 ( z ) =
( z ˆ2 − i ) / ( ( z − 1)∗( z − 2 ) )
✂ ✁
Una operación importante en matemáticas es el cálculo de lı́mites, la cual también
se puede realizar con la herramienta simbólica de M ATLAB . En particular, el comando
Para más deta- limit(f,z,a) calcula el lı́mite lı́mz→a f (z).Observe el siguiente ejemplo donde se calcula
lles del comando
limit, teclee
el lı́mite
help limit en
el escritorio de z + z + 2 Re z (Im z + 1)i
M ATLAB lı́m = 1 + i.
✄ z→0 z+z
>> syms z
>> f ( z ) = ( z+c o n j ( z)+2∗ r e a l ( z ) ∗ ( imag ( z )+1)∗ i ) / ( z+c o n j ( z ) )
f (z) =
( z + c o n j ( z ) + r e a l ( z ) ∗ ( imag ( z ) + 1)∗2∗ i ) / ( z + c o n j ( z ) )
>> l i m i t ( f , z , 0 )
ans ( z ) =
1 + i
✂ ✁
La derivada de una función f (z) se calcula usando el comando diff. Especı́ficamente,
diff(f,z,n) calcula la derivada enésima de f (z). Por ejemplo, la primera, segunda y
tercera derivada de
2i − z 4
f (z) = 3 ,
z + iz 2 − z + 1
se calculan como sigue:

>> syms z
>> f ( z )=(2∗ i −z ˆ 4 ) / ( zˆ3+ i ∗ zˆ2−z+1)
f (z) =
−(z ˆ4 − 2 i ) / ( z ˆ3 + z ˆ2∗1 i − z + 1)
>> d i f f ( f , z )
ans ( z ) =
− (4∗ z ˆ 3 ) / ( z ˆ3 + z ˆ2∗1 i − z + 1) +
( ( z ˆ4 − 2 i )∗(3∗ z ˆ2 + z ∗2 i − 1 ) ) / ( z ˆ3 + z ˆ2∗1 i − z + 1)ˆ2
>> d i f f ( f , z , 2 )
ans ( z ) =
− (12∗ z ˆ 2 ) / ( z ˆ3 + z ˆ2∗1 i − z + 1) − (2∗( z ˆ4 − 2 i )∗(3∗ z ˆ2 + z ∗2 i − 1)ˆ2)/
( z ˆ3 + z ˆ2∗1 i − z + 1)ˆ3 + (8∗ z ˆ3∗(3∗ z ˆ2 + z ∗2 i − 1 ) ) /
( z ˆ3 + z ˆ2∗1 i − z + 1)ˆ2 + ((6∗ z + 2 i ) ∗ ( z ˆ4 − 2 i ) ) /
( z ˆ3 + z ˆ2∗1 i − z + 1)ˆ2
>> d i f f ( f , z , 3 )
ans ( z ) =
(6∗( z ˆ4 − 2 i ) ) / ( z ˆ3 + z ˆ2∗1 i − z + 1)ˆ2 − (24∗ z ) / ( z ˆ3 + z ˆ2∗1 i − z + 1) +
(6∗( z ˆ4 − 2 i )∗(3∗ z ˆ2 + z ∗2 i − 1 ) ˆ 3 ) / ( z ˆ3 + z ˆ2∗1 i − z + 1)ˆ4 +
(12∗ z ˆ3∗(6∗ z + 2 i ) ) / ( z ˆ3 + z ˆ2∗1 i − z + 1)ˆ2 + (36∗ z ˆ2∗(3∗ z ˆ2 + z ∗2 i − 1 ) ) /
( z ˆ3 + z ˆ2∗1 i − z + 1)ˆ2 − (24∗ z ˆ3∗(3∗ z ˆ2 + z ∗2 i − 1)ˆ2)/
( z ˆ3 + z ˆ2∗1 i − z + 1)ˆ3 − (6∗(6∗ z + 2 i ) ∗ ( z ˆ4 − 2 i )∗(3∗ z ˆ2 + z ∗2 i − 1 ) ) /
( z ˆ3 + z ˆ2∗1 i − z + 1)ˆ3
✂ ✁
En este caso, las expresiones obtenidas de la segunda y tercera derivada de f (z) no están
simplificadas. Para buscar la forma más simple de una expresión simbólica se utiliza el
comando simplify. Utilicemos tal comando para hallar las formas simplificadas de la
primera, segunda y tercera derivada de f (z) en el ejemplo anterior.
CAPÍTULO 2. FUNCIONES DE VARIABLE COMPLEJA 63


>> d2f=s i m p l i f y ( d i f f ( f , z , 2 ) ) ; d2f
d2f ( z ) =
(6∗( z ˆ5∗(1 + 1 i ) − z ˆ4∗(1 − 3 i ) − (8∗ z ˆ3)/3 − z ˆ2∗(2 + 4 i ) +
z ∗(2 − 2 i ) + (2/3 + 2 i / 3 ) ) ) / ( z ˆ3 + z ˆ2∗1 i − z + 1)ˆ3
>> d3f=s i m p l i f y ( d i f f ( f , z , 3 ) ) ; d3f
d3f ( z ) =
−(( z ˆ7∗(2 − 2 i ) + z ˆ6∗(8 + 3 i ) + z ˆ5∗10 i − z ˆ4∗(19 − 10 i ) − z ˆ3∗(18 + 18 i ) +
z ˆ2∗(10 − 13 i ) + z ∗(8 + 2 i ) + 2 i )∗12 i )/(− z ˆ3∗1 i + z ˆ2 + z ∗1 i − 1 i )ˆ4
✂ ✁

M ATLAB está provisto de un gran número de funciones elementales. Algunas de estas


funciones se pueden apreciar en la Tabla ?? del Capı́tulo A. Los argumentos de las funciones
elementales pueden ser (siempre que tenga sentido) reales o complejos y el resultado se
devuelve en el mismo tipo del argumento. En los siguientes ejemplos mostramos como se
utilizan algunas funciones elementales en M ATLAB . Observe el siguiente ejemplo:

>> c o s (1− i )
ans =
0.8337 + 0.9889 i
>> l o g (1+ i )
ans =
0.3466 + 0.7854 i
>> exp ( p i ∗ i )
ans =
−1.0000 + 0.0000 i
✂ ✁

Con el comando cos(1-i) √ se obtiene el valor de cos(1 − i), con log(1+i) se obtiene el
π
valor de Log (1 + i) = ln 2 + i 4 , y con exp(pi*i) se obtiene eπi = −1.
Con las funciones elementales se pueden crear cualquier tipo de funciones simbólicas.
Observe el siguiente ejemplo donde se crea la función

sen (1 − Log z)
f (z) =
ez 2 +1 − 2

y se obtienen los valores |f (i)| y |f ′ (i)|.



>> syms z
>> f ( z)= s i n (1− l o g ( z ) ) / ( exp ( zˆ2+1)−2)
f (z) =
−s i n ( l o g ( z ) − 1 ) / ( exp ( z ˆ2 + 1) − 2)
>> abs ( f ( i ) )
ans =
( cosh ( p i /2)ˆ2∗ s i n (1)ˆ2 + s i n h ( p i /2)ˆ2∗ c o s ( 1 ) ˆ 2 ) ˆ ( 1 / 2 )
>> d f ( z ) = d i f f ( f , z ) ;
>> abs ( d f ( i ) )
ans =
( ( cosh ( p i /2)∗ c o s ( 1 ) + 2∗ cosh ( p i /2)∗ s i n ( 1 ) ) ˆ 2 +
(2∗ s i n h ( p i /2)∗ c o s ( 1 ) − s i n h ( p i /2)∗ s i n ( 1 ) ) ˆ 2 ) ˆ ( 1 / 2 )
✂ ✁

M ATLAB también puede emplearse para hallar los conjuntos transformados de circun-
ferencias y polı́gonos, bajo el mapeo lineal w = bz + c. El Programa 2.1 muestra la función
MapeoPoligono que halla el transformado de un polı́gono con vértices z1 , z2 , . . . , zn ∈ C,
bajo el mapeo lineal w = bz+c. El Programa 2.2 muestra la función MapeoCircunferencia
que halla el transformado de una circunferencia de centro z0 ∈ C y radio r > 0, bajo el
mapeo lineal w = bz + c.
64 2.8. FUNCIONES, LÍMITE, DERIVADA Y MAPEOS CON MATLAB

Programa 2.1. Función MapeoPoligono.m

f u n c t i o n w = MapeoPoligono ( b , c , z )
%MapeoPoligono H a l l a h a l l a e l t r a n s f o r m a d o de un p o lı́ g o n o
% cu yos v é r t i c e s son l o s elementos d e l a r r e g l o z ,
% b a j o e l mapeo w = bz + c .
% w e s un a r r e g o que c o n t i e n e l o s v é r t i c e s d e l
% p o lı́ g o n o t r a n s f o r m a d o
w = b∗ z + c ;
subplot (1 ,2 ,1)
p l o t ( [ z , z ( 1 ) ] , ’ r ’ , ’ LineWidth ’ , 2 )
xlabel ( ’ x ’ )
ylabel ( ’ y ’ )
g r i d on
xmin = min ( [ min ( r e a l ( z ) ) min ( r e a l (w) ) ] ) − . 5 ;
xmax = max( [ max( r e a l ( z ) ) max( r e a l (w) ) ] ) + . 5 ;
ymin = min ( [ min ( imag ( z ) ) min ( imag (w) ) ] ) − . 5 ;
ymax = max( [ max( imag ( z ) ) max( imag (w) ) ] ) + . 5 ;
a x i s ( [ xmin xmax ymin ymax ] )
a x i s equal
t i t l e ( ’ P o lı́ g o n o ’ )
%
subplot (1 ,2 ,2)
p l o t ( [w,w( 1 ) ] , ’ b ’ , ’ LineWidth ’ , 2 )
xlabel ( ’u ’ )
ylabel ( ’ v ’ )
g r i d on
a x i s ( [ xmin xmax ymin ymax ] )
a x i s equal
t e x t o = s p r i n t f ( ’w=(%s ) z+%s ’ , num2str ( b ) , num2str ( c ) ) ;
t e x t o = [ ’ P o lı́ g o n o t r a n s f o r m a d o b a j o e l mapeo ’ , t e x t o ] ;
t i t l e ( texto )
end

Programa 2.2. Función MapeoCircunferencia.m

f u n c t i o n [w0, l ] = M a p e o C i r c u n f e r e n c i a ( b , c , z0 , r )
%M a p e o C i r c u n f e r e n c i a H a l l a e l t r a n s f o r m a d o de una c i r c u n f e −
% r e n c i a con c e n t r o z0 y r a d i o r , b a j o e l
% mapeo w = bz + c .
% w0 e s e l c e n t r o y l e s e l r a d i o de l a c i r c u n f e r e n c i a
% transformada
w0 = b∗ z0 + c ;
l = abs ( b )∗ r ;
t = l i n s p a c e (0 ,2∗ p i ) ;
subplot (1 ,2 ,1)
x t = r ∗ c o s ( t )+ r e a l ( z0 ) ;
y t = r ∗ s i n ( t )+imag ( z0 ) ;
p l o t ( xt , yt , ’ r ’ , ’ LineWidth ’ , 2 )
xlabel ( ’ x ’ )
ylabel ( ’ y ’ )
g r i d on
rmax = max( [ r l ] ) ;
xmin = min ( [ r e a l ( z0 ) r e a l (w0)]) −rmax −.2;
xmax = max( [ r e a l ( z0 ) r e a l (w0)])+rmax +.2;
ymin = min ( [ imag ( z0 ) imag (w0)]) −rmax −.2;
ymax = max( [ imag ( z0 ) imag (w0)])+rmax +.2;
CAPÍTULO 2. FUNCIONES DE VARIABLE COMPLEJA 65

a x i s ( [ xmin xmax ymin ymax ] )


a x i s equal
t i t l e ( ’ Cricunferencia ’ )
%
subplot (1 ,2 ,2)
u t = l ∗ c o s ( t )+ r e a l (w0) ;
v t = l ∗ s i n ( t )+imag (w0) ;
p l o t ( ut , vt , ’ b ’ , ’ LineWidth ’ , 2 )
xlabel ( ’u ’ )
ylabel ( ’ v ’ )
g r i d on
a x i s ( [ xmin xmax ymin ymax ] )
a x i s equal
t e x t o = s p r i n t f ( ’w=(%s ) z+%s ’ , num2str ( b ) , num2str ( c ) ) ;
t e x t o = [ ’ C i r c u n f e r e n c i a t r a n s f o r m a d a b a j o e l mapeo ’ , t e x t o ] ;
t i t l e ( texto )
end

A continuación usamos la función MapeoPoligono para hallar el transformado del


polı́gono con vértices 1 + 2i, 2i, 2, 2 − 2i, 0 y −2 − i, bajo el mapeo w = (−1 + i)z + i.

>> z=[−1+2 i 2 i 2 2−2 i 0 −2−i ] ; b=−1+i ; c=i ;
>> w = MapeoPoligono ( b , c , z )
w=
Columns 1 throu gh 4
−1.0000 − 2.0000 i −2.0000 − 1.0000 i −2.0000 + 3.0000 i 0 + 5.0000 i
Columns 5 throu gh 6
0 + 1.0000 i 3.0000
✂ ✁

Polígono Polígono transformado bajo el mapeo w=(−1+1i)z+0+1i

5 5

4 4

3 3

2 2
y

1 1

0 0

−1 −1

−2 −2

−2 0 2 −2 −1 0 1 2 3
x u

Seguidamente usamos la función


√ MapeoCircunferencia para hallar el transformado
de la circunferencia |z − 1 + i| = 2, bajo el mapeo w = (−2 − i)z + 4.

>> z0=1−i ; r=s q r t ( 2 ) ; b=−2−i ; c =4;
>> [w0, l ] = M a p e o C i r c u n f e r e n c i a ( b , c , z0 , r )
w0 =
1.0000 + 1.0000 i
l =
3.1623
✂ ✁
66 2.9. PROBLEMAS RESUELTOS

Cricunferencia Circunferencia transformada bajo el mapeo w=(−2−1i)z+4

4 4

3 3

2 2

1 1

0 0
y

v
−1 −1

−2 −2

−3 −3

−4 −4
−2 0 2 4 −2 0 2 4
x u

2.9 Problemas Resueltos


Problema 2.1. Pruebe que los siguientes lı́mites no existen en punto alguno z0 ∈ C.
z − z0
a) lı́m .
z→z0 z − z0

Re z − Re z0
b) lı́m .
z→z0 z − z0
Solución. Demostremos que los lı́mites dados alcanzan valores distintos cuando z tiende
a z0 por los siguientes caminos: i) una recta horizontal que pasa por z0 , esto es, tomando
z = x + iy0 ; y ii) una recta vertical que pasa por z0 , esto es, tomando z = x0 + iy.
a) El lı́mite de z−z
z−z0 cuando z tiende a z0 por el camino i) está dado por:
0

z − z0 x + iy0 − x0 + iy0 x − x0
lı́m = lı́m = lı́m = 1.
z→z0 z − z0 z→z 0 x + iy0 − x0 − iy0 z→z 0 x − x0

z−z0
Ahora, el lı́mite de z−z0 cuando z tiende a z0 por el camino ii) está dado por:

z − z0 x0 + iy − x0 + iy0 i(−y + y0 )
lı́m = lı́m = lı́m = −1.
z→z0 z − z0 z→z0 x0 + iy − x0 − iy0 z→z0 i(y − y0 )
z−z0
Por lo tanto, el lı́mite lı́m no existe.
z→z0 z−z0
Re z−Re z0
b) El lı́mite de z−z0 cuando z tiende a z0 por el camino i) está dado por:
Re z − Re z0 Re (x + iy0 ) − Re (x0 + iy0 ) x − x0
lı́m = lı́m = lı́m = 1.
z→z0 z − z0 z→z0 x + iy0 − x0 − iy0 z→z0 x − x0

Re z−Re z0
El lı́mite de z−z0 cuando z tiende a z0 por el camino ii) está dado por:
Re z − Re z0 Re (x0 + iy) − Re (x0 + iy0 ) x0 − x0
lı́m = lı́m = lı́m = 0.
z→z0 z − z0 z→z0 x0 + iy − x0 − iy0 z→z0 i(y − y0 )

Re z−Re z0
Por lo tanto, el lı́mite lı́m z−z0 no existe. 
z→z0
CAPÍTULO 2. FUNCIONES DE VARIABLE COMPLEJA 67

Problema 2.2. Sea f (z) una función de variable compleja definida como

2
z ,
 Im z < Re z,
f (z) = 2(Re z)2 i, Im z = Re z,


(Im z)z, Im z > Re z.

Determine el conjunto de todos los números complejos z donde la función f (z) es continua.

Solución. Es claro que f (z) está bien definida en todo z ∈ C. Como f (z) = z 2 para
Im z < Re z y z 2 es un polinomio, entonces f (z) es continua en z ∈ C tal que Im z < Re z.
Ahora, para z = x + iy tal que Im z > Re z, se tiene que f (z) = (Im z)z = yx + iy 2 ; de lo
cual se deduce que las funciones componentes de f (z) son u(x, y) = yx y v(x, y) = y 2 , para
y > x, que son continuas en todo (x, y) ∈ R2 , en particular, en todo (x, y) tal que y > x.
De esta forma, f (z) es continua en todo z tal que Im z > Re z o Im z < Re z. Veamos ahora
que f (z) no es continua en z tal que Im z = Re z 6= 0. Sea z0 tal que Im z0 = Re z0 6= 0.
Calculemos el lı́mite lı́m f (z) en la dirección de la recta Im z = Re z0 − Re z + Im y0 , con
z→z0
Im z > Re z. Ası́,

lı́m f (z) = lı́m (Im z)z = (Im z0 )z0 = (Re z0 )2 + i(Re z0 )2 6= 2(Re z0 )2 i = f (z0 ),
z→z0 z→z0

Por lo cual f (z) no es continua en z tal que Im z = Re z 6= 0. Es claro que f (z) es continua
z0 = 0. De todo lo anterior, se tiene que el conjunto de todos los números complejos z
donde f (z) es continua está dado por:

C − {z ∈ C : Im z = Re z 6= 0}

Problema 2.3. Si u y v se expresan en términos de las coordenadas polares (r, θ), muestre
que las ecuaciones de Cauchy-Riemann pueden escribirse de la forma, para r > 0,

 ∂u 1 ∂v

 =
∂r r ∂θ
 1 ∂u ∂v


= −
r ∂θ ∂r

denominada forma polar de las ecuaciones de Cauchy-Riemann.

Solución. Asumamos que u(x, y) y v(x, y) satisfacen las ecuaciones de Cauchy-Riemann


en su forma cartesiana, esto es

∂u ∂v
 ∂x = ∂y

 ∂u
 ∂v
 = −
∂y ∂x
68 2.9. PROBLEMAS RESUELTOS

Ahora, haciendo el cambio de variable x = r cos θ e y = r sen θ y aplicando la regla de la


cadena, obtenemos:

∂u ∂u ∂x ∂u ∂y ∂u ∂u
= + = cos θ + sen θ,
∂r ∂x ∂r ∂y ∂r ∂x ∂y
∂u ∂u ∂x ∂u ∂y ∂u ∂u
= + = −r sen θ + r cos θ,
∂θ ∂x ∂θ ∂y ∂θ ∂x ∂y
∂v ∂v ∂x ∂v ∂y ∂v ∂v
= + = cos θ + sen θ,
∂r ∂x ∂r ∂y ∂r ∂x ∂y
∂v ∂v ∂x ∂v ∂y ∂v ∂v
= + = −r sen θ + r cos θ.
∂θ ∂x ∂θ ∂y ∂θ ∂x ∂y

De esta forma, usando las ecuaciones anteriores conjuntamente con las ecuaciones de
Cauchy-Riemann en su forma cartesiana, podemos escribir:
 
∂u 1 ∂v ∂u ∂u 1 ∂v ∂v
− = cos θ + sen θ − −r sen θ + r cos θ
∂r r ∂θ ∂x ∂y r ∂x ∂y
   
∂u ∂v ∂u ∂v ∂u 1 ∂v
= − cos θ + + sen θ = 0 ⇒ = ,
∂x ∂y ∂y ∂x ∂r r ∂θ
 
1 ∂u ∂v 1 ∂u ∂u ∂v ∂v
+ = −r sen θ + r cos θ + cos θ + sen θ
r ∂θ ∂r r ∂x ∂y ∂x ∂y
   
∂u ∂v ∂u ∂v 1 ∂u ∂v
= − + cos θ + + sen θ = 0 ⇒ =− .
∂x ∂y ∂y ∂x r ∂θ ∂r

Problema 2.4. Sea f (z) una función de variable compleja definida como
 
1
f (z) = log ,
z−1

donde log z = ln |z| + iarg z, |z| > 0 y −π/4 < arg z < 7π/4. Determine el conjunto de
todos los números complejos z donde f (z) es analı́tica.

Solución. Por las condiciones del enunciado del problema, se tiene que la función log z
es analı́tica en todo el plano complejo excepto en el conjunto {z ∈ C : Re z ≥ 0, Im z =
−Re z}. Ahora, como
   
1 Re z − 1 1 −Im z
Re = 2 2
e Im = ,
z−1 (Re z − 1) + (Im z) z−1 (Re z − 1)2 + (Im z)2
 
1
entonces la función f (z) = log z−1 es analı́tica en todo el plano complejo excepto en el
conjunto
 
Re z − 1 −Im z Re z − 1
z∈C: ≥ 0, =−
(Re z − 1)2 + (Im z)2 (Re z − 1)2 + (Im z)2 (Re z − 1)2 + (Im z)2
CAPÍTULO 2. FUNCIONES DE VARIABLE COMPLEJA 69

o, equivalentemente, {z ∈ C : Re z ≥ 1, Im z = Re z − 1}. Por lo tanto, el conjunto de to-


dos los números complejos z donde f (z) es analı́tica está dado por:
C − {z ∈ C : Re z ≥ 1, Im z = Re z − 1}


Problema 2.5. Sea u(x, y) una función definida de R2 en R dada por


(
Re (z 2 + 1), si x ≤ 0,
u(x, y) =
x−3 y 3 , si x > 0.

Determine el conjunto de todos los pares ordenados (x, y) donde u(x, y) es armónica,
además, construya una función f (z) = u(x, y) + iv(x, y) y el mayor dominio D ⊂ C tales
que f (z) es analı́tica en D y f (−1 − i) = 1 − i.

Solución. Se tiene que la función z 2 + 1 es entera; por tanto, sus funciones componentes
Re (z 2 + 1) e Im (z 2 + 1) son funciones armónicas en todo R2 . Como u(x, y) = Re (z 2 + 1) =
x2 − y 2 + 1, para cada z = x + iy tal que x < 0, entonces u(x, y) es armónica en el conjunto
{(x, y) ∈ R2 : x ≤ 0}. Ahora, para (x, y) ∈ R2 tal que x > 0, se tiene que u(x, y) = x−3 y 3 ;
ası́
∂2u ∂2u
∇2 u(x, y) = (x, y) + (x, y)
∂x2 ∂y 2
= 12x−5 y 3 + 6x−3 y

= 6x−3 y x−2 y 2 + 1 = 0 ⇔ y = 0,
es decir, u(x, y) también es armónica en el conjunto {(x, y) ∈ R2 : x > 0, y = 0}, que no
es un dominio. De esta forma, u(x, y) es armónica en el conjunto
{(x, y) ∈ R2 : x ≤ 0} ∪ {(x, y) ∈ R2 : x > 0, y = 0}.
En consecuencia, el dominio D ⊂ C donde la función f (z) = u(x, y) + iv(x, y) es analı́tica
es
D = {z = x + iy : x < 0}.
Como u(x, y) = Re (z 2 + 1) = x2 − y 2 + 1 para todo z ∈ D, entonces es claro que la función
v(x, y) = Im (z 2 + 1) + c = 2xy + c es armónica conjugada de u(x, y), donde c es una
constante real. Por lo tanto, forzando f (−1 − i) = 1 − i, se obtiene
1 − i = f (−1 − i) = u(−1, −1) + iv(−1, −1) = 1 + (2 + c)i ⇒ c = −3.
Es decir, la función
f (z) = (x2 − y 2 + 1) + i(2xy − 3) = z 2 + 1 − 3i
es analı́tica en D y f (−1 − i) = 1 − i. 

Problema 2.6. Sean α un número real y z0 un número complejo tal que Im z0 > 0. Demo-
strar que el mapeo bilineal  
iα z − z0
w=e ,
z − z0
transforma al semiplano superior Im z ≥ 0 sobre en el disco unitario |w| ≤ 1.
70 2.9. PROBLEMAS RESUELTOS

Solución. Sean z = x + iy y z0 = x0 + iy0 . Se tiene que


 2 2
2 iα z − z0 iα 2 z − z0 |z − z0 |2 (x − x0 )2 + (y − y0 )2
|w| = e = e = = .
z − z0 z − z0 |z − z 0 |2 (x − x0 )2 + (y + y0 )2
Como Im z0 = y0 > 0 e Im z = y ≥ 0, entonces de la ecuación anterior se deduce que
|w|2 ≤ 1,
la cual implica que |w| ≤ 1. 

Problema 2.7. Sea A ⊂ C el conjunto sombreado que se muestra en la siguiente figura.


y
1
A

1
2

-1 − 21 1
1 x
2

Determine la expresión analı́tica y luego grafique el transformado del conjunto A bajo el


mapeo
z−1
w= .
i−z
Solución. El conjunto A se puede expresar analı́tica a través del siguiente sistema de
inecuaciones: 
 |z| ≤ 1,
A: |z| ≥ 21 , (2.37)

y ≥ 0.
z−1
Ahora, despejemos z = x + iy en función w = u + iv, a partir de la ecuación w = .
i−z
Se tiene que
iw + 1
iw − wz = z − 1 ⇔ z(w + 1) = iw + 1 ⇔ z= , (2.38)
w+1
de donde se deduce
i(u + iv) + 1 [(1 − v) + iu] [(u + 1) − iv]
x + iv = ⇔ x + iv = ,
u + iv + 1 (u + 1)2 + v 2
lo que implica que
u−v+1 u2 + v 2 + u − v
x= , y= . (2.39)
(u + 1)2 + v 2 (u + 1)2 + v 2
Sustituyendo convenientemente en (2.37) las expresiones de x, y y z dadas en (2.38) y
(2.39), y luego operando, se obtienen las ecuaciones que describen el transformado de A,

 iw + 1 2
 ≤ 1, 
|iw + 1|2 ≤ |w + 1|2 ,



 w+1 


 

iw + 1 2
 
f (A) : ≥ 1
4 , ⇔ |iw + 1|2 ≥ 14 |w + 1|2 , ⇔


 w + 1 


 
 2
u + v 2 + u − v ≥ 0,


 u2 + v 2 + u − v

 ≥ 0,
(u + 1)2 + v 2
CAPÍTULO 2. FUNCIONES DE VARIABLE COMPLEJA 71

 

 |(1 − v) + iu|2 ≤ |(u + 1) + iv|2 , 
 (1 − v)2 + u2 ≤ (u + 1)2 + v 2 ,

 

  
|(1 − v) + iu|2 ≥ 1
4 |(u + 1) + iv|2 , ⇔ 4 (1 − v)2 + u2 ≥ (u + 1)2 + v 2 ,

 


 

 (u + 1 )2 + (v − 1 )2 ≥ 1 ,
u2 + v 2 + u − v ≥ 0,

2 2 2
 

 v 2 − 2v + 1 + u2 ≤ u2 + 2u + 1 + v 2 , 
 v ≥ −u,

 

 
4v 2 − 8v + 4 + 4u2 ≥ u2 + 2u + 1 + v 2 , ⇔ u2 + v 2 − 32 u − 83 v + 1 ≥ 0,

 


 

 (u + 1 )2 + (v − 1 )2 ≥ 1 .  (u + 21 )2 + (v − 12 )2 ≥ 12 .
2 2 2

De esta forma, el transformado de A está dado por:




 v ≥ −u,



f (A) : (u − 31 )2 + (v − 43 )2 ≥ 89 ,




 (u + 1 )2 + (v − 1 )2 ≥ 1 .
2 2 2

cuya representación gráfica se muestra en la siguiente figura.


v
..

4 f (A)
.

3
..
.

-4 -3 -2 -1 1 2 3 4 u
-1
..
.

2.10 Problemas Propuestos


2.1. Encuentre el dominio de definición de cada una de las siguientes funciones.
1 |z|2 − 1
a) f (z) = e) g(z) =
1 − |z|2 |z + i|
1 1
b) f (z) = 2 f) h(z) =
z +1 2 − (z − i)2
z
c) g(z) = g) h(z) = Arg (1/z)
z+z
1−z z 2 − z + (1 − z)i
d) g(z) = 3 h) h(z) =
z + iz 2 + 3z − i z3 + z
72 2.10. PROBLEMAS PROPUESTOS

2.2. Encuentre las funciones componentes, u(x, y) y v(x, y), de las siguientes funciones
f (z), donde z = x + iy.
1 c) f (z) = z 2 − 3z + 2.
a) f (z) = .
z2
z+1 z+2
b) f (z) = . d) f (z) = .
2z − 5 2z − 1 + i
z+2
2.3. Dada la función f (z) = , establecer en cada caso el conjunto de puntos
2z − 1
z ∈ C tales que:

a) f (z) = i b) f (z) = z c) f (z) = 2

2.4. Calcule los siguientes lı́mites:


1−z z2 + 1
a) lı́m f) lı́m
z→2i 1 + z z→i z6 + 1
b) lı́m (2z 3 − 3z 2 + z + i) z
z→2i g) lı́m
z→(3−4i) z+z
z2
c) lı́m
πi z4 + z + 1 |z|2 − 1
z→e 4 h) lı́m
z→(1−i) |z + i|
z 2 − 2i z
d) lı́m
z→2+i z 2 + 4 i) lı́m Arg (1/z)
z→i
z 2 − 2i z
e) lı́m j) lı́m (ln |z| + i Arg z)
z→2i z 2 + 4 z→−i

2.5. Dada la función f : C → C definida como

(z + z) + 2(Re z)(Im z + 1)i


f (z) = ,
z+z
calcule lı́mite de f (z) cuando z tiende a 0.

2.6. Determine el conjunto de números complejos donde las siguientes funciones son
continuas:
 2
 3z − 5iz + 2
 , z 6= 2i;
a) f (z) = z 2 + iz + 6

7
5, z = 2i.
 2
 z +1
 , z 6= −i;
b) g(z) = z+i


−2i, z = −i.
 Re z

 , Re z 6= Im z, Im z 6= 0;
 Im z



c) h(z) = 1, Re z = Im z, Im z 6= 0;






Re z, Im z = 0.
CAPÍTULO 2. FUNCIONES DE VARIABLE COMPLEJA 73

2.7. Sea f (z) una función definida por



(z + i)2 , si y 6= x,
f (z) =
(−2x − 1) + i 2(x2 + x), si y = x,

donde z = x + iy. Determine el conjunto de todos los números complejos z donde


f (z) es continua.

2.8. Pruebe que cada una de las siguientes funciones satisfacen las ecuaciones de
Cauchy-Riemann:

a) f (z) = ex (−sen y + i cos y).


b) f (z) = cos x cosh y − i sen x senh y.
c) f (z) = sen x cosh y + i cos x senh y.
2 −y 2
d) f (z) = ex (cos(2xy) + i sen (2xy)).

2.9. Determine en qué conjunto D de números complejos z = x + i y, las siguientes


funciones son derivables y calcule su derivada.

a) f (z) = x2 + i y 2 c) f (z) = 2xy + i (x + 23 y 3 )


b) f (z) = x2 + 2x − i y

2.10. Utilizando las condiciones necesarias y suficientes para la existencia de derivada,


demostrar que f ′ (z) no existe en punto alguno del plano complejo si f (z) está
dada por:

a) f (z) = z. c) f (z) = 2x + ixy 2 .


b) f (z) = z − z. d) f (z) = ex e−iy .

2.11. Determine el conjunto D de todos los números complejos donde cada una de las
siguientes funciones f (z) es analı́tica y calcule su derivada:

a) f (z) = (z + 1)2 z4
e) f (z) =
1 1 + z4
b) f (z) = z +
z Log (z + 4)
3z − 1 f) f (z) =
c) f (z) = z2 + i
3−z g) f (z) = ee
z

2z + 1
d) f (z) = h) f (z) = sen (ez )
z(z 2 + 1)

2.12. Comprobar que cada una de las siguientes funciones son enteras:

a) f (z) = 3x + y + i(3y − x).


b) f (z) = e−y eix .
c) f (z) = (z 2 − 2)e−z .
d) f (z) = sen (x2 − y 2 ) cosh(2xy) + i cos(x2 − y 2 ) senh(2xy).
74 2.10. PROBLEMAS PROPUESTOS

2.13. Determine el conjunto de todos los números complejos z = x+iy donde la función
y 2 − x2
f (z) = + i|1 − xy| es analı́tica.
2
2.14. Determine en qué conjunto D ⊂ R2 cada una de las siguientes funciones son
armónicas, y encuentre de ser posible su armónica conjugada.

a) u(x, y) = Im (z 2 + 3z + 1) c) v(x, y) = Im (z + 1/z)


x−1 −y − 2
b) u(x, y) = 2 d) v(x, y) = +3
x + y 2 − 2x + 1 (x + 1)2 + (y + 2)2

2.15. Demostrar las siguientes identidades.

a) e(2±3πi) = −e2
b) ez+πi = −ez
c) exp(z) = exp(iz), para z ∈ C
d) senh(z + πi) = − senh z, para z ∈ C
e) cosh(z + πi) = − cosh z, para z ∈ C
f) senh 2z = 2 senh z cosh z, para z ∈ C
g) Log (ei) = 1 + (π/2)i
h) Log (1 + i) = 21 (ln 2 + (π/2)i)
i) log 1 = 2nπi, para n = 0, ±1, ±2, . . .
j) log(−1) = (2n + 1)πi, para n = 0, ±1, . . .

k) log i1/2 = (n + 1/4)πi, para n = 0, ±1, . . .
l) (1 + i)i = exp(−π/4 + 2nπ) exp((i/2) ln 2), para n = 0, ±1, ±2, . . .
m) (−1)1/π = exp((2n + 1)i), para n = 0, ±1, ±2, . . .

2.16. Demostrar que:

a) si log z = ln r + iθ, para todo z tal que r = |z| > 0 y π/4 < θ = arg z < 9π/4,
entonces log(i2 ) = 2 log i.
b) si log z = ln r + iθ, para todo z tal que r = |z| > 0 y 3π/4 < θ = arg z < 11π/4,
entonces log(i2 ) 6= 2 log i.

2.17. Encontrar todas las raı́ces de cada una de las siguientes ecuaciones:

a) cosh z = 1/2 c) tanh z = −2 e) ez = −3


b) senh z = i d) log z = (π/2)i

2.18. Encontrar el valor principal de:

a) ii b) (1 − i)4i c) (−i)3πi
 
1
2.19. Sea f (z) = Log . Determine el conjunto D de todos los números comple-
z+i
jos z donde f (z) es analı́tica.
CAPÍTULO 2. FUNCIONES DE VARIABLE COMPLEJA 75

2.20. Sea f (z) = Log (z 2 + 1). Determine el conjunto D de todos los números complejos
z donde f (z) es analı́tica.

2.21. Determine la rama principal y su derivada, para cada una de las siguientes fun-
ciones multivaluadas:
√ 2
a) f (z) = ez + 1 d) f (z) = z log z ≡ e(log z)
b) f (z) = cos(log z) e) f (z) = icos z ≡ ecos z log i
c) f (z) = log(ez + 1) f) f (z) = z sen z ≡ esen z log z

2.22. Sea f (z) una función de variable compleja definida como


z z
f (z) = ie ≡ ee log i
,

donde se utiliza un rama tal que se satisface f ′ (iπ) = 3π/2. Determine f ′ (z) y
f ′ (πi/2).

2.23. Demostrar que la transformación w = iz + i mapea el semiplano x > 0 sobre el


semiplano v > 1.

2.24. Demostrar que si c1 < 0, la imagen del semiplano x < c1 bajo la transformación
w = 1/z es el interior de un cı́rculo. ¿Cuál es la imagen cuando c1 = 0?

2.25. Demostrar que la imagen del semiplano y > c2 bajo la transformación w = 1/z es
el interior de un cı́rculo, siempre y cuando c2 > 0. Encontrar la imagen cuando
c2 < 0; también cuando c2 = 0.

2.26. Sean α un número real y z0 un número complejo tal que |z0 | ≤ 1. Demostrar que
la transformación bilineal  
iα z − z0
w=e ,
zz 0 − 1
mapea el disco |z| ≤ 1 sobre disco |w| ≤ 1.

2.27. Considere el conjunto D de números complejos que se muestra en la siguiente


figura.

y
1

-1 1 x

-1 D

z+1+i
Determine el conjunto transformado de D bajo el mapeo w = .
z+i
76 2.10. PROBLEMAS PROPUESTOS

2.28. Sea S el conjunto de números complejos que se muestra en la siguiente figura.

y
1 S

x
−1 1

Determine el transformado de S bajo cada uno de los siguientes mapeos:


1 i−z
a) w = . c) w = .
z i+z
1−z z+2
b) w = . d) w = .
i+z z − 2i
3
Series de Potencias y Singularidades
Aisladas
En este capı́tulo se estudian los conceptos básicos de las series de números complejos,
particularmente, la representación en serie de potencias de las funciones de variable com-
pleja, hecho éste de gran importancia para la resolución de problemas en Ingenierı́a, por
ejemplo, ecuaciones diferenciales. Inicialmente se dan los conceptos de sucesión y serie de
números complejos; después se presenta un resultado fundamental de la convergencia de
las series de potencias y se describen los desarrollos de Taylor y de Laurent. Finalmente,
se caracterizan los puntos singulares aislados de una función de variable compleja.

3.1 Serie de Números Complejos

Definición 3.1 (Sucesión de números complejos). El conjunto de números complejos


{z0 , z1 , z3 , . . .}, se denomina sucesión de números complejos y se denota por {zn }∞
n=0 .

Definición 3.2 (Sucesión convergente). La sucesión de números complejos {zn }∞ n=0


tiene lı́mite o converge a un número complejo z, si para todo ε > 0 existe un número
entero N > 0 tal que
|zn − z| < ε siempre que n ≥ N .
Cuando la sucesión {zn }∞
n=0 converge a z, se escribe

lı́m zn = z.
n→∞

Observación 3.1. Para cada sucesión de números complejos {zn }∞


n=0 , existen sucesiones de
∞ ∞
números reales {xn }n=0 y {yn }n=0 tales que

zn = xn + i yn , para n = 0, 1, 2, . . .

Ejemplo 3.1. Sea {zn }∞


n=0 la sucesión de números complejos definida por

1
zn = .
(1 + i)n

Entonces, las sucesiones de números reales {xn }∞ ∞


n=0 y {yn }n=0 tales que zn = xn + i yn
están definidas como

xn = 2−n/2 cos(nπ/4) y yn = −2−n/2 sen (nπ/4).

77
78 3.1. SERIE DE NÚMEROS COMPLEJOS

El siguiente teorema permite estudiar la convergencia de las sucesiones de números


complejos a través de la convergencia de sucesiones de números reales. La demostración
de este teorema se establece usando la definición de convergencia de sucesiones.

Teorema 3.1. Sean zn = xn + i yn (n = 0, 1, 2, . . .) con xn , yn ∈ R, y z = x + i y con


x, y ∈ R. Entonces,
lı́m zn = z
n→∞

si, y sólo si
lı́m xn = x y lı́m yn = y.
n→∞ n→∞

En el siguiente ejemplo se emplea el Teorema 3.1 para calcular el lı́mite de sucesión de


números complejos.
Ejemplo 3.2. Determinar si la sucesión
 
1 (−1)n
zn = + i 1 + (n = 1, 2, . . .)
n n
converge y halle el lı́mite si es el caso.
Solución. Se tiene que zn = xn + i yn , donde
1 (−1)n
xn = y yn = 1 + .
n n
Como lı́m xn = 0, y lı́m yn = 1, entonces por el Teorema 3.1 la sucesión {zn }∞
n=0 co-
n→∞ n→∞
nverge y su lı́mite es
 
1 (−1)n
lı́m zn = lı́m xn + i lı́m yn = lı́m + i lı́m 1 + = i.
n→∞ n→∞ n→∞ n→∞ n n→∞ n


Definición 3.3 (Serie de Números Complejos). La suma de los términos de una sucesión
de números complejos {zn }∞ n=0 , se denomina serie de números complejos, esto es


X
zn = z0 + z1 + z2 + · · ·
n=0

Un ejercicio in-
teresantes es de-
finir las sumas P∞
parciales de la Definición 3.4 (Serie Convergente). Se dice que una serie n=0 zn converge a un
serie
P∞ armónica número complejo S, si la sucesión de sumas parciales,
n
n ρ , con
0 < ρ < 1, y de- N
X
mostrar que esta
serı́a converge a
SN = zn ,
1/(1 − ρ) n=0

converge a S. En este caso se escribe



X
zn = lı́m SN = S.
N →∞
n=0
CAPÍTULO 3. SERIES DE POTENCIAS Y SINGULARIDADES AISLADAS 79

Un hecho muy importante que relaciona las series de números complejos


P con las series
de números reales es el siguiente: toda serie de números complejos ∞ n=0 zn se puede
escribir como:
X∞ X∞ ∞
X
zn = xn + i yn ,
n=0 n=0 n=0
P∞ P∞
donde n=0 xn y n=0 yn son series P de números reales, además, estas series se denominan
parte real y parte imaginaria de ∞ n=0 zn , respectivamente.
El hecho descrito arriba permite estudiar la convergencia de las series de números
complejos a través de la convergencia de series de números reales. En el siguiente teorema
se establece que una serie de números complejos converge si, y sólo si las series de sus
partes real e imaginaria convergen.

Teorema 3.2. Sean zn = xn + i yn (n = 0, 1, 2, . . .) con xn , yn ∈ R, y S = X + i Y con


X, Y ∈ R. Entonces,
X∞
zn = S
n=0

si, y sólo si

X ∞
X
xn = X y yn = Y.
n=0 n=0

Demostración. Sean {XN }∞ ∞


n=0 y {XN }n=0 las sucesiones de sumas parciales definidas res-
pectivamente por
XN N
X
XN = xn y Y N = yn .
n=0 n=0

Ası́,
N
X N
X N
X
SN = zn = xn + i yn = XN + i YN .
n=0 n=0 n=0

Como S = X + i Y = lı́mN →∞ SN = lı́mN →∞ XN + i lı́mN →∞ YN si, y sólo si

lı́m XN = X y lı́m YN = Y,
N →∞ N →∞
P
entonces,
P∞podemos concluir que la serie ∞
P∞ n=0 zn converge a S = X + i Y si, y sólo si las
series n=0 xn y n=0 yn convergen respectivamente a X e Y .

3.1.1 Serie de Potencias


Definición 3.5 (Serie de potencias). Dada una sucesión {an }∞
n=0 de números complejos
y z0 ∈ C, a la serie
X∞
an (z − z0 )n (3.1)
n=0

se le llama serie de potencias, donde z ∈ C. Los números an se denominan coeficientes


de la serie y z0 se denomina centro de la serie.

El siguiente teorema da una idea completa del campo de convergencia de las series de
potencias. La demostración de este teorema se puede apreciar en [8].
80 3.1. SERIE DE NÚMEROS COMPLEJOS

Teorema 3.3 (Teoremap de Cauchy-Hadamard). Consideremos la serie de potencias (3.1).


n
Sea α = lı́mn→∞ |an |. Entonces: si α = ∞, la serie es convergente en el único punto
z = z0 ; si 0 < α < ∞, la serie es absolutamente convergente en el cı́rculo |z − z0 | < 1/α y
es divergente en el exterior de este cı́rculo; y si α = 0, la serie es absolutamente convergente
en todo el plano complejo.
El Teorema 3.3
es conocido
como crite-
De este modo, cuando 0 < α < ∞, existe un cı́rculo con centro en el punto z = z0 ,
rio de la raı́z; en el interior del cual la serie (3.1) es absolutamente convergente y en el exterior del cual
cuando α se de- la serie es divergente. Éste se llama cı́rculo de convergencia de la serie de potencias y su
fine por α =
lı́m
|an+1 |
,
radio R = 1/α, radio de convergencia de la misma. Los casos α = ∞ y α = 0 se pueden
n→∞ |an |
este teorema se considerar como casos lı́mites. En el primero de ellos, el cı́rculo de convergencia se reduce
conoce como cri- a un punto z0 y su radio R es igual a cero. En el segundo, el cı́rculo de convergencia se
terio del cociente
extiende a todo el plano, de modo que se puede considerar que su radio es igual a ∞.
Llamando en los tres casos al número R radio de convergencia de la serie de potencias, el
contenido de la fórmula de Cauchy-Hadamard puede expresarse por la fórmula
1
R= .
α
Esta última se llama fórmula de Cauchy-Hadamard. Para las aplicaciones de la fórmula de
Cauchy-Hadamard, en muchos casos suele ser útil la relación siguiente:
r
n n! 1
lı́m n
= . (3.2)
n→∞ n e
La demostración de esta relación se deja como ejercicio para el lector.

Ejemplo 3.3. Hallar el cı́rculo y el radio de convergencia de la serie



X nn
zn.
n!
n=1

Solución. Se tiene que


nn
an = y z0 = 0.
n!
Ahora, utilizando la ecuación (3.2) obtenemos
r
nn
α = lı́m n = e.
n→∞ n!
Por lo tanto, el radio de convergencia de la serie es
1
R= ,
e
y el cı́rculo de convergencia de la misma es
1
|z| < .
e

CAPÍTULO 3. SERIES DE POTENCIAS Y SINGULARIDADES AISLADAS 81

En muchos casos resulta conveniente determinar el radio de convergencia de una serie


de potencias mediante el criterio del cociente, es decir, tomando
an+1
α = lı́m .
n→∞ an
Ası́, para la serie del Ejemplo 3.3 se tiene
(n+1)n+1  n
an+1 (n+1)! 1
α = lı́m = lı́m nn = lı́m 1+ = e,
n→∞ an n→∞ n→∞ n
n!

por consiguiente, el radio de convergencia de la serie es R = 1/e.


Ejemplo 3.4. Determine el cı́rculo de convergencia de la serie de potencias

X
zn.
n=0

También determine a qué converge esta serie.


Solución. Como an = 1, para n ≥ 0, y z0 = 0, entonces el radio de convergencia de la serie
es R = 1 y su cı́rculo de convergencia es |z| < 1. Ahora, para determinar a qué converge
la serie dada, consideremos la sucesión de sumas parciales
N
X
SN (z) = zn
n=0
= 1 + z + z2 + · · · + zN
1 − z N +1
= , z 6= 1.
1−z
Por lo tanto, utilizando la ecuación anterior podemos escribir

X
z n = lı́m SN (z)
N →∞
n=0
1 − z N +1
= lı́m
N →∞ 1−z
1
= , |z| < 1.
1−z


3.1.2 Serie de Taylor


En esta parte se estudia una propiedad muy importante de las funciones analı́ticas, a
saber: toda función analı́tica se puede expresar como una serie de potencias que converge a
dicha función en algún dominio.
En el siguiente teorema se establece que si una función f (z) es analı́tica en un cı́rculo
centrado en un punto z0 ∈ C, entonces f (z) se puede representar como una serie de
potencias en dicho cı́rculo. Para la descripción formal del siguiente teorema, es necesario
suponer que f (z) es infinitamente diferenciable, lo cual, como veremos más adelante en el
Capı́tulo 4, es una propiedad que posee toda función analı́tica.
82 3.1. SERIE DE NÚMEROS COMPLEJOS

Teorema 3.4 (Teorema de Taylor). Si f (z) es una función analı́tica en todo punto del
disco |z − z0 | < r0 , entonces f (z) se expresa como

X f (n) (z0 )
f (z) = (z − z0 )n , (3.3)
n=0
n!

para |z − z0 | < r0 .

Observación 3.2.
P f (n) (z0 )
• El Teorema de Taylor garantiza que la serie ∞ n=0 n! (z − z0 )n converge a f (z),
para todo z tal que |z − z0 | < r0 . El nombre de Taylor es en honor al matemático
británico Brook Taylor (1685-1731).

• La serie de potencias (3.3) se denomina desarrollo en serie de Taylor o, simplemente,


desarrollo de Taylor de f (z) centrado en el punto z0 .

• Si z0 = 0, entonces el desarrollo de Taylor adquiere la forma



X f (n) (0)
f (z) = zn
n!
n=0

y se denomina desarrollo de MacLaurin de f (z). El nombre de MacLaurin es en honor


al matemático escocés Colin MacLaurin (1698-1746).
El siguiente teorema nos garantiza que el desarrollo de Taylor de una función f (z)
alrededor de z0 , es la única serie de potencias que converge a f (z) en un disco centrado
en z0 .

Teorema 3.5. El desarrollo en serie de Taylor de una función f (z) alrededor de z0 es la


única serie en potencias de (z − z0 ) que converge a f (z) en todo punto de un disco centrado
en z0 .

Demostración. Por el absurdo. Supongamos que f (z) posee dos desarrollos de Taylor dis-
tintos alrededor de z0 válidos en un mismo dominio D ⊂ C, a saber:

X ∞
X
f (z) = an (z − z0 )n y f (z) = bn (z − z0 )n
n=0 n=0

Como estos desarrollos son distintos, entonces se debe satisfacer que an 6= bn , para algún
n ≥ 0. Ahora, también tenemos que, para todo z 6= z0 ,

X
0 = f (z) − f (z) = (an − bn )(z − z0 )n ⇐⇒ a n = bn , para todo n ≥ 0.
n=0

Esto es una contradicción.

Observación 3.3. Sean f (z) y g(z) funciones tales que su desarrollo de Taylor centrado en
z0 está dado respectivamente por

X
f (z) = an (z − z0 )n , para z tal que |z − z0 | < r1 ,
n=0
CAPÍTULO 3. SERIES DE POTENCIAS Y SINGULARIDADES AISLADAS 83

y

X
g(z) = bn (z − z0 )n , para z tal que |z − z0 | < r2 .
n=0

Entonces del desarrollo de Taylor de f (z) + g(z) centrado en z0 es



X
f (z) + g(z) = (an + bn ) (z − z0 )n , para z tal que |z − z0 | < mı́n(r1 , r2 ).
n=0

El siguiente teorema nos permite calcular el radio de convergencia del mayor cı́rculo
para el cual existe la serie de Taylor de una función f (z).

Teorema 3.6. Consideremos el desarrollo en serie de Taylor (3.3) de una función f (z)
alrededor de z0 . Entonces, |z − z0 | < r es el mayor cı́rculo dentro del cual la serie converge
a f (z) para cada z en éste cı́rculo, donde r es la distancia entre z0 y el punto singular de
f (z) más cercano.

Demostración. Como f (z) es analı́tica en z0 , entonces existe r0 > 0 tal que para todo z en
el disco |z − z0 | < r0 se tiene que f (z) es derivable. Afirmamos que r0 = r, donde r es la
distancia entre z0 y el punto singular de f (z) más cercano, ya que de lo contrario, si r0 > r,
esto serı́a una contradicción al Teorema de Taylor.

Obsérvese que este teorema no afirma que la serie de Taylor no converja fuera de
|z − z0 | = r. Sólo asevera que éste es el mayor cı́rculo en todo punto del cual la serie
converge a f (z). El cı́rculo en todo punto del cual la serie de Taylor (3.3) converge a
f (z) y el cı́rculo en todo punto del cual la serie converge no son necesariamente iguales. El
segundo de ellos puede tener un radio mayor. No obstante, se puede mostrar que cuando el
punto singular más cercano a z0 es tal que |f (z)| se hace infinito, los dos cı́rculos coinciden.
Éste es el caso en la mayor parte de las funciones que consideraremos.

Ejemplo 3.5. Comprobar cada uno de los siguientes desarrollos de Taylor.



X 1 n
a) ez = z , |z| < ∞.
n=0
n!

X (−1)n 2n+1
b) sen z = z , |z| < ∞.
(2n + 1)!
n=0

X (−1)n
c) cos z = z 2n , |z| < ∞.
(2n)!
n=0

X 1
d) senh z = z 2n+1 , |z| < ∞.
n=0
(2n + 1)!

X 1
e) cosh z = z 2n , |z| < ∞.
(2n)!
n=0

1 X
f) = zn, |z| < 1.
1−z
n=0
84 3.1. SERIE DE NÚMEROS COMPLEJOS


1 X
g) = (−1)n z n , |z| < 1.
1+z
n=0

1 X
h) = (−1)n (z − 1)n , |z − 1| < 1.
z n=0

X1h ∞ i
1
i) = (−1)n + 2−(n+1) (z − 1)n , |z − 1| < 1.
z(3 − z) 3
n=0

Solución.
a) Se tiene que f (z) = ez es una función entera y f (n) (z) = ez . Ası́, por los Teoremas 3.4 y
3.6, el desarrollo de MacLaurin de ez es

z
X 1 n
e = z , |z| < ∞.
n!
n=0

eiz − e−iz
b) Se tiene que f (z) = sen z es una función entera, además, sen z = . De esta
2i
forma, utilizando el desarrollo de Macalurin de ez podemos escribir:
∞ ∞
X 1 n
X 1
(iz) − (−iz)n ∞
eiz − e−iz n=0
n! n=0
n! 1 X 1 n
sen z = = = (i − (−i)n )z n .
2i 2i 2i n!
n=0

Ahora, utilizando la expresión anterior y el hecho que


(
0, si n es par,
(in − (−i)n ) = m
2i(−1) , si n = 2m + 1 es impar,
obtenemos que el desarrollo de MacLaurin de sen z es

X (−1)n 2n+1
sen z = z , |z| < ∞.
(2n + 1)!
n=0

La verificación de c), d) y e), se deje como ejercicio pare el lector.


1
f) Se tiene que f (z) = 1−z es analı́tica en todo z ∈ C tal que z 6= 1, entonces por el
1
Teorema 3.6 el desarrollo de MacLaurin de f (z) = 1−z es válido para todo |z| < 1. Ahora,
1
por el Ejemplo 3.4, se tiene que el desarrollo de MacLaurin de f (z) = 1−z es

1 X
= zn , |z| < 1.
1−z
n=0
1
g) Como f (z) = es analı́tica en todo z ∈ C tal que z 6= 1, entonces por el Teorema 3.6
1+z
1
el desarrollo de Maclaurin de f (z) = 1+z es válido para todo |z| < 1. A continuación
1
hallamos el desarrollo de MacLaurin de f (z) = 1+z empleando el desarrollo de Taylor de
1
1−z dado en f). Se tiene que
∞ ∞
1 1 X X
= = (−z)n = (−1)n z n , |z| < 1.
1+z 1 − (−z)
n=0
| {z } n=0
|−z|<1
CAPÍTULO 3. SERIES DE POTENCIAS Y SINGULARIDADES AISLADAS 85

h) Aquı́ utilizamos el desarrollo de Taylor dado en g) para hallar el desarrollo de Taylor de


f (z) = 1/z centrado en z0 = 1. Se tiene que

1 1 X
= = (−1)n (z − 1)n , |z − 1| < 1.
z 1 + (z − 1)
n=0
1
i) Para hallar el desarrollo de Taylor de f (z) = z(3−z) centrado en z0 = 1, utilizamos
fracciones simples, desarrollos de Taylor conocidos y operaciones algebraicas simples. Se
tiene que
1 1/3 −1/3 1/3 −1/3
= + = +
z(3 − z) z z−3 1 + (z − 1) (z − 1) − 2
1 1 1 1 1
= · + · ·
3 1 + (z − 1) 3 2 1 − (z−1)
2
∞ ∞
1X 1 1 X (z − 1)n
= (−1)n (z − 1)n + ·
3 3 2 2n
n=0 n=0
∞ ∞
X 1 n n
X 1 −(n+1)
= (−1) (z − 1) + 2 (z − 1)n
n=0
3 3
| {z } |n=0 {z }
|z−1|<1 |z−1|<2

X 1h i
= (−1)n + 2−(n+1) (z − 1)n , |z − 1| < 1.
n=0
3

3.1.3 Serie de Laurent


El desarrollo en serie de Laurent o, simplemente, desarrollo de Laurent de una función
f (z) centrado en el punto z0 es de la forma

X
f (z) = cn (z − z0 )n ,
n=−∞

donde la serie converge a f (z) en cierto dominio o región y cn ∈ C. El nombre de Laurent


es en honor al matemático y oficial militar francés Pierre Alphonse Laurent (1813-1854).
Ası́ pues, un desarrollo de Laurent, a diferencia del desarrollo de Taylor, puede contener
uno o más términos con (z − z0 ) elevado a una potencia negativa. También puede contener
potencias positivas de (z − z0 ).
Normalmente los desarrollos de Laurent se obtienen a partir de los desarrollos de Tay-
lor. Por ejemplo, para encontrar el desarrollo de Laurent de f (z) = e1/z centrado en z0 = 0,
se considera el desarrollo de MacLaurin de es ,

s
X 1 n
e = s , |s| < ∞
n!
n=0

después, se hace s = 1/z en la ecuación anterior para obtener



1/z
X 1 −n
e = z , |z| > 0,
n!
n=0
86 3.1. SERIE DE NÚMEROS COMPLEJOS

que es, efectivamente, el desarrollo de Laurent de e1/z centrado en z0 = 0, válido en todo


el plano complejo excepto en z = 0.
¿Qué clase de funciones pueden representarse por medio de series de Laurent y en qué
región del plano complejo será válida dicha representación? La respuesta se encuentra en
el siguiente teorema. Antes de dar el teorema, definiremos anillo o dominio anular.

Definición 3.6 (Anillo o dominio anular). Sean r1 > 0 y r2 > 0 tales que r1 < r2 . Se
denomina anillo o dominio anular centrado en z0 ∈ C, al conjunto de números complejos
z tales que r1 < |z − z0 | < r2 .

Teorema 3.7 (Teorema de Laurent). Si f (z) es una función analı́tica en el anillo r1 <
|z − z0 | < r2 , centrado en z0 , entonces f (z) se puede expresar como

X ∞
X
f (z) = an (z − z0 )n + bn (z − z0 )−n ,
n=0 n=1

para todo z tal que r1 < |z − z0 | < r2 .

Observación 3.4.
P P∞
• El Teorema de Laurent garantiza que la serie ∞ n
n=0 an (z − z0 ) + n=1 bn (z − z0 )
−n

converge a f (z) para todo z tal que r1 < |z − z0 | < r2 .

• Si f (z) es analı́tica en todos los puntos de la región |z − z0 | < r2 , entonces el desa-


rrollo de Laurent de f (z) centrado en z0 , se convierte en el desarrollo de Taylor de
f (z) centrado en z0 .

• Si f (z) es analı́tica en todos los puntos de la región |z − z0 | < r2 excepto en en el


punto z0 , entonces el desarrollo de Laurent es válido en toda la región 0 < |z − z0 | <
r2 .

• Los coeficientes de la serie de Laurent se pueden definir como una integral que invo-
lucra a la función f (z). Este último hecho permite calcular integrales utilizando los
resultados de series de potencias, lo cual se estudiará en el Capı́tulo 4.

Ejemplo 3.6. Determine el desarrollo de Laurent en potencias de z de la función

1
f (z) = .
1+z

Solución. Observamos que el único punto singular de f (z) es z = −1, entonces existen dos
regiones donde f (z) posee desarrollos de Laurent centrados en z0 = 0, a saber: a) |z| < 1,
y b) |z| > 1.
a) Como f (z) es analı́tica en todo punto del disco |z| < 1, el desarrollo de Laurent de f (z)
centrado en z0 = 0 válido en el dominio |z| < 1, coincide con su desarrollo de Maclaurin.
En otras palabras, el desarrollo de Laurent de f (z) centrado en z0 = 0 válido en el dominio
|z| < 1, es:

X
f (z) = (−1)n z n , |z| < 1.
n=0
CAPÍTULO 3. SERIES DE POTENCIAS Y SINGULARIDADES AISLADAS 87

b) Ahora bien, el desarrollo de Laurent de f (z) centrado en z0 = 0 válido en el dominio


|z| > 1, está dado por:
∞ ∞
1 1 1 X X
= · −1 = z −1 (−1)n z −n = (−1)n z −(n+1) , |z| > 1.
1+z z z +1
| n=0 {z } n=0
|z −1 |<1

Ejercicio 3.1. Compruebe que todos los desarrollos de Laurent centrados en z0 = i de la


función
1
f (z) =
(z − 1)(z − 2)
están dados por:
∞ h
X i √
• f (z) = (1 − i)−(n+1) − (2 − i)−(n+1) (z − i)n , |z − i| < 2,
n=0

∞ ∞
X
n −(n+1)
X √ √
• f (z) = − (1 − i) (z − i) − (2 − i)−(n+1) (z − i)n , 2 < |z − i| < 5,
n=0 n=0


X √
• f (z) = [(2 − i)n − (1 − i)n ] (z − i)−(n+1) , |z − i| > 5.
n=0

3.1.4 Propiedades Adicionales de las Series


Diferenciación término a término

Si una función f (z) está representada con una serie de potencias en una región anular
R, la serie que se obtiene por diferenciación término a término converge a f ′ (z) dentro de
R. Este procedimiento puede repetirse un número indefinido de veces.
Los siguientes ejemplos utilizan la propiedad de diferenciación término a término para
encontrar desarrollos de Taylor.

Ejemplo 3.7. Usando el desarrollo de Taylor de 1/z centrado en z0 = 1, obtenga el desa-


rrollo de 1/z 2 centrado en el mismo punto.
Solución. Primero determinemos el desarrollo de Taylor de 1/z centrado en z0 = 1, lu-
ego diferenciando término a término éste desarrollo obtendremos el desarrollo de 1/z 2
centrado en z0 = 1. Se tiene que

1 1 X
= = (−1)n (z − 1)n , |z| < 1,
z 1 + (z − 1) n=0

ahora, como
d 1 1
= − 2,
dz z z
88 3.2. SINGULARIDADES AISLADAS

entonces utilizando el desarrollo de Taylor de 1/z centrado en z0 = 1, podemos escribir:


∞ ∞
1 d 1 X d n n
X
= − = − [(−1) (z − 1) ] = (−1)n+1 n (z − 1)n−1 , |z| < 1.
z2 dz z dz
| n=0 {z } n=0
|z|<1

Ejemplo 3.8. Calcular el desarrollo de MacLaurin de


z
f (z) = .
(1 − z)2

Solución. Se tiene que el desarrollo de MacLaurin de 1/(1 − z) está dado por:



1 X
= zn, |z| < 1.
(1 − z)
n=0

Como
d 1 1
= ,
dz (1 − z) (1 − z)2
entonces utilizando el desarrollo de MacLaurin de 1/(1 − z), podemos escribir:
∞ ∞
z d 1 X d n X n
=z =z z = nz , |z| < 1.
(1 − z)2 dz (1 − z) dz
n=0 n=0
| {z }
|z|<1

3.2 Singularidades Aisladas

Definición 3.7 (Punto singular aislado). Se dice que z0 ∈ C es un punto singular aislado
de una función f (z), si z0 es un punto singular de f (z) y existe una vecindad de z0 en
todo punto de la cual f (z) es analı́tica excepto en z0 .

Ejemplo 3.9. Cada punto z0 indicado es un punto singular aislado de la función dada.
1
• f (z) = , z0 = 0
z
z+1
• f (z) = , z0 = 1, z1 = i
(z − 1)(z − i)

• f (z) = cot z, zn = nπ, n = 0, ±1, ±2, . . .


CAPÍTULO 3. SERIES DE POTENCIAS Y SINGULARIDADES AISLADAS 89

Ejemplo 3.10. El punto z0 = 0 no es un punto singular aislado de la función


1
f (z) = .
sen (1/z)


¿z0 = 0 es un
punto de acu-
mulación de los Ejercicio 3.2. Verificar lo afirmado en los Ejemplos 3.9 y 3.10.
puntos singula-
res de f (z) =
1/sen (1/z)? Definición 3.8 (Parte principal). Sea z0 un punto singular aislado de f (z). Sea

X ∞
X
f (z) = an (z − z0 )n + bn (z − z0 )−n , 0 < |z − z0 | < r0 , (3.4)
n=0 n=1

el desarrollo de Laurent de f (z) centrado en z0 válido en el anillo 0 < |z − z0 | < r0 . Se


denomina parte principal de f (z) en z0 , a la parte
P∞ del desarrollo de Laurent (3.4) que
−n
posee potencias negativas de (z − z0 ), esto es, n=1 bn (z − z0 ) .

Ejemplo 3.11. Determine la parte principal de la función


1
f (z) =
z(z − 1)
en cada uno de sus puntos singulares.
Solución. Los puntos singulares aislados de f (z) son z1 = 0 y z2 = 1. Como
1 −1 1
= + ,
z(z − 1) z (z − 1)
el desarrollo de Laurent de f (z) centrado en z1 = 0 válido en 0 < |z| < 1, está dado por:

1 1 1 X
=− − = (−1)z n + (−1)z −1 , 0 < |z| < 1,
z(z − 1) (1 − z) z n=0

por lo tanto, la parte principal de f (z) en z1 = 0 es: (−1)z −1 .


Ahora, se tiene que
1 1 1 1
= − + =− + (z − 1)−1
z(z − 1) z (z − 1) 1 + (z − 1)
X∞
= (−1)n+1 (z − 1)n + (z − 1)−1 ,
| {z }
|n=0 {z } |z−1|>0
|z−1|<1

ası́, el desarrollo de Laurent de f (z) centrado en z2 = 1 válido en 0 < |z − 1| < 1, está


dado por:

1 X
= (−1)n+1 (z − 1)n + (z − 1)−1 , 0 < |z − 1| < 1;
z(z − 1) n=0

por lo tanto, la parte principal de f (z) en z2 = 1 es (z − 1)−1 . 


90 3.2. SINGULARIDADES AISLADAS

Los puntos singulares aislados se pueden caracterizar según la forma que adquiere
la parte principal de la función en tales puntos. Se distinguen tres tipos de singularidades
aisladas: polo de orden m, punto singular esencial y punto singular removible. A continua-
ción se describen cada una de ellas.

3.2.1 Polo de Orden m


Definición 3.9. Sea z0 un punto singular aislado de f (z). Se dice que z0 es un polo de
orden m de f (z), si la parte principal de f (z) en z0 tiene un número finito de términos,
esto es, el desarrollo de Laurent de f (z) centrado en z0 , válido en el anillo 0 < |z − z0 | <
r0 , tiene la forma

X b1 b2 bm
f (z) = an (z − z0 )n + + + ··· + ,
(z − z0 ) (z − z0 )2 (z − z0 )m
n=0

donde bm 6= 0 y bm+1 = bm+2 = · · · = 0. Los polos de orden m = 1 se llaman polos


simples.

La definición anterior nos indica que para determinar si z0 es un polo de f (z), se


debe observar el desarrollo de Laurent de f (z) centrado en z0 ; pero, en general, este
procedimiento no es práctico. Existen otros procedimientos más adecuados para verificar
si un punto es o no un polo. El siguiente teorema nos da un procedimiento para determinar
si un punto es o no un polo de una función, sin construir su serie de Laurent.

Teorema 3.8. Si f (z) tiene un polo en z0 , entonces lı́mz→z0 |f (z)| = ∞.

Demostración. Como z0 es un polo de orden m de f (z), el desarrollo de Laurent de f (z)


centrado en z0 , válido en el anillo 0 < |z − z0 | < r0 , tiene la forma

X b1 b2 bm
f (z) = an (z − z0 )n + + 2
+ ··· + ,
(z − z0 ) (z − z0 ) (z − z0 )m
n=0

donde bm 6= 0. Multiplicando en ambos lados de la ecuación anterior por (z−z0 )m tenemos



X
(z − z0 )m f (z) = an (z − z0 )n+m + b1 (z − z0 )m−1 + b2 (z − z0 )m−2 + · · · + bm ,
n=0

de lo cual se deduce que

lı́m (z − z0 )m f (z) = bm 6= 0, ∞. (3.5)


z→z0

Como lı́mz→z0 |(z − z0 )m | = 0, entonces por (3.5) se tiene que lı́mz→z0 |f (z)| = ∞.

El teorema anterior no solo nos permite identificar si un punto es un polo, sino también
el orden del mismo. Basándose en este teorema y, particularmente, en la ecuación (3.5),
las siguientes reglas nos permiten identificar si un punto singular aislado z0 es un polo y,
además, calcular el orden del mismo.

Regla I Si existe el lı́mz→z0 (z − z0 )m f (z) y si dicho lı́mite no es cero ni infinito, entonces


z0 es un polo de orden m de f (z).
CAPÍTULO 3. SERIES DE POTENCIAS Y SINGULARIDADES AISLADAS 91

Regla II Si z0 es un polo es un polo de orden m de f (z), entonces


(
n 0, si n > m,
lı́m (z − z0 ) f (z) =
z→z0 ∞, si n < m.

1
Ejemplo 3.12. Sea f (z) = . Probar que 0 y 1 son polos simples de f (z).
z(z − 1)
Solución. Utilicemos el Teorema 3.8 y las Reglas I y II, para determinar que los puntos 0 y
1 son polos simples de f (z). Se tiene que

lı́m |f (z)| = ∞ y lı́m |f (z)| = ∞,


z→0 z→1

luego, los puntos 0 y 1 son polos de f (z). Además,



z n−1 0,
 si n > 1,
n
lı́m z f (z) = lı́m = −1, si n = 1,
z→0 z→0 (z − 1) 

∞, si n < 1,

y 
(z − 1)n−1 0,
 si n > 1,
lı́m (z − 1)n f (z) = lı́m = 1, si n = 1,
z→1 z→1 z 

∞, si n < 1.
Por lo tanto, los puntos 0 y 1 son polos simples de f (z). 

Es común encontrarse con problemas en los que se quiere determinar el orden de los
polos de una función de la forma f (z) = p(z)/q(z), por ello les dedicaremos una atención
especial. El siguiente teorema nos permite identificar si un punto es un polo, y además su
orden, para una función f (z) = p(z)/q(z).

Teorema 3.9. Sea z0 ∈ C. Sea f (z) una función tal que se puede escribir como

p(z)
f (z) = ,
q(z)

donde p(z) y q(z) son analı́ticas en z0 y p(z0 ) 6= 0. Entonces, z0 es un polo de orden m de


f (z) si, y sólo si
q(z0 ) = q ′ (z0 ) = · · · = q (m−1) (z0 ) = 0
y
q (m) (z0 ) 6= 0.

Demostración. (⇒) Supongamos que z0 es un polo de orden m de f (z) y demostremos que


q(z0 ) = q ′ (z0 ) = · · · = q (m−1) (z0 ) = 0 y q (m) (z0 ) 6= 0. Como z0 es un polo de orden m de
f (z), el desarrollo de Laurent de f (z) centrado en z0 , válido en el anillo 0 < |z − z0 | < r0 ,
tiene la forma

p(z) X b1 b2 bm
f (z) = = an (z − z0 )n + + 2
+ ··· + ,
q(z) (z − z0 ) (z − z0 ) (z − z0 )m
n=0
92 3.2. SINGULARIDADES AISLADAS

donde bm 6= 0. Como q(z) es analı́tica en z0 , entonces posee desarrollo de Taylor centrado


en z0 de la forma

X q n (z0 )
q(z) = (z − z0 )n .
n!
n=0

Ası́, obtenemos que



! ∞
!
X q n (z0 ) X b 1 b m
p(z) = (z − z0 )n an (z − z0 )n + + ··· +
n! (z − z0 ) (z − z0 )m
n=0 n=0

! ∞ ! m ∞
X q n (z0 ) X X X q n (z0 )
n n
= (z − z0 ) an (z − z0 ) + bk (z − z0 )n−k .
n=0
n! n=0 n=0
n!
k=1

Como p(z) es analı́tica en z0 , entonces la expresión anterior es el desarrollo de Taylor de


p(z) centrado z0 , por lo que no puede tener potencias negativas de (z − z0 ). Para que esto
sea cierto, se debe cumplir que q(z0 ) = q ′ (z0 ) = · · · = q (m−1) (z0 ) = 0. Además, haciendo
z = z0 en el desarrollo de Taylor de p(z), podemos escribir:

q (m) (z0 )
0 6= p(z0 ) = bm ,
m!
de donde se deduce que q (m) (z0 ) 6= 0.
(⇐) Supongamos que q(z0 ) = q ′ (z0 ) = · · · = q (m−1) (z0 ) = 0 y q (m) (z0 ) 6= 0, y demos-
tremos que z0 es un polo de orden m de f (z). Como p(z) y q(z) son analı́ticas en z0 con
p(z0 ) 6= 0 y q(z0 ) = 0, entonces z0 es un punto singular aislado de f (z). Ahora utilizando
el desarrollo de Taylor de q(z) centrado en z0 se tiene que

q(z) q (m) (z0 ) X q n (z0 )
= + (z − z0 )n−m ,
(z − z0 )m m! n=m+1
n!

por tanto,
q(z) q (m) (z0 )
lı́m m
= 6= 0, ∞,
z→z0 (z − z0 ) m!
lo cual implica que
  
m (z − z0 )m
lı́m (z − z0 ) f (z) = lı́m p(z) lı́m 6= 0, ∞.
z→z0 z→z0 z→z0 q(z)

Según la Regla I, lo anterior indica que z0 es un polo de orden m de f (z).

Ejemplo 3.13. Verificar que todos los puntos singulares aislados de la función
ez
f (z) =
sen z
son polos simples.
Solución. Se tiene que f (z) = p(z)/q(z), donde p(z) = ez y q(z) = sen z. Ahora, los
puntos singulares aislados de f (z) son: zn = nπ, para n = 0, ±1, ±2, . . .. Como p(zn ) 6= 0,
q(zn ) = 0 y q ′ (zn ) = cos(nπ) 6= 0, para todo n, entonces por el Teorema 3.9 todos los
puntos singulares de f (z) son polos simples. 
CAPÍTULO 3. SERIES DE POTENCIAS Y SINGULARIDADES AISLADAS 93

Ejercicio 3.3. Verifique la existencia de los polos y el orden de los mismos para cada una
de las funciones dadas.
1
a) El punto z0 = 0 es un polo de orden m = 2 de la función f (z) = .
z(ez − 1)

(z + 1)
b) Los puntos z0 = 3i y z1 = −3i son polos simples de la función f (z) = .
(z 2 + 9)

z 2 − 2z + 3
c) El punto z0 = 2 es un polo simple de la función f (z) = .
(z − 2)
senh z
d) El punto z0 = 0 es un polo de orden m = 3 de la función f (z) = .
z4

3.2.2 Punto Singular Esencial

Definición 3.10. Sea z0 un punto singular aislado de f (z). Se dice que z0 es un punto
singular esencial de f (z), si la parte principal de f (z) en z0 tiene un número infinito de
términos diferentes de cero.

De la definición de punto singular esencial se deduce que z0 es un punto singular ese-


ncial de f (z) si, y sólo si lı́mz→z0 f (z) no existe (ni finito ni infinito). Observe el siguiente
ejemplo.

Ejemplo 3.14. Verificar que z0 = 0 es un punto singular esencial de f (z) = e1/z .


Solución. El desarrollo de Laurent de e1/z centrado en z0 = 0 es

X 1 −n
e1/z = 1 + z , |z| > 0.
n!
n=1

Se observa que la parte principal de e1/z en z0 = 0 tiene un número infinito de términos


diferentes de cero; por lo tanto, z0 = 0 es un punto singular esencial de e1/z .
Otra manera de ver que z0 = 0 es un punto singular esencial de e1/z , es probar que
el lı́mite, lı́mz→0 e1/z , no existe. Si nos acercamos al origen por la recta y = 0, x > 0,
obtenemos que
lı́m e1/z = lı́m e1/x = ∞.
z→0 x→0

Ahora, si nos acercamos al origen por la recta x = 0, y > 0, tenemos que

e1/z = cos(1/y) − i sen (1/y) = e−i/y ,

es un número complejo de módulo 1 para todo valor de y. Luego,

lı́m e1/z = lı́m e−i/y 6= ∞.


z→0 y→0

Por lo tanto, el lı́mite lı́mz→0 e1/z no existe. En consecuencia, z0 = 0 es un punto singular


esencial de e1/z . 
94 3.3. SERIES DE POTENCIAS Y SINGULARIDADES CON MATLAB

3.2.3 Punto Singular Removible

Definición 3.11. Sea z0 un punto singular aislado de f (z). Se dice que z0 es un punto
singular removible de f (z), si todos los coeficientes de la parte principal de f (z) en z0
son cero.

Si z0 es un punto singular removible de f (z), el desarrollo de Laurent de f (z) centrado


en z0 , válido en el anillo 0 < |z − z0 | < r0 , tiene la forma

X
f (z) = an (z − z0 )n ,
n=0

de donde se deduce que


lı́m f (z) = a0 .
z→z0

Por lo tanto, para determinar si un punto singular aislado z0 es un punto singular removible
de f (z), basta con verificar que el lı́mz→z0 f (z) existe y es finito.

Ejemplo 3.15. Verificar que z0 = 0 es un punto singular removible de la función


sen z
f (z) = .
z
Solución. Se tiene que
sen z L′ H
lı́m = lı́m cos z = 1,
z→0 z z→0
sen z
por tanto, z0 = 0 es un punto singular removible de f (z) = . 
z

3.3 Series de Potencias y Singularidades con M ATLAB


En esta sección también usaremos la caja de herramientas de matemática simbólica
(Symbolic Math Toolbox) de M ATLAB . En particular el comando symsum(expr,n,a,b) que
permite evaluar la suma una la serie, donde expr define los términos de una serie con
respecto de z y n, y n varı́a de a hasta b. Observe el siguiente ejemplo donde se calcula:

X 1
zn = , |z| < 1.
1−z
n=0

>> syms z n
>> symsum( z ˆn , n , 0 , I n f )
ans =
p i e c e w i s e ( [ 1 <= z , I n f ] , [ abs ( z ) < 1 , −1/(z − 1 ) ] )
✂ ✁
En esta caso, la respuesta que da M ATLAB , piecewise([1 <= z, Inf], [abs(z) < 1,
-1/(z - 1)]), indica que la serie no converge para |z| ≥ 1, pero si converge a 1/(1 − z)
cuando |z| < 1. Veamos otros ejemplos. Seguidamente se calcula:

X zn
• = −Log (1 − z), |z| < 1
n
n=1
CAPÍTULO 3. SERIES DE POTENCIAS Y SINGULARIDADES AISLADAS 95


X n n z
• z = 2 , |z| < 2
2n 2 z
−1
n=1 2

X n(z + 2)n 2z + 4
• = , |z + 2| < 2
2n z2
n=0

>> syms z n
>> symsum( z ˆn/n , n , 1 , I n f )
ans =
p i e c e w i s e ( [ 1 <= z , I n f ] , [ abs ( z ) <= 1 & z ˜= 1 , −l o g (1 − z ) ] )
>> symsum( ( n∗ z ˆn )/2ˆn , n , 1 , I n f )
ans =
p i e c e w i s e ( [ abs ( z ) < 2 , z / ( 2 ∗ ( z /2 − 1 ) ˆ 2 ) ] )
>> symsum( ( n ∗( z+2)ˆn )/2ˆn , n , 1 , I n f )
ans =
p i e c e w i s e ( [ abs ( z /2 + 1) < 1 , (2∗( z + 2 ) ) / z ˆ 2 ] )
✂ ✁
Hemos visto que el comando symsum permite calcular a qué y en dónde converge una
serie de potencias. Ahora utilicemos el comando taylor para determinar el desarrollo
de Taylor de una función f (z). Con taylor(f,z,a) se calculan, por defecto, los cinco
primeros términos (f (a) + f ′ (a)(z − a) + f ′′ (a)(z − a)2 + f ′′′ (a)(z − a)3 + f (iv) (a)(z − a)4 )
del desarrollo de Taylor de f (z) al rededor de z0 = a. El desarrollo de Maclaurin de f (z)
se calcula usando taylor(f). Observe el siguiente ejemplo donde se calculan los cinco
primeros términos de los desarrollos de Maclaurin de ez , cos z y 1/(1 − z).

>> syms z
>> t a y l o r ( exp ( z ) )
ans =
z ˆ5/120 + z ˆ4/24 + z ˆ3/6 + z ˆ2/2 + z + 1
>> t a y l o r ( c o s ( z ) )
ans =
z ˆ4/24 − z ˆ2/2 + 1
>> t a y l o r (1/(1− z ) )
ans =
z ˆ5 + z ˆ4 + z ˆ3 + z ˆ2 + z + 1
✂ ✁
La opción taylor(f,z,a,’Order’,m) permite calcular los m primeros términos del de-
sarrollo de Taylor de f (z) al rededor de z0 = a. Observe el siguiente ejemplo donde se
calculan los 8 primeros términos del desarrollo de Taylor de f (z) = (z + 1)/(z − 5)6 al
rededor de z0 = −1.

>> syms z
>> t a y l o r ( ( z +1)/(z −5)ˆ6 ,z , −1 , ’ Order ’ , 8 )
ans =
z /46656 + ( z + 1)ˆ2/46656 + (7∗( z + 1)ˆ3)/559872 +
(7∗( z + 1)ˆ4)/1259712 + (7∗( z + 1)ˆ5)/3359232 +
(7∗( z + 1)ˆ6)/10077696 + (77∗( z + 1)ˆ7)/362797056 + 1/46656
✂ ✁
M ATLAB no tiene comandos que permiten hallar directamente el desarrollo de Laurent.
Para ello se deben utilizar desarrollos de Taylor conocidos. Para funciones particulares
como las funciones racionales propias,
b1 z m + b2 z m−1 + b3 z m−2 + · · · + bm
f (z) = ,
a1 z n + a2 z n−1 + a3 z n−2 + · · · + an
con n > m, hemos diseñado la función LaurentRacional (ver Programa 3.1), que permite
hallar el desarrollo de Laurent de f (z) alrededor de uno de sus puntos singulares z0 ∈ C.
96 3.3. SERIES DE POTENCIAS Y SINGULARIDADES CON MATLAB

Programa 3.1. Función LaurentRacional.m

f u n c t i o n [ d s l ]= L a u r e n t R a c i o n a l ( b , a , z0 ,m)
%L a u r e n t R a c i o n a l H a l l a e l d e s a r r o l l o de L a u r e n t de l a f u n c i ó n
% r a c i o n a l propia
% b ( 1 ) z ˆm + b ( 2 ) z ˆ(m−1) +...+ b (m)
% f ( z ) = −−−−−−−−−−−−−−−−−−−−−−−−−−−−−−−−−−−−
% a ( 1 ) z ˆn + a ( 2 ) z ˆ( n−1) +...+ a ( n )
% a l rededor de uno de s u s pu ntos s i n g u l a r e s z0
syms z ;
d s l = sym ( 0 ) ; d s l 1 = sym ( 0 ) ;
[ r , p , k] = r e sidu e (b , a ) ;
np = l e n g t h ( r ) ;
f i = sym ( z e r o s ( s i z e ( r ) ) ) ;
f i ( 1 ) = sym ( r ( 1 ) ) / ( z−sym ( p ( 1 ) ) ) ;
mr = 1 ;
i f np>1
f o r t =2:np
i f p ( t ) == p ( t −1)
f i ( t ) = sym ( r ( t ) ) / ( z−sym ( p ( t ) ) ) ˆ ( sym (mr+1));
mr = mr+1;
else
f i ( t ) = sym ( r ( t ) ) / ( z−sym ( p ( t ) ) ) ;
mr = 1 ;
end
end
end
f o r t =1:np
i f sym ( p ( t ) ) == z0
dsl = dsl + f i ( t );
else
dsl1 = dsl1 + f i ( t ) ;
end
end
d s l = d s l + t a y l o r ( dsl1 , z , z0 , ’ Order ’ ,m) ;
i f ˜ isempty (k )
d s l = d s l + sym ( k ) ;
end
end

Observe el siguiente ejemplo donde se calcula el desarrollo de Laurent de


z+1 z+1
f (z) = = 3
(z − 1)2 (z + 3) z + z 2 − 5z + 3
alrededor de z0 = −3, usando la función LaurentRacional.

>> b = [1 1 ] ; a = [1 1 −5 3 ] ;
>> d s l = L a u r e n t R a c i o n a l ( b , a , −3 ,5)
dsl =
z /128 − 1/(8∗( z + 3 ) ) + ( z + 3)ˆ2/256 + (3∗( z + 3)ˆ3)/2048 +
( z + 3)ˆ4/2048 + 3/128
✂ ✁

De esta forma, el desarrollo de Laurent de f (z) alrededor de z0 = −3 obtenido previa-


mente, se expresa como:
1 (z + 3) (z + 3)2 3 (z + 3)3 (z + 3)4
f (z) = − + + + + + ···
8 (z + 3) 128 256 2048 2048
CAPÍTULO 3. SERIES DE POTENCIAS Y SINGULARIDADES AISLADAS 97

Por otra parte, el comando limit puede usarse para determinar la naturaleza de una
singularidad aislada, es decir, podemos identificar si es un polo, un punto singular remo-
vible o un punto singular esencial. A continuación verificamos cada una de las siguientes
afirmaciones:

a) El punto z0 = 0 es un polo de orden m = 2 de la función f (z) = 1/(z(ez − 1)).

b) El punto z0 = i es un punto singular esencial de f (z) = e1/(z−i) .

c) El punto z0 = π/2 es un punto singular removible de f (z) = cos(z)/(2z − π).

a) Se tiene

>> syms z
>> f ( z ) = 1/( z ∗( exp ( z ) −1))
f (z) =
1/( z ∗( exp ( z ) − 1 ) )
>> l i m i t ( abs ( f ) , z , 0 )
ans ( z ) =
Inf
✂ ✁

Esto nos indica que z0 = 0 es un polo de f (z) = 1/(z(ez − 1)). Ahora,



>> l i m i t ( z ˆ2∗ f , z , 0 )
ans ( z ) =
1
✂ ✁

indica que z0 = 0 es un polo de orden 2 de f (z) = 1/(z(ez − 1)).


b) Se tiene

>> syms z
>> f ( z ) = exp ( 1 / ( z−i ) )
f (z) =
exp ( 1 / ( z − i ) )
>> l i m i t ( f , z , i )
ans ( z ) =
NaN
✂ ✁

En este caso, el sı́mbolo NaN indica que el lı́mite no existe. Por lo tanto, z0 = i es un punto
singular esencial de f (z) = e1/(z−i) .
c) Se tiene

>> syms z
>> f ( z ) = c o s ( z ) / ( 2 ∗ z−p i )
f (z) =
−c o s ( z ) / ( p i − 2∗ z )
>> l i m i t ( f , z , p i /2)
ans ( z ) =
−1/2
✂ ✁

Por lo tanto, z0 = π/2 es un punto singular removible de f (z) = cos(z)/(2z − π).


98 3.4. PROBLEMAS RESUELTOS

3.4 Problemas Resueltos


Problema 3.1. Determine la región de convergencia de la serie de potencias
∞ h
X i
(−1)n − 2−(n+1) (z − 1)n .
n=0

Solución. Es claro que la serie de potencias dada se puede escribir como la resta de dos
series,
∞ h
X i ∞
X ∞
X
n −(n+1) n n n
(−1) − 2 (z − 1) = (−1) (z − 1) − 2−(n+1) (z − 1)n .
n=0 n=0 n=0

De esta forma, la región de convergencia de la serie de potencias Pinicial es la intersec-


ciónPde las regiones de convergencia de las series de potencias: a) ∞ n
n=0 (−1) (z − 1) y
n
∞ −(n+1) (z − 1)n .
b) n=0 2
P
a) Los coeficientes y el centro de la serie ∞ n n
n=0 (−1) (z − 1) son:

an = (−1)n y z0 = 1.

Ası́, aplicando el criterio de la raı́z obtenemos:


p
n
p
n
α = lı́m |an | = lı́m |(−1)n | = 1.
n→∞ n→∞
P
Luego, |z − 1| < 1 es el disco de convergencia de la serie ∞ n n
n=0 (−1) (z − 1) .
P
b) Los coeficientes y el centro de la serie ∞ n=0 2
−(n+1) (z − 1)n son: b = 2−(n+1) y z = 1.
n 0
Ası́, aplicando el criterio del cociente obtenemos:

|bn+1 | 2−(n+2) 1
α = lı́m = lı́m −(n+1) = .
n→∞ |bn | n→∞ 2 2
P
Luego, |z − 1| < 2 es el disco de convergencia de la serie ∞n=0 2
−(n+1) (z − 1)n .
P  
Por lo tanto, la región de convergencia de la serie ∞ n
n=0 (−1) − 2
−(n+1) (z − 1)n es el

disco |z − 1| < 1. 

Problema 3.2. Determine el desarrollo de Taylor de

2z + 1
f (z) = ,
z(2z − 1)

centrado en z0 = − 12 .

Solución. Expresemos a f (z) en fracciones simples:

2 1
f (z) = 1 − .
z− 2
z
CAPÍTULO 3. SERIES DE POTENCIAS Y SINGULARIDADES AISLADAS 99

Hallemos el desarrollo de Taylor de cada uno de los sumandos, centrado en z0 = − 12 . Se


tiene:
!
2 −1
=2
z − 21 1 − (z + 12 )
∞  
X 1 n 1
= (−2) z+ , z+ < 1,
2 2
n=0

!
1 −1
=2
z 1 − 2(z + 12 )
∞  
X
n 1 n 1 1
= (−2) 2 z+ , z+ < .
2 2 2
n=0

De esta forma, considerando los desarrollos de Taylor hallados previamente, obtenemos


que el desarrollo de Taylor de f (z) centrado en z0 = − 12 , está dado por:
∞   ∞  
X 1 n X
n 1 n
f (z) = (−2) z+ − (−2) 2 z+
n=0
2 n=0
2
| {z } | {z }
|z+ 21 |<1 |z+ 21 |< 21
∞  n
X
n+1 1 1 1
= (2 − 2) z + , z+ < .
2 2 2
n=0

Problema 3.3. Determine el desarrollo de Maclaurin de


ez
f (z) = .
1−z
Solución. Considerando el desarrollo de Macalurin de ez y el de 1/(1 − z), podemos
escribir:
 
z1
f (z) = e
1−z

! ∞
!
X 1 X
n n
= z z
n!
n=0 n=0
| {z } | {z }
|z|<∞ |z|<1
     
1 1 1 2 1 1 1
= 1+ 1+ z+ 1+ + z + 1+ + + z3 + · · ·
1! 1! 2! 1! 2! 3!
∞ n
!
X X 1
= zn, |z| < 1.
k!
n=0 k=0


100 3.4. PROBLEMAS RESUELTOS

Problema 3.4. Determine el desarrollo de Taylor de


i
f (z) = ,
z2
centrado en z0 = −i.
Solución. Primero hallemos el desarrollo de Taylor de g(z) = i/z, centrado en z0 = −i. Se
tiene que
 
i −1
g(z) = =i
z i − (z + i)

−1 X
= =− i−n (z + i)n
1 − i−1 (z + i)
| n=0 {z }
|i−1 (z+i)|<1

X
= − i−n (z + i)n , |z + i| < 1.
n=0

Como f (z) = −g′ (z), entonces el desarrollo de Taylor de f (z) centrado en z0 = −i, está
dado por:
" ∞ # ∞
d X
−n n
X
f (z) = − − i (z + i) = i−n n (z + i)n−1 , |z + i| < 1.
dz
n=0 n=0

Problema 3.5. Determine todos los posibles desarrollos de Laurent de


1
f (z) = ,
z(z + i)
centrados en z0 = 1.
Solución. Primero expresemos a f (z) en fracciones simples:
i i
f (z) = − .
z+i z
La función f (z) dada tiene tres desarrollos de Laurent centrados en z√
0 = 1, cuyos dominios

de validez son los siguientes anillos: a) |z − 1| < 1, b) 1 < |z − 1| < 2, y c) |z − 1| > 2.
a) El desarrollo de Laurent de f (z) centrado en z0 = 1, válido en |z − 1| < 1, está dado
por:
i i
f (z) = −
z+i z   
i 1 1
= −i
1 + i 1 + (1 + i)−1 (z − 1) 1 + (z − 1)
∞ ∞
i X X
= (−1) (1 + i) (z − 1) − i (−1)n (z − 1)n
n −n n
1+i
|n=0 {z } |n=0 {z }

|z−1|< 2 |z−1|<1

X h i
= i(−1)n (1 + i)−(n+1) − 1 (z − 1)n , |z − 1| < 1.
n=0
CAPÍTULO 3. SERIES DE POTENCIAS Y SINGULARIDADES AISLADAS 101


b) El desarrollo de Laurent de f (z) centrado en z0 = 1, válido en 1 < |z − 1| < 2, está
dado por:

i i
f (z) = −
z+i z   
i 1 i 1
= −
1 + i 1 + (1 + i)−1 (z − 1) z − 1 1 + (z − 1)−1

X ∞
X
n −(n+1) n
= i(−1) (1 + i) (z − 1) − i(−1)n (z − 1)−(n+1)
|n=0 {z } |n=0 {z }

|z−1|< 2 |z−1|>1
∞ ∞
X
n −(n+1) n
X √
= i(−1) (1 + i) (z − 1) + i(−1)n+1 (z − 1)−(n+1) , 1 < |z − 1| < 2.
n=0 n=0


c) El desarrollo de Laurent de f (z) centrado en z0 = 1, válido en |z − 1| > 2, está dado
por:

i i
f (z) = −
z+i z   
i 1 i 1
= −
z − 1 1 + (1 + i)(z − 1)−1 z − 1 1 + (z − 1)−1

X ∞
X
n n −(n+1)
= i(−1) (1 + i) (z − 1) − i(−1)n (z − 1)−(n+1)
|n=0 {z } |n=0 {z }

|z−1|> 2 |z−1|>1

X √
= i(−1)n [(1 + i)n − 1] (z − 1)−(n+1) , |z − 1| > 2.
n=0

Problema 3.6. Determine todos los posibles desarrollos de Laurent, centrados en z0 = 0,


de la función
1
f (z) = ,
z (z − 1) (z − 2)

Solución. Primero expresemos a f (z) en fracciones simples:

1/2 1 1/2
f (z) = − + .
z−2 z−1 z

La función f (z) tiene tres desarrollos de Laurent centrados en z0 = 0, cuyos dominios de


validez son los siguientes anillos: a) 0 < |z| < 1, b) 1 < |z| < 2, y c) |z| > 2.
102 3.4. PROBLEMAS RESUELTOS

a) El desarrollo de Laurent de f (z) centrado en z0 = 0, válido en 0 < |z| < 1, está dado
por:

1/2 1 1/2
f (z) = − +
z − 2 z−1  z
1 1 1 1
= − + + z −1
4 1 − 2−1 z 1−z 2
∞ ∞
1 X −n n X n 1 −1
= − 2 z + z + z
4 |2 {z }
n=0 n=0
| {z } | {z } |z|>0
|z|<2 |z|<1
∞ h i
X 1 −1
= 1 − 2−(n+2) z n + z , 0 < |z| < 1.
2
n=0

b) El desarrollo de Laurent de f (z) centrado en z0 = 0, válido en |z| > 2, está dado por:

1/2 1 1/2
f (z) = − +
z − 2 z−1  z  
1 1 −1 1 1
= − −1
−z −1
+ z −1
4 1−2 z 1−z 2
∞ ∞
1 X −n n X −(n+1) 1 −1
= − 2 z − z + z
4 |2 {z }
n=0 n=0
| {z } | {z } |z|>0
|z|<2 |z|>1
∞ ∞
X X 1 −1
= (−1)2−(n+2) z n + (−1)z −(n+1) + z , 1 < |z| < 2.
2
n=0 n=0

c) El desarrollo de Laurent de f (z) centrado en z0 = 0, válido en 1 < |z| < 2, está dado
por:

1/2 1 1/2
f (z) = − +
z−2 z−1 z
   
z −1 1 −1 1 1
= −1
−z −1
+ z −1
2 1 − 2z 1−z 2
∞ ∞
X X 1
= 2n−1 z −(n+1) − z −(n+1) + z −1
n=0 |2 {z }
| {z } |n=0 {z } |z|>0
|z|>2 |z|>1

X   1
= 2n−1 − 1 z −(n+1) + z −1 , |z| > 2.
2
n=0


CAPÍTULO 3. SERIES DE POTENCIAS Y SINGULARIDADES AISLADAS 103

Problema 3.7. Determine y clasifique las singularidades aisladas de la función


1
f (z) = .
(z + 2)3 e1/(z+2)3

Solución. El único punto singular de f (z) es z0 = −2, el cual es un punto singular esencial.
En efecto, se tiene que el desarrollo de Laurent de f (z) centrado en z0 = −2, está dado
por:
3
f (z) = (z + 2)−3 e−1/(z+2)

X (−1)n
= (z + 2)−3 (z + 2)−3n
n!
n=0

X (−1)n
= (z + 2)−3(n+1) , |z + 2| > 0.
n=0
n!

Por lo que la parte principal de f (z) en z0 = −2 posee infinitos términos distintos de cero;
por lo tanto, z0 = −2 es un punto singular esencial de f (z). 

Problema 3.8. Determine y clasifique las singularidades aisladas de la función


sen z
f (z) = .
z5
Solución. El único punto singular de f (z) es z0 = 0, el cual es un polo de orden 4. En
efecto, f (z) se puede escribir como
p(z)
f (z) = ,
q(z)
donde ( sen z
, z 6= 0,
p(z) = z y q(z) = z 4 .
1, z = 0,
Además, se tiene que: p(z) y q(z) son analı́ticas en z0 = 0, p(0) = 1 6= 0, q(0) = q ′ (0) =
q ′′ (0) = q ′′′ (0) = 0 y q (iv) (0) = 24 6= 0. Entonces, z0 = 0 es un polo de orden 4 de f (z). 

Problema 3.9. Determine y clasifique las singularidades aisladas de la función


1
f (z) = .
z 2 ez 2
Solución. El único punto singular de f (z) es z0 = 0, el cual es un polo de orden 2. En
efecto, f (z) se puede escribir como
p(z)
f (z) = ,
q(z)
2
donde p(z) = e−z y q(z) = z 2 . Además, se tiene que: p(z) y q(z) son analı́ticas en z0 = 0,
p(0) = 1 6= 0, q(0) = q ′ (0) = 0 y q ′′ (0) = 2 6= 0. Por lo tanto, z0 = 0 es un polo de orden 2
de f (z). 
104 3.5. PROBLEMAS PROPUESTOS

3.5 Problemas Propuestos


3.1. Determine el dominio de convergencia de cada una de las siguientes series:
∞ ∞
X zn X n! n
a) g) z
n nn
n=1 n=1
∞ ∞
X zn X
b) h) cos(in)z n
n!
n=0 n=0
X∞ X∞
c) nn z n i) an z n , a ∈ R
n=1 n=0
∞ ∞
X n n X (z − 1)−n
d) z j)
2n 2n n3
n=1 n=1
∞ ∞
X  −n  X n(z + 2)n
e) 2 + (−i)n (z + i)n k)
2n
n=0 n=0
∞ ∞
X n
X (−1)n−1 n
f) z2 l) z
n=0 n=1
n

3.2. Encuentre todas las representaciones en serie de potencias de (z − z0 ) para cada


una de las siguientes funciones, según el centro z0 dado.

a) f (z) = 1/z, z0 = 2i. z+1


f) f (z) = , z0 = 1.
(z − 1)2 (z + 3)
1
b) f (z) = , z0 = 2. cos z
z−3 g) f (z) = 2 , z0 = 0.
z
1  
c) f (z) = 2 , z0 = 0. 3 1
z +1 h) f (z) = z cos , z0 = 2.
z−2
1
d) f (z) = 2 , z0 = 0. sen (2z)
(z + 1)2 i) f (z) = , z0 = −1.
(z + 1)3
z
e) f (z) = , z0 = −1. j) f (z) = (1 + z + z 2 )e1/(z−1) , z0 = 1.
3z − z 2 − 2

3.3. Determine y clasifique todas las singularidades aisladas de las siguientes funciones
complejas.
1 cos z
a) f (z) = . e) f (z) = .
z 2 (z 2 − 4z + 5) z2
z+i f) f (z) = z 3 e1/z .
b) f (z) = 3 .
z + z 2 − 8z − 12
z−1 z
c) f (z) = 4 . g) f (z) = .
z − z 2 (1 + i) + i sen z
cos(z − 1) ez
d) f (z) = 4 . h) f (z) =
z − z 2 (1 + i) + i z(1 − e−z )
4
Integración Compleja
Es de notar que en los cursos de cálculo elemental, particularmente, en el tema de
integración de funciones definidas de R en R, el concepto de integral tiene la siguiente in-
Rb
terpretación geométrica: el valor de la integral a h(x) dx corresponde al área comprendida
entre el eje real y la gráfica de la función h(x) en el intervalo [a, b]. Desafortunadamente,
esta interpretación geométrica a través de áreas no se aplica para funciones de variable
compleja. Por ello se hace necesario interpretar de forma diferente la integral de una fu-
nción de variable compleja. En sı́, la integración de funciones de variable compleja es
una herramienta esencial para el desarrollo teórico de las ideas del cálculo operacional, en
particular, el estudio de las transformadas de Laplace, Fourier y z, que son temas indispen-
sables para la compresión de ciertos conceptos estudiados en Ingenierı́a Eléctrica. En este
capı́tulo se describen los conceptos básicos de la integración compleja, comenzando con la
integral definida, pasando luego por la integral de lı́nea y la primitiva de una función, para
finalizar con la teorı́a de residuos.

4.1 Integral Definida

Definición 4.1. Sea F (t) una función de variable real con valores complejos definida
como
F (t) = U (t) + i V (t),
donde U : [a, b] → R y V : [a, b] → R son funciones continuas a trozos definidas en el
intervalo acotado y cerrado a ≤ t ≤ b. Bajo estas condiciones, la función F es continua
a trozos y la integral definida de F (t) en el intervalo a ≤ t ≤ b se define como:
Z b Z b Z b
F (t) dt = U (t) dt + i V (t) dt
a a a

y se dice que F (t) es integrable en [a, b].

Ejemplo 4.1. Calcular la integral


Z π/4
eit dt.
0

Solución. Como eit = cos t + i sen t, se tiene que


Z π/4 Z π/4 Z π/4
√ √
it 2 2− 2
e dt = cos t dt + i sen t dt = +i .
0 0 0 2 2


105
106 4.2. INTEGRACIÓN DE LÍNEA

Propiedades de la integral definida

Sean F (t) = U (t) + i V (t), F1 (t) = U1 (t) + i V1 (t) y F2 (t) = U2 (t) + i V2 (t), integrables
en [a, b]. A partir de la definición de integral definida se deducen fácilmente las siguientes
propiedades:
Z b  Z b
i) Re F (t) dt = Re [F (t)] dt.
a a
Z b  Z b
ii) Im F (t) dt = Im [F (t)] dt.
a a
Z b Z b
iii) c F (t) dt = c F (t) dt, para todo c ∈ C.
a a
Z b Z b Z b
iv) [F1 (t) + F2 (t)] dt = F1 (t) dt + F2 (t) dt.
a a a
Z b Z b
v) F (t) dt ≤ |F (t)| dt.
a a

Ejercicio 4.1. Demostrar las propiedades de la integral definida.

4.2 Integración de Lı́nea


4.2.1 Contornos
A continuación presentamos conjuntos de puntos muy particulares denominados con-
tornos que nos permitirán estudiar la integral de una función de variable compleja.

Definición 4.2 (Curva). Una curva C es un conjunto de puntos z = x + i y en el plano


complejo tales que x = x(t), y = y(t) (a ≤ t ≤ b), donde x(t) y y(t) son funciones
continuas en el intervalo [a, b]. Los puntos de C se pueden describir mediante la función
continua
z(t) = x(t) + i y(t), a ≤ t ≤ b
denominada parametrización de C (ver Figura 4.1).

y
C
β = z(b)
α = z(a) b b
b
z(t)

b b b

a t b x

Figura 4.1. Representación gráfica de una curva C


CAPÍTULO 4. INTEGRACIÓN COMPLEJA 107

En la Figura 4.1 se aprecia la representación gráfica de una curva. El valor α = z(a)


se denomina extremo inicial de C y β = z(b) extremo final. Ahora, si x(t) y y(t) son
diferenciables, entonces la parametrización z(t) = x(t) + i y(t) también es diferenciable,
cuya derivada está dada por

z ′ (t) = x′ (t) + i y ′ (t), a ≤ t ≤ b.

Definición 4.3 (Curva suave). Una curva C se llama curva suave, si la derivada de la
parametrización z ′ (t) existe, es continua y nunca se hace cero en el intervalo a ≤ t ≤ b.

Ejemplo 4.2. A continuación se muestran las gráficas de las curvas C1 y C2 , y sus respec-
tivas parametrizaciones z1 (t) y z2 (t). La curva C1 es suave, en cambio la curva C2 no lo es
(se deja al lector verificar esta aseveración).

y y
1 C1 1 C2

−1 1 x −1 1 x

−1

(
z1 (t) = eit , 0 ≤ t ≤ 2π −t + i(1 + t), −1 ≤ t ≤ 0,
z2 (t) =
−t + i(1 − t), 0 < t ≤ 1.

Definición 4.4 (Contorno). Un contorno o curva suave a tramos, es una curva que consta
de un número finito de curvas suaves unidas por sus extremos.

Ejemplo 4.3. Seguidamente se aprecia una representación gráfica de un contorno confor-


mado por seis curvas suaves C1 , C2 , . . . , C6 .

C2

C3
C1
C4

C6 C5

Definición 4.5 (Contorno cerrado simple). Se dice que C es un contorno cerrado simple,
si C es un contorno y z(a) = z(b) y z(t1 ) 6= z(t2 ), para todo t1 6= t2 ∈ (a, b).
108 4.2. INTEGRACIÓN DE LÍNEA

Ejemplo 4.4. En la siguiente gráfica se muestran dos curvas C1 y C2 . La curva C1 es un


contorno cerrado simple, en cambio C2 no lo es.

C1 z2 (a) = z2 (b)
C2

z2 (t1 ) = z2 (t2 )
z1 (a) = z1 (b)

Observación 4.1. A todo contorno C representado por la ecuación

z(t) = x(t) + i y(t), a≤t≤b

se le asocia el contorno −C, cuya parametrización está definida por la ecuación z = z(−t),
donde −b ≤ t ≤ −a. Gráficamente, el contorno −C es el mismo contorno C pero recorrido
en sentido contrario.

4.2.2 Integral de Lı́nea

Definición 4.6. Sean f (z) una función y C un contorno representado por la ecuación

z(t) = x(t) + i y(t), a ≤ t ≤ b,

con extremo inicial α = z(a) y extremo final β = z(b). Supongamos que f (z) = u(x, y)+
i v(x, y) es continua a trozos en C, es decir, las partes real e imaginaria, u(x(t), y(t)) y
v(x(t), y(t)), de f (z(t)) son funciones de t continuas a trozos. Bajo estas condiciones, se
define la integral de lı́nea de f (z) a lo largo de C como:
Z Z b
f (z) dz = f (z(t)) z ′ (t) dt,
C a

donde z ′ (t) = x′ (t) + i y ′ (t).

Propiedades de la Integral de Lı́nea


Sean f (z) y g(z) funciones de variable compleja continuas a trozos sobre un contorno
C con parametrización z(t) = x(t) + i y(t), a ≤ t ≤ b. A partir de la definición de integral
de lı́nea se deducen fácilmente las siguientes propiedades.
Z Z
i) k f (z) dz = k f (z) dz, para k ∈ C.
C C
Z Z Z
ii) [f (z) + g(z)] dz = f (z) dz + g(z) dz.
C C C
CAPÍTULO 4. INTEGRACIÓN COMPLEJA 109

Z Z −a Z
iii) f (z) dz = f (z(−t)) z ′ (−t) dt = − f (z) dz.
−C −b C

iv) Si C consta de una curva C1 desde α1 hasta β1 y de la curva C2 desde α2 hasta β2 ,


donde α2 = β1 , se cumple:
Z Z Z
f (z) dz = f (z) dz + f (z) dz.
C C1 C2

Z Z b
v) f (z) dz ≤ |f (z(t)) z ′ (t)| dt.
C a

Ejercicio 4.2. Demostrar las propiedades de la integral de lı́nea.

Ejemplo 4.5. Calcular la integral Z


1
dz,
|z|=1 z
donde |z| = 1 es la circunferencia de centro 0 y radio 1, recorrida en sentido positivo.
Solución. Una parametrización de la circunferencia |z| = 1 es:

z(t) = eit = cos t + i sen t, 0 ≤ t ≤ 2π.

Ası́,
Z Z 2π Z 2π
1 1 it
dz = ie dt = i dt = 2πi.
|z|=1 z 0 eit 0

Ejemplo 4.6. Calcular la integral Z


(z − i) dz,
C

donde C es el contorno descrito por la ecuación


(
−t + i(1 + t), −1 ≤ t ≤ 0,
z(t) =
−t + i(1 − t), 0 < t ≤ 1.

Solución. Utilizando la propiedad iv) de la integral de lı́nea, podemos escribir:


Z Z 0 Z 1
(z − i) dz = (−t + i(1 + t) − i) (−1 + i) dt + (−t + i(1 − t) − i) (−1 − i) dt
C −1 0
Z 0 Z 1
= (−1 + i) t(−1 + i) dt + (−1 − i) t(−1 − i) dt
−1 0
Z 0 Z 1
2 2 −1 1
= (−1 + i) t dt + (−1 − i) t dt = −2i · + 2i · = 2i.
−1 0 2 2


110 4.3. TEOREMA DE CAUCHY-GOURSAT

4.3 Teorema de Cauchy-Goursat

El siguiente resultado se conoce como Teorema de Cauchy-Goursat. El nombre de


Cauchy es en honor al matemático francés Augustin Louis Cauchy Mel (1789-1857), quien
descrubrió el teorema en 1825, y el nombre Goursat es en honor al matemático francés
Edouard Goursat (1858-1936), quien dio una demostración que no requerı́a de la hipótesis
de que la derivada de la función fuera continua. La demostración del Teorema de Cauchy-
Goursat se puede ver en [8] y es esencialmente la misma que la demostración dada por
Goursat.
Teorema 4.1 (Teorema de Cauchy-Goursat). Sea C un contorno cerrado simple. Sea f (z)
una función analı́tica sobre y en el interior de C. Entonces,
Z
f (z) dz = 0.
C

El Teorema de Cauchy-Goursat es uno de los resultados más importantes del análisis


complejo. Desde el punto de vista práctico, este teorema puede ahorrar una gran cantidad
de trabajo al realizar cierto tipo de integraciones. Por ejemplo, integrales como
Z Z Z
sen z dz, cosh z dz y ez dz
C C C

deben anularse para cualquier contorno cerrado simple C. En todos estos casos, el in-
tegrando es una función entera. Ahora, desde el punto de vista teórico, el Teorema de
Cauchy-Goursat permite introducir el concepto de primitiva de una función (lo cual se
estudiará más adelante), entre otros conceptos.
R
Note que la dirección de integración en la integral C f (z) dz = 0 no afecta el resultado,
pues
Z Z
f (z) dz = − f (z) dz.
−C C

En el siguiente ejemplo se verifica la validez del Teorema de Cauchy-Goursat.

Ejemplo 4.7. Verifique que


Z
z n dz = 0,
C

donde n es un entero positivo y C es la circunferencia |z| = r, con r > 0.


Solución. Como n > 0, la función f (z) = z n es entera, luego por el Teorema de Cauchy-
Goursat
Z
z n dz = 0.
C

Veamos que esto es efectivamente cierto. Una parametrización de |z| = r es:

z(t) = r eit , 0 ≤ t ≤ 2π.


CAPÍTULO 4. INTEGRACIÓN COMPLEJA 111

Luego,
Z Z 2π Z 2π
n it n it n+1
z dz = (r e ) (ir e ) dt = ir e(n+1)it dt
C 0 0
Z 2π
= ir n+1 [cos((n + 1)t) + i sen ((n + 1)t)] dt
0
 
sen ((n + 1)t) − i cos((n + 1)t) 2π
= ir n+1 = 0.
(n + 1) 0

Ası́, hemos verificado el Teorema de Cauchy-Goursat para un caso particular. 

4.3.1 Extensión del Teorema de Cauchy-Goursat

Pasemos ahora a describir la extensión del Teorema de Cauchy-Goursat. Para ello ne-
cesitamos definir algunos dominios muy particulares.

Definición 4.7 (Dominio simplemente conexo). Un dominio D se dice simplemente co-


nexo si todo contorno cerrado simple dentro del mismo encierra sólo puntos de D.

Definición 4.8 (Dominio multiplemente conexo). Un dominio D se dice multiplemente


conexo si no es simplemente conexo.

Ejemplo 4.8. En la siguiente gráfica se muestran dos dominios D1 y D2 , y tres contornos


cerrados simples C, C1 y C2 . El Dominio D1 es simplemente conexo, en cambio D2 es
multiplemente conexo.

D1 D2

C C1
C2

El siguiente teorema es la extensión del Teorema de Cauchy-Goursat para dominios


simplemente conexos. La demostración de esta teorema se obtiene de forma inmediata al
emplear el Teorema de Cauchy-Goursat.

Teorema 4.2. Si f (z) es analı́tica en un dominio simplemente conexo D, entonces para


todo contorno cerrado simple C, dentro de D, se cumple
Z
f (z) dz = 0.
C
112 4.3. TEOREMA DE CAUCHY-GOURSAT

Veamos ahora un ejemplo de aplicación del Teorema de Cauchy-Goursat el cual nos


permitirá extenderlo para dominios multiplemente conexos.

Ejemplo 4.9. Pruebe que Z


1
dz = 0,
B z 2 (z 2 − 1)
donde B consta de la circunferencia |z| = 2 descrita en la dirección positiva, y de las
circunferencias |z + 1| = 1/2, |z| = 1/2 y |z − 1| = 1/2, descritas en la dirección negativa.
Solución. Sea f (z) la función definida por

1
f (z) = .
z 2 (z 2 − 1)

Sean los contornos cerrados simples C1 y C2 indicados respectivamente con los colores
azul y rojo, que se muestran en las siguientes figuras.

y y
2 2
C1
1 1

x x
−2 −1 1 2 −2 −1 1 2
−1 −1
C2
−2 −2

Es claro que Z Z Z
f (z) dz = f (z) dz + f (z) dz.
B C1 C2

Como f (z) es una función analı́tica sobre y en el interior de los contornos cerrados simples
C1 y C2 , entonces por el Teorema de Cauchy-Goursat obtenemos:
Z Z
f (z) dz = 0 y f (z) dz = 0.
C1 C2

Por lo tanto, podemos concluir que


Z
1
dz = 0.
B z 2 (z 2 − 1)

El procedimiento realizado en el Ejemplo 4.9 permite definir la extensión del Teorema


de Cauchy-Goursat para dominios multiplemente conexos, lo cual se expresa en el Teo-
rema 4.3. La demostración de este teorema se deja como ejercicio para el lector. (Ayuda:
utilice un razonamiento similar al empleado en el Ejemplo 4.9.)
CAPÍTULO 4. INTEGRACIÓN COMPLEJA 113

Teorema 4.3. Se denota a C como un contorno cerrado simple y a Cj (j = 1, 2, . . . , n)


como un número finito de contornos cerrados simples interiores a C tales que los conjuntos
interiores a cada Cj no tienen puntos en común. R es la región cerrada que consta de
todos los puntos dentro y sobre C excepto los puntos interiores a cada Cj (R es un dominio
multiplemente conexo). Se denota por B la frontera completa orientada de R que consta
de C y todos los Cj , descrita en una dirección tal que los puntos de R se encuentran a la
izquierda de B. En este caso, si una función f (z) es analı́tica en R, entonces
Z
f (z) dz = 0.
B

4.4 Integral Indefinida


El Teorema de Cauchy-Goursat es una herramienta valiosa cuando se trata de integrar
una función analı́tica alrededor de un contorno cerrado. En caso de que el contorno no
sea cerrado, existen métodos que se pueden deducir a partir de dicho teorema y que faci-
litan el cálculo de la integral considerada. El siguiente teorema se conoce como principio
de independencia de la trayectoria, cuya demostración se fundamenta en el Teorema de
Cauchy-Goursat.

Teorema 4.4 (Principio de independencia de la trayectoria). Sea f (z) analı́tica en un


dominio simplemente conexo D y sean z1 , z2 ∈ D. Entonces, el valor de la integral
Z z2
f (z) dz
z1

no depende del contorno, dentro de D, utilizado para ir de z1 a z2 .

Demostración. Podemos suponer, sin pérdida de generalidad, que los contornos C1 y C2


van de z1 a z2 sin intersección en puntos intermedios. Se tiene que los contornos C1 y
−C2 forman un contorno cerrado simple, que denominamos C. Luego, por el Teorema de
Cauchy-Goursat
Z
f (z) dz = 0,
C

pero
Z Z Z Z Z
f (z) dz = f (z) dz + f (z) dz = f (z) dz − f (z) dz,
C C1 −C2 C1 C2

por lo tanto,
Z Z
f (z) dz = f (z) dz,
C1 C2

lo cual indica que la integral desde z1 hasta z2 es ası́ independiente del contorno seguido,
en tanto ese contorno se encuentre dentro de D.

Del principio de la independencia de la trayectoria se define la primitiva de una función


de variable compleja.
114 4.4. INTEGRAL INDEFINIDA

Definición 4.9 (Integral indefinida o primitiva). Sea f (z) una función analı́tica en un
dominio simplemente conexo D ⊂ C. Sea z0 ∈ D. La función F (z) definida en D por
Z z
F (z) = f (s) ds + c, (4.1)
z0

se denomina integral indefinida o primitiva de f (z), donde c es un constante compleja.

En realidad, f (z) posee un número infinito de primitivas. Dichas primitivas difieren en


valores constantes y son analı́ticas en D, y satisfacen

F ′ (z) = f (z), para todo z ∈ D.

Usamos la notación de integral indefinida


Z
f (z) dz

para indicar todas las posibles primitivas de f (z). Note que la primitiva de cada una de las
funciones elementales complejas, son las mismas primitivas de las funciones elementales
de variable real; por ejemplo, la primitiva de ez es, por supuesto, ella misma, y la primitiva
de sen z es − cos z, etc.
Por otra parte, el valor de la constante c correspondiente a una primitiva especı́fica
Z z
f (s) ds
z0

queda determinado por el lı́mite inferior de integración, como se muestra en el siguiente


ejemplo.

Ejemplo 4.10. a) Encuentre las primitivas de f (z) = z sen z.


R π/2
b) Encuentre la primitiva necesaria para calcular la integral π z sen z dz y halle el valor
de la integral.
Solución.
a) Usando integración por partes obtenemos
Z Z
z sen z dz = −z cos z + cos z dz = −z cos z + sen z + c = F (z).

b) Usando el resultado de a) tenemos


Z z
s sen s ds = −z cos z + sen z + c.
π

Para determinar el valor de c observemos que el lado izquierdo de esta ecuación es cero
cuando z = π. Por lo tanto,
0 = −π cos π + sen π + c,
de donde se deduce que c = −π. De esta forma,
Z z
s sen s ds = −z cos z + sen z − π.
π
CAPÍTULO 4. INTEGRACIÓN COMPLEJA 115

Utilizando esta última ecuación tenemos que


Z π/2
z sen z dz = −(π/2) cos(π/2) + sen (π/2) − π = 1 − π.
π

De la ecuación (4.1) se infiere que una integral definida se puede evaluar de igual
forma que en el cálculo elemental:
Z β β
f (z) dz = F (β) − F (α) = F (z) ,
α α

es decir, usando la Regla de Barrow, que es una aplicación del Teorema Fundamental del
Cálculo.

4.5 Fórmula Integral de Cauchy


En esta sección veremos que si una función es analı́tica en un punto, sus derivadas de
todos los órdenes existen en ese punto y son también analı́ticas ahı́. Previo a este resultado
veremos un resultado curioso que se obtiene a través del Teorema de Cauchy-Goursat, a
saber: si consideramos una función analı́tica sobre y en el interior de un contorno cerrado
simple, basta con conocer los valores que ella toma sobre ese contorno, para determinar los
valores que toma en el interior del mismo. Este resultado se conoce como fórmula integral
de Cauchy. La demostración de este teorema se puede ver en [8].

Teorema 4.5 (Fórmula Integral de Cauchy). Sea f (z) una función analı́tica en un domi-
nio simplemente conexo D ⊂ C. Sea C un contorno cerrado simple C dentro de D. Sea
z0 ∈ D un punto interior a C. Entonces,
Z
1 f (z)
f (z0 ) = dz. (4.2)
2πi C (z − z0 )

La fórmula (4.2) se denomina fórmula integral de Cauchy. El siguiente ejemplo aclara


el uso de esta fórmula en la evaluación de integrales.
Ejemplo 4.11. Hallar el valor de la integral
Z
1
dz.
|z−i|=2 z2 + 4

Solución. Factorizando el integrando tenemos


1 1
= .
z2 +4 (z − 2i)(z + 2i)
Observamos que el factor (z −2i) se anula dentro del contorno de integración y que (z +2i)
no se anula ni sobre el contorno ni en su interior. Escribiendo la integral considerada en la
forma
1
Z
(z + 2i)
dz
|z−i|=2 (z − 2i)
116 4.5. FÓRMULA INTEGRAL DE CAUCHY

notamos que la función f (z) = 1/(z + 2i) es analı́tica tanto en la circunferencia |z − i| = 2


como en su interior. Por tanto, podemos usar la fórmula integral de Cauchy dada en el
Teorema 4.5, podemos escribir:
Z Z
1 f (z) 1 1
f (2i) = dz = 2
dz.
2πi |z−i|=2 (z − 2i) 2πi |z−i|=2 z + 4

1
Como f (2i) = , entonces el valor de la integral considerada es
4i
Z
1 π
2+4
dz = 2πif (2i) = .
|z−i|=2 z 2

Pasemos ahora a ver que si una función es analı́tica en un punto, entonces sus derivadas
de todos los órdenes existen en ese punto y son también analı́ticas. Tal resultado se conoce
como extensión de la fórmula integral de Cauchy.

Teorema 4.6 (Extensión de la Fórmula Integral de Cauchy). Sea f (z) una función
analı́tica en un dominio simplemente conexo D ⊂ C. Sea C un contorno cerrado sim-
ple C dentro de D. Sea z0 ∈ D un punto interior a C. Entonces, f (z) es infinitamente
diferenciable en D y Z
(n) n! f (z)
f (z0 ) = dz.
2πi C (z − z0 )n+1
Además, f (n) (z) es analı́tica en D para cada n.

El siguiente ejemplo aclara el uso de la extensión de la fórmula integral de Cauchy en


la evaluación de integrales.
Ejemplo 4.12. Hallar el valor de la integral
Z
1
dz.
|z−i|=2 (z 2 + 4)2

Solución. Factorizando el integrando tenemos


1 1
= .
(z 2 + 4)2 (z − 2i) (z + 2i)2
2

Tomando f (z) = 1/(z + 2i)2 y z0 = 2i, por el Teorema 4.6 podemos escribir:
Z Z
′ 1 f (z) 1 1
f (2i) = 2
dz = dz.
2πi |z−i|=2 (z − 2i) 2πi |z−i|=2 (z + 4)2
2

−2
Como f ′ (z) = , entonces el valor de la integral considerada es
(z + 2i)3
Z
1 π
2 2
dz = 2πif ′ (2i) = .
|z−i|=2 (z + 4) 16


CAPÍTULO 4. INTEGRACIÓN COMPLEJA 117

4.6 Residuo
La teorı́a de residuos proporciona una técnica de evaluación (rápida y) efectiva de
integrales de la forma Z
f (z) dz,
C
donde C es un contorno cerrado simple. El punto esencial de la teorı́a es el concepto de
residuo y su relación con la serie de Laurent.

Definición 4.10. Sean C un contorno cerrado simple y z0 ∈ C un punto interior a C.


Sea f (z) una función analı́tica sobre C y en todo punto de su interior, salvo en z0 . El
residuo de f (z) en z0 , denotado por Res [f (z)], se define como
z=z0
Z
1
Res [f (z)] = f (z) dz.
z=z0 2πi C

El siguiente teorema describe la relación que existe entre el residuo Res [f (z)] y una y
z=z0
sólo una serie de Laurent de f (z) centrada en z0 ; además, dice que Res [f (z)] es igual al
z=z0
coeficiente b1 de esa serie de Laurent.

Teorema 4.7. Sea z0 un punto singular aislado de una función f (z). Entonces, el residuo
de f (z) en z0 es igual al coeficiente de (z − z0 )−1 en la serie de Laurent que representa a
f (z) en el anillo
0 < |z − z0 | < r,
para cierto número real r > 0.

Demostración. Como z0 es un punto singular aislado de f (z), entonces existe r > 0 tal que
el desarrollo de Laurent de f (z) centrado en z0 , válido en el anillo 0 < |z − z0 | < r, está
dado por:
X∞ X∞
n
f (z) = an (z − z0 ) + bn (z − z0 )−n .
n=0 n=1
Sea C un contorno cerrado simple contenido en el anillo 0 < |z − z0 | < r tal que z0 es un
punto interior de C. Ası́, integrando en ambas partes de la ecuación anterior obtenemos:
Z Z "X∞ ∞
X
#
f (z) dz = an (z − z0 )n + bn (z − z0 )−n dz
C C n=0 n=1

X Z X∞ Z
n
= an (z − z0 ) dz + bn (z − z0 )−n dz,
n=0 C n=1 C

además, aplicando convenientemente el Teorema de Cauchy-Goursat, la fórmula integral


de Cauchy o la extensión de la fórmula integral de Cauchy, tenemos:
Z
(z − z0 )n dz = 0, para n ≥ 0,
Z C
(z − z0 )−1 dz = 2πi,
ZC
(z − z0 )−n dz = 0, para n ≥ 2.
C
118 4.6. RESIDUO

Por lo tanto, Z
f (z) dz = b1 2πi,
C
de donde se deduce que
Res [f (z)] = b1 ,
z=z0

que era lo que deseábamos demostrar.

Ejercicio 4.3. Utilice el Teorema 4.7 para calcular los siguientes residuos:
 
a) Res e1/z
z=0
 
1
b) Res
z=i (z − i)2
 
c) Res z 4 sen (1/z)
z=0

4.6.1 Cálculo del Residuo


Como observamos en el Ejercicio 4.3, el Teorema 4.7 se puede aplicar para calcular
el residuo de una función f (z) en un punto singular aislado z0 . De esta forma, por el
Teorema 4.7 se tiene:
• si z0 es un punto singular removible de la función f (z), entonces su residuo en ese
punto es cero;
• si z0 es un punto singular esencial de f (z), la única manera para determinar el resi-
duo en dicho punto, consiste en obtener el desarrollo de Laurent centrado z0 , válido
en el anillo dado en el Teorema 4.7, y tomar el residuo igual al coeficiente b1 .
Pero, si la función tiene un polo en z0 , no es necesario obtener todo el desarrollo
de Laurent centrado en z0 para encontrar el coeficiente que buscamos. Existen diversos
métodos que podemos usar siempre y cuando sepamos que la singularidad es un polo, los
cuales estudiaremos a continuación.

Cálculo del Residuo en un Polo


El siguiente teorema nos permite identificar si un punto z0 es un polo de f (z) y, además,
nos dice como calcular el residuo Res [f (z)].
z=z0

Teorema 4.8. Sea z0 un punto singular aislado de una función f (z). Si para cierto entero
positivo m, la función
φ(z) = (z − z0 )m f (z)
se puede definir en z0 de modo que sea analı́tica ahı́ y φ(z0 ) 6= 0, entonces f (z) tiene un
polo de orden m en z0 y, además,


 φ(z0 ) = lı́m (z − z0 )m f (z), m = 1;

 z→z0
Res [f (z)] = (4.3)
z=z0 
 φ(m−1) (z0 )

 , m > 1.
(m − 1)!
CAPÍTULO 4. INTEGRACIÓN COMPLEJA 119

Demostración. Como φ(z) = (z − z0 )m f (z) es analı́tica en z0 y φ(z0 ) 6= 0, entonces la


función f (z) se puede escribir como

p(z)
f (z) = ,
q(z)

donde p(z) = φ(z) y q(z) = (z − z0 )m , además,

q(z0 ) = q ′ (z0 ) = · · · = q (m−1) (z0 ) = 0, y q (m) (z0 ) = m! 6= 0.

Luego, por el Teorema 3.9 z0 es un polo de orden m de f (z).


Pasemos ahora a demostrar la fórmula (4.3). Como φ(z) es analı́tica en z0 ella posee
desarrollo de Taylor centrado en z0 , válido en cierto disco |z − z0 | < r, dado por

φ(m) (z0 )
φ(z) = (z − z0 )m f (z) = φ(z0 ) + φ′ (z0 )(z − z0 ) + · · · + (z − z0 )m + · · · .
m!
Ası́, en cada punto z del disco |z − z0 | < r excepto en z0 , se tiene que

φ(z0 ) φ′ (z0 ) φ(m−1) (z0 ) φ(m) (z0 )


f (z) = + + · · · + + + ··· ,
(z − z0 )m (z − z0 )m−1 (m − 1)!(z − z0 ) m!

que es el desarrollo de Laurent de f (z) centrado en z0 válido en el anillo 0 < |z − z0 | < r,


con
φ(m−1) (z0 )
b1 = .
(m − 1)!
Entonces, por el Teorema 4.7 la ecuación (4.3) es cierta.

En el siguiente ejemplo se muestra la utilidad del teorema anterior en el cálculo del


residuo en un polo.
ez
Ejemplo 4.13. Hallar el residuo de f (z) = en cada uno de sus puntos singulares.
sen z
ez
Solución. Los puntos singulares aislados de f (z) = son:
sen z
zn = nπ, n = 0, ±1, ±2, . . . ,

además, cada zn es un polo simple de f (z). Definamos la función φn (z) como




 lı́m (z − zn )f (z) = (−1)n enπ , si z = zn ,
z→zn
φn (z) = (z − zn )f (z) =
z
 (z − zn )e ,


si z 6= zn .
sen z
Es claro que φn (z) es analı́tica en zn y φn (zn ) 6= 0. Ası́. utilizando la ecuación (4.3) se tiene
ez
que el residuo de f (z) = en cada uno de sus puntos singulares zn es
sen z
 z 
e
Res = (−1)n enπ , n = 0, ±1, ±2, . . .
z=zn sen z


120 4.6. RESIDUO

Ejercicio 4.4. Calcule los siguientes residuos:


 
ez
a) Res 2 2 .
z=0 z (z + 1)
" #
z 1/2
b) Res . (Utilizando la rama principal de z 1/2 )
z=2 z(z − 2)2

 
Log z
c) Res 4 .
z=1 z (z − 1)2
 
sen z − z
d) Res , para n = 0, ±1, ±2, . . .
z=nπi z senh z

4.6.2 Teorema de los Residuos


El siguiente teorema nos permite calcular la integral de una función f (z) a lo largo de
un contorno cerrado simple C, tal que f (z) es analı́tica en C y en su interior, salvo en un
número finito de puntos singulares interiores a C.

Teorema 4.9 (Teorema de los Residuos). Sea C un contorno cerrado simple orientado
positivamente, dentro y sobre el cual una función f (z) es analı́tica excepto en un número
finito de puntos z1 , z2 , . . . , zn interiores a C. Entonces,
Z n
X
f (z) dz = 2πi Res [f (z)] .
C z=zk
k=1

Demostración. Se deja como ejercicio para el lector. Ayuda:R defina convenientemente


Pn R n
contornos cerrados simples, C1 , . . . , Cn , de manera que C f (z)dz = k=1 Ck f (z)dz,
luego
R aplique la fórmula integral de Cauchy para calcular cada una de las integrales
Ck f (z)dz.

En el siguiente ejemplo utilizamos el Teorema de los Residuos para calcular la integral


de una función f (z), a lo largo de un contorno cerrado simple que posee en su interior un
número finito de puntos singulares de f (z).

Ejemplo 4.14. Calcular la integral


Z
z−2
dz,
C (z − 1)z

donde C es la circunferencia |z| = 3 orientada positivamente.


z−2
Solución. Sea f (z) = . Los puntos singulares de f (z) son z0 = 0 y z1 = 1, que son
(z − 1)z
puntos interiores a C, además, z0 y z1 son polos simples de f (z). Luego,

Res [f (z)] = 2 y Res [f (z)] = −1.


z=z0 z=z1
CAPÍTULO 4. INTEGRACIÓN COMPLEJA 121

Como f (z) es analı́tica dentro y sobre C excepto en z0 y z1 , entonces por el Teorema de los
Residuos obtenemos
Z  
z−2
dz = 2πi Res [f (z)] + Res [f (z)] = 2πi.
C (z − 1)z z=z0 z=z1

4.6.3 Expansión en Fracciones Parciales


Una aplicación de gran importancia del cálculo de residuos, es la expansión en frac-
ciones parciales de algunas funciones racionales particulares. La expansión en fracciones
parciales se aplica a funciones racionales propias, esto es, a funciones del tipo
b0 + b1 z + · · · + bM z M
f (z) = ,
a0 + a1 z + · · · + z N
donde M < N . La expansión en fracciones parciales consiste en expresar la función racio-
nal propia f (z) como una suma de fracciones simples. El siguiente teorema nos muestra
explı́citamente la forma de la expansión en fracciones parciales, dependiendo de la multi-
plicidad de los polos de f (z).

Teorema 4.10. Sea f (z) una función racional propia dada por
c0 + c1 z + · · · + cM z M
f (z) = ,
d0 + d1 z + · · · + dN −1 z N −1 + z N
donde M < N . Sean pk los polos de f (z) y rk sus respectivas
P multiplicidades, para k =
1, 2, . . . , T , donde T es un entero positivo tal que N = Tk=1 rk . Entonces:
(i) Si todos los polos de f (z) son simples, la expansión en fracciones parciales de f (z) es:
A1 A2 AN
f (z) = + + ··· + ,
(z − p1 ) (z − p2 ) (z − pN )
donde los números complejos Ak , denominados coeficientes, se calculan como
Ak = Res [f (z)] , para k = 1, 2, . . . , N .
z=pk

(ii) Si todos los polos de f (z) son simples, excepto el polo pℓ que es de orden rℓ , la ex-
pansión en fracciones parciales de f (z) es:
A1 A2 Aℓ−1
f (z) = + + ··· +
(z − p1 ) (z − p2 ) (z − pℓ−1 )
Aℓ,1 Aℓ,2 Aℓ,rℓ
+ + + ··· +
(z − pℓ ) (z − pℓ )2 (z − pℓ )rℓ
Aℓ+1 Aℓ+2 AT
+ + + ··· + ,
(z − pℓ+1 ) (z − pℓ+2 ) (z − pT )
donde
Ak = Res [f (z)] , 6 ℓ;
para k = 1, 2, . . . , T , y k =
z=pk
" #
1 d(rℓ −)
Aℓ, = · [(z − pℓ )rℓ f (z)] , para  = 1, 2, . . . , rℓ .
(rℓ − )! dz (rℓ −)
z=pℓ
122 4.6. RESIDUO

Demostración. Sólo demostraremos la parte (i), la parte (ii) se deja como ejercicio para el
lector. Realicemos la demostración por inducción en el número de polos simples de f (z).
Supongamos que f (z) posee un sólo polo simple p1 y demostremos que

A1
f (z) = (4.4)
(z − p1 )
con
A1 = Res [f (z)] .
z=p1

Como f (z) es una función racional propia de la forma

c0 + c1 z + · · · + cM z M
f (z) = ,
d0 + d1 z + · · · + dN −1 z N −1 + z N

entonces todos los polos de f (z) son las raı́ces del polinomio d0 +d1 z+· · ·+dN −1 z N −1 +z N ;
pero f (z) posee un sólo polo simple p1 , luego el polinomio a0 + a1 z + · · · + z N adquiere la
forma (z − p1 ) y el polinomio c0 + c1 z + · · · + cM z M debe ser de grado 0, es decir, c0 6= 0 y
c1 = c2 = · · · = cM = 0. Por lo tanto, f (z) tiene la forma
c0
f (z) = ,
z − p1

además, utilizando el desarrollo de Laurent de f (z) centrado en p1 obtenemos:

Res [f (z)]
c0 b1 z=p1
= f (z) = a0 + = a0 + , ⇒ a0 = 0 y c0 = Res [f (z)] ,
z − p1 z − p1 z − p1 z=p1

en otras palabras, (4.4) es cierta.


Como hipótesis inductiva supongamos que una función h(z) posee N = L polos simples
p1 , . . . , pL y que además se cumple

A1 A2 AL
h(z) = + + ··· + , (4.5)
(z − p1 ) (z − p2 ) (z − pL )

donde
Ak = Res [h(z)] , para k = 1, 2, . . . , L.
z=pk

Demostremos que si f (z) posee N = L + 1 polos simples p1 , . . . , pL , pL+1 , entonces se


cumple
A1 A2 AL AL+1
f (z) = + + ··· + + , (4.6)
(z − p1 ) (z − p2 ) (z − pL ) (z − pL+1 )
donde
AL+1 = Res [f (z)] .
z=pL+1

Como f (z) es una función racional propia y p1 , . . . , pL , pL+1 son polos simples, entonces
f (z) tiene la forma

p(z) c
f (z) = + ,
(z − p1 )(z − p1 ) · · · (z − pL ) (z − pL+1 )
CAPÍTULO 4. INTEGRACIÓN COMPLEJA 123

donde p(z) es un polinomio de grado < L y c ∈ C. Tomando h1 (z) y h2 (z) definidas


respectivamente como:
p(z) c
h1 (z) = y h2 (z) = ,
(z − p1 )(z − p1 ) · · · (z − pL ) (z − pL+1 )
se tiene
f (z) = h1 (z) + h2 (z),
además, por (4.5) podemos escribir:
A1 A2 AL AL+1
h1 (z) = + + ··· + y h2 (z) = ,
(z − p1 ) (z − p2 ) (z − pL ) (z − pL+1 )
donde
Ak = Res [h(z)] , para k = 1, 2, . . . , L + 1.
z=pk

Por lo tanto, de todo lo anterior obtenemos:


A1 A2 AL AL+1
f (z) = + + ··· + + ,
(z − p1 ) (z − p2 ) (z − pL ) (z − pL+1 )
es decir, (4.6) es cierta.

El teorema anterior se puede extender a funciones racionales propias que tienen dos o
más polos cuyo orden es mayor que 1. En el siguiente ejemplo se muestra tal extensión.
Ejemplo 4.15. Halle la expansión en fracciones parciales de
144 z 2 + 144 z + 144
f (z) = .
(z − 3)2 (z − 2)2 (z + 1)
Solución. Los puntos p1 = −1, p2 = 2 y p3 = 3 son los polos de f (z). Se observa que p1 es
de orden 1 y p2 y p3 son de orden 2. De esta forma, la expansión en fracciones parciales
de f (z) es de la forma
A1 A2,1 A2,2 A3,1 A3,2
f (z) = + + 2
+ + .
(z + 1) (z − 2) (z − 2) (z − 3) (z − 3)2
Se tiene que

A1 = Res [f (z)] = 1,
z=−1
 
A2,2 = (z − 2)2 f (z) z=2 = 336,
 
d 2
A2,1 = [(z − 2) f (z)] = 800,
dz z=2
 
A3,2 = (z − 3)2 f (z) z=3 = 468,
 
d
A3,1 = [(z − 3)2 f (z)] = −801,
dz z=3
ası́, la expansión en fracciones parciales de la función dada es
1 800 336 801 468
f (z) = + + 2
− + .
(z + 1) (z − 2) (z − 2) (z − 3) (z − 3)2

124 4.7. INTEGRACIÓN Y RESIDUOS CON MATLAB

4.7 Integración y Residuos con M ATLAB


La herramienta de matemática simbólica de M ATLAB cuenta con el comando int que
permite calcular integrales de funciones complejas. Con int(f,z) se calcula la primitiva
de la función f (z). En el siguiente ejemplo se calculan las primitivas de las siguientes
funciones:
z+1 z5
f (z) = sen (z + 1), g(z) = , h(z) = .
(z − 2)2 ez−1

>> syms z
>> f ( z ) = s i n ( z +1); i n t ( f , z )
ans ( z ) =
−c o s ( z + 1)
>> g ( z ) = ( z +1)/(z −2)ˆ2; i n t ( g , z )
ans ( z ) =
l o g ( z − 2) − 3/( z − 2)
>> h ( z ) = z ˆ5/ exp ( z −1); i n t ( h , z )
ans ( z ) =
−exp (1 − z ) ∗ ( z ˆ5 + 5∗ z ˆ4 + 20∗ z ˆ3 + 60∗ z ˆ2 + 120∗ z + 120)
✂ ✁
Ası́, las primitivas de las funciones dadas son:

3
F (z) = − cos(z + 1) + c, G(z) = Log (z − 2) − + c,
z−2
z 5 + 5 z 4 + 20 z 3 + 60 z 2 + 120 z + 120
H(z) = − + c.
ez−1

Con el comando int(f,z,a,b) se calcula la integral definida


Z b
f (z) dz.
a

donde a, b ∈ C. Observe el siguiente ejemplo donde se calcula la integral


Z −1
ez + z 3 − i
dz = i e + e2 (−2 + i) + cos(1) e2 (2 − 3i) + sen (1)e2 (3 + 2i).
−1+i ez−1

>> syms z
>> f ( z ) = ( exp ( z)+zˆ3− i ) / exp ( z −1); i n t ( f , z,−1−i , −1)
ans ( z ) =
exp (1)∗ i + exp(2)∗( − 2 + i ) + c o s (1)∗ exp ( 2 ) ∗ ( 2 − 3∗ i )
+ exp (2)∗ s i n ( 1 ) ∗ ( 3 + 2∗ i )
✂ ✁
En M ATLAB no existe una comando que permita calcular, en forma directa, una integral
de lı́nea. Nosotros hemos diseñado la función IntegralLinea (ver Programa 4.1), que
permite calcular la integral de lı́nea
Z Z b
f (z) dz = f (z(t)) z ′ (t) dt,
C a

donde C es una curva con parametrización z(t) = x(t) + iy(t), a ≤ t ≤ b y f (z) es una
función de variable compleja.
CAPÍTULO 4. INTEGRACIÓN COMPLEJA 125

Programa 4.1. Función IntegralLinea.m

f u n c t i o n [ v i ]= I n t e g r a l L i n e a ( f , zt , a , b )
%I n t e g r a l L i n e a C a l c u l a e l v a l o r de l a i n t e g r a l de f ( z ) a l o
% l a r g o de l a cu rva C con p a r a m e t r i z a c i ó n
% z ( t ) , a <= t <= b
% v i : v a l o r de l a i n t e g r a l de lı́ n e a
%
syms z t ;
gt = f ( zt ) ;
d z t = d i f f ( zt , t ) ;
v i = i n t ( g t ∗ dzt , t , a , b ) ;
end

En el siguiente ejemplo, se utiliza la función IntegralLinea para calcular las integra-


les:
Z Z    
z3 1 i 1 i
dz = 0, Log z dz = π − + ln(2) − − 1 + 2 i,
C1 z2 C2 4 4 2 2

donde C1 es el arco de la circunferencia de centro 0 y radio 2, desde z = 2i a z = −2, y C2


es el segmento de recta que va de z = i a z = 1 − i.

>> syms z t
>> f ( z)= c o n j ( z )ˆ3/ z ˆ 2 ; z t =2∗( c o s ( t )+ i ∗ s i n ( t ) ) ;
>> v i= I n t e g r a l L i n e a ( f , zt , p i /2 , p i )
vi =
0
>> f ( z)= l o g ( z ) ; z t=t +(1−2∗ t )∗ i ;
>> v i= I n t e g r a l L i n e a ( f , zt , 0 , 1 )
vi =
p i ∗(1/4 − i /4) + l o g (2)∗(1/2 − i /2) − 1 + 2∗ i
✂ ✁

Por otra parte, M ATLAB no cuenta con un comando que calcule directamente el residuo
de una función f (z) en z0 . Hemos diseñado la función ResiduoPolo (ver Programa 4.2),
que calcula el residuo Res [f (z)], cuando z0 es un polo de orden m.
z=z0

Programa 4.2. Función ResiduoPolo.m

f u n c t i o n [ r e s]= Residu oPolo ( f , z0 ,m)


%Residu oPolo C a l c u l a e l r e s i u d o de f ( z ) en e l polo
% z0 de orden m
% res : valor del residuo
%
syms z ;
p h i = ( z−z0 )ˆm∗ f ;
i f m==1
r e s = l i m i t ( phi , z , z0 ) ;
else
dphi = d i f f ( phi , z ,m−1);
r e s = l i m i t ( dphi , z , z0 ) / f a c t o r i a l (m−1);
end
end
126 4.7. INTEGRACIÓN Y RESIDUOS CON MATLAB

En el siguiente ejemplo, se utiliza la función ResiduoPolo para calcular los residuos:


 
ez
a) Res 2 2 = 1.
z=0 z (z + 1)
 
Log z
b) Res 4 = 0.
z=1 z (z − 1)2
 
sen z − z
c) Res = π−senh(π)
π .
z=πi z senh z

>> syms z
>> f ( z ) = exp ( z ) / ( z ˆ2∗( z ˆ2+1)); z0 = 0 ; m = 2 ;
>> r e s = Residu oPolo ( f , z0 ,m)
res (z) =
1
>> f ( z ) = l o g ( z ) / ( z ˆ4∗( z −1)); z0 = 1 ; m = 2 ;
>> r e s = Residu oPolo ( f , z0 ,m)
res (z) =
0
>> f ( z ) = ( s i n ( z)−z ) / ( z ∗ s i n h ( z ) ) ; z0 = p i ∗ i ; m = 1 ;
>> r e s = Residu oPolo ( f , z0 ,m)
res (z) =
( pi − sinh ( pi ))/ pi
✂ ✁
Ahora bien, para calcular el residuo de una función f (z) en un punto singular esencial
z0 , es necesario hallar el desarrollo de Laurent para obtener tal residuo. Para ello podemos
usar convenientemente el comando taylor para determinar solo aquellos coeficientes del
desarrollo de Laurent necesarios para hallar Res [f (z)]. Tenga presente que el procedi-
z=z0
miento que explicaremos a continuación, se aplica a funciones f (z) que pueden expresarse
como f (z) = f1 (z)f2 (z) (o un producto finito de funciones), donde z0 es un punto singular
esencial de f2 (z) y f1 (z) es analı́tica en z0 .
Suponga que se desea hallar el residuo de f (z) = (1 + z 2 )e1/(z−2i) en z0 = 2i, que, por
supuesto, es un punto singular esencial de f (z). Es claro que

f (z) = f1 (z)f2 (z),

donde
f1 (z) = 1 + z 2 y f2 (z) = e1/(z−2i) .
Primero, hallamos el desarrollo de Taylor de f1 (z) centrado en z0 = 2i (en este caso se
hallan todos los términos, que son tres),

>> syms z
>> f 1 ( z ) = 1+z ˆ 2 ; d t f 1 ( z ) = t a y l o r ( f1 , z , 2 i , ’ Order ’ , 3 )
dtf1 ( z) =
z ∗4∗ i + ( z − 2∗ i )ˆ2 + 5
✂ ✁
o, equivalentemente,
f1 (z) = −3 + 4i(z − 2i) + (z − 2i)2 . (4.7)
Luego, hallamos los primeros 5 términos del desarrollo de Maclaurin de h(w) = ew ,

>> syms w
>> h (w) = exp (w) ; dth (w)= t a y l o r ( h , w, 0 , ’ Order ’ , 5 )
dth (w) =
wˆ4/24 + wˆ3/6 + wˆ2/2 + w + 1
✂ ✁
CAPÍTULO 4. INTEGRACIÓN COMPLEJA 127

Seguidamente realizamos la operación dth(1/(z-2i)),



>> dth ( 1 / ( z−2i ) )
ans =
1/( z − 2∗ i ) + 1/(2∗( z − 2∗ i ) ˆ 2 ) + 1/(6∗( z − 2∗ i ) ˆ 3 ) +
1/(24∗( z − 2∗ i ) ˆ 4 ) + 1
✂ ✁

que corresponden a los primeros 5 términos del desarrollo de Laurent de f2 (z) centrado
en z0 = 2i,
1 1 1 1
+ 2 + 3 + +1 (4.8)
z − 2 i 2 (z − 2 i) 6 (z − 2 i) 24 (z − 2 i)4

Multiplicando las ecuaciones (4.7) y (4.8), obtenemos que el coeficiente b1 del desarrollo
de Laurent de f (z) centrado en z0 = 2i, está dado por:

4i 1 17
b1 = −3 + + = − + 2i,
2 6 6

en otras palabras, el residuo de f (z) en z0 = 2i es

17
Res [f (z)] = − + 2i.
z=2i 6

Para finalizar esta sección, mostramos la función ExpFracParc (ver Programa 4.3) que
halla la expansión en fracciones parciales de una función racional propia del tipo

c0 + c1 z + · · · + cM z M
f (z) = ,
d0 + d1 z + · · · + dN −1 z N −1 + z N

donde M < N .
En el siguiente ejemplo, se utiliza la función ExpFracParc para hallar la expansión en
fracciones parciales de
z+1
f (z) = .
(z + 3)(z + 2)3 (z − 1)2

>> syms z
>> f ( z)=( z +1)/(( z +3)∗( z +2)ˆ3∗(z −1)ˆ2); p=[−3 −2 1 ] ; v=[1 3 2 ] ;
>> e f p = E x p F r a c P a rc ( f , p , v )
efp =
1/(54∗(z − 1)ˆ2) − 1/(72∗( z − 1 ) ) − 1/(9∗( z + 2 ) ) +
4/(27∗( z + 2)ˆ2) + 1/(8∗( z + 3 ) ) − 1/(9∗( z + 2)ˆ3)
✂ ✁

o, equivalentemente,

1/9 4/27 1/9 1/8 1/72 1/54


f (z) = − + 2 − 3 + − +
(z + 2) (z + 2) (z + 2) (z + 3) (z − 1) (z − 1)2
128 4.8. PROBLEMAS RESUELTOS

Programa 4.3. Función ExpFracParc.m

f u n c t i o n [ e f p]= E x p F r a c P a r c ( f , p , v )
%E x p F r a c P a rc H a l l a l a expansi ón en f r a c c i o n e s p a r c i a l e s
% de l a f u n c i ó n r a c i o n a l p r o p i a f ( z )
% p : v e c t o r de p o l o s
% v : v e c t o r de m u t i p l i c i d a d e s de l o s p o l o s
% e f p : expansi ón en f r a c c i o n e s p a r c i a l e s
syms z ;
np = l e n g t h ( p ) ;
efp = 0;
f o r t =1:np
i f v ( t ) == 1
dphi ( z ) = ( z−p ( t ) ) ∗ f ( z ) ;
r = dphi ( p ( t ) ) ;
e f p = e f p + sym ( r / ( z−p ( t ) ) ) ;
else
f o r s =1:v ( t )
dphi ( z ) = d i f f ( ( z−p ( t ) ) ˆ ( v ( t ) ) ∗ f ( z ) , z , v ( t )−s ) ;
r = dphi ( p ( t ) ) / f a c t o r i a l ( v ( t )− s ) ;
e f p = e f p + sym ( r / ( z−p ( t ) ) ˆ s ) ;
end
end
end
end

4.8 Problemas Resueltos


Problema 4.1. Determine el valor de la integral
Z
Re (z(z − 1)) dz,
C

donde C es el arco desde z = −1 + i hasta z = 1 + i a lo largo de la curva y = |x|.

Solución. La parametrización de C es
(
t − it, −1 ≤ t ≤ 0,
z(t) =
t + it, 0 < t ≤ 1.

Ası́, usando la definición y las propiedades de la integral de lı́nea podemos escribir:


Z Z 1
Re (z(z − 1)) dz = Re (z(t)(z(t) − 1))z ′ (t) dt
C −1
Z 0 Z 1
2
= (1 − i) (2t − t) dt + (1 + i) (2t2 − t) dt
−1 0
7 1 8
= (1 − i) + (1 + i) = − i.
6 6 6

CAPÍTULO 4. INTEGRACIÓN COMPLEJA 129

Problema 4.2. Sea f (z) una función de variable compleja definida por
(
1, si Re z < 0;
f (z) =
4 Re z, si Re z ≥ 0.

Determine el valor de la integral Z


f (z) dz,
C
donde C es el arco desde z = −1 − i hasta z = 1 + i, a lo largo de la curva y = x.

Solución. La parametrización de C es z(t) = t + it, −1 ≤ t ≤ 1. Ası́, considerando la


definición de f (z) y las propiedades de la integral de lı́nea, podemos escribir:
Z Z 1
f (z) dz = f (z(t)) z ′ (t) dt
C −1
Z 0 Z 1 
= (1 + i) dt + 4t dt = 3 + 3i.
−1 0

Problema 4.3. Sea f (z) = Log (z + 1). Determine la expresión analı́tica de la primitiva
Rz
F (z) = f (s) ds + c, en el dominio simplemente conexo
i−1

D = {z ∈ C : |z + 1| > 0, −π < arg (z + 1) < π} .

Solución. Las primitivas de f (z) = Log (z + 1), en el dominio D, tienen la forma

F (z) = (Log (z + 1) − 1)(z + 1) + c

La primitiva pedida debe satisfacer F (i − 1) = 0, en otras palabras,


π π
0 = F (i − 1) = (Log (i) − 1)i + c = − −i+c ⇒ c= + i.
2 2
Por lo tanto, la expresión analı́tica de la primitiva pedida es
π
F (z) = (Log (z + 1) − 1)(z + 1) + + i.
2


Problema 4.4. Sea f (z) una función de variable compleja definida como

f (z) = z i ≡ ei log z ,

donde log z = ln |z| + i arg z, con |z| > 0 y −π/2 < arg z < 3π/2. Determine el valor de la
integral Z
f (z) dz,
C
donde C es el arco de la circunferencia |z| = 1 desde z = −1 hasta z = 1 situado en el
semiplano superior.
130 4.8. PROBLEMAS RESUELTOS

Solución. La función f (z) = ei log z es analı́tica en el dominio simplemente conexo

D = {z ∈ C : |z| > 0, −π/2 < arg z < 3π/2}.

Por lo tanto, f (z) tiene primitiva en D dada por:

z i+1 e(i+1) log z


F (z) = ≡ + c,
i+1 i+1
además, la curva C está dentro de D. Luego, podemos escribir:
Z Z 1 " #1  0 
i e(i+1) log z e − e−π eπi 1 + e−π
f (z) dz = z dz = +c = = (1 − i).
C −1 i+1 i+1 2
−1

Problema 4.5. Determine el valor de la integral


Z
sen (z + 1)
34
dz.
|z+1|=1 (z + 1)

Solución. Sea f (z) = sen (z+1). Es claro que f (z) es analı́tica en la circunferencia |z+1| =
1 y en el interior de la misma; además, z0 = −1 es un punto interior de esta circunferencia.
Ası́, usando la extensión de la fórmula integral de Cauchy, podemos escribir:
Z
sen (z + 1) 2πi f (33) (−1)
34
dz = .
|z+1|=1 (z + 1) 33!

Hallemos f (33) (−1). Para ello utilizaremos el desarrollo de Taylor de f (z) centrado en
z0 = −1. Tal desarrollo tiene la forma

X
f (z) = an (z + 1)n , |z + 1| < ∞,
n=0

f (n) (−1)
donde an = n! . Ası́, f (33) (−1) = 33! a33 . Como

X (−1)n
f (z) = sen (z + 1) = (z + 1)2n+1 , |z + 1| < ∞,
n=0
(2n + 1)!

entonces f (33) (−1) = 1. Por lo tanto, el valor de la integral dada está dado por:
Z
sen (z + 1) 2πi
34
dz = .
|z+1|=1 (z + 1) 33!

Problema 4.6. Evalúe la integral


Z
(z + 1) cos(1/(z − 1))
dz.
|z|=2 (1 − z)2
CAPÍTULO 4. INTEGRACIÓN COMPLEJA 131

Solución. Sea f (z) = (z+1) (1−z)


cos(1/(z−1))
2 . Es claro que f (z) es analı́tica en la circunferencia
|z| = 2 y en el interior de la misma, salvo en el punto z0 = 1. Ası́, aplicando el teorema de
los residuos tenemos:
Z
(z + 1) cos(1/(z − 1))
dz = 2πi Res [f (z)] .
|z|=2 (1 − z)2 z=1

Como z0 = 1 es un punto singular esencial de f (z), entonces debemos hallar el desarrollo


de Laurent para obtener el residuo en tal punto. Se tiene que el desarrollo de Laurent de
f (z) centrado en z0 = 1, válido en el anillo |z − 1| > 0, está dado por:
 
1 1
f (z) = (z + 1) cos
(1 − z)2 1−z

1 X (−1)n
= (2 + (z − 1)) 2
(z − 1)−2n
(1 − z) (2n)!
n=0
∞ ∞
X 2(−1)n X (−1)n
= (z − 1)−2(n+1) + (z − 1)−2n−1 , |z − 1| > 0,
(2n)! (2n)!
n=0 n=0

en consecuencia, el residuo de f (z) en z0 = 1 es: Res [f (z)] = 1. Por lo tanto, el valor de


z=1
la integral dada es Z
(z + 1) cos(1/(z − 1))
dz = 2πi.
|z|=2 (1 − z)2


Problema 4.7. Evalúe la integral


Z
z3
z e1/(z+i) + dz.
|z|=3/2 (z + 2)2 (z − 1)

Solución. Por las propiedades de la integral tenemos:


Z Z Z
z3 z3
z e1/(z+i) + dz = z e1/(z+i) dz + dz.
(z + 2)2 (z − 1) (z + 2)2 (z − 1)
|z|=3/2 |z|=3/2 |z|=3/2

Sean f1 (z) y f2 (z) funciones definidas como:

z3
f1 (z) = z e1/(z+i) , f2 (z) = .
(z + 2)2 (z − 1)

Es claro que f1 (z) y f2 (z) son funciones analı́ticas en la circunferencia |z| = 3/2 y en el
interior de la misma, salvo en los puntos z0 = −i y z1 = 1 y z2 = −2, respectivamente.
Observamos que z2 = −2 no es punto interior de la circunferencia |z| = 3/2. Ası́, aplicando
el teorema de los residuos se tiene
Z Z
1/(z+i) z3
ze dz = 2πi Res [f1 (z)] , dz = 2πi Res [f2 (z)] .
z=z0 (z + 2)2 (z − 1) z=z1
|z|=3/2 |z|=3/2
132 4.8. PROBLEMAS RESUELTOS

Ahora, como z0 = −i es un punto singular esencial de f1 (z), entonces debemos hallar el


desarrollo de Laurent para calcular el residuo en tal punto. Se tiene que el desarrollo de
Laurent de f1 (z) centrado en z0 = −i, válido en el anillo |z + i| > 0, está dado por:

1/(z+i)
X 1
f1 (z) = z e = (−i + (z + i)) (z + i)−n
n=0
n!
∞ ∞
X −i −n
X 1
= (z + i) + (z + i)−n+1 , |z + i| > 0,
n! n!
n=0 n=0

luego,
1
Res [f1 (z)] = − i.
z=z0 2
Por otra parte, como z1 = 1 es un polo simple de f2 (z), se tiene que
 
z3 1
Res [f2 (z)] = = .
z=z1 (z + 2)2 z=1 9

En consecuencia, el valor de la integral dada es


Z    
1/(z+i) z3 1 1 π
ze + 2
dz = 2πi − i + 2πi = (18 + 11 i) .
(z + 2) (z − 1) 2 9 9
|z|=3/2

Problema 4.8. Calcular el valor de la integral


Z
(1 + z + z 2 )(e1/z + e1/(z−1) + e1/(z−2) ) dz,
C

donde C es un contorno cerrado simple que contiene en su interior a los puntos 0, 1 y 2.

Solución. Sean f1 (z), f2 (z) y f3 (z) funciones definidas como:

f1 (z) = (1 + z + z 2 ) e1/z , f2 (z) = (1 + z + z 2 ) e1/(z−1) , f3 (z) = (1 + z + z 2 ) e1/(z−2) .

Ası́,
f (z) = f1 (z) + f2 (z) + f3 (z),
luego,
Z Z Z Z
2 1/z 1/(z−1) 1/(z−2)
(1 + z + z )(e + e +e ) dz = f1 (z)dz + f2 (z)dz + f3 (z)dz.
C C C C

Ahora, los únicos puntos singulares de f1 (z), f2 (z) y f3 (z) son, respectivamente, 0, 1 y 2,
además, cada uno de ellos son puntos singulares esenciales, lo cual nos obliga a determinar
el desarrollo de Laurent para calcular el residuo en tales puntos. Se tiene que el desarrollo
de Laurent de f1 (z) centrado 0, válido en el anillo |z| > 0, está dado por
∞ ∞ ∞ ∞
2
X 1 −n X 1 −n X 1 1−n X 1 2−n
f1 (z) = (1 + z + z ) z = z + z + z ,
n! n! n! n!
n=0 n=0 n=0 n=0
CAPÍTULO 4. INTEGRACIÓN COMPLEJA 133

de donde se deduce que


1 1 10
+ = .
Res [f1 (z)] = 1 +
z=0 2 6 6
Asimismo, el desarrollo de Laurent de f2 (z) centrado 1, válido en el anillo |z − 1| > 0, está
dado por

X 1
2
f2 (z) = (3 + 3(z − 1) + (z − 1) ) (z − 1)−n
n!
n=0
∞ ∞ ∞
X 3 −n
X 3 1−n
X 1
= (z − 1) + (z − 1) + (z − 1)2−n ,
n=0
n! n=0
n! n=0
n!

que implica
3 1 28
Res [f2 (z)] = 3 +
+ = .
z=1 2 6 6
Ahora, el desarrollo de Laurent de f3 (z) centrado 2, válido en el anillo |z − 2| > 0, está
dado por

X 1
f3 (z) = (7 + 5(z − 2) + (z − 2)2 ) (z − 2)−n
n=0
n!
∞ ∞ ∞
X 7 X 5 X 1
= (z − 2)−n + (z − 2)1−n + (z − 2)2−n ,
n! n! n!
n=0 n=0 n=0

luego
5 1 58
+ = .
Res [f2 (z)] = 7 +
z=1 2 6 6
De esta forma, utilizando el Teorema de los Residuos obtenemos
Z
10
f1 (z)dz = πi,
3
ZC
28
f2 (z)dz = πi,
C 3
Z
58
f3 (z)dz = πi,
C 3
en consecuencia, el valor de la integral dada es
Z
10 28 58
(1 + z + z 2 )(e1/z + e1/(z−1) + e1/(z−2) ) dz = πi + πi + πi = 32πi.
C 3 3 3

 
(z + i)2 z
Problema 4.9. Sea f (z) = + z cos .
(z − i − 1)(ez−1 − 1) z+1

a) Halle el residuo de f (z) en cada uno de sus puntos singulares aislados.


Z
b) Determine el valor de la integral f (z) dz , donde C es el contorno que se muestra
C
en la siguiente figura. El punto inicial y el fine de C coinciden en α = 2.
134 4.8. PROBLEMAS RESUELTOS

2 C

x
−2 −1 1 2
−1

Solución. a) Los puntos singulares de f (z) son: z0 = 1, z1 = 1 + i, z2 = −1. Como z0 y z1


son polos simples de la función

(z + i)2
h(z) =
(z − i − 1)(ez−1 − 1)

y la función  
z
g(z) = z cos
z+1
es analı́tica en z0 y z1 , entonces el residuo de f (z) en los puntos z0 = 0 y z1 = 1 es:
 
(z + i)2 (z − 1)(z + i)2
Res [f (z)] = Res = lı́m = −2
z=z0 z=z0 (z − i − 1)(ez−1 − 1) z→1 (z − i − 1)(ez−1 − 1)
 
(z + i)2 (z + i)2 −3 + 4i
Res [f (z)] = Res = lı́m = i
z=z1 z=z1 (z − i − 1)(ez−1 − 1) z→1+i (e z−1 − 1) e −1

El residuo de f (z) en z2 = −1, está dado por:


  
z
Res [f (z)] = Res z cos .
z=z2 z=z2 z+1
 
z
Ahora, el desarrollo de Laurent de g(z) = z cos centrado en z2 = −1 es, para
z+1
|z + 1| > 0:
 
(z + 1) − 1
g(z) = ((z + 1) − 1) · cos
z+1
    
1 1
= ((z + 1) − 1) cos(1) cos + sen (1)sen
z+1 z+1
∞ n ∞
X (−1) X (−1)n
= cos(1) (z + 1)−2n+1 + sen (1) (z + 1)−2n
n=0
(2n)! n=0
(2n + 1)!
∞ ∞
X (−1)n+1 X (−1)n+1
+ cos(1) (z + 1)−2n + sen (1) (z + 1)−(2n+1) .
(2n)! (2n + 1)!
n=0 n=0

Por tanto, z2 es un punto singular esencial de g(z) y

1 1
Res [g(z)] = − cos(1) − sen (1) ⇒ Res [f (z)] = − cos(1) − sen (1)
z=z2 2 z=z2 2
CAPÍTULO 4. INTEGRACIÓN COMPLEJA 135

b) Es claro que C no es un contorno cerrado simple, y


pero éste se puede escribir como la unión de dos con-
2 C2
tornos cerrados simples: C1 (trazado de color azul re-
corrido en sentido negativo) y C2 (trazado de color
rojo recorrido en sentido positivo), que se muestran C1 1
en la figura adjunta. Luego,
Z Z Z
x
f (z) dz = f (z) dz + f (z) dz. −2 −1 1 2
C C1 C2 −1

Los puntos singulares de f (z), z0 = 1 y z1 = 1 + i, son puntos interiores del contorno C2 .


Entonces, por el Teorema de los Residuos se tiene
Z    
−3 + 4i
f (z) dz = 2πi Res [f (z)] + Res [f (z)] = 2πi −2 + i .
C2 z=z0 z=z1 e −1

Ahora, el punto singular de f (z), z2 = −1, es un punto interior del contorno C1 . De esta
forma, por el Teorema de los Residuos se tiene
Z  
1
f (z) dz = −2πi Res [f (z)] = 2πi cos(1) + sen (1) .
C1 z=z2 2

De todo lo anterior se obtiene que el valor de la integral es:


Z  
1 −3 + 4i
f (z) dz = 2πi −2 + cos(1) + sen (1) + i .
C 2 e −1

4.9 Problemas Propuestos


Z
4.1. Calcule la integral 1/z dz si la curva C es:
C

a) el segmento de recta que va de z = i a z = 1;


b) la semicircunferencia |z| = 1, −π/2 ≤ arg z ≤ π/2 (el camino se inicia en el
punto z = −i).

4.2. Evalúe la integral


Z
z3
dz,
C z2
donde C es la circunferencia |z| = 1 tomada en sentido positivo.

4.3. Evalúe la integral Z


Log z dz,
C

donde C es la curva z(t) = eit , 0 ≤ t ≤ π.


136 4.9. PROBLEMAS PROPUESTOS

4.4. Evalúe la integral Z


f (z) dz,
|z|=2

donde f (z) es una función de variable compleja definida por


 √

0, si Re z < − 2;


z 2 , √
si − 2 ≤ Re z ≤ 0;
f (z) = √

z, si 0 < Re z ≤ 2;


0, √
si Re z > 2.

4.5. Para cada una de las siguientes integrales, diga las caracterı́sticas del contorno
cerrado simple C para que el valor de la integral sea cero, según el Teorema de
Cauchy-Goursat. (Justifique su respuesta.)
Z Z
cos z 1
a) dz c) dz
C z+2 C 1 + ez
Z Z
1
b) Log z dz d) dz
C C 1 − ez

1
4.6. Para la función de f (z) = determine la primitiva de f (z) tal que:
(z − i)2

a) esté definida en el dominio Re z > 0.


b) sea igual a cero cuando z = i + 1.

4.7. Evalúe las siguientes integrales. Use la fórmula integral de Cauchy (o su ex-
tensión) o bien el Teorema de Cauchy-Goursat donde sea necesario. En cada
una de las integrales, C es un contorno cerrado simple.
Z
z
a) 4
dz, a > 1.
|z−a|=a z − 1

Z
1 x2 y2
b) dz, donde C es el contorno + = 1.
C ez (z − 2) 9 16
Z
sen (ez + cos z) x2
c) dz, donde C es el contorno + y 2 = 1.
C (z − 1)2 (z + 3) 2
Z
cos(2z)
d) dz.
|z|=1 z 21
Z
ez
e) dz, donde C es un contorno cerrado simple que contiene a |z| ≤ a.
C z 2 + a2
Z
z ez
f) dz, donde a es un punto interior del contorno cerrado simple C.
C (z − a)3
CAPÍTULO 4. INTEGRACIÓN COMPLEJA 137

4.8. En cada una de las siguientes integrales determine el mı́nimo valor de la constante
positiva M que satisface Z
f (z) dz ≤ M,
C
para cada función f (z) considerada en cada caso.
Z
1
a) dz
z
|z|=1
Z
1
b) dz
z2 +1
|z−i|=1/2
Z
1
c) dz
ez −1
|z|=2
Z
z
d) dz, donde C está conformado con los puntos del cuadrado con vértices
C z−i
en los puntos −1 + 2i, 1 + 2i, 1 y −1.

4.9. Determine los residuos de las siguientes funciones complejas en cada polo o en el
polo indicado:
2z + 1 z+1
a) f (z) = . e)
z2 −z−2 (z − 1)2 (z + 3)

cos z
1 f) , z0 = 0
b) f (z) = 2 . z
z (1 − z)
z4 − 1 π
g) 4
, z0 = e 4 i
3z 2 + 2 z +1
c) f (z) = .
(z − 1)(z 2 + 9) sen z π
h) , z0 = e 3 i
z4 2
+z +1
z3 − z2 + z + 1 z
d) i) , z0 = π
z 2 + 4z sen z

4.10. Use el Teorema de los Residuos para evaluar las siguientes integrales:
Z
z
a) dz, para
z2 + 1
C (
i) |z| = 1/2
C=
ii) |z| = 2

Z
z 2 + 3i − 2
b) dz, para
z 3 + 9z
C (
i) |z| = 1
C=
ii) |z| = 4
138 4.9. PROBLEMAS PROPUESTOS

Z
(z 2 + 2)(z 2 + 4)
c) dz para
(z 2 + 1)(z 2 + 6)
C


 i) |z| = 2

C= ii) |z − i| = 1



iii) |z| = 4

Z
1
d) dz, para
z 2 (1 + z 2 )2
C
(
i) |z| = 1/2
C=
ii) |z| = 2

Z
3z 2 + 2
e) dz, para
(z 2 + 4)(z − 1)
C
(
i) |z − 2| = 2
C=
ii) |z| = 4

Z
z 2 − 2z
f) dz, para
(z 2 + 4)(z − 1)2
C
(
i) |z| = 3
C=
ii) |z + i| = 2

Z
1
g) dz, para
(z + 1)3 (z − 1)(z − 2)
C
(
i) |z + 1| = 1
C=
ii) es el rectángulo de vértices en ± i, 3 ± i
Parte II

Cálculo Operacional

139
5
Funciones de Dominios Continuo y
Discreto
Los conceptos de de funciones de dominio continuo (señales de tiempo continuo) o
funciones de dominio discreto (señales de tiempo discreto) surgen en casi cualquier área
de las ciencias e ingenierı́a, en los circuitos eléctricos, dispositivos para comunicaciones,
robótica y automatización, dispositivos biomédicos, procesos quı́micos, entre otros. En este
capı́tulo se da una introducción al cálculo operacional, comenzando con la caracterización
de las funciones de dominio continuo, pasando luego por la convolución en el dominio
continuo. En esta parte se estudia con interés la función impulso unitario. Finalmente,
se describen las funciones de dominio discreto y se define la convolución de funciones de
dominio discreto.

5.1 Funciones de Dominio Continuo


Definición 5.1. Una función f que depende de una variable t se dice de dominio con-
tinuo, si t toma valores en el conjunto de los números reales; en otras palabras, toda
función definida en el conjunto de números reales es una función de dominio continuo.

Ejemplo 5.1. Las siguientes funciones son de dominio continuo:

• f (t) = et , para todo t ∈ R.

• f (t) = sen (t), para todo t ∈ R.

• f (t) = cos(t), para todo t ∈ R.


(
0, t < 0;
• f (t) =
1, t > 0.

Note que normalmente en los libros textos de señales y sistemas, a una función de
dominio continuo se le denomina señal continua o señal en tiempo continuo. Tal deno-
minación corresponde a que la representación matemática de una señal continua es una
función definida en el conjunto de números reales. Recuerde que una señal es cualquier
fenómeno o magnitud fı́sica que puede ser representado de manera cuantitativa. En este
material no se usa el concepto de señal, pero el lector debe tener presente la relación que
existe entre señal en tiempo continuo y funciones de dominio continuo. A continuación se
describen las funciones de dominio continuo de uso más frecuente.

140
CAPÍTULO 5. FUNCIONES DE DOMINIOS CONTINUO Y DISCRETO 141

5.1.1 Impulso Unitario

El impulso unitario o delta de Dirac no es una función ordinaria. El nombre de Dirac


es en honor a Paul Adrien Maurice Dirac (1902-1984), ingeniero eléctrico, matemático y
fı́sico teórico británico. Las funciones ordinarias se especifican definiendo la relación para
obtener el valor de la función (un número) para cada valor de su argumento, en algún
conjunto especı́fico. Pero, el impulso unitario se define por el efecto que produce en la in-
teracción con una función ordinaria de dominio continuo. Hay muchos fenómenos fı́sicos
que se pueden modelar como funciones delta, por ejemplo, fuentes puntuales, cargas pun-
tuales, cargas concentradas en estructuras, fuentes puntuales de tensión y de corriente que
actúan durante intervalos de tiempo extremadamente cortos. Matemáticamente la función
impulso se define de la siguiente manera.

Definición 5.2. La función impulso unitario es una función generalizada o distribución,


denotada con δ(t), que satisface:
Z ∞
ϕ(t)δ(t) dt = ϕ(0),
−∞

para toda función ϕ(t) continua en R.

En la Figura 5.1, se aprecia la representación gráfica de δ(t). Por supuesto, el impulso


unitario no es la “flecha vertical con altura 1”, sino que esto es simplemente la represen-
tación usual de δ(t), que no puede ser justificada matemáticamente. Esto se usa como una
ayuda nemotécnica.

δ(t)

Figura 5.1. Representación gráfica de δ(t)

La representación gráfica de la función

f (t) = A δ(t − t0 ), con A ∈ R y t0 ∈ R,

se aprecia en la Figura 5.2. Aquı́, A δ(t − t0 ) se representa como una flecha vertical con
altura A, cuyo extremo inicial está localizado en t0 .
142 5.1. FUNCIONES DE DOMINIO CONTINUO

Aδ(t − t0 ) Aδ(t − t0 )

t0 A
t

t0 t
A
a) A < 0 b) A > 0
Figura 5.2. Representación gráfica de A δ(t − t0 ). a) A < 0; b) A > 0.

Propiedades del impulso unitario


Seguidamente describimos algunas propiedades de la función δ(t).

i) lı́m δ(t) = ∞.
t→0

ii) δ(t) = 0, para todo t 6= 0.


Z ∞
iii) δ(t) dt = 1.
−∞

iv) δ(t) = δ(−t), en otras palabras, δ(−t) se comporta como δ(t) (si δ(t) se toma como
una función ordinaria, esta propiedad indicarı́a que el impulso unitario es una
función par).

v) Desplazamiento. Si ϕ(t) es continua en t0 ∈ R, entonces


Z ∞
ϕ(t)δ(t − t0 ) dt = ϕ(t0 ).
−∞

Esta propiedad también se puede aplicar en un intervalo [t1 , t2 ],


Z t2 (
ϕ(t0 ), si t0 ∈ [t1 , t2 ],
ϕ(t)δ(t − t0 ) dt =
t1 0, si t0 6∈ [t1 , t2 ].

vi) Muestreo. Si ϕ(t) es continua en t0 ∈ R, entonces

ϕ(t) δ(t − t0 ) = ϕ(t0 ) δ(t − t0 ).

vii) Escalado. Para a ∈ R se cumple


1
δ(at) = δ(t).
|a|

Particularmente, para a, t0 ∈ R se cumple


 
1 t0
δ(at − t0 ) = δ t− .
|a| a
CAPÍTULO 5. FUNCIONES DE DOMINIOS CONTINUO Y DISCRETO 143

viii) Derivada enésima. Si ϕ(t) es n veces continuamente diferenciable en t0 ∈ R, en-


tonces Z ∞
ϕ(t)δ(n) (t − t0 ) dt = (−1)n ϕ(n) (t0 ).
−∞
Esta propiedad también se puede aplicar en un intervalo [t1 , t2 ],
Z t2 (
(−1)n ϕ(n) (t0 ), si t0 ∈ [t1 , t2 ],
ϕ(t)δ(n) (t − t0 ) dt =
t1 0, si t0 6∈ [t1 , t2 ].

Ejercicio 5.1. Demostrar cada una de las propiedades del impulso unitario.
Ejemplo 5.2. Evalúe las siguientes expresiones:
a) 3t4 δ(t − 1)
b) tδ(3 − 2t)
Z ∞
c) sen (t2 ) δ′ (t − π 1/2 ) dt
−∞
Solución. a) Aplicando la propiedad de muestreo se tiene:
3t4 δ(t − 1) = [3t4 ] t=1
δ(t − 1) = 3δ(t − 1).
b) Se tiene que   
3
tδ(3 − 2t) = t δ −2 t − .
2
Ahora, aplicando la propiedad de escalado seguido de la propiedad de muestreo, se ob-
tiene:
       
1 3 t 3 3 3
tδ(3 − 2t) = t δ t− = δ t− = δ t− .
| − 2| 2 2 t=3/2 2 4 2
c) Aplicando la propiedad de la derivada n-ésima se tiene:
Z ∞  
2 ′ 1/2 d 2
  √
sen (t )δ (t − π ) dt = (−1) sen (t ) = (−1) 2t cos(t2 ) t=π 1/2
= 2 π.
−∞ dt t=π 1/2


5.1.2 Escalón Unitario


La función escalón unitario o función escalón de Heaviside es una función cuyo valor es 0
para cualquier argumento negativo y es 1 si el argumento es positivo. El nombre Heaviside
es en honor a Oliver Heaviside (1850-1925), fı́sico, ingeniero eléctrico, radiotelegrafista y
matemático inglés. Matemáticamente el escalón unitario se define de la siguiente forma.

Definición 5.3. El escalón unitario, denotado por u(t), se define como


(
0, t < 0,
u(t) =
1, t > 0,

La representación gráfica de u(t) se aprecia en la Figura 5.3. La función u(t − t0 )


representa un desplazamiento en tiempo de la función escalón unitario, para t0 ∈ R, lo cual
se muestra con un ejemplo gráfico en la Figura 5.4.
144 5.1. FUNCIONES DE DOMINIO CONTINUO

u(t)
1

Figura 5.3. Representación gráfica de u(t)

u(t − t0 )

t0 t

Figura 5.4. Representación gráfica de u(t − t0 )

5.1.3 Pulso Rectangular

A partir de la definición del escalón unitario se pueden definir otras funciones básicas
de gran utilidad en análisis de sistemas, como lo es el pulso rectangular. Un pulso rectangu-
lar de duración 2T con T > 0, se puede concebir como una función que escoge dos valores
especı́ficos: inicialmente el pulso rectangular es igual a cero hasta cierto argumento −T ,
luego cambia abruptamente a 1 manteniéndose en este valor hasta un argumento T , para
luego cambiar su valor a 0. Matemáticamente, la definción del pulso rectangular es la
siguiente.

Definición 5.4. El pulso rectangular, denotado por pT (t), se define para T > 0 como
(
1, |t| < T,
pT (t) =
0, |t| > T.

La representación gráfica de pT (t) se muestra en la Figura 5.5.

pT (t)
1

−T T t

Figura 5.5. Representación gráfica de pT (t)

El pulso rectangular se puede escribir como una suma de dos funciones escalones:

pT (t) = u(t + T ) − u(t − T ).


CAPÍTULO 5. FUNCIONES DE DOMINIOS CONTINUO Y DISCRETO 145

En efecto, las funciones u(t + T ) y u(t − T ) están definidas respectivamente por:


( (
0, t < −T, 0, t < T,
u(t + T ) = y u(t − T ) =
1, t > −T, 1, t > T,
luego, se sigue que
u(t + T ) − u(t − T ) = pT (t).

5.1.4 Pulso Triangular


El pulso triangular tiene un comportamiento similar al pulso rectangular pT (t). El
pulso triangular inicialmente es cero hasta llegar a −T , luego toma el valor de una recta
con pendiente T −1 hasta llegar a 0, luego toma el valor de una recta con pendiente −T −1
hasta llegar a T , para luego cambiar su valor a 0. Matemáticamente, el pulso triangular se
define de la siguiente forma.

Definición 5.5. El pulso triangular, denotado por qT (t), se define para T > 0 como

 |t|
1 − , |t| ≤ T,
qT (t) = T
0, |t| > T,

La representación gráfica de qT (t) se muestra en la Figura 5.6.

qT (t)
1

−T T t

Figura 5.6. Representación gráfica de qT (t)

El pulso triangular se puede expresar a través de funciones escalones o pulsos rectan-


gulares, multiplicadas por una recta. He aquı́ algunas representaciones útiles.
   
t t
pT (t) = 1 + (u(t + T ) − u(t)) + 1 − (u(t) − u(t − T ))
T T
o        
t T t T
pT (t) = 1+ pT /2 t+ + 1− pT /2 t−
T 2 T 2
Se deja como ejercicio para el lector demostrar que las representaciones anteriores son
ciertas.

5.1.5 Función Signo


La función signo adquiere el signo del argumento, es decir, ella adquiere el valor −1
cuando el argumento es negativo y es igual a 1 si el argumento es positivo; en cero la
función signo no está definida, por supuesto, ya que el cero no tiene signo. La definición
matemática de la función signo es la siguiente.
146 5.1. FUNCIONES DE DOMINIO CONTINUO

Definición 5.6. La función signo, denotada por sgn(t), se define como


(
−1, t < 0,
sgn(t) =
1, t > 0.

La representación gráfica de sgn(t) se aprecia en la Figura 5.7. La definición de sgn(t)


permite inferir que ella se puede expresar como la suma de dos funciones escalón, a saber:

sgn(t) = u(t) − u(−t) o, equivalentemente, sgn(t) = −1 + 2 u(t).

sgn(t)

t
−1

Figura 5.7. Representación gráfica de sgn(t)

También el escalón unitario se puede definir utilizando la función signo:


1 1
u(t) = + sgn(t).
2 2

5.1.6 Pulso Exponencial


El pulso exponencial se obtiene como la multiplicación de una función exponencial por
el escaló unitario. Matemáticamente, el pulso exponencial se define de la siguiente forma.

Definición 5.7. El pulso exponencial, denotado por Exp (t), se define, para A > 0 y
α > 0, como (
0, t<0
Exp (t) = −αt
A e , t ≥ 0,

La representación gráfica de Exp (t) se aprecia en la Figura 5.8.

Exp (t)
A

Figura 5.8. Representación gráfica de Exp (t)


CAPÍTULO 5. FUNCIONES DE DOMINIOS CONTINUO Y DISCRETO 147

5.1.7 Función Rampa

La función rampa tiene un comportamiento similar al escalón unitario. La función


rampa inicialmente es cero hasta llegar a 0, luego toma el valor de la recta t. La función
rampa se puede ver como la multiplicación de la función t por el escaló unitario.

Definición 5.8. La función rampa, denotada por r(t), se define como


(
0, t < 0,
r(t) =
t, t ≥ 0,

La representación gráfica de r(t) se aprecia en la Figura 5.9.

r(t)
2

1 2 t

Figura 5.9. Representación gráfica de r(t)

Es claro que la función rampa se puede expresar como:

r(t) = tu(t).

Además, de las definiciones de r(t) y u(t) se obtienen las siguientes relaciones:

Z t
d
r(t) = u(τ ) dτ y u(t) = r(t).
−∞ dt

5.1.8 Funciones Sa y sinc

La función de muestreo es de mucha utilidad en análisis de sistemas y, particularmente,


en el muestreo de señales. La definición matemática de la función de muestreo es la
siguiente.

Definición 5.9. La función de muestreo, denotada por Sa (t), se define como



1, t = 0,
Sa (t) = sen t
 , t 6= 0,
t

La representación gráfica de Sa (t) se aprecia en la siguiente figura.


148 5.1. FUNCIONES DE DOMINIO CONTINUO

Sa (t)

t
−6π −5π −4π −3π −2π −π π 2π 3π 4π 5π 6π

Figura 5.10. Representación gráfica de Sa (t)

La función seno cardinal o simplemente función sinc está estrechamente relacionada


con la función de muestreo. La definición matemática de la función seno cardinal es la
siguiente.

Definición 5.10. La función sinc (t) está definida como



1, t = 0,
sinc (t) = sen (πt)
 , t 6= 0.
πt

Es claro que sinc (t) = Sa (πt). La representación gráfica de sinc (t) se muestra en la
siguiente figura.

sinc (t)

t
−6 −5 −4 −3 −2 −1 1 2 3 4 5 6

Figura 5.11. Representación gráfica de sinc (t)

5.1.9 Relación entre u(t) y δ(t)


Las funciones escalón unitario e impulso unitario están relacionadas de la siguiente
manera: Z t
d
u(t) = δ(τ ) dτ y δ(t) = u(t).
−∞ dt
Demostremos estas relaciones. Primero verifiquemos que la integral
Z t
h(t) = δ(τ ) dτ
−∞

se comporta como el escalón unitario, en otras palabras, verificaremos que h(t) = 0, para
t < 0 y h(t) = 1, para t > 0. Como δ(t) = 0 para todo t 6= 0, entonces es claro que h(t) = 0,
CAPÍTULO 5. FUNCIONES DE DOMINIOS CONTINUO Y DISCRETO 149

para t < 0. Ahora, para t > 0 podemos escribir:


Z t Z 0− Z 0+ Z t
h(t) = δ(τ ) dτ = δ(τ ) dτ + δ(τ ) dτ + δ(τ ) dτ,
−∞ −∞ 0− 0+

pero δ(t) = 0 para todo t 6= 0 y


Z 0+
δ(τ ) dτ = 1,
0−
luego
h(t) = 1, para t > 0.
Por lo tanto, todo lo anterior nos indica que h(t) se comporta como el escalón unitario, es
decir, Z t
u(t) = δ(τ ) dτ.
−∞

Ahora, derivando en ambos lados de la ecuación anterior se tiene que


d
δ(t) = u(t).
dt

5.1.10 Derivada Generalizada


La expresión de la derivada generalizada de una función se deduce del hecho de que
toda función diferenciable a trozos o, en general, toda función continua a trozos, puede
expresarse como una suma de funciones que involucran escalones unitarios.

Definición 5.11. Sea f (t) una función diferenciable a trozos en R, esto es, f (t) es
derivable en todo R excepto en t1 , t2 , . . . , tn ∈ R, además, en cada uno de estos puntos
f (t) tiene saltos h1 , h2 , . . . , hn ∈ R, respectivamente. La derivada generalizada de f (t)
se define como
n
′ d X
f (t) = f (t) + hk δ(t − tk ). (5.1)
dt t6=tk k=1

Observe el siguiente ejemplo.

Ejemplo 5.3. Sea f (t) la función dada por




 0, t ≤ −1,

 −t −1 < t ≤ 0,
e ;


f (t) = t, 0 < t ≤ 2,




 3, 2 < t ≤ 3,


1, t > 3.

cuya gráfica se muestra en la Figura 5.12.


Note que f (t) no es diferenciable en los puntos t1 = −1, t2 = 0, t3 = 2 y t4 = 3, con
saltos h1 = e, h2 = −1, h3 = 1 y h4 = −2, respectivamente. Ası́, por (5.1) la derivada
generalizada de f (t) es

f ′ (t) = g(t) + e δ(t + 1) − δ(t) + δ(t − 2) − 2δ(t − 3), (5.2)


150 5.1. FUNCIONES DE DOMINIO CONTINUO

f (t)
3

-1 1 2 3 4 t

Figura 5.12. Representación gráfica de f (t)

donde

−t
−e , −1 < t < 0,

g(t) = 1, 0 < t < 2,


0, en otros casos.

Determinemos la derivada generalizada de f (t) por otro procedimiento, usando el he-


cho que f (t) puede expresarse equivalentemente como:

f (t) = e−t (u(t + 1) − u(t)) + t(u(t) − u(t − 2)) + 3(u(t − 2) − u(t − 3)) + u(t − 3).

Recuerde la regla Ahora, derivando la expresión anterior se tiene:


de derivación del
producto

d  −t  d d
f ′ (t) = e (u(t + 1) − u(t)) + [t(u(t) − u(t − 2))] + [3(u(t − 2) − u(t − 3))]
dt dt dt
d
+ u(t − 3)
dt
= − e−t (u(t + 1) − u(t)) + e−t (δ(t + 1) − δ(t)) + (u(t) − u(t − 2))
+ t(δ(t) − δ(t − 2)) + 3(δ(t − 2) − δ(t − 3)) + δ(t − 3)

= − e−t (u(t + 1) − u(t)) + e δ(t + 1) − δ(t) + (u(t) − u(t − 2)) − 2 δ(t − 2)


+ 3 δ(t − 2) − 3 δ(t − 3) + δ(t − 3)

= − e−t (u(t + 1) − u(t)) + (u(t) − u(t − 2)) + e δ(t + 1) − δ(t) + δ(t − 2) − 2δ(t − 3),

que es equivalente a la expresión dada en (5.2). 


CAPÍTULO 5. FUNCIONES DE DOMINIOS CONTINUO Y DISCRETO 151

5.1.11 Convolución en el Dominio Continuo


La convolución es una operación entre funciones de dominio continuo de mucha utili-
dad en aplicaciones en ingenierı́a y, especialmente, en análisis de sistemas. La definición
matemática de la convolución de funciones de dominio continuo es la siguiente.

Definición 5.12. Sean f y g dos funciones de dominio continuo. La convolución entre f


y g es una función de dominio continuo, denotada por f ∗ g, la cual está definida como:
Z ∞
f ∗ g (t) = f (τ ) g(t − τ ) dτ, para t ∈ R.
−∞

En la siguiente proposición se establecen algunas propiedades de la convolución.

Proposición 5.1. Sean f , g y h funciones de dominio continuo. Entonces, las siguientes


propiedades se cumplen.

i) f ∗ g = g ∗ f . (Conmutatividad)

ii) f ∗ (g + h) = f ∗ g + f ∗ h. (Distributiva con respecto a la suma)

iii) Si h(t) = f ∗ g(t) es la convolución de las funciones de dominio continuo f (t) y g(t),
entonces
f (t − t0 ) ∗ g(t) = h(t − t0 ).

Demostración. i) Se tiene que


Z ∞
f ∗ g(t) = f (τ ) g(t − τ ) dτ ;
−∞

ahora, haciendo el cambio de variable λ = t − τ se obtiene:


Z −∞ Z ∞
f ∗ g(t) = − f (t − λ) g(λ) dλ = f (t − λ) g(λ) dλ = g ∗ f (t).
∞ −∞

Demostración de ii). Se tiene que


Z ∞
f ∗ (g + h)(t) = f (τ ) [g(t − τ ) + h(t − τ )] dτ
−∞
Z ∞ Z ∞
= f (τ ) g(t − τ ) dτ + f (τ ) h(t − τ ) dτ
−∞ −∞
= f ∗ g(t) + f ∗ h(t).

Demostración de iii). Sean h(t) = f ∗ g(t) y t0 . Por definición, la convolución f (t − t0 ) ∗ g(t)


está dada por: Z ∞
f (t − t0 ) ∗ g(t) = f (τ − t0 ) g(t − τ ) dτ ;
−∞
ahora, haciendo el cambio de variable λ = τ − t0 se obtiene:
Z ∞
f (t − t0 ) ∗ g(t) = f (λ) g((t − t0 ) − λ) dλ = h(t − t0 ).
−∞

Esto completa la demostración.


152 5.1. FUNCIONES DE DOMINIO CONTINUO

En los siguientes ejemplos se calcula la convolución entre dos funciones de dominio


continuo.

Ejemplo 5.4. Sean f (t) = p1 (t) y g(t) = u(t). Determinar la convolución f ∗ g.


Solución. Tomemos h(t) = f ∗ g (t). Considerando las expresiones de f (t) y g(t), se puede
escribir:
Z ∞
h(t) = f ∗ g (t) = p1 (τ ) u(t − τ ) dτ
−∞
Z 1−
= u(t − τ ) dτ, para cada t ∈ R. (5.3)
−1+

Determinemos la expresión de h(t) para: t ≤ −1, −1 < t < 1 y t ≥ 1.

• Para t ≤ −1, la gráfica de la función u(t − τ ) es

u(t − τ )

-2 t -1 1 2 τ

Entonces, por (5.3), h(t) = 0, para t ≤ −1.

• Para −1 < t < 1, la gráfica de u(t − τ ) es

u(t − τ )

-2 -1 t 1 2 τ

Z t
Entonces, por (5.3), h(t) = dτ = t + 1, para −1 < t < 1.
−1+

• Para t ≥ 1, la gráfica de u(t − τ ) es

u(t − τ )
1

-2 -1 1 t 2 τ

Z 1−
Entonces, por (5.3), h(t) = dτ = 2, para t ≥ 1.
−1+
CAPÍTULO 5. FUNCIONES DE DOMINIOS CONTINUO Y DISCRETO 153

De esta forma, la expresión analı́tica de la convolución entre f y g está dada por



0,
 t ≤ −1,
h(t) = t + 1, −1 < t < 1,


2, t ≥ 1,

cuya representación gráfica es


h(t)
2

-2 -1 1 2 t

Ejemplo 5.5. Sean f y g las funciones de dominio continuo cuyas gráficas se muestran en
la siguiente figura.
f (t) g(t)

1 1

-1 1 t -1 1 t

Determinar la convolución f ∗ g.
Solución. Tomemos h(t) = f ∗ g (t). Se tiene que f y g se pueden expresar como

f (t) = (1 − t)(u(t) − u(t − 1)) y g(t) = u(t) − u(t − 1).

Luego, considerando la expresión de f (t), se puede escribir, para cada t ∈ R:


Z ∞ Z 1−
h(t) = f ∗ g (t) = f (τ ) g(t − τ ) dτ = (1 − τ )g(t − τ ) dτ. (5.4)
−∞ 0+

Además, para cada t ∈ R, la función g(t − τ ) se puede escribir como

g(t − τ ) = u(t − τ ) − u((t − 1) − τ ), para τ ∈ R.

Determinemos la expresión de h(t) para: t ≤ 0, 0 < t < 1, 1 ≤ t < 2 y t ≥ 2.

• Para t ≤ 0, la gráfica de g(t − τ ) es

g(t − τ )
1

t−1 t 1 2 τ
154 5.1. FUNCIONES DE DOMINIO CONTINUO

Entonces, por (5.4), h(t) = 0, para t ≤ 0.

• Para 0 < t < 1, la gráfica de g(t − τ ) es

g(t − τ )
1

t−1 t 1 2 τ

Entonces, por (5.4)


Z t  t
(τ − 1)2 1 
h(t) = (1 − τ ) dτ = − = 1 − (t − 1)2 , para 0 < t < 1.
0+ 2 0 2

• Para 1 ≤ t < 2, la gráfica de g(t − τ ) es

g(t − τ )
1

t−1 1 t 2 τ

Entonces, por (5.4)


Z 1−  1
(τ − 1)2 (t − 2)2
h(t) = (1 − τ ) dτ = − = , para 1 ≤ t < 2.
t−1 2 t−1 2

• Para t ≥ 2, la gráfica de g(t − τ ) es

g(t − τ )
1

1t−12 t τ

Entonces, por (5.4), h(t) = 0, para t ≥ 2.

De esta forma, la expresión analı́tica de la convolución h(t) = f ∗ g (t) está dada por:


 0, t ≤ 0,


 
 1 1 − (t − 1)2 , 0 < t < 1,

2
h(t) =
1 2
2 (t − 2) , 1 ≤ t < 2,







0, t ≥ 2,
cuya representación gráfica se aprecia en la siguiente figura.
CAPÍTULO 5. FUNCIONES DE DOMINIOS CONTINUO Y DISCRETO 155

h(t)

1
1/2

-3 -2 -1 1 2 3 t

5.2 Funciones de Dominio Discreto


Una función f que depende de una variable n se dice de dominio discreto, si n toma
valores en el conjunto de los números enteros; en otras palabras, toda función definida en
el conjunto de números enteros es una función de dominio discreto. Una de las formas
más comunes en las que surgen funciones de dominio discreto es discretizando funciones
de dominio continuo. Sean t0 , t1 , . . . .
La función
g(n) = f (tn )
Ejemplo 5.6. Seguidamente mostramos funciones de dominio discreto que se obtienen se denomina dis-
respectivamente como la discretización de las funciones de dominio continuo: f (t) = et , cretización de la
función f (t)
f (t) = sen (t), f (t) = ln(t), f (t) = t2 y u(t).

• f (n) = en , para todo n ∈ Z.

• f (n) = sen (n), para todo n ∈ Z.

• f (n) = ln(n), para todo n ≥ 0.

• f (n) = n2 , para todo n ∈ Z.


(
0, n < 0,
• f (n) =
1, n ≥ 0.

5.2.1 Impulso Unitario Discreto


El impulso unitario discreto es la contraparte discreta del impulso unitario continuo,
pero el impulso unitario discreto no puede verse como la discretización de δ(t), ya que
δ(0) no está definido.

Definición 5.13. El impulso unitario discreto, denotado por δ(n), se define como
(
1, n = 0,
δ(n) =
0, n 6= 0,

La gráfica de δ(n) se aprecia en la Figura 5.13.


156 5.2. FUNCIONES DE DOMINIO DISCRETO

δ(n)
b
1

b b b b b b b b

-4 -3 -2 -1 1 2 3 4 n

Figura 5.13. Gráfica de δ(n)

5.2.2 Escalón Unitario Discreto


El escalón unitario discreto puede obtenerse discretizando el impulso unitario, a pesar
de que u(0) no está definido. En este caso, el escalón unitario discreto alcanza el valor 1
en 0, es decir, toma el valor del lı́mite por la derecha de u(t) cuando t tiende a 0.

Definición 5.14. El escalón unitario discreto, denotado por u(n), se define como
(
0, n < 0,
u(n) =
1, n ≥ 0,

La gráfica de u(n) se muestra en la Figura 5.14.

u(n)
b b b b b
1
···
b b b b

-4 -3 -2 -1 1 2 3 4 n

Figura 5.14. Gráfica de u(n)

5.2.3 Función Rampa Discreta


La función rampa discreta se obtiene discretizando la función rampa continua.

Definición 5.15. La función rampa discreta, denotada por r(n), se define como
(
0, n < 0,
r(n) =
n, n ≥ 0,

La gráfica de r(n) se aprecia en la Figura 5.15. Las funciones escalón unitario discreto
y la rampa discreta están relacionadas de la siguiente manera:
n
X
r(n) = u(k) y u(n) = r(n) − r(n − 1).
k=−∞

Se deja como ejercicio para el lector demostrar estas relaciones.


CAPÍTULO 5. FUNCIONES DE DOMINIOS CONTINUO Y DISCRETO 157

r(n)
b
3

b
2
···
b
1

b b b b b

-4 -3 -2 -1 1 2 3 n

Figura 5.15. Gráfica de r(n)

5.2.4 Función Signo Discreto

La función signo discreto puede obtenerse discretizando de la función signo de dominio


continuo, a pesar de que sgn(0) no está definido. En este caso la función signo discreto
discreto alcanza el valor 0 en 0.

Definición 5.16. La función signo discreto, denotado por sgn(n), se define como

−1, n < 0,

sgn(n) = 0, n = 0,


1, n > 0,

La gráfica sgn(n) se aprecia en la siguiente figura.

sgn(n)
b b b b
1
···
−4 −3 −2 −1 b

1 2 3 4 n
···
b b b b
-1

Figura 5.16. Gráfica de sgn(n)

Igual que en el dominio continuo, las funciones u(n) y sgn(n) también están estrecha-
mente relacionadas. La definición de sgn(n) permite inferir que ella se puede expresar
como la suma de dos funciones escalón, a saber:

sgn(n) = u(n) − u(−n).

Se deja como ejercicio para el lector demostrar esta relación.


158 5.2. FUNCIONES DE DOMINIO DISCRETO

5.2.5 Relación entre u(n) y δ(n)

Las funciones escalón unitario discreto e impulso unitario discreto están relacionadas
de la siguiente manera:

n
X
u(n) = δ(k) y δ(n) = u(n) − u(n − 1).
k=−∞

Demostremos estas relaciones. Primero veamos que la suma

n
X
f (n) = δ(k)
k=−∞

se comporta como el escalón unitario discreto. Como δ(k) = 0 para todo k 6= 0, entonces
es claro que f (n) = 0 para n < 0. Para n = 0, se tiene que

−1
X
f (0) = δ(k) + δ(0) = 1.
k=−∞

Ahora, para n > 0 se obtiene

−1
X n
X
f (n) = δ(k) + δ(0) + δ(k) = 1.
k=−∞ k=1

Por lo tanto, todo lo anterior nos dice que f (n) se comporta como el escalón unitario
discreto, es decir,
n
X
u(n) = δ(k).
k=−∞

Veamos ahora que la función g(n) = u(n) − u(n − 1) se comporta como el impulso
unitario discreto. Como u(n) = 0 para todo n < 0, entonces es claro que g(n) = 0 para
n < 0. Para n = 0, se tiene que

g(0) = u(0) − u(−1) = 1.

Ahora, se tiene que u(n) = 1 y u(n − 1) = 1 para n ≥ 1; luego, g(n) = 0 para n ≥ 1. Por lo
tanto, todo lo anterior nos indica que g(n) se comporta como el impulso unitario discreto,
es decir,
δ(n) = u(n) − u(n − 1).
CAPÍTULO 5. FUNCIONES DE DOMINIOS CONTINUO Y DISCRETO 159

5.2.6 Convolución en el Dominio Discreto


La convolución en el dominio discreto es un concepto de mucha utilidad en análisis de
sistemas, especialmente, en el procesamiento digital de señales. La definición matemática
de la convolución de funciones de dominio discreto es la siguiente.

Definición 5.17. Sean f y g dos funciones de dominio discreto. La convolución entre f


y g, denotada por f ∗ g, es una función de dominio discreto definida como

X
f ∗ g (n) = f (k) g(n − k), para n ∈ Z.
k=−∞

En la siguiente proposición se establecen algunas propiedades de la convolución de


funciones de dominio discreto.

Proposición 5.2. Sean f , g y h funciones de dominio discreto. Entonces, las siguientes


propiedades se cumplen.

i) f ∗ g = g ∗ f . (Conmutatividad)

ii) f ∗ (g + h) = f ∗ g + f ∗ h. (Distributiva con respecto a la suma)

iii) Si h(n) = f ∗ g(n) es la convolución de las funciones de dominio discreto f (n) y g(n),
entonces
f (n − n0 ) ∗ g(n) = h(n − n0 ).

Demostración. i) Se tiene que



X
f ∗ g(n) = f (k) g(n − k);
k=−∞

ahora, haciendo el cambio m = n − k en la sumatoria, se obtiene:


−∞
X ∞
X
f ∗ g(n) = f (n − m) g(m) = g(m) f (n − m) = g ∗ f (n).
m=∞ m=−∞

Demostración de ii). Se tiene que



X
f ∗ (g + h)(n) = f (k) [g(n − k) + h(n − k)]
k=−∞
X∞ ∞
X
= f (k) g(n − k) + f (k) h(n − k)
k=−∞ k=−∞
= f ∗ g(n) + f ∗ h(n).

Demostración de iii). Sean h(n) = f ∗ g(n) y n0 . Por definición, la convolución f (n − n0 ) ∗


g(n) está dada por:

X
f (n − n0 ) ∗ g(n) = f (k − n0 ) g(n − k);
k=−∞
160 5.2. FUNCIONES DE DOMINIO DISCRETO

ahora, haciendo el cambio m = k − n0 en la sumatoria, se obtiene:



X
f (n − n0 ) ∗ g(n) = f (m) g((n − n0 ) − m) = h(n − n0 ).
k=−∞

Esto completa la demostración.

En los siguientes ejemplos se calcula la convolución entre dos funciones de dominio


discreto.

Ejemplo 5.7. Determinar la convolución f ∗ g, donde

f (n) = u(n − 2) − u(n − 4) y g(n) = u(n).

Solución. Considerando las expresiones de f (n) y g(n), se tiene que la expresión analı́tica
de la convolución h(n) = f ∗ g (n), para cada n ∈ Z, es:

X
h(n) = f ∗ g (n) = f (k)g(n − k)
k=−∞
3
X
= g(n − k)
k=2
= g(n − 2) + g(n − 3)
= u(n − 2) + u(n − 3),

cuya representación gráfica se muestra en la siguiente figura.


h(n)
b b b b
2

b
1
··· ···
b b b b b

-3 -2 -1 1 2 3 4 5 6
n

Ejemplo 5.8. Si f (n) = (1/3)n u(n) y g(n) = 2n u(−n), entonces determine la convolución
f ∗ g.
Solución. Tomemos h(n) = f ∗ g (n). Considerando las expresiones de f (n) y g(n), se tiene
que

X ∞
X
h(n) = f ∗ g (n) = f (k)g(n − k) = 3−k 2n−k u(k − n)
k=−∞ k=0

X
n −k
= 2 6 u(k − n), para todo n ∈ Z. (5.5)
k=0

Determinemos la expresión de h(n) para: n ≤ 0 y n > 0.


CAPÍTULO 5. FUNCIONES DE DOMINIOS CONTINUO Y DISCRETO 161

• Para n ≤ 0, la gráfica de u(n − k) es

u(k − n)
1
b b b b b

··· ···
b b b

n−1 n −1 1 2 k

Entonces, por (5.5)



!
X
n 1
h(n) = 2 (1/6)k = 2n 1 = (6/5)2n , para n ≤ 0.
k=0
1− 6

• Para n > 0, la gráfica de u(n − k) es

u(k − n)
b b b
1
··· ···
b b b b b b

1 n−1 n n+1 n+2 k

Entonces, por (5.5)



X
h(n) = 2n (1/6)k
k=n
"∞ n−1
#
X X
= 2n (1/6)k − (1/6)k
k=0 k=0
" #
n 1 1 − (1/6)n
= 2 1 −
1− 6 1 − 16
= (6/5)2n (1/6)n = (6/5)3−n , para n > 0.

De esta forma, la expresión analı́tica de la convolución h(n) = f ∗ g (n) está dada por

h(n) = (6/5)2n u(−1 − n) + (6/5)3−n u(n),

cuya representación gráfica se aprecia en la siguiente figura.


h(n)
b

1
b
b
b
b b
b b b b b

-5 -4 -3 -2 -1 1 2 3 4 5 n


162 5.3. INTRODUCCIÓN AL CÁLCULO OPERACIONAL CON MATLAB

5.3 Introducción al Cálculo Operacional con M ATLAB

Comencemos utilizando la herramienta de matemática simbólica de M ATLAB para tra-


bajar con funciones de dominio continuo y calcular la convolución en el dominio continuo.
Una función de dominio continuo f (t) se puede crear como una función simbólica. Pri-
mero se crea el objeto simbólico t, luego se define la función simbólica f(t) según su
forma algebraica. Observe el siguiente ejemplo, donde creamos la función

f (t) = t2 (sen (t) + e−t ).


>> syms t
>> f ( t ) = t ˆ2∗( s i n ( t )+exp(− t ) )
f(t) =
t ˆ2∗( exp(− t ) + s i n ( t ) )
✂ ✁

El comando ezplot(f,[tmin,tmax]) dibuja la función f (t) en el intervalo [tmin, tmax].


Por ejemplo, la gráfica de la función f (t) = t2 (sen (t) + e−t ), creada previamente, en el in-
tervalo [−5, 5] se obtiene de la siguiente forma:

>> e z p l o t ( f , [ − 5 , 5 ] ) , t i t l e ( ’ t ˆ2( s i n ( t )+exp(− t ) ) ’ )
✂ ✁

t2(sin(t)+exp(−t))

1000

800

600

400

200

−5 −4 −3 −2 −1 0 1 2 3 4 5
t

Las funciones simbólicas creadas en M ATLAB se pueden operar entre sı́, diferenciar,
integrar, etc. Ahora, las funciones elementales ya están predefinidas y las funciones como
el impulso unitario y el escalón unitario, también están predefinidas. El impulso unitario,
δ(t), es dirac(t), y el escalón unitario, u(t), es heaviside(t). La gráfica de u(t) en el
intervalo [−10, 10], se obtiene de la siguiente forma:

>> syms t
>> e z p l o t ( h e a v i s i d e ( t ) ,[ −10 ,10])
✂ ✁
CAPÍTULO 5. FUNCIONES DE DOMINIOS CONTINUO Y DISCRETO 163

heaviside(t)

0.8

0.6

0.4

0.2

−10 −8 −6 −4 −2 0 2 4 6 8 10
t

Las otras funciones de dominio continuo de uso común, por ejemplo, la función signo
sgn(t), la rampa r(t), el pulso rectangular pT (t) o el pulso triangular qT (t), se pueden
definir a través de su relación con el escalón unitario. Observe el siguiente ejemplo donde
se definen las funciones sgn(t), r(t), p1 (t) y q1 (t). Utilice el co-
mando ezplot
para obtener las
✄ gráficas de las
>> syms t funciones sgn(t),
>> u ( t ) = h e a v i s i d e ( t ) ; r(t), p1 (t) y
>> sgn ( t ) = u ( t )−u(− t ) q1 (t), en el in-
sgn ( t ) = tervalo [−10, 10]
h e a v i s i d e ( t ) − h e a v i s i d e (− t )
>> r ( t ) = t ∗u ( t )
r(t) =
t ∗heaviside ( t )
>> p1 ( t ) = u ( t +1)−u ( t −1)
p1 ( t ) =
h e a v i s i d e ( t + 1) − h e a v i s i d e ( t − 1)
>> q1 ( t ) = (1+ t ) ∗ ( u ( t +1)−u ( t ))+(1− t ) ∗ ( u ( t )−u ( t −1))
q1 ( t ) =
( t − 1)∗( h e a v i s i d e ( t − 1) − h e a v i s i d e ( t ) ) +
( t + 1)∗( h e a v i s i d e ( t + 1) − h e a v i s i d e ( t ) )
✂ ✁

También se puede definir la composición de funciones de dominio continuo. Observe


el siguiente ejemplo donde se define la función f (t) = u(cos(t)) y se obtiene su gráfica en
el intervalo [−6π, 6π].


>> syms t
>> f ( t ) = h e a v i s i d e ( c o s ( t ) ) ;
>> e z p l o t ( f ,[ −6∗ pi , 6 ∗ p i ] )
✂ ✁
164 5.3. INTRODUCCIÓN AL CÁLCULO OPERACIONAL CON MATLAB

heaviside(cos(t))

0.8

0.6

0.4

0.2

−15 −10 −5 0 5 10 15
t

Con el comando int se pueden calcular integrales que involucren a la función impulso
unitario. Seguidamente se calculan los valores de las integrales
Z 3 Z 3
1 1
e(t−2) δ(2t − 4) dt = , e(t−2) δ′′ (2t − 4) dt = .
0 2 0 2

>> syms t
>> f 1 ( t ) = exp ( t −2)∗ d i r a c (2∗ t −4);
>> i n t ( f1 , t , 0 , 3 )
ans ( t ) =
1/2
>> f 2 ( t ) = exp ( t −2)∗ d i f f ( d i r a c (2∗ t −4) , t , 2 ) ;
>> i n t ( f2 , t , 0 , 3 )
ans ( t ) =
1/2
✂ ✁
M ATLAB no tiene un comando que permita calcular en forma explı́cita la convolución de
dos funciones de demonio continuo. En el Programa 5.1 se muestra la función ConvCont,
que permite calcular el valor de la convolución h(t) = f ∗ g(t) en t = t0 , en otras palabras,
halla el valor h(t0 ). Veamos un ejemplo. Consideremos las funciones f (t) = p1 (t) y g(t) =
u(t) del Ejemplo 5.4, donde hallamos la convolución

h(t) = f ∗ g(t) = (t + 1)(u(t + 1) − u(t − 1)) + 2u(t − 1).

Programa 5.1. Función ConvCont.m

f u n c t i o n [ h0]= ConvCont ( f , g , t 0 )
%ConvCont H a l l a e l v a l o r de l a c o n v o l u c i ó n h ( t ) = f ∗g ( t )
% en t 0
% h0 = h ( t 0 )
syms t tau ;
h0 = i n t ( f ( tau )∗ g ( t0−tau ) , tau ,− I n f , I n f ) ;
end

Seguidamente se utiliza la función ConvCont para hallar el valor h(1/2) = 3/2.



>> syms t
>> f ( t ) = h e a v i s i d e ( t +1)−h e a v i s i d e ( t −1);
CAPÍTULO 5. FUNCIONES DE DOMINIOS CONTINUO Y DISCRETO 165

>> g ( t ) = h e a v i s i d e ( t ) ;
>> h0 = ConvCont ( f , g , 1 / 2 )
h0 =
3/2
✂ ✁

La función ConvCont no sólo permite hallar el valor de f ∗ g(t) en t = t0 , sino que


también podemos usarla para obtener una gráfica de f ∗ g(t). A continuación, con las
funciones f (t) y g(t) creadas previamente, se construye la gráfica de h(t) = f ∗ g(t) en el
intervalo [−4, 4].

>> t 0 = −4:.1:4;
>> h0 = ConvCont ( f , g , t 0 ) ;
>> p l o t ( t0 , h0 , ’ LineWidth ’ , 2 )
>> a x i s ([ −4 ,4 , −1 ,3]); g r i d on
>> t i t l e ( ’ h ( t ) = f ∗g ( t ) ’ )
✂ ✁

h(t) = f*g(t)
3

2.5

1.5

0.5

−0.5

−1
−4 −3 −2 −1 0 1 2 3 4

En este caso, t0 es un vector, por ello ConvCont arroja como salida un vector h0, cuyas
componentes son los valores que alcanza h(t) en cada componente del vector t0.
En cuanto a las funciones de dominio discreto, con M ATLAB también se pueden manejar,
pero en forma limitada. Una función f (n) de dominio discreto es en sı́ una sucesión de
números reales. Este hecho nos permite definir funciones de dominio discreto como un
arreglo de números reales. Por ejemplo, la función sen (πn/2) puede definirse en M ATLAB
como una discretización de la función sen (πt), primero creando n como un arreglo de
números enteros y después evaluando la función sen (πt/2) en ese arreglo. Esto puede
observarlo a continuación.

>> n=−10:10;
>> f = s i n ( p i ∗n / 2 ) ;
✂ ✁

Con n=-10:10 se crea el arreglo [-10,-9,-8,...,8,9,10]. Ası́, el resultado de las opera-


ciones anteriores es un arreglo f cuyas componentes son sen (πn/2), para n = −10, . . . , 10;
en otras palabras, se ha definido la función f (n) = sen (πn/2), para n = −10, −9, . . . , 10.
La gráfica de esta función puede generarse con el comando stem(n,f,’fill’,’--’).

>> stem ( n , f , ’ f i l l ’ , ’−− ’ )
>> t i t l e ( ’ sen ( n\ p i /2) ’ )
✂ ✁
166 5.3. INTRODUCCIÓN AL CÁLCULO OPERACIONAL CON MATLAB

sen(nπ/2)
1

0.8

0.6

0.4

0.2

−0.2

−0.4

−0.6

−0.8

−1
−10 −8 −6 −4 −2 0 2 4 6 8 10

El proceso anterior se puede aplicar para generar todas las funciones elementales de
dominio discreto (cos(n), en , tan(n), etc.). En cuanto a las funciones impulso unitario
discreto δ(n) y escalón unitario discreto u(n), hemos definido las funciones impulso (ver
Programa 5.2) y escalon (ver Programa 5.3), que hallan los valores que alcanzan δ(n)
y u(n) en las componentes del arreglo n, respectivamente. A continuación usamos las
funciones impulso y escalon, para generar la gráfica de δ(n) y la de u(n) en el intervalo
[−10, 10].


>> n=−10:10;
>> s u b p l o t ( 2 , 1 , 1 ) , stem (n , impu lso ( n ) , ’ f i l l ’ , ’−− ’ )
>> t i t l e ( ’ \ de lta (n) ’ )
>> s u b p l o t ( 2 , 1 , 2 ) , stem (n , e s c a l o n ( n ) , ’ f i l l ’ , ’−− ’ )
>> t i t l e ( ’ u(n) ’ )
✂ ✁

Programa 5.2. Función impulso.m

f u n c t i o n [ d ] = impu lso ( n )
%impu lso H a l l a l o s v a l o r e s de d e l t a ( n )
% d : a r r e g l o de v a l o r e s de d e l t a ( n )
d = zeros ( s i z e (n ) ) ;
d ( n==0) = 1 ;
end

Programa 5.3. Función escalon.m

f u nct i on [u] = escalon (n)


%e s c a l o n H a l l a l o s v a l o r e s de u ( n )
% u : a r r e g l o de v a l o r e s de u (n )
u = zeros ( s i z e (n ) ) ;
u (n>=0) = 1 ;
end
CAPÍTULO 5. FUNCIONES DE DOMINIOS CONTINUO Y DISCRETO 167

δ(n)
1

0.8

0.6

0.4

0.2

0
−10 −8 −6 −4 −2 0 2 4 6 8 10

u(n)
1

0.8

0.6

0.4

0.2

0
−10 −8 −6 −4 −2 0 2 4 6 8 10

Los Programas 5.4, 5.5 y 5.6, muestran las funciones signo, rampa y pulsorectangular,
que definen respectivamente las funciones signo discreto,


−1, n < 0,

sgn(n) = 0, n = 0,


1, n > 0,

rampa discreta,

r(n) = n u(n),

y pulso rectangular discreto


0,
 n < n1 ,
pn1 ,n2 (n) = 1, n1 ≤ n ≤ n2 ,


0, n > n2 ,

donde n1 , n2 ∈ Z tales que n1 < n2 . Utilice el co-


mando stem
para obtener
Programa 5.4. Función signo.m las gráficas de
las funciones
f u nct i on [ s ] = signo (n) sgn(n), r(n)
%s i g n o H a l l a l o s v a l o r e s de sgn ( n ) y p−7,−1 (n),
% s : a r r e g l o de v a l o r e s de sgn (n ) en el intervalo
[−10, 10]
s = zeros ( s i z e (n ) ) ;
s (n<0) = −1;
s (n>0) = 1 ;
end
168 5.3. INTRODUCCIÓN AL CÁLCULO OPERACIONAL CON MATLAB

Programa 5.5. Función rampa.m

f u n c t i o n [ r ] = rampa( n )
%rampa H a l l a l o s v a l o r e s de r ( n )
% r : a r r e g l o de v a l o r e s de r ( n )
r = zeros ( s i z e (n ) ) ;
r (n>=0) = n ( n>=0);
end

Programa 5.6. Función pulsorectangular.m

f u n c t i o n [ p ] = p u l s o r e c t a n g u l a r (n , n1 , n2 )
%p u l s o r e c t a n g u l a r H a l l a l o s v a l o r e s de un p u l s o
% r e c t a n g u l a r e n t r e n1 y n2
% p : a r r e g l o de v a l o r e s d e l p u l s o r e c t a n g u l a r
p = ones ( s i z e ( n ) ) ;
p (n<n1 ) = 0 ;
p (n>n2 ) = 0 ;
end

Con las funciones impulso, escalon, signo, rampa y pulsorectangular, se pueden


definir nuevas funciones, por ejemplo, las funciones

f1 (n) = u(4 − n), f2 (n) = n (u(n + 2) − u(n − 3)),



−1, −5 ≤ n ≤ −1,

f3 (n) = u(−n) − δ(n − 1) − δ(n − 2), f4 (n) = 1, 0 ≤ n ≤ 4,


0, en otros casos.

se definen, en el intervalo [−10, 10], como:


>> n = −10:10;
>> f 1 = e s c a l o n ( n−4);
>> f 2 = n . ∗ ( e s c a l o n ( n+2)−e s c a l o n ( n −3));
>> f 3 = e s c a l o n (−n)−impu lso ( n−1)−impu lso ( n−2);
>> f 4 = p u l s o r e c t a n g u l a r ( n ,0 ,4) − p u l s o r e c t a n g u l a r ( n, −5 , −1);
✂ ✁

cuyas gráficas se obtienen de la siguiente forma:


>> subplot (2 ,2 ,1) , stem (n , f1 , ’ fill ’ , ’−− ’ ) ; title ( ’f 1 (n) ’)
>> subplot (2 ,2 ,2) , stem (n , f2 , ’ fill ’ , ’−− ’ ) ; title ( ’f 2 (n) ’)
>> subplot (2 ,2 ,3) , stem (n , f3 , ’ fill ’ , ’−− ’ ) ; title ( ’f 3 (n) ’)
>> subplot (2 ,2 ,4) , stem (n , f4 , ’ fill ’ , ’−− ’ ) ; title ( ’f 4 (n) ’)
✂ ✁
CAPÍTULO 5. FUNCIONES DE DOMINIOS CONTINUO Y DISCRETO 169

f (n) f (n)
1 2
1 2

0.8
1

0.6
0
0.4

−1
0.2

0 −2
−10 −5 0 5 10 −10 −5 0 5 10

f (n) f (n)
3 4
1 1

0.5 0.5

0 0

−0.5 −0.5

−1 −1
−10 −5 0 5 10 −10 −5 0 5 10

M ATLAB no tiene un comando que permita calcular en forma explı́cita la convolución de


dos funciones cualesquiera de demonio discreto. Pero, M ATLAB si cuenta con el comando
conv(f,g) que calcula la convolución de funciones f (n) y g(n) definidas en intervalos
finitos, es decir, f (n) debe estar definida en un intervalo [n1 , n2 ] y g(n) debe estar definida
en un intervalo [n3 , n4 ], donde n1 , n2 , n3 , n4 ∈ Z tales que n1 < n2 y n3 < n4 . Cuando se
aplica el comando h = conv(f,g), se obtiene la convolución h(n) = f ∗ g(n) definida en
el intervalo n ∈ [n1 + n3 , n2 + n4 ]. Veamos un ejemplo. Consideremos las funciones

f (n) = u(n − 2) − u(n − 4), para n ∈ [0, 5]

y
g(n) = u(n + 2) − u(n − 5), para n ∈ [−2, 8]

En esta caso, la convolución h(n) = f ∗ g(n) está dada por:

h(n) = u(n − 1) + u(n − 2) − u(n − 7) − u(n − 8).

Utilicemos el comando h = conv(f,g) para hallar los valores de h(n), por supuesto, en el
intervalo [−2, 13].

>> n1=0; n2=5; n3=−2; n4=8;
>> n=n1 : n2 ; f=e s c a l o n ( n−2)−e s c a l o n ( n−4);
>> n=n3 : n4 ; g=e s c a l o n ( n+1)−e s c a l o n ( n−5);
>> h = conv ( f , g ) ;
✂ ✁

Las gráficas de f (n), g(n) y h(n) se obtienen de la siguiente forma:



>> s u b p l o t ( 2 , 2 , 1 ) , stem ( n1 : n2 , f , ’ f i l l ’ , ’−− ’ ) ; t i t l e ( ’ f ( n ) ’ )
>> s u b p l o t ( 2 , 2 , 2 ) , stem ( n3 : n4 , g , ’ f i l l ’ , ’−− ’ ) ; t i t l e ( ’ g ( n ) ’ )
>> s u b p l o t ( 2 , 2 , 3 : 4 ) , stem ( n1+n3 : n2+n4 , h , ’ f i l l ’ , ’−− ’ ) ; t i t l e ( ’ h ( n ) ’ )
✂ ✁
170 5.4. PROBLEMAS RESUELTOS

f(n) g(n)
1 1

0.8 0.8

0.6 0.6

0.4 0.4

0.2 0.2

0 0
0 1 2 3 4 5 −2 0 2 4 6 8

h(n)
2

1.5

0.5

0
−2 0 2 4 6 8 10 12 14

5.4 Problemas Resueltos


Problema 5.1. Sean f (t) y g(t) funciones de dominio continuo definidas respectivamente
por:
f (t) = pa (t − a) y g(t) = u(t).
Determinar la convolución h(t) = f ∗ g(t).

Solución. Tenemos que la convolución h(t) = f ∗ g(t) se define como:


Z ∞
h(t) = f ∗ g(t) = f (τ )g(t − τ ) dτ, para t ∈ R.
−∞

Ahora bien, considerando las expresiones de f (t) y g(t) dadas en el enunciado del pro-
blema, obtenemos:
Z ∞ Z 2a−
h(t) = pa (τ − a)u(t − τ ) dτ = u(t − τ ) dτ, para t ∈ R. (5.6)
−∞ 0+

Pasemos a determinar la expresión matemática de h(t) considerando la ecuación (5.6).


Debemos tener presente que esta ecuación es válida para todo t ∈ R. El procedimiento
que seguiremos para hallar la expresión de h(t) es el siguiente: fijar valores de t, luego
construir la gráfica de la función u(t − τ ) (en función de τ ) y, por último, determinar el
valor de la integral de (5.6), para cada valor de t.

• Para t ≤ 0, la gráfica de u(t − τ ) es:

u(t − τ )

τ
t 2a
CAPÍTULO 5. FUNCIONES DE DOMINIOS CONTINUO Y DISCRETO 171

Lo que implica, según (5.6), que h(t) = 0, para t ≤ 0.

• Para 0 < t < 2a, la gráfica de u(t − τ ) es:

u(t − τ )
1

τ
t 2a

Ası́, usando (5.6) obtenemos:


Z t
h(t) = dτ = t, para t ∈ (0, 2a).
0+

• Para t ≥ 2a, la gráfica de u(t − τ ) es:

u(t − τ )
1

τ
2a t

Ası́, usando (5.6) obtenemos:


Z 2a+
h(t) = dτ = 2a, para t ≥ 2a.
0+

De todo lo anterior se tiene que la expresión matemática de la convolución h(t) = f ∗g(t)


está dada por:
h(t)

0,
 t ≤ 0, 2a
h(t) = t, 0 < t < 2a,


2a, t ≥ 2a.
2a t

Problema 5.2. Sean f (t) y g(t) funciones de dominio continuo definidas respectivamente
por:
f (t) = r(t) y g(t) = sgn(t) + u(−t − 1).

Determinar la convolución h(t) = f ∗ g(t).


172 5.4. PROBLEMAS RESUELTOS

Solución. Usando la distributividad de la convolución podemos escribir, para t ∈ R,

h(t) = f ∗ g(t) = r(t) ∗ sgn(t) + r(t) ∗ u(−t − 1).

Calculemos cada una de las convoluciones r(t) ∗ sgn(t) y r(t) ∗ u(−t − 1).
Usando la definición de convolución obtenemos:
Z ∞
r(t) ∗ sgn(t) = r(τ ) sgn(t − τ ) dτ, para t ∈ R. (5.7)
−∞

• Para t ≤ 0, la gráfica de la función r(τ ) sgn(t − τ ) es:

r(τ ) sgn(t − τ )

−τ

Ası́, por (5.7) la convolución r(t) ∗ sgn(t) no existe, para t ≤ 0.

• Para t > 0, la gráfica de la función r(τ ) sgn(t − τ ) es:

r(τ ) sgn(t − τ )

t τ
−t
−τ

Ası́, usando (5.7) obtenemos:


Z t Z ∞
r(t) ∗ sgn(t) = τ dτ + (−τ )dτ
0+ t
 2 s
t2 τ
= − lı́m = −∞, para t > 0;
2 s→∞ 2 t

en otras palabras, la convolución r(t) ∗ sgn(t) no existe, para t ≥ 0.

De esta forma, la convolución r(t) ∗ sgn(t) no existe para todo t ∈ R. En consecuencia,


podemos concluir que la convolución f ∗ g(t) no existe para todo t ∈ R, sin considerar el
resultado de la convolución r(t) ∗ u(−t − 1). Se deja como ejercicio para el lector, verificar
que la convolución r(t) ∗ u(−t − 1) no existe para todo t ∈ R. 
CAPÍTULO 5. FUNCIONES DE DOMINIOS CONTINUO Y DISCRETO 173

Problema 5.3. Sean f (t) y g(t) funciones de dominio continuo definidas respectivamente
por:
f (t) = e−t u(t) y g(t) = e−2t u(t).
Determinar la convolución h(t) = f ∗ g(t).

Solución. Considerando la definición de la convolución h(t) = f ∗ g(t) y las expresiones


de f (t) y g(t) dadas en el enunciado del problema, obtenemos:
Z ∞ Z ∞
−τ −2(t−τ ) −2t
h(t) = e e u(t − τ ) dτ = e eτ u(t − τ ) dτ, para t ∈ R. (5.8)
−∞ 0+

• Para t ≤ 0, la gráfica de eτ u(t − τ ) es:

eτ u(t − τ )
et

τ
t

Ası́, por (5.8), h(t) = 0, para t ≤ 0.

• Para t > 0, la gráfica de eτ u(t − τ ) es:

eτ u(t − τ )

et

τ
t

Ası́, por (5.8) obtenemos:


Z t 
h(t) = e−2t eτ dτ = e−2t et − 1 , para t > 0.
0+

De todo lo anterior se tiene que la expresión matemática de la convolución h(t) = f ∗ g(t)


está dada por:
h(t)

h(t) = e−2t et − 1 u(t)

t

174 5.4. PROBLEMAS RESUELTOS

Problema 5.4. Determinar la convolución h(t) = f ∗ g(t) de las funciones de dominio


continuo f (t) y g(t) que se muestran en la siguiente figura. Tome en cuenta que solo se
muestra la gráfica de las funciones donde ellas son distintas de cero.
f (t) g(t)
1

−1 1

−1 1 t t

−1

Solución. Considerando la definición de la convolución h(t) = f ∗ g(t) y la gráfica de f (t)


dada en el enunciado del problema, obtenemos:
Z 0 Z 1
h(t) = (τ + 1)g(t − τ ) dτ + g(t − τ ) dτ, para t ∈ R. (5.9)
−1 0

Determinemos los valores de las integrales de (5.9) para cada t ∈ R.

• Para t ≤ −2, las gráficas de g(t − τ ) y (τ + 1)g(t − τ ) son:

g(t − τ ) g(t − τ )(τ + 1)


(τ + 1)(τ − t)

(τ + 1)(t − τ )
t−1 t t+1
τ t−1 t t+1 τ
−1 −1
−1

Ası́, por (5.9), h(t) = 0, para t ≤ −2.

• Para −2 < t ≤ −1, las gráficas de g(t − τ ) y (τ + 1)g(t − τ ) son:

g(t − τ ) g(t − τ )(τ + 1)

(τ + 1)(τ − t)

t−1 t t+1 t t+1


−2 −1 τ t − 1 −2 −1 τ
(τ + 1)(t − τ )
−1

Ası́, por (5.9) obtenemos:


Z t+1
(t − 1) (t + 2)2
h(t) = (τ + 1)(t − τ ) dτ = , para − 2 < t ≤ −1.
−1 6
CAPÍTULO 5. FUNCIONES DE DOMINIOS CONTINUO Y DISCRETO 175

• Para −1 < t ≤ 0, las gráficas de g(t − τ ) y (τ + 1)g(t − τ ) son:


g(t − τ ) g(t − τ )(τ + 1)

(τ + 1)(τ − t)
t−1 t t+1 t t+1
−1 1 τ t−1 1 τ

−1 −1 (τ + 1)(t − τ )

Ası́, por (5.9) obtenemos:


Z t Z 0 Z t+1
h(t) = (τ + 1)(τ − t) dτ + (τ + 1)(t − τ ) dτ + (t − τ ) dτ
−1 t 0
t 2 (t + 1)3 t2 (t + 3) 1
= − − − , para − 1 < t ≤ 0.
2 6 6 2
• Para 0 < t ≤ 1, las gráficas de g(t − τ ) y (τ + 1)g(t − τ ) son:
g(t − τ ) g(t − τ )(τ + 1)

t−1 t t+1 t−1 t t+1


1 2 τ 1 2 τ
(τ + 1)(τ − t)
−1

(τ + 1)(t − τ )

Ası́, por (5.9) obtenemos:


Z 0 Z t Z 1
h(t) = (τ + 1)(τ − t) dτ + (τ − t) dτ + (t − τ ) dτ
t−1 0 t

(t − 1) t2 + 4 t + 1 (t − 1)2 t2
= − − , para 0 < t ≤ 1.
6 2 2
• Para 1 < t ≤ 2, las gráficas de g(t − τ ) y (τ + 1)g(t − τ ) son:
g(t − τ ) g(t − τ )(τ + 1)

t−1 t t+1 t−1 t t+1


1 2 3 τ 1 2 3 τ

−1
(τ + 1)(τ − t)

(τ + 1)(t − τ )
176 5.4. PROBLEMAS RESUELTOS

Ası́, por (5.9) obtenemos:


Z 1
t (t − 2)
h(t) = (τ − t) dτ = , para 1 < t ≤ 2.
t−1 2

• Para t > 2, las gráficas de g(t − τ ) y (τ + 1)g(t − τ ) son:

g(t − τ ) g(t − τ )(τ + 1)

t−1 t t+1 t−1 t t+1


1 2 3 4 τ 1 2 3 4 τ

−1

(τ + 1)(τ − t)

(τ + 1)(t − τ )

Ası́, por (5.9), h(t) = 0, para t > 2.

De todo lo anterior se tiene que la convolución h(t) = f ∗ g(t) está dada por:

(t−1) (t+2)2


 6 , −2 < t ≤ −1,
 (t+1)3 t2 (t+3)
 t2 1
 2 − 6 − 6 − 2, −1 < t ≤ 0,


h(t) = (t−1) (t +4 t+1) − (t−1) − t2 ,
2 2

 6 2 2 0 < t ≤ 1,

 t (t−2)


 2 , 1 < t ≤ 2,


0, en otros casos.

h(t)

−2 2
t

−2/3

Problema 5.5. Sean f (n) y g(n) funciones de dominio discreto definidas respectivamente
por:
f (n) = u(n + 1) − u(n − 2) y g(n) = u(n + 1) − u(n − 2).
Determinar la convolución h(n) = f ∗ g(n).
CAPÍTULO 5. FUNCIONES DE DOMINIOS CONTINUO Y DISCRETO 177

Solución. Considerando la expresión de f (n) dada en el enunciado del problema y la


definición de la convolución de funciones de dominio discreto, podemos escribir:

X 1
X
h(n) = f ∗ g(n) = f (k)g(n − k) = g(n − k), para n ∈ Z. (5.10)
k=−∞ k=−1

Calculemos los valores que toma h(n) para cada n ∈ Z. Seguidamente construiremos
gráficas de g(n − k) para cada n. Aquı́ solo se mostrará el gráfico de

g(n − k) = u((n + 1) − k) − u((n − 2) − k)

donde ella es distinta de cero.

• Para n ≤ −3, la gráfica de g(n − k) es:

g(n − k)
b b b
1

n−1 n n+1 ··· −1 1 k

Ası́, por (5.10), h(n) = 0, para n ≤ −3.

• Para n = −2, la gráfica de g(n − k) es:

g(n − k)
b b b
1

-5 -4 -3 -2 -1 1 2 3 4 5 k

Ası́, por (5.10), h(−2) = 1.

• Para n = −1, la gráfica de g(n − k) es:

g(n − k)
b b b
1

-5 -4 -3 -2 -1 1 2 3 4 5 k

Ası́, por (5.10), h(−1) = 2.

• Para n = 0, la gráfica de g(n − k) es:

g(n − k)
b b b
1

-5 -4 -3 -2 -1 1 2 3 4 5 k
178 5.4. PROBLEMAS RESUELTOS

Ası́, por (5.10), h(0) = 3.

• Para n = 1, la gráfica de g(n − k) es:

g(n − k)
b b b
1

-5 -4 -3 -2 -1 1 2 3 4 5 k

Ası́, por (5.10), h(1) = 2.

• Para n = 2, la gráfica de g(n − k) es:

g(n − k)
b b b
1

-5 -4 -3 -2 -1 1 2 3 4 5 k

Ası́, por (5.10), h(2) = 1.

• Para n ≥ 3, la gráfica de g(n − k) es:

g(n − k)
b b b
1

−1 1 ··· n−1 n n+1 k

Ası́, por (5.10), h(n) = 0, para n ≥ 3.

De todo lo anterior se tiene que la convolución h(n) = f ∗ g(n) está dada por:

h(n) = (3 − |n|)(u(n + 2) − u(n − 3))

h(n)
b
3

b b
2

b b
1

b b b b b b

-5 -4 -3 -2 -1 1 2 3 4 5 n


CAPÍTULO 5. FUNCIONES DE DOMINIOS CONTINUO Y DISCRETO 179

Problema 5.6. Sean f (n) y g(n) funciones de dominio discreto definidas respectivamente
por:
f (n) = u(n + 2) − u(n − 4) y g(n) = nu(n).
Determinar la convolución h(n) = f ∗ g(n).

Solución. Considerando las expresiones de f (n) y g(n) dadas en el enunciado del problema
y la definición de la convolución de funciones de dominio discreto, podemos escribir para
n ∈ Z:

X 3
X
h(n) = f ∗ g(n) = f (k)g(n − k) = (n − k) u(n − k).
k=−∞ k=−2

Haciendo el cambio de ı́ndices m = n−k y luego sustituyendo m por k, la ecuación anterior


adquiere la forma:
n+2
X
h(n) = k u(k), para n ∈ Z. (5.11)
k=n−3

Calculemos los valores que toma h(n) para cada n ∈ Z. Para ello construiremos la gráfica
de ku(k) y usaremos la ecuación (5.11) variando los valores de n.
La gráfica de k u(k) es:

ku(k)
b
6

5 b

4 b

b
3

b
2

b
1

b b b b b b

-5 -4 -3 -2 -1 1 2 3 4 5 6 k

• Para n ≤ −2, por (5.11), h(n) = 0.

• Para n = −1, por (5.11), h(−1) = 1.

• Para n = 0, por (5.11), h(0) = 1 + 2 = 3.

• Para n = 1, por (5.11), h(1) = 1 + 2 + 3 = 6.

• Para n = 2, por (5.11), h(2) = 1 + 2 + 3 + 4 = 9.

• Para n = 3, por (5.11), h(3) = 1 + 2 + 3 + 4 + 5 = 14.

• Para n = 4, por (5.11), h(4) = 1 + 2 + 3 + 4 + 5 + 6 = 20.


180 5.4. PROBLEMAS RESUELTOS

• Para n = 5, por (5.11), h(5) = 2 + 3 + 4 + 5 + 6 + 7 = 27.

• Para n = 6, por (5.11), h(6) = 3 + 4 + 5 + 6 + 7 + 8 = 33.

..
.

De todo lo anterior se tiene que la convolución h(n) = f ∗ g(n) está dada por:


 0, n ≤ −2,


(n+2)(n+3)
h(n) = 2 , −1 ≤ n ≤ 4,



 (n+2)(n+3) (n−4)(n−3)
2 − 2 , n ≥ 5.

h(n) b

···
b

b
b b b b

-5 -4 -3 -2 -1 1 2 3 4 5 6
n

4
X (−1)m
Problema 5.7. Determinar la convolución f ∗ g, donde f (n) = δ(n − m) y g(n)
m
m=1
es una función cuya gráfica se muestra en la siguiente figura.

g(n)

b b
1

1 2 3 4 ···
b b b b b

· · · −4 −3 −2 −1 n
b b
-1

Solución. Se tiene que la función g(n) se puede expresar como g(n) = g1 (n) − g2 (n),
donde

g1 (n) = u(−1 − n) − u(−3 − n) y g2 (n) = u(n − 1) − u(n − 3).

Verifique que
g(n) = g1 (n) −
g2 (n), para las
funciones g1 (n)
y g2 (n) dadas
CAPÍTULO 5. FUNCIONES DE DOMINIOS CONTINUO Y DISCRETO 181

Ası́, considerando las expresiones de f (n) y g(n), y aplicando la propiedad distributiva de


la convolución, se puede escribir, para todo n ∈ Z:

h(n) = f ∗ g (n)
= f ∗ g1 (n) − f ∗ g2 (n)
4 4
X (−1)m X (−1)m
= δ(n − m) ∗ g1 (n) − δ(n − m) ∗ g2 (n). (5.12)
m=1
m m=1
m

Calculemos las convoluciones δ(n − m) ∗ g1 (n) y δ(n − m) ∗ g2 (n), para m = 1, 2, 3, 4. Se


tiene que, para todo n ∈ Z:

δ(n − m) ∗ g1 (n) = δ(n − m) ∗ u(−1 − n) − δ(n − m) ∗ u(−3 − n)


X∞ X∞
= δ(k − m)u(−1 − n + k) − δ(k − m)u(−3 − n + k)
k=−∞ k=−∞

= u((m − 1) − n) − u((m − 3) − n), para m = 1, 2, 3, 4,

δ(n − m) ∗ g2 (n) = δ(n − m) ∗ u(n − 1) − δ(n − m) ∗ u(n − 3)


X∞ X∞
= δ(k − m)u(n − k − 1) − δ(k − m)u(n − k − 3)
k=−∞ k=−∞

= u(n − (m + 1)) − u(n − (m + 3)), para m = 1, 2, 3, 4.

Ahora, usando convenientemente estas últimas expresiones en (5.12), se obtiene que la


expresión analı́tica de la convolución h(n) = f ∗ g (n) está dada por:

4
X (−1)m
h(n) = [u((m − 1) − n) − u((m − 3) − n) − u(n − (m + 1)) + u(n − (m + 3))]
m
m=1
1 2 1 1 1
=u(−2 − n) − u(−1 − n) − u(−n) + u(1 − n) − u(2 − n) + u(3 − n)
2 3 4 3 4
1 2 1 1 1
+ u(n − 2) − u(n − 3) − u(n − 4) + u(n − 5) − u(n − 6) + u(n − 7),
2 3 4 3 4
cuya representación gráfica se aprecia en la siguiente figura.

h(n)
1 b
b
b

· · · −3 −2 −1 b 4 6
b b b b
b

b
1 2 3 5 b
7 8··· n
b
-1


182 5.5. PROBLEMAS PROPUESTOS

5.5 Problemas Propuestos

5.1. Dibujar las siguientes funciones de dominio continuo:

a) f1 (t) = u(t) + 3u(t − 1) − 2u(t − 2) f) f6 (t) = p1/2 (t2 + 2t + 1)


b) f2 (t) = r(t) − r(t − 1) − u(t − 2) 
1

g) f7 (t) = f1 (t)f2 t +
c) f3 (t) = u(t)u(t − a), a>0 2
d) f4 (t) = u(t)u(a − t), a>0 
t 1

e) f5 (t) = u(cos t) h) f8 (t) = f1 − + + f2 (t)
3 2

5.2. Calcular las siguientes integrales:

Z 1 Z 3
2
a) (t + t )δ(t − 3) dt c) e(t−2) δ(2t − 4) dt
−2 0
Z 4 Z 4  
2 2 ′ 1 1
b) (t + t )δ(t − 3) dt d) (t − 2) δ − t+ dt
−2 −4 3 2

5.3. Determinar la convolución h(t) = f ∗ g (t) de la parejas de funciones f (t) y g(t)


de dominio continuo que se enumeran a continuación:

a) f (t) = pa (t − a), e) f (t) = e−t u(t),


g(t) = δ(t − b), b > a g(t) = e−2t u(t) + u(−t)
b) f (t) = u(t), f) f (t) = t e−t u(t),
g(t) = sgn(t) g(t) = u(t)
c) f (t) = t[u(t) − u(t − 1)], g) f (t) = u(t),
g(t) = u(t) g(t) = e−2t u(t) + δ(t)
d) f (t) = [u(t + 1) − u(t − 1)] sgn(t), h) f (t) = δ(t − 1) + e−t u(t),
g(t) = u(t) g(t) = e−2t u(t)

5.4. Determinar la convolución h(t) = f ∗ g (t) de la pareja de funciones f (t) y g(t),


cuyas gráficas se muestran en siguiente figura.

f (t) g(t)
1 1

a) −1
−1 1 t 1 t
−1
CAPÍTULO 5. FUNCIONES DE DOMINIOS CONTINUO Y DISCRETO 183

f (t) g(t)
1 1

b) 1
−1 1 t −1 t
−1

f (t) g(t)
2

c) 1
1

−1 2 t −1 1 t
f (t) g(t)

d) 1 1

−1 1 t −1 1 t

5.5. Sea f (n) la función de dominio discreto cuya gráfica se muestra en la siguiente
figura.

f (n)
b
2

b
1

−2 1 3
−1 2 n
b
-1

b b
-2

Dibujar las siguientes funciones:

a) g(n) = f (n − 2) d) g(n) = f (n)u(n)


b) g(n) = f (3n − 4) e) g(n) = u(f (n))
 
n+8
c) g(n) = f − f) g(n) = δ(f (n))
4
184 5.5. PROBLEMAS PROPUESTOS

5.6. Determine la convolución h(n) = f ∗ g (n) de la pareja de funciones f (n) y g(n)


de dominio discreto que se enumeran a continuación:

a) f (n) = u(n − 1), e) f (n) = u(n + 2) − u(n − 4),



−1, −5 ≤ n ≤ −1,
 g(n) = nu(n)
g(n) = 1, 0 ≤ n ≤ 4, f) f (n) = n (u(n + 2) − u(n − 3)),


0, en otros casos. g(n) = u(−n)−(δ(n − 1) + δ(n − 2))
 n
1 X2
(−1)k
b) f (n) = u(n), g) f (n) = δ(n − k)
2 2k+2
 n k=−2
1
g(n) = δ(n) + δ(n − 1) + u(n) 3
3 X
g(n) = (−1)k δ(n − k)
c) f (n) = u(n + 1) − u(n − 2), k=0

g(n) = u(n + 1) − u(n − 2) 5


X (−1)k+1
h) f (n) = δ(n − k)
d) f (n) = −n(1/4)n u(−n − 1), k+1
k=0
g(n) = u(n) g(n) = u(n) − u(n − 5)
6
Transformada de Fourier
Este capı́tulo presenta la Transformada de Fourier, también conocida como la Integral
de Fourier. El nombre de Fourier es en honor al fı́sico francés Jean Baptiste Fourier (1768-
1830). En Ingenierı́a, la transformada de Fourier es una herramienta sumamente útil.
Generalmente se emplea para pasar una señal al dominio de frecuencia para obtener in-
formación que no es evidente en el dominio del tiempo. Por ejemplo, se puede usar en el
ámbito de procesamiento de señales, para determinar en qué ancho de banda se concentra
la energı́a de una señal o también en el procesamiento digital de imágenes, para mejorar
ciertas zonas de una imagen fotográfica. El capı́tulo comienza con la definición matemática
de la Transforma de Fourier, luego se prueban todos los teoremas y propiedades. Segui-
damente se derivan las transformadas de Fourier de funciones comunes. Finalmente, se
calculan la magnitud y la fase de una función de dominio continuo.

6.1 Definición
Pasemos definir una de las herramientas matemáticas más importantes para la inge-
nierı́a como lo es la Transformada de Fourier.

Definición 6.1. Sea f (t) una función de dominio continuo definida de R en C. La


transformada de Fourier de f (t) es una transformada integral lineal definida por:
Z ∞
F (ω) = F{f (t)} = f (t) e−jωt dt,
−∞

para ω ∈ R, donde j denota la unidad imaginaria. A la variable ω, generalmente, se le


denomina frecuencia.

La notación de la definición anterior la usaremos en este y en los subsiguientes capı́tulos.


La transformada de Fourier, F (ω), es una función definida de R en C, es decir, es una fun-
ción que toma valores complejos. Por ello, se puede expresar como la suma de su parte
real e imaginaria, o en su forma polar, esto es,
F (ω) = Re {F (ω)} + j Im {F (ω)} = |F (ω)| ejφ(ω) ,
donde |F (ω)| se denomina magnitud y φ(ω) se denomina fase.
Si f (t) es una función generalizada, entonces F (ω) se denomina transformada de Fou-
rier generalizada. Para que la transformada de Fourier de una función f (t) exista en forma
ordinaria (no generalizada), f (t) debe satisfacer las siguientes propiedades denominadas
condiciones de Dirichlet:
• f (t) es absolutamente integrable, esto es,
Z ∞
|f (t)| dt < ∞;
−∞

185
186 6.1. DEFINICIÓN

• f (t) posee un número finito de discontinuidades en cualquier intervalo de longitud


finita.
En el siguiente ejemplo mostramos una función que no posee transformada de Fourier
ordinaria, pero si generalizada.
Ejemplo 6.1. Sea f (t) dada por
(
1, |t| ≤ 1,
f (t) = = 2u(−t − 1) + u(t + 1) + u(t − 1)
2, |t| > 1.

La transformada de Fourier ordinaria de f (t) no existe, ya que f (t) no es absolutamente


integrable. En cambio, f (t) si tiene transformada de Fourier generalizada, la cual está
Intente hallar dada por 
la transformada
de Fourier de
eω j j e−2 ω j − j
F (ω) = 4 π δ(ω) − .
f (t) usando la ω
definición


En el resto del capı́tulo, llamaremos transformada de Fourier de una función a su trans-


formada de Fourier generalizada. No seremos tan rigurosos exigiendo el cumplimiento de
las condiciones de Dirichlet.

6.1.1 Transformada Inversa de Fourier


La transformada de Fourier nos permite pasar del plano del tiempo al plano de la
frecuencia y realizar un análisis en frecuencia; pero luego es necesario trasladarnos nueva-
mente al plano del tiempo. Para esto último se utiliza la Transformada Inversa de Fourier.

Definición 6.2. Sea f (t) una función con transformada de Fourier F (ω). La transfor-
mada inversa de Fourier está definida como
Z ∞
−1 1
f (t) = F {F (ω)} = F (ω) ejωt dω,
2π −∞

para todo t ∈ R.

Ejemplo 6.2. Determine la transformada inversa de Fourier de F (ω) = δ(ω).


Solución. Por definición de la transformada inversa de Fourier podemos escribir:
Z ∞ Z ∞
1 jωt 1
f (t) = F (ω)e dω = δ(ω)ejωt dω
2π −∞ 2π −∞
Ahora, como ejωt = cos ωt + j sen ωt, entonces se obtiene que
Z ∞
1
f (t) = δ(ω)(cos ωt + j sen ωt) dω
2π −∞
Z ∞ Z ∞
1 1
= δ(ω) cos ωt dω + j δ(ω)sen ωt dω
2π −∞ 2π −∞
1 1
= (cos(0) + jsen (0)) = .
2π 2π

CAPÍTULO 6. TRANSFORMADA DE FOURIER 187

Observación 6.1. En general y con mucha frecuencia, para el cálculo de la transformada


inversa de Fourier se utiliza el procedimiento Inversión por Tablas, que se estudiará en el
Capı́tulo 7.
En el resto del capı́tulo usaremos la siguiente notación. El sı́mbolo

F
f (t) ←→ F (ω),

que se lee par de transformadas, denota que F (ω) es la transformada de Fourier de f (t) y
que f (t) es la transformada inversa de Fourier de F (ω).

6.2 Propiedades de la Transformada de Fourier


A continuación damos las propiedades de la transformada de Fourier. Cada una estas
propiedades la enunciaremos como un teorema.

Teorema 6.1 (Linealidad). Si F (ω) y G(ω) son las transformadas de Fourier de f (t) y
g(t), respectivamente, entonces
F
af (t) + bg(t) ←→ aF (ω) + bG(ω),

donde a y b son constantes reales.

Demostración. Por la definición de la transformada de Fourier se tiene que


Z ∞
F{af (t) + bg(t)} = (af (t) + bg(t)) e−jωt dt
−∞
Z ∞ Z ∞
−jωt
= a f (t) e dt + b g(t) e−jωt dt
−∞ −∞
= aF{f (t)} + bF{g(t)} = aF (ω) + bG(ω),

que era lo que deseábamos demostrar.

Teorema 6.2 (Desplazamiento en tiempo). Si F (ω) es la transformada de Fourier de


f (t), entonces
F
f (t − t0 ) ←→ e−jωt0 F (ω),
para todo t0 ∈ R. En otras palabras, la propiedad de desplazamiento de tiempo de la
transformada de Fourier establece que si desplazamos la función f (t) por una constante t0 ,
la magnitud de la transformada de Fourier no cambia, pero el término ωt0 se añade a su
ángulo de fase.

Demostración. Por la definición de la transformada de Fourier se tiene que


Z ∞
F{f (t − t0 )} = f (t − t0 ) e−jωt dt.
−∞
188 6.2. PROPIEDADES DE LA TRANSFORMADA DE FOURIER

Haciendo el cambio de variable r = t − t0 , la integral anterior adquiere la forma:


Z ∞
F{f (t − t0 )} = f (r) e−jω(r+t0 ) dr
−∞
Z ∞
−jωt0
= e f (r) e−jωr dr
−∞
= e−jωt0 F{f (t)}
= e−jωt0 F (ω),

con lo cual queda demostrado el teorema.

Teorema 6.3 (Desplazamiento en frecuencia). Si F (ω) es la transformada de Fourier de


f (t), entonces
F
ejω0 t f (t) ←→ F (ω − ω0 ),
para todo ω0 ∈ R. En otras palabras, la multiplicación de la función f (t) por ejω0 t , resulta
en un desplazamiento en la transformada de Fourier por ω0 .

Demostración. Se tiene que


Z ∞ Z ∞
jω0 t jω0 t −jωt
F{e f (t)} = e f (t) e dt = f (t) e−j(ω−ω0 )t dt = F (ω − ω0 ).
−∞ −∞

Teorema 6.4 (Escalamiento en tiempo). Si F (ω) es la transformada de Fourier de f (t),


entonces
F 1
f (at) ←→ F (ω/a),
|a|
para todo a ∈ R distinto de cero. En otras palabras, la propiedad de escalamiento en
tiempo establece que si se reemplaza la variable t por at, se debe reemplazar la variable ω
en el dominio de la frecuencia por ω/a y dividir a F (ω/a) por el valor absoluto de a.

Demostración. Consideremos los casos a > 0 y a < 0.


Para a > 0, Z ∞
F{f (at)} = f (at) e−jωt dt. (6.1)
−∞
Haciendo el cambio de variable r = at, la integral de (6.1) adquiere la forma:
Z
1 ∞ ω
F{f (at)} = f (r) e−j( a )r dr.
a −∞
Cambiando r por t en la integral se tiene:
Z
1 ∞ ω 1 1
F{f (at)} = f (t) e−j( a )t dt = F (ω/a) = F (ω/a).
a −∞ a |a|
Para a < 0, haciendo el cambio de variable r = at, la integral de (6.1) adquiere la forma:
Z Z
1 −∞ ω 1 ∞ ω
F{f (at)} = f (r) e−j( a )r dr = − f (r) e−j( a )r dr.
a ∞ a −∞
CAPÍTULO 6. TRANSFORMADA DE FOURIER 189

Cambiando r por t en la integral y considerando que a < 0, se tiene:


Z ∞
1 ω 1
F{f (at)} = f (t) e−j( a )t dt = F (ω/a).
(−a) −∞ |a|

Teorema 6.5 (Dualidad). Si F (ω) es la transformada de Fourier de f (t), entonces


F
F (t) ←→ 2π f (−ω).

Demostración. Como Z ∞
1
f (t) = F (ω) ejωt dω,
2π −∞
entonces Z ∞
2πf (−t) = F (ω) e−jωt dω.
−∞
Intercambiando t y ω, se obtiene
Z ∞
2πf (−ω) = F (t) e−jωt dt = F{F (t)}.
−∞

Teorema 6.6 (Conjugación). Si F (ω) es la transformada de Fourier de f (t), entonces


F
f (t) ←→ F (−ω).

Demostración. Se tiene que


n o Z ∞ Z ∞ Z ∞
−jωt
F f (t) = f (t) e dt = f (t) e−jωt dt = f (t) ejωt dt = F (−ω).
−∞ −∞ −∞

Teorema 6.7 (Convolución). Si F (ω) y G(ω) son las transformadas de Fourier de f (t) y
g(t), respectivamente, entonces
F
f ∗ g (t) ←→ F (ω) G(ω).

En otras palabras, la convolución en el dominio del tiempo corresponde a la multiplicación


en el dominio de la frecuencia.

Demostración. Se tiene que


Z ∞
F{f ∗ g (t)} = f ∗ g (t) e−jωt dt
−∞
Z ∞ Z ∞ 
= f (τ )g(t − τ ) dτ e−jωt dt
−∞ −∞
Z ∞ Z ∞ 
= f (τ ) g(t − τ ) e−jωt dt dτ.
−∞ −∞
190 6.2. PROPIEDADES DE LA TRANSFORMADA DE FOURIER

Haciendo el cambio de variable σ = t − τ , la ecuación anterior adquiere la forma:


Z ∞ Z ∞ 
−jω(σ+τ )
F{f ∗ g (t)} = f (τ ) g(σ) e dσ dτ
−∞ −∞
Z ∞  Z ∞ 
−jωτ −jωσ
= f (τ ) e dτ g(σ) e dσ = F (ω) G(ω).
−∞ −∞

Teorema 6.8 (Multiplicación). Si F (ω) y G(ω) son las transformadas de Fourier de f (t)
y g(t), respectivamente, entonces

F 1
f (t) g(t) ←→ F ∗ G (ω).

Es decir, la multiplicación en el dominio del tiempo, corresponde a la convolución en el
dominio de la frecuencia multiplicada por la constante 1/2π.

Demostración. Se tiene que


Z ∞
F{f (t) g(t)} = (f (t) g(t)) e−jωt dt
−∞
Z ∞ Z ∞ 
1
= F (σ) e dσ g(t) e−jωt dt
jσt
−∞ 2π −∞
Z ∞ Z ∞ 
1 −j(ω−σ)t
= F (σ) g(t) e dt dσ
2π −∞ −∞
Z ∞
1
= F (σ)G(ω − σ) dσ
2π −∞
1
= F ∗ G (ω).

Teorema 6.9 (Diferenciación en tiempo). Si F (ω) es la transformada de Fourier de f (t),


entonces
d F
f (t) ←→ jω F (ω).
dt
Demostración. Se tiene que
Z ∞
1
f (t) = F (ω) ejωt dω.
2π −∞

Derivando con respecto a t en ambos lados de la ecuación anterior, se obtiene:


 Z ∞ 
d d 1 jωt
f (t) = F (ω) e dω
dt dt 2π −∞
Z ∞
1 d  
= F (ω) ejωt dω
2π −∞ dt
Z ∞
1
= [jω F (ω)] ejωt dω = F −1 {jω F (ω)},
2π −∞
CAPÍTULO 6. TRANSFORMADA DE FOURIER 191

de donde se deduce que


d F
f (t) ←→ jω F (ω).
dt

Teorema 6.10 (Diferenciación en frecuencia). Si F (ω) es la transformada de Fourier de


f (t), entonces
F d
t f (t) ←→ j F (ω).

Demostración. Se tiene que Z ∞
F (ω) = f (t) e−jωt dt.
−∞
Derivando con respecto a ω en ambos lados de la ecuación anterior, se obtiene:
Z ∞ 
d d −jωt
F (ω) = f (t) e dt
dω dω
Z ∞ −∞
d  
= f (t) e−jωt dt
−∞ dω
Z ∞ 
−jωt
= (−j) [t f (t)] e dt = F{t f (t)},
−∞
de donde se deduce que
F d
t f (t) ←→ j F (ω).

Teorema 6.11 (Integración). Si F (ω) es la transformada de Fourier de f (t), entonces


Z t
F 1
f (τ ) dτ ←→ F (ω) + πF (0)δ(ω).
−∞ jω

Demostración. Para la prueba usaremos la transformada de Fourier del escalón unitario, a


saber:
1
F{u(t)} = + πδ(ω),

la cual la hallaremos más adelante. Ahora, se tiene que
Z ∞ Z t
f (t) ∗ u(t) = f (t)u(t − τ )dτ = f (τ ) dτ, (6.2)
−∞ −∞
para toda función f (t) de dominio continuo. De esta forma, usando la propiedad de con- Se deja al lector
la prueba de la
volución en la ecuación (6.2), obtenemos: ecuación (6.2)
F{f (t) ∗ u(t)} = F{f (t)} F{u(t)}.
Ahora, usando la expresión de la transformada de Fourier del escalón unitario y la propie-
dad de muestreo del impulso unitario, se obtiene
Z t   
1 1
F f (τ ) dτ = F (ω) + πδ(ω) = F (ω) + πF (0)δ(ω).
−∞ jω jω
192 6.2. PROPIEDADES DE LA TRANSFORMADA DE FOURIER

Teorema 6.12 (Modulación). Si F (ω) es la transformada de Fourier de f (t), entonces

F 1
cos(ω0 t) f (t) ←→ [F (ω + ω0 ) + F (ω − ω0 )].
2
y
F j
sen (ω0 t) f (t) ←→ [F (ω + ω0 ) − F (ω − ω0 )].
2
Demostración. Para la prueba usaremos las transformadas de Fourier de cos(ω0 t) y sen (ω0 t),
a saber:
F{cos(ω0 t)} = π[δ(ω − ω0 ) + δ(ω + ω0 )]
y
π
F{sen (ω0 t)} = [δ(ω − ω0 ) − δ(ω + ω0 )],
j
las cuales la hallaremos más adelante. Aplicando la propiedad de multiplicación
1
F{cos(ω0 t) f (t)} = F{cos(ω0 t)} ∗ F{f (t)}.

Ahora, usando la expresión de la transformada de Fourier de cos(ω0 t) y la propiedad dis-
tributiva de la convolución, se obtiene
1
F{cos(ω0 t) f (t)} = [δ(ω − ω0 ) ∗ F (ω) + δ(ω + ω0 ) ∗ F (ω)]
2
1
= [F (ω − ω0 ) + F (ω + ω0 )] .
2
Por un razonamiento similar se demuestra que
j
F{sen (ω0 t) f (t)} = [F (ω + ω0 ) − F (ω − ω0 )].
2
(Se deja al lector la prueba.)

Teorema 6.13 (Teorema de Parseval). Si F (ω) es la transformada de Fourier de f (t),


entonces Z ∞ Z ∞
1
|f (t)|2 dt = |F (ω)|2 dω.
−∞ 2π −∞

Demostración. Aplicando la propiedad de multiplicación, se puede escribir:


Z ∞
(f1 (t)f2 (t)) e−jσt dt = F{f1 (t)f2 (t)}
−∞
1
= F{F1 } ∗ F{F2 }
2π Z

1
= F1 (ω)F2 (σ − ω) dω. (6.3)
2π −∞
Como (6.2) se cumple para todo σ ∈ R, ella también es cierta para σ = 0, y bajo esta
condición (6.2) se reduce a
Z ∞ Z ∞
1
f1 (t)f2 (t) dt = F1 (ω)F2 (−ω) dω. (6.4)
−∞ 2π −∞
CAPÍTULO 6. TRANSFORMADA DE FOURIER 193

Ahora, tomando f1 (t) = f (t) y f2 (t) = f (t), y luego aplicando la propiedad de conjugación
F
f (t) ←→ F (−ω), de la ecuación (6.4) se deduce
Z ∞ Z ∞ Z ∞
2 1 1
|f (t)| dt = F (ω)F (ω) dω = |F (ω)|2 dω,
−∞ 2π −∞ 2π −∞

que era lo que deseábamos demostrar.

Las propiedades de la transformada de Fourier se resumen en la Tabla 6.1.

Tabla 6.1. Propiedades de la Transformada de Fourier

Propiedad Descripción Matemática


Linealidad F {a1 f (t) + a2 g(t)} = a1 F (ω) + a2 G(ω)

Desplazamiento en tiempo F {f (t − t0 )} = e−jωt0 F (ω)



Desplazamiento en frecuencia F ejω0 t f (t) = F (ω − ω0 )

1
Escalamiento en tiempo F {f (at)} = F (ω/a)
|a|

Dualidad F {F (t)} = 2πf (−ω)


n o
Conjugación F f (t) = F (−ω)

Convolución F {f ∗ g (t)} = F (ω)G(ω)

1
Multiplicación F {f (t)g(t)} = F ∗ G (ω)

 
dn
Diferenciación en tiempo F f (t) = (jω)n F (ω)
dtn
dn
Diferenciación en frecuencia F {tn f (t)} = j n F (ω)
dω n
 t 
Z  1
Integración F f (λ) dλ = F (ω) + πF (0)δ(ω)
  jω
−∞
1
Modulación F {cos(ω0 t) f (t)} = [F (ω + ω0 ) + F (ω − ω0 )]
2
j
F {sen (ω0 t) f (t)} = [F (ω + ω0 ) − F (ω − ω0 )]
2
Z ∞ Z ∞
1
Teorema de Parseval |f (t)|2 dt = |F (ω)|2 dω
−∞ 2π −∞
194 6.3. ALGUNOS PARES DE TRANSFORMADAS

6.3 Algunos Pares de Transformadas


En esta sección se calcula la transformada de Fourier de funciones comunes.

1.
F
δ(t) ←→ 1

Se tiene que
Z ∞
F{δ(t)} = δ(t) e−jωt dt
Z−∞

= δ(t)(cos(−ωt) + jsen (−ωt)) dt
Z−∞
∞ Z ∞
= δ(t) cos(ωt) dt − j δ(t) sin(ωt) dt
−∞ −∞
= cos(0) − jsen (0) = 1.

F
En la Figura 6.1 se aprecia gráficamente el par de transformadas δ(t) ←→ 1.

δ(t) F (ω)
1
1 F
←→

t ω

Figura 6.1. Transformada de Fourier de δ(t)

2.
F
1 ←→ 2π δ(ω)

Aplicando la propiedad de dualidad y considerando que δ(−t) = δ(t), se tiene

F{1} = 2πδ(−ω) = 2πδ(ω).

F
En la Figura 6.2 se aprecia gráficamente el par de transformadas 1 ←→ 2πδ(ω).

f (t) = 1 F (ω)
1 2πδ(ω)
F
←→

t ω

Figura 6.2. Transformada de Fourier de 1


CAPÍTULO 6. TRANSFORMADA DE FOURIER 195

3.
F
ejω0 t ←→ 2π δ(ω − ω0 )
Aplicando la propiedad de desplazamiento en frecuencia con f (t) = 1, se tiene
F{ejω0 t } = F{1}(ω − ω0 ) = 2πδ(ω − ω0 ).

4.
F 2
sgn(t) ←→

Para obtener la transformada de Fourier de la función signo, es necesario expresar a
sgn(t) como el siguiente lı́mite de exponenciales (la prueba de esta ecuación se deja
como ejercicio para el lector):
 
sgn(t) = lı́m e−at u(t) − eat u(−t) .
a→0

De esta forma, por la continuidad de la transformada de Fourier, podemos escribir:


n  o
F{sgn(t)} = F lı́m e−at u(t) − eat u(−t)
a→0
 
= lı́m F{e−at u(t)} − F{eat u(−t)}
a→0
Z ∞ Z ∞ 
= lı́m e−at u(t) e−jωt dt − eat u(−t) e−jωt dt
a→0 −∞ −∞
Z ∞ Z 0 
= lı́m e−(a+jω)t dt − e(a−jω)t dt
a→0
 0  −∞
1 1 1 1 2
= lı́m − = − = .
a→0 a + jω a − jω jω −jω jω

5.
F 1
u(t) ←→ + πδ(ω)

Para establecer la transformada de Fourier del escalón unitario, emplearemos la
siguiente relación (se deja al lector la verificación de la relación):
sgn(t) = 2u(t) − 1,
de donde se deduce:
sgn(t) 1
u(t) = + .
2 2
Ası́, empleando la última ecuación, se obtiene:
1 1
F{u(t)} = F{sgn(t)} + F{1}
2  2
1 2 1 1
= + (2πδ(ω)) = + πδ(ω).
2 jω 2 jω

6.
F 1
e−αt u(t) ←→
α + jω
Se tiene que
Z ∞ Z ∞
1
F{e−αt u(t)} = e−αt u(t) e−jωt dt = e−(α+jω)t dt = .
−∞ 0 α + jω
196 6.3. ALGUNOS PARES DE TRANSFORMADAS

7.
F π
sen (ω0 t) ←→ [δ(ω − ω0 ) − δ(ω + ω0 )]
j

Aplicando la propiedad de desplazamiento en frecuencia, se obtiene:

 
ejω0 t − e−jω0 t
F{sen (ω0 t)} = F
2j
1    
= F ejω0 t − F e−jω0 t
2j
1 π
= [2πδ(ω − ω0 ) − 2πδ(ω + ω0 )] = [δ(ω − ω0 ) − δ(ω + ω0 )].
2j j

En la Figura 6.3 se aprecia gráficamente la parte imaginaria de la transformada de


Fourier de sen (ω0 t).

sen (ω0 t) Im {F (ω)}

F
←→ ω0
t −ω0 ω

−π

Figura 6.3. Parte imaginaria de la transformada de Fourier de sen (ω0 t)

8.
F
cos(ω0 t) ←→ π [δ(ω − ω0 ) + δ(ω + ω0 )]

Aplicando la propiedad de desplazamiento en frecuencia, se obtiene:

 
ejω0 t + e−jω0 t
F{sen (ω0 t)} = F
2
1   jω0 t  
= F e + F e−jω0 t
2
1
= [2πδ(ω − ω0 ) + 2πδ(ω + ω0 )] = π [δ(ω − ω0 ) + δ(ω + ω0 )].
2

En la Figura 6.4 se aprecia gráficamente la transformada de Fourier de cos(ω0 t).


CAPÍTULO 6. TRANSFORMADA DE FOURIER 197

cos(ω0 t) F (ω)

π π

F
←→
−ω0 ω0 ω
t

Figura 6.4. Transformada de Fourier de cos(ω0 t)

9.
F sen (ωT )
pT (t) ←→ 2T
ωT
Como
pT (t) = u(t + T ) − u(t − T ),

entonces

F{pT (t)} = F{u(t + T )} − F{u(t − T )}


= ejωT F{u(t)} − e−jωT F{u(t)}
 
 jωT −jωT
 1
= e −e + πδ(ω)

 
1
= 2jsen (ωT ) + πδ(ω)

sen (ωT )
= 2 + 2πj sen (ωT )δ(ω)
ω
sen (ωT )
= 2T .
ωT

En la Figura 6.7 se aprecia gráficamente la transformada de Fourier de pT (t).

pT (t) F (ω)
1 1
F
←→

−T T t ω

Figura 6.5. Transformada de Fourier de pT (t)

10.
F 2a
e−a|t| ←→ , para a > 0
ω2 + a2
Como
e−a|t| = eat u(−t) + e−at u(t),
198 6.3. ALGUNOS PARES DE TRANSFORMADAS

entonces

F{e−a|t| } = F eat u(−t) + e−at u(t)
Z ∞ Z ∞
at −jωt
= e u(−t) e dt + e−at u(t) e−jωt dt
−∞ −∞
Z 0 Z ∞
at −jωt
= e e dt + e−at e−jωt dt
−∞ 0
Z 0 Z ∞
= e(a−jωt) dt + e−(a+jωt) dt
−∞ 0
1 1 2a
= + = 2 .
a − jω a + jω ω + a2

En la Figura 6.6 se aprecia gráficamente la transformada de Fourier de e−a|t| .

e−a|t| F (ω)
1 2/a
F
←→

t ω

Figura 6.6. Transformada de Fourier de e−a|t|

11.
1 F π −a|ω|
←→ e , a>0
a2 + t2 a
Usando las propiedades de linealidad y dualidad, se tiene
    
1 1 2a 1  −a|ω|
 π
F = F = 2πe = e−a|ω| .
a2 + t 2 2a a2 + t2 2a a

1
En la Figura 6.7 se aprecia gráficamente la transformada de Fourier de .
a2 + t2

F (ω)
1
a2 +t2
π/a
F
1/a 2
←→

t ω

1
Figura 6.7. Transformada de Fourier de
a2 + t2

En la Tabla 6.1 se muestra un resumen de los pares de transformadas de Fourier de


funciones comunes.
CAPÍTULO 6. TRANSFORMADA DE FOURIER 199

Tabla 6.2. Algunos Pares de Transformadas de Fourier

Función Transformada de Fourier


1 2πδ(ω)

δ(t) 1

1
u(t) πδ(ω) +

δ(t − t0 ) e−jωt0

2
sgn(t)

ejω0 t 2πδ(ω − ω0 )

cos (ω0 t) π[δ(ω − ω0 ) + δ(ω + ω0 )]

π
sen (ω0 t) [δ(ω − ω0 ) − δ(ω + ω0 )]
j
π jω
cos (ω0 t)u(t) [δ(ω − ω0 ) + δ(ω + ω0 )] + 2
2 ω0 − ω 2
π ω0
sen (ω0 t)u(t) [δ(ω − ω0 ) − δ(ω + ω0 )] + 2
2j ω0 − ω 2
1
e−at u(t), Re {a} > 0
a + jω
1
te−at u(t), Re {a} > 0
(a + jω)2
tn−1 −at 1
e u(t), Re {a} > 0
(n − 1)! (a + jω)n
2a
e−a|t| , a > 0
a2 + ω2
4ajω
|t|e−a|t| , Re {a} > 0
a2 + ω2
1 π −a|ω|
, Re {a} > 0 e
a2 + t2 a
t −jπωe−a|ω|
, Re {a} > 0
a + t2
2 2a
r
2 π −ω2 /4a
e−at , a > 0 e
a
200 6.4. MAGNITUD Y FASE

6.4 Magnitud y Fase


Sea f (t) una función de dominio continuo cuya transformada de Fourier es F (ω). Re-
cordemos que F (ω) es un número complejo para cada ω ∈ R, por lo que puede expresarse
en coordenadas polares,
F (ω) = |F (ω)| ejarg (F (ω)) ,

donde |F (ω)| y arg (F (ω)) son el valor absoluto y el argumento de la transformada de


Fourier de f (t), respectivamente. De esta representación en coordenadas polares surgen
las definiciones de magnitud y fase.

Definición 6.3 (Magnitud). Sea F (ω) la transformada de Fourier de f (t). La magnitud


o amplitud de f (t) se define como el valor absoluto de su transformada de Fourier, esto
es,
A(ω) = |F (ω)|, para cada ω ∈ R.
La función A(ω) se denomina espectro de magnitud de f (t).

Definición 6.4 (Fase). Sea F (ω) la transformada de Fourier de f (t). La fase de f (t) se
define como el argumento de su transformada de Fourier, esto es,

φ(ω) = arg (F (ω)), para cada ω ∈ R.

La función φ(ω) se denomina espectro de fase de f (t).

Para calcular la fase es necesario fijar una determinación de arg (F (ω)). En este escrito
la fase se calculará como el argumento principal, es decir, φ(ω) = Arg (F (ω)).
La siguiente proposición es un resultado de mucha utilidad para el cálculo de la mag-
nitud y la fase de una función. Esta proposición establece que si f (t) es una función real,
entonces la magnitud es una función par y la fase es una función impar.

Proposición 6.1. Si f (t) es una función real, entonces el espectro de magnitud A(ω) es
una función par, y el espectro de fase φ(ω) es una función impar.

Demostración. Por la definición de F (ω) se tiene que

F (ω) = Re {F (ω)} + jIm {F (ω)} ,

donde Z ∞
Re {F (ω)} = f (t) cos(ωt) dt (6.5)
−∞

e
Z ∞
Im {F (ω)} = − f (t) sen (ωt) dt. (6.6)
−∞

Ası́, los espectros de magnitud y fase de f (t) están dados respectivamente por:
p
A(ω) = |F (ω)| = (Re {F (ω)})2 + (Im {F (ω)})2 (6.7)
CAPÍTULO 6. TRANSFORMADA DE FOURIER 201

y



 0,   Re {F (ω)} > 0, Im {F (ω)} = 0,

 Im {F (ω)}


 arctan , Re {F (ω)} > 0, Im {F (ω)} > 0,


 Re {F (ω)}




 π/2,   Re {F (ω)} = 0, Im {F (ω)} > 0,

 Im {F (ω)}

arctan
 + π, Re {F (ω)} < 0, Im {F (ω)} > 0,
Re {F (ω)}
φ(ω) = (6.8)


 π,   Re {F (ω)} < 0, Im {F (ω)} = 0,

 Im {F (ω)}


 arctan − π, Re {F (ω)} < 0, Im {F (ω)} < 0,


 Re {F (ω)}




 −π/2,   Re {F (ω)} = 0, Im {F (ω)} < 0,

 Im {F (ω)}

arctan
 , Re {F (ω)} > 0, Im {F (ω)} < 0.
Re {F (ω)}

Veamos que A(ω) es par. Como la función real | · | es una función par y, por (6.5) y (6.6),
Re {F (ω)} y Im {F (ω)} son funciones reales (por ser f (t) una función real), entonces por
(6.7) la función A(ω) es par.
Veamos ahora que φ(ω) es impar. Por (6.5) y (6.6) se tiene que

Re {F (−ω)} = Re {F (ω)} e Im {F (−ω)} = −Im {F (ω)} .

De esta forma, por (6.8), se obtiene




0,   Re {F (ω)} > 0, Im {F (ω)} = 0,

 Im {F (ω)}


− arctan , Re {F (ω)} > 0, Im {F (ω)} < 0,


 Re {F (ω)}




π/2,   Re {F (ω)} = 0, Im {F (ω)} < 0,

 Im {F (ω)}

− arctan
 + π, Re {F (ω)} < 0, Im {F (ω)} < 0,
Re {F (ω)}
φ(−ω) =


π,   Re {F (ω)} < 0, Im {F (ω)} = 0,

 Im {F (ω)}


− arctan − π, Re {F (ω)} < 0, Im {F (ω)} > 0,


 Re {F (ω)}




−π/2,   Re {F (ω)} = 0, Im {F (ω)} > 0,

 Im {F (ω)}

− arctan
 , Re {F (ω)} > 0, Im {F (ω)} > 0.
Re {F (ω)}

= −φ(ω),

en otras palabras, φ(ω) es una función impar.

En los siguientes ejemplos se calculan los espectros de magnitud y fase de diferentes


funciones comunes.
202 6.4. MAGNITUD Y FASE

Ejemplo 6.3. Determine los espectros de magnitud y fase de la función

f (t) = e−2t u(t).

Solución. Se tiene que la transformada de Fourier de f (t) es

1
F (ω) = .
2 + jω

Luego,
A(ω)

1 1
A(ω) = =√
|2 + jω| 4 + ω2
ω
y
φ(ω)
π/2

 
1
φ(ω) = Arg = − arctan(ω/2).
2 + jω ω

−π/2

Ejemplo 6.4. Determine los espectros de magnitud y fase de la función

f (t) = u(t).

Solución. Se tiene que la transformada de Fourier de f (t) es

1
F (ω) = + π δ(ω).

Ası́,
A(ω)

1 1
A(ω) = + π δ(ω) = , para ω 6= 0
jω |ω|

ω
y
CAPÍTULO 6. TRANSFORMADA DE FOURIER 203

φ(ω)
 
1 π/2
φ(ω) = Arg + π δ(ω)

 
1
= Arg , ω 6= 0

( ω
π/2, ω < 0,
=
−π/2, ω > 0.
−π/2

Ejemplo 6.5. Determine los espectros de magnitud y fase de la función

f (t) = sgn(t).

Solución. Se tiene que la transformada de Fourier de f (t) es

2
F (ω) = .

Ası́,
A(ω)

2 2
A(ω) = = , para ω 6= 0
jω |ω|

ω
y
φ(ω)
π/2
 
2
φ(ω) = Arg , ω 6= 0

(
π/2, ω < 0, ω
=
−π/2, ω > 0.

−π/2

Ejemplo 6.6. Determine los espectros de magnitud y fase de la función

f (t) = δ(t).

Solución. Se tiene que la transformada de Fourier de f (t) es

F (ω) = 1.

Ası́,
204 6.4. MAGNITUD Y FASE

A(ω)
1
A(ω) = 1, para ω ∈ R

ω
y
φ(ω)

φ(ω) = Arg (1) = 0, para ω ∈ R.

ω


Ejemplo 6.7. Determine los espectros de magnitud y fase de la función

f (t) = e−|t| .

Solución. Se tiene que la transformada de Fourier de f (t) es

2
F (ω) =
1 + ω2

Ası́,
A(ω)
1
2
A(ω) = , para ω ∈ R
1 + ω2

ω
y
φ(ω)
 
2
φ(ω) = Arg = 0, para ω ∈ R.
1 + ω2
ω

CAPÍTULO 6. TRANSFORMADA DE FOURIER 205

6.5 Transformada de Fourier con M ATLAB


La herramientas de matemática simbólica de M ATLAB cuenta con el comando fourier
que permite calcular la transformada de Fourier de f (t). Observe el siguiente ejemplo
donde se calcula la transformada de Fourier de las siguientes funciones:

f1 (t) = e−2t u(t), f2 (t) = e−|t| , f3 (t) = p1 (t).


>> syms t w
>> f 1 ( t )=exp(−2∗ t )∗ h e a v i s i d e ( t ) ; F1 (w)= f o u r i e r ( f 1 )
F1 (w) =
1/(w∗ i + 2)
>> f 2 ( t )=exp(−abs ( t ) ) ; F2 (w)= f o u r i e r ( f 1 )
F2 (w) =
1/(w∗ i + 2)
>> f 3 ( t )= h e a v i s i d e ( t +1)−h e a v i s i d e ( t −1); F2 (w)= f o u r i e r ( f 3 )
F2 (w) =
− ( c o s (w)∗ i − s i n (w) ) /w + ( c o s (w)∗ i + s i n (w) ) /w
✂ ✁

Las transformadas de Fourier obtenidas son:


1 2 2 sin(ω)
F1 (ω) = , F2 (ω) = , F3 (ω) =
jω ω2 +1 ω

Programa 6.1. Función magnitudfase.m

f u n c t i o n [A , p h i ] = magnitu dfase ( f , w0)


%magnitu dfase H a l l a e l v a l o r de l a magnitud A(w) y de l a
% f a s e p h i (w) , en w = w0
% p = A(w0) , p h i = p h i (w0)
syms t w;
F (w) = f o u r i e r ( f ) ;
A = abs ( dou ble ( F (w0 ) ) ) ;
p h i = a n g l e ( dou ble ( F (w0 ) ) ) ;
end

El Programa 6.1 muestra la función magnitudfase que calcula la magnitud y la fase


de f (t) en ω = ω0 , es decir, calcula los valores A(ω0 ) y φ(ω0 ). Calculemos el valor de la
magnitud y la fase en ω = 1/2, de cada una de las funciones f1 (t), f2 (t) y f3 (t), dadas
arriba, en otras palabras, hallemos los valores de A(1/2) y φ(1/2) para cada una de estas
funciones.

>> syms t
>> w0 = 1/2;
>> f 1 ( t )=exp(−2∗ t )∗ h e a v i s i d e ( t ) ; [A1 , phi1 ] = magnitu dfase ( f1 , w0)
A1 =
0.4851
phi1 =
−0.2450
>> f 2 ( t )=exp(−abs ( t ) ) ; [ A2 , phi2 ] = magnitu dfase ( f2 , w0)
A2 =
1.6000
phi2 =
0
>> f 3 ( t )= h e a v i s i d e ( t +1)−h e a v i s i d e ( t −1); [A3 , phi3 ] = magnitu dfase ( f3 , w0)
206 6.5. TRANSFORMADA DE FOURIER CON MATLAB

A3 =
1.9177
phi3 =
0
✂ ✁
La función magnitudfase no sólo permite hallar el valor de la magnitud y la fase en
ω = ω0 , sino que también podemos usarla para obtener una gráfica de los espectros de
magnitud y fase de una función. A continuación, con las funciones f1 (t), f2 (t) y f3 (t)
creadas previamente, se hallan los espectros de magnitud y fase, para w ∈ (0, 10].

A (ω) φ (ω)
1 1
0.8 2

0.6 1

0.4 0

0.2 −1

0 −2
−10 −5 0 5 10 −10 −5 0 5 10

A (ω) φ (ω)
2 2
2 1

1.5 0.5

1 0

0.5 −0.5

0 −1
−10 −5 0 5 10 −10 −5 0 5 10

A (ω) φ (ω)
3 3
2 4

1.5 2

1 0

0.5 −2

0 −4
−10 −5 0 5 10 −10 −5 0 5 10

Para la construcción de las gráficas anteriores, tomamos en cuenta que las funciones con-
sideradas todas eran reales. Por ello, el espectro de magnitud es una función par y el
espectro de fase es impar. Seguidamente mostramos el código usado para la generación de
las gráficas.

>> syms t
>> w0 = 0 . 0 1 : 0 . 1 : 1 0 ;
>> f 1 ( t )=exp(−2∗ t )∗ h e a v i s i d e ( t ) ; [A1 , phi1]=magnitu dfase ( f1 , w0) ;
>> f 2 ( t )=exp(−abs ( t ) ) ; [ A2 , phi2]=magnitu dfase ( f2 , w0 ) ;
>> f 3 ( t )= h e a v i s i d e ( t +1)−h e a v i s i d e ( t −1); [A3 , phi3 ] = magnitu dfase ( f3 , w0) ;
>> s u b p l o t ( 3 , 2 , 1 ) , p l o t (−w0, A1 , ’ b ’ , ’ LineWidth ’ , 2 ) , hold on
>> p l o t (w0, A1 , ’ b ’ , ’ LineWidth ’ , 2 ) , t i t l e ( ’ A 1 (\ omega ) ’ ) , g r i d on
>> s u b p l o t ( 3 , 2 , 2 ) , p l o t (−w0,−phi1 , ’ b ’ , ’ LineWidth ’ , 2 ) , hold on
>> p l o t (w0, phi1 , ’ b ’ , ’ LineWidth ’ , 2 ) , t i t l e ( ’ \ p h i 1 (\ omega ) ’ ) , g r i d on
>> s u b p l o t ( 3 , 2 , 3 ) , p l o t (−w0, A2 , ’ b ’ , ’ LineWidth ’ , 2 ) , hold on
>> p l o t (w0, A2 , ’ b ’ , ’ LineWidth ’ , 2 ) , t i t l e ( ’ A 2 (\ omega ) ’ ) , g r i d on
>> s u b p l o t ( 3 , 2 , 4 ) , p l o t (−w0,−phi2 , ’ b ’ , ’ LineWidth ’ , 2 ) , hold on
>> p l o t (w0, phi2 , ’ b ’ , ’ LineWidth ’ , 2 ) , t i t l e ( ’ \ p h i 2 (\ omega ) ’ ) , g r i d on
>> s u b p l o t ( 3 , 2 , 5 ) , p l o t (−w0, A3 , ’ b ’ , ’ LineWidth ’ , 2 ) , hold on
>> p l o t (w0, A3 , ’ b ’ , ’ LineWidth ’ , 2 ) , t i t l e ( ’ A 3 (\ omega ) ’ ) , g r i d on
>> s u b p l o t ( 3 , 2 , 6 ) , p l o t (−w0,−phi3 , ’ b ’ , ’ LineWidth ’ , 2 ) , hold on
>> p l o t (w0, phi3 , ’ b ’ , ’ LineWidth ’ , 2 ) , t i t l e ( ’ \ p h i 3 (\ omega ) ’ ) , g r i d on
✂ ✁
CAPÍTULO 6. TRANSFORMADA DE FOURIER 207

M ATLAB también cuenta con el comando ifourier(F) que permite calcular la trans-
formada inversa de Fourier de F (ω). Observe el siguiente ejemplo donde se calcula la
transformada inversa de Fourier de las siguientes transformadas:
F1 (ω) = q1 (10ω), F2 (ω) = δ(ω) + 2 p1 (−ω), F3 (ω) = j sgn(2ω).

>> syms w t
>> u (w) = h e a v i s i d e (w) ;
>> F1=(1+10∗w) ∗ ( u(10∗w+1)−u(10∗w))+(1−10∗w) ∗ ( u(10∗w)−u(10∗w−1));
>> f 1 = s i m p l e ( i f o u r i e r ( F1 ) )
f1 =
(20∗ s i n ( x / 2 0 ) ˆ 2 ) / ( p i ∗ x ˆ2)
>> F2=d i r a c (w)+2∗(u(−w+1)−u(−w−1));
>> f 2 = s i m p l e ( i f o u r i e r ( F2 ) )
f2 =
((2∗ s i n ( x ) ) / x + 1/2)/ p i
>> F3=i ∗( u (2∗w)−u(−2∗w) ) ;
>> f 3 = s i m p l e ( i f o u r i e r ( F3 ) )
f3 =
−1/( p i ∗x )
✂ ✁
Las transformadas inversas de Fourier obtenidas son:
20 sin2 (t/20) 2 sin(t) 1 1
f1 (t) = , f2 (t) = + , f3 (t) = −
π t2 πt 2π πt

6.6 Problemas Resueltos


Problema 6.1. Determinar la transformada de Fourier del pulso rectangular p1 (t).

Solución. Hallaremos la transformada de Fourier por dos procedimientos: i) por defi-


nición, y ii) por propiedades.
i) Por definición, la transformada de Fourier de p1 (t) está dada por:
Z ∞ Z 1−  −jωt 1
e
F (ω) = p1 (t)e−jωt dt = e−jωt dt = −
−∞ −1+ jω −1
ejω − e−jω 2 sen ω
= =
jω ω

ii) Como p1 (t) = u(t + 1) − u(t − 1), entonces usando convenientemente las propiedades
de linealidad y desplazamiento en tiempo, obtenemos:
F (ω) = F {p1 (t)} = F {u(t + 1) − u(t − 1)}
= F {u(t + 1)} − F {u(t − 1)}
= ejω F {u(t)} − e−jω F {u(t)}
   
jω 1 −jω 1
= e + πδ(ω) − e + πδ(ω)
jω jω
ejω − e−jω 2 sen ω
= =
jω ω

208 6.6. PROBLEMAS RESUELTOS

Problema 6.2. Determinar la transformada de Fourier del pulso triangular q1 (t).

Solución. Hallaremos la transformada de Fourier por dos procedimientos: i) por defi-


nición, y ii) por propiedades.
i) Por definición, la transformada de Fourier de q1 (t) está dada por:
Z ∞
F (ω) = q1 (t)e−jωt dt
−∞
Z 0 Z 1
−jωt
= (t + 1)e dt + (1 − t)e−jωt dt
−1 0
1 − ejω + jω e−jω − 1 + jω
= −
ω2 ω2
2(1 − cos ω)
=
ω2
ii) Como

q1 (t) = (t + 1) [u(t + 1) − u(t)] + (1 − t) [u(t) − u(t − 1)]


= (t + 1)u(t + 1) − 2t u(t) + (t − 1)u(t − 1),

entonces usando convenientemente las propiedades de linealidad, desplazamiento en ti-


empo y diferenciación en frecuencia, obtenemos:

F (ω) = F {q1 (t)} = F {(t + 1)u(t + 1) − 2t u(t) − (1 − t)u(1 − t)}


= F {(t + 1)u(t + 1)} − 2F {tu(t)} + F {(t − 1)u(t − 1)}
= ejω F {tu(t)} − 2F {tu(t)} + e−jω F {tu(t)}
 
−jω −jω
 1 ′
= e −2+e − 2 + πjδ (ω)
ω
2(1 − cos ω)
=
ω2


Problema 6.3. Determinar la transformada de Fourier de la función f (t) = u(t) − u(t − 5).

Solución. Usando convenientemente las propiedades de linealidad y desplazamiento en


tiempo, obtenemos:

F (ω) = F {u(t) − u(t − 5)}


= F {u(t)} − F {u(t − 5)}
 
1 −5jω 1
= + πδ(ω) − e + πδ(ω)
jω jω
1 e−5jω
= + πδ(ω) − − e−5jω πδ(ω)
jω jω
1 − e−5jω
= .


CAPÍTULO 6. TRANSFORMADA DE FOURIER 209

Problema 6.4. Determinar la transformada de Fourier de la función f (t) = q1/2 (t − 1).

Solución. Primero calculemos por definición la transformada de Fourier de q1/2 (t).


Aplicando la definición de la trasformada de Fourier y considerando la identidad trigo-
nométrica 1 − cos(2α) = 2 sen 2 α, podemos escribir:
Z ∞

F q1/2 (t) = q1/2 (t)e−jωt dt
−∞
Z 0 Z 1/2
−jωt
= (2t + 1) e dt + (1 − 2t) e−jωt dt
−1/2 0

− jω
2 − 2e + jω 2e
2 − 2 + jω 2
= 2

ω ω2
1  jω jω

= 4 − 2e 2 − 2e− 2
ω2
4 8 sen 2 (ω/4)
= (1 − cos(ω/2)) = .
ω2 ω2
Ahora, usando la propiedad de desplazamiento en tiempo obtenemos:
  8 e−jω sen 2 (ω/4)
F (ω) = F q1/2 (t − 1) = e−jω F q1/2 (t) = .
ω2


Problema 6.5. Determinar la transformada de Fourier de la función f (t) = p1/2 ((t − 2)/2).

Solución. Primero calculemos por definición la transformada de Fourier de p1/2 (t). Se


tiene que
Z ∞ Z 1/2−
 −jωt 2 sen (ω/2)
F p1/2 (t) = p1/2 (t)e dt = e−jωt dt = .
−∞ −1/2+ ω

Ahora, usando convenientemente las propiedades de desplazamiento en tiempo y escala-


miento en tiempo, obtenemos:
     
1 −2jω t
F (ω) = F p1/2 (t − 2) =e F p1/2
2 2
 2e−2jω sen (ω)
= e−2jω 2 F p1/2 (t) (2ω) = .
ω


Problema 6.6. Determinar la transformada de Fourier de la función

f (t) = e2t u(−t) + u(t).

Solución. Usando la propiedad de linealidad obtenemos:


 
F (ω) = F e2t u(−t) + u(t) = F e2t u(−t) + F {u(t)}
 1
= F e2t u(−t) + + πδ(ω). (6.9)

210 6.6. PROBLEMAS RESUELTOS


Ahora, calculemos la transformada F e2t u(−t) . Sea g(t) la función definida como
g(t) = e−2t u(t).
Por definición, la transformada de Fourier de g(t) está dada por:
Z ∞ Z ∞ " #r
e−(2+jω)t 1
G(ω) = e−2t u(t)e−jωt dt = e−(2+jω)t dt = lı́m − = .
−∞ 0+ r→∞ 2 + jω 2 + jω
0

Ası́, usando la propiedad de escalamiento en tiempo, podemos escribir:


 1
F e2t u(−t) = F {g(−t)} = G(−ω) = . (6.10)
2 − jω
Usando convenientemente las ecuaciones (6.9) y (6.10), obtenemos:
1 1 2
F (ω) = + + πδ(ω) = + πδ(ω).
2 − jω jω jω(2 − jω)


Problema 6.7. Sea f (t) una función de dominio continuo cuya transformada de Fourier
está dada por:
F (ω) = p1 ((ω − 1)/2).
Determine la transformada de Fourier de g(t) = f (2t − 1) e−2jt .

Solución. Se tiene que la función g(t) se puede escribir como


 
1
g(t) = h t − e−2jt ,
2
donde h(t) = f (2t). Ası́, aplicando la propiedad de desplazamiento en frecuencia, se
obtiene   
1
G(ω) = F {g(t)} = F h t − (ω + 2).
2
Ahora, aplicando convenientemente las propiedades de desplazamiento en tiempo y esca-
lamiento en tiempo, podemos escribir:
  
1
F h t− = e−jω/2 F {h(t)}
2

= e−jω/2 F {f (2t)}
 
−jω/2 1
= e F (ω/2)
2

1 −jω/2
= e p1 ((ω − 2)/4).
2
De esta forma, la transformada de Fourier de g(t) es
1 −j(ω+2)/2
G(ω) = e p1 (ω/4)
2

CAPÍTULO 6. TRANSFORMADA DE FOURIER 211

Problema 6.8. Determine la transformada inversa de Fourier de F (ω) = u(ω) − δ(ω/2).

Solución. Aplicando la propiedad de dualidad podemos escribir:

F {F (t)} = 2πf (−ω). (6.11)

Por otra parte, aplicando convenientemente las propiedades de linealidad y escalamiento


en tiempo, tenemos:

F {F (t)} = F {u(t) − δ(t/2)} = F {u(t)} − F {δ(t/2)}


1
= + πδ(ω) − 2. (6.12)

Ahora, usando las ecuaciones (6.11) y (6.12), obtenemos:
1
2πf (−ω) = + πδ(ω) − 2.

Evaluando la expresión anterior en ω = −t y luego despejando f (t), tenemos que la trans-
formada inversa de Fourier de F (ω) está dada por:
 
1 1
f (t) = πδ(t) − 2 − .
2π jt


Problema 6.9. Determine la transformada inversa de Fourier de F (ω) = q1 (10ω).

Solución. Aplicando convenientemente la propiedad de escalamiento en tiempo, tenemos:


1 20(1 − cos(ω/10))
F {F (t)} = F {q1 (10t)} = F {q1 (t)} (ω/10) = . (6.13)
10 ω2
Ahora, usando las ecuaciones (6.11) y (6.13), obtenemos:
20(1 − cos(ω/10))
2πf (−ω) = .
ω2
Evaluando la expresión anterior en ω = −t y luego despejando f (t), tenemos que la trans-
formada inversa de Fourier de F (ω) está dada por:
10(1 − cos(t/10))
f (t) = .
πt2


Problema 6.10. Determine la transformada inversa de Fourier de F (ω) = δ(ω) + 2p1 (−ω).

Solución. Aplicando convenientemente las propiedades de linealidad y escalamiento en


tiempo, tenemos:

F {F (t)} = F {δ(t) + 2p1 (−t)} = F {δ(t)} + 2F {p1 (−t)}


= 1 + 2F {p1 (t)} (−ω)
4 sen ω
= 1+ . (6.14)
ω
212 6.6. PROBLEMAS RESUELTOS

Ahora, usando las ecuaciones (6.11) y (6.14), obtenemos:


4 sen ω
2πf (−ω) = 1 + .
ω
Evaluando la expresión anterior en ω = −t y luego despejando f (t), tenemos que la trans-
formada inversa de Fourier de F (ω) está dada por:
1 2 sen t
f (t) = +
2π πt


Problema 6.11. Determine la transformada inversa de Fourier de F (ω) = j sgn(2ω).

Solución. Aplicando convenientemente las propiedades de linealidad y escalamiento en


tiempo, tenemos:

F {F (t)} = F {j sgn(2t)} = jF {sgn(2t)}


j 2
= F {sgn(t)} (ω/2) = . (6.15)
2 ω
Ahora, usando las ecuaciones (6.11) y (6.15), obtenemos:
2
2πf (−ω) = .
ω
Evaluando la expresión anterior en ω = −t y luego despejando f (t), tenemos que la trans-
formada inversa de Fourier de F (ω) está dada por:
1
f (t) = − t−1 .
π


Problema 6.12. Determine los espectros de magnitud y fase del pulso rectangular p1 (t).

Solución. La transformada de Fourier de p1 (t) (ver Problema 6.1) es:


2 sen ω
F (ω) = .
ω
Luego, el espectro de magnitud de p1 (t) es:
2 |sen ω|
A(ω) = , para ω 6= 0.
|ω|

A(ω)
2

ω
−6π −5π −4π −3π −2π −π π 2π 3π 4π 5π 6π
CAPÍTULO 6. TRANSFORMADA DE FOURIER 213

Ahora, se tiene que

arg (F (ω)) = arg (2 sen ω) − arg (ω), para ω 6= 0.

Luego, el espectro de fase de p1 (t) es:





 0, ω < 0, sen ω < 0,

−π, ω < 0, sen ω > 0,
φ(ω) =


 π, ω > 0, sen ω < 0,

0, ω > 0, sen ω > 0,

φ(ω)

π
···
ω
−6π −5π −4π −3π −2π −π π 2π 3π 4π 5π 6π
···
−π

Problema 6.13. Determine los espectros de magnitud y fase del pulso triangular q1 (t).

Solución. La transformada de Fourier de q1 (t) (ver Problema 6.2) es:

2(1 − cos ω)
F (ω) = .
ω2
Luego, el espectro de magnitud de q1 (t) es:

2 (1 − cos ω)
A(ω) = , para ω 6= 0.
ω2

A(ω)
1

ω
−6π −5π −4π −3π −2π −π π 2π 3π 4π 5π 6π

Como 2(1 − cos ω) > 0 y ω 2 > 0, para todo ω ∈ R, entonces el espectro de fase de q1 (t) es:
 
2(1 − cos ω)
φ(ω) = Arg (F (ω)) = Arg = 0, para ω 6= 0.
ω2

214 6.6. PROBLEMAS RESUELTOS

Problema 6.14. Determine los espectros de magnitud y fase de f (t) = u(t) − u(t − 5).
Solución. La transformada de Fourier de f (t) = u(t) − u(t − 5) (ver Problema 6.3) es:
1 − e−5jω 1 − cos(5ω) + j sen (5ω)
F (ω) = = .
jω jω
Luego, el espectro de magnitud de f (t) es:
p p
(1 − cos(5ω))2 + sen 2 (5ω) 2 − 2 cos(5ω)
A(ω) = = , para ω 6= 0.
|ω| |ω|
A(ω)
5

ω
−6π −5π −4π −3π −2π −π π 2π 3π 4π 5π 6π

Ahora, se tiene que


arg (F (ω)) = arg (1 − e5jω ) − Arg (jω), para ω 6= 0.
ası́, (
arg (1 − cos(5ω) + j sen (5ω)) + π2 , ω < 0,
arg (F (ω)) =
arg (1 − cos(5ω) + j sen (5ω)) − π2 , ω > 0.
Además, 1 − cos(5ω) > 0, para todo ω ∈ R. De esta forma, el espectro de fase de f (t) es:
  
 sen (5ω) π
arctan
 + , ω < 0,
 1 − cos(5ω) 2
φ(ω) =  

 sen (5ω) π
arctan
 − , ω > 0.
1 − cos(5ω) 2
φ(ω)
π

···
ω
···
−π

Problema 6.15. Determine los espectros de magnitud y fase de f (t) = q1/2 (t − 1).
Solución. La transformada de Fourier de f (t) = q1/2 (t − 1) (ver Problema 6.4) es:
8 e−jω sen 2 (ω/4)
F (ω) = .
ω2
Luego, el espectro de magnitud de f (t) es:
8 sen 2 (ω/4)
A(ω) = , para ω 6= 0.
ω2
CAPÍTULO 6. TRANSFORMADA DE FOURIER 215

A(ω)

1/2

ω
−3π −2π −π π 2π 3π

8 sen 2 (ω/4)
Como ω2
> 0, para ω 6= 0, podemos escribir:
 
8 sen 2 (ω/4)
arg (F (ω)) = arg (e−jω ) + Arg = −ω, para ω 6= 0.
ω2
ası́, el espectro de fase de f (t) es, para ω 6= 0:

X ∞
X
φ(ω) = (2kπ − ω)pπ (ω − 2kπ) − (ω + 2kπ)pπ (ω + 2kπ).
k=0 k=1

φ(ω)
π

··· ···
ω
−5π −4π −3π −2π −π π 2π 3π 4π 5π

−π

Problema 6.16. Determine los espectros de magnitud y fase de f (t) = e2t u(−t) + u(t).

Solución. La transformada de Fourier de f (t) = e2t u(−t) + u(t) (ver Problema 6.6) es:
2
F (ω) = + πδ(ω).
jω(2 − jω)
Luego, el espectro de magnitud de f (t) es:
2
A(ω) = √ , para ω 6= 0.
|ω| 4 + ω 2

A(ω)

ω
−3π −2π −π π 2π 3π
216 6.7. PROBLEMAS PROPUESTOS

Ahora, se tiene que


arg (F (ω)) = Arg (2) − Arg (jw) − Arg (2 − jω)
= −Arg (jw) − Arg (2 − jω), para ω 6= 0.
Como (
−π/2, ω < 0,
Arg (jw) = y Arg (2 − jω) = − arctan(ω/2),
π/2, ω > 0,
entonces el espectro de fase de f (t) es, para ω 6= 0:
(
π/2 + arctan(ω/2), ω < 0,
φ(ω) =
−π/2 + arctan(ω/2), ω > 0,

φ(ω)
π
2

− π2

6.7 Problemas Propuestos


6.1. Obtener la transformada de Fourier de las siguientes funciones:

a) f (t) = et/2 u(−t) g) f (t) = π [δ(t − π) + δ(t + π)]


b) f (t) = p1 (t + 1) h) f (t) = u(−t − 1/2) q1 (t)
c) f (t) = q1/4 (t) i) f (t) = e−a|t| , a > 0
(
t + 4 sgn(t), si |t| > 0; 1
d) f (t) = j) f (t) = 2 , a > 0.
0, si t = 0 a + t2
e) f (t) = |t|u(t) k) f (t) = 1 + j sgn(t)
f) f (t) = 2πδ(t − t0 ), donde t0 ∈ R l) f (t) = p1 (t) + jq1/2 (t)

6.2. Sea f (t) una función de dominio continuo cuya transformada de Fourier es F (ω).
Pruebe que la transformada de Fourier de la función g(t) = f (t) cos ω0 t, con
ω0 ∈ R, está dada por
1
F{g(t)} = {f (t) cos ω0 t} = [F (ω − ω0 ) + F (ω + ω0 )].
2
6.3. Sea f (t) una función de dominio continuo cuya transformada de Fourier es
F (ω) = p1 ((ω − 1)/2).
Determine la transformada de Fourier de las siguientes funciones de dominio con-
tinuo, utilizando las propiedades de la transformada de Fourier:
CAPÍTULO 6. TRANSFORMADA DE FOURIER 217

a) g1 (t) = f (−t) h) g8 (t) = f (2t − 1)e−2jt


b) g2 (t) = t f (t) i) g9 (t) = f (t)e−2jt
c) g3 (t) = f (t + 1) j) g10 (t) = tf (t)e−2jt
d) g4 (t) = f (3 − 5t) k) g11 (t) = (t − 1)f (t − 1)e−2jt
Z t
e) g5 (t) = (t − 1) f (t + 1)
l) g12 (t) = f (τ ) dτ
d −∞
f) g6 (t) = f (t)
dt m) g13 (t) = f (t) sen (πt)
d
g) g7 (t) = t f (t) n) g14 (t) = f (t) ∗ δ(t − 1)
dt

6.4. Utilizando la relación


Z ∞ Z ∞
1
f (t)g(t) dt = F (ω)G(ω) dω
−∞ 2π −∞

y un par de transformadas conocida demostrar que


Z ∞
1 π
2 2 2
dt = 3 , a > 0.
0 (a + t ) 4a

6.5. Hallar la transformada inversa de Fourier de las siguientes transformadas:

a) F (ω) = u(ω) − δ(−ω/2).


b) F (ω) = q1 (ω/2).
c) F (ω) = δ(−ω) + 2p1 (−ω).
d) F (ω) = j sgn(−2ω).

6.6. Calcular los espectros de magnitud y fase de las siguientes funciones de dominio
continuo:

a) f (t) = et/2 u(−t) e) f (t) = p1/2 ((t − 2)/2)


b) f (t) = p1 (t + 1) f) f (t) = |t|u(t)
(
c) f (t) = q1/4 (t) t + 4 sgn(t), si |t| > 0;
g) f (t) =
d) f (t) = r(t) 0, si t = 0
7
Transformada de Laplace
En este capı́tulo se presenta la transformada de Laplace. El nombre de Laplace es
en honor a Pierre Simnon Laplace (1749-1827), matemático y astrónomo francés. Se
incluyen su definición, sus propiedades y los teoremas de valor inicial y final. Se derivan las
transformadas de Laplace de funciones comunes. Finalmente, se calcula la transformada
inversa de Laplace a través de dos métodos: integración de contornos e inversión por
tablas.

7.1 Definición
La transformada de Laplace puede considerarse como una generalización de la trans-
formada de Fourier; en forma más precisa, la adición de un factor exponencial al inte-
grando de la definición de la transformada de Fourier da como resultado la transformada
de Laplace bilateral o de dos lados.

Definición 7.1 (Transformada de Laplace Bilateral). Sea f (t) una función de dominio
continuo definida de R en C. La transformada de Laplace bilateral de f (t) se define como
Z ∞
F (s) = L[f (t)] = f (t) e−st dt,
−∞

para todo s ∈ C tal que |F (s)| < ∞.

Cambiando los lı́mites de integración en la definición de la transformada de Laplace


bilateral obtenemos las transformadas de Laplace unilaterales.

Definición 7.2 (Transformadas de Laplace Unilaterales). Sea f (t) una función de domi-
nio continuo definida de R en C. La transformada de Laplace unilateral derecha de f (t)
se define como Z ∞
LD (s) = f (t) e−st dt,
0
para todo s ∈ C tal que |LD (s)| < ∞.
La transformada de Laplace unilateral izquierda de f (t) se define como
Z 0
LI (s) = f (t) e−st dt,
−∞

para todo s ∈ C tal que |LI (s)| < ∞.

Observación 7.1. En general, la expresión matemática de la transformada de Laplace se


calcula utilizando la teorı́a de integración compleja.

218
CAPÍTULO 7. TRANSFORMADA DE LAPLACE 219

A lo largo del capı́tulo nos referiremos con transformada de Laplace a la transformada


de Laplace bilateral, en caso contrario lo indicaremos explı́citamente.
En la siguiente proposición se establece que si la transformada de Laplace existe, en-
tonces ella es una función analı́tica en su región de convergencia, de la cual hablaremos
más adelante. Para la demostración de este resultado, simplemente verificaremos que F (s)
satisface las condiciones necesarias y suficientes para ser derivable, esto es, las funciones
componentes de F (s) poseen derivadas parciales continuas, que satisfacen las ecuaciones
de Cauchy-Riemann.

Proposición 7.1. Si F (s) es la transformada de Laplace de la función f (t), entonces F (s)


es analı́tica en todo número complejo s tal que |F (s)| < ∞.

Demostración. Tomemos s = x+jy. Entonces, por definición de la transformada de Laplace


se tiene que
Z ∞ Z ∞ Z ∞
−st −xt
F (s) = f (t) e dt = f (t) e cos(yt) dt − j f (t) e−xt sen (yt) dt.
−∞ −∞ −∞

Esta ecuación nos indica que las funciones componentes de F (s) = u(x, y) + jv(x, y), son:
Z ∞ Z ∞
−xt
u(x, y) = f (t) e cos(yt) dt y v(x, y) = − f (t) e−xt sen (yt) dt.
−∞ −∞

Como u(x, y) y v(x, y) envuelven en su definición la integral de funciones exponenciales


y trigonométricas, entonces u(x, y) y v(x, y) poseen primeras derivadas parciales, con res-
pecto a x e y, continuas en todo s tal que |F (s)| < ∞. Demostremos que u(x, y) y v(x, y)
satisfacen las ecuaciones de Cauchy-Riemann. Se tiene que La derivada de
una integral es
Z ∞ Z ∞ igual a la inte-
∂u −xt ∂u
(x, y) = − [tf (t)] e cos(yt) dt, (x, y) = − [tf (t)] e−xt sen (yt) dt gral de la deri-
∂x −∞ ∂y −∞
vada del inte-
grando, por su-
Z Z puesto, cuando
∞ ∞
∂v −xt ∂v el integrando es
(x, y) = [tf (t)] e sen (yt) dt, (x, y) = − [tf (t)] e−xt cos(yt) dt continuamente
∂x −∞ ∂y −∞ diferenciable

Es claro que
∂u ∂v ∂u ∂v
(x, y) = (x, y) y (x, y) = − (x, y),
∂x ∂y ∂y ∂x
es decir, u(x, y) y v(x, y) satisfacen las ecuaciones de Cauchy-Riemann en todo s tal que
|F (s)| < ∞. Todo lo anterior nos indica que F (s) es derivable en el dominio {s ∈ C :
|F (s)| < ∞}. Por lo tanto, F (s) analı́tica en todo s ∈ C tal que |F (s)| < ∞.

7.1.1 Región de Convergencia

Definición 7.3. La región de convergencia de la transformada de Laplace de f (t), es el


conjunto de números complejos s donde F (s) existe, en otras palabras, son todos los
s ∈ C tales que Z ∞
|F (s)| = f (t) e−st dt < ∞. (7.1)
−∞
220 7.2. PROPIEDADES DE LA TRANSFORMADA DE LAPLACE

Para hallar la región de convergencia de la transformada de Laplace se realiza el si-


guiente procedimiento. Si tomamos s = σ + jω, entonces la ecuación (7.1) adquiere la
forma
Z ∞ Z ∞
−(σ+jω)
|F (s)| = f (t) e dt ≤ |f (t)| e−σt dt < ∞. (7.2)
−∞ −∞

Por lo tanto, los valores de σ = Re s que satisfacen la ecuación (7.2), determinan explı́cita-
mente la región de convergencia de F (s). Note que, por la Proposición 7.1, la transformada
de Laplace es analı́tica en su región de convergencia.

7.1.2 Transformada Inversa de Laplace

La transformada de Laplace nos permite pasar del plano complejo. Ahora para retornar
al plano del tiempo se utiliza la Transformada Inversa de Laplace.

Definición 7.4. Sea F (s) = L[f (t)], para todo s ∈ D ⊂ C, la transformada de Laplace
de f (t). La transformada inversa de Laplace es el proceso de obtener f (t) a través de
F (s) y se define como
Z σ+j∞
1
f (t) = F (s) est ds. (7.3)
2πj σ−j∞

La integral de la ecuación (8.1) se evalúa en la recta del plano complejo σ + jω, desde
σ − j∞ hasta σ + j∞, siendo σ un número real fijo tal que la recta Re s = σ esté en el
interior de D. En este trabajo, se utilizarán dos métodos para calcular la transformada
inversa de Laplace: Integración de Contornos e Inversión por Tablas, los cuales trataremos
más adelante.
En el resto del capı́tulo usaremos la siguiente notación. El sı́mbolo

L
f (t) ←→ F (s), s∈D

que se lee par de transformadas, denota que F (s) es la transformada de Laplace de f (t)
con región de convergencia el conjunto D ⊂ C, y que f (t) es la transformada inversa de
Laplace de F (s).

7.2 Propiedades de la Transformada de Laplace

A continuación damos las propiedades de la transformada de Laplace.

Teorema 7.1 (Linealidad). Si las transformadas de Laplace de las funciones f (t) y g(t)
son respectivamente F (s), para todo s ∈ Df ⊂ C y G(s), para todo s ∈ Dg ⊂ C, entonces

L
af (t) + bg(t) ←→ aF (s) + bG(s), s ∈ Df ∩ Dg ,

donde a y b son constantes reales.


CAPÍTULO 7. TRANSFORMADA DE LAPLACE 221

Demostración. Por la definición de la transformada de Laplace se tiene que


Z ∞
L{af (t) + bg(t)} = (af (t) + bg(t)) e−st dt
−∞
Z ∞ Z ∞
−st
= a f (t) e dt + b g(t) e−st dt
−∞ −∞
= aL{f (t)} + bL{g(t)} = aF (s) + bG(s).

Ahora, la región de convergencia de L{af (t) + bg(t)} son todos los s = σ + jω tales que
Z ∞
|af (t) + bg(t)| e−σt dt < ∞,
−∞

pero
Z ∞ Z ∞ Z ∞
|af (t) + bg(t)| e−σt dt, ≤ |a| |f (t)| e−σt dt, +|b| |g(t)| e−σt dt. (7.4)
−∞ −∞ −∞

Como Df y Dg son las regiones de convergencia de f y g, respectivamente, entonces se


tiene que Z ∞
|f (t)| e−σt dt < ∞, para todo s = σ + jω ∈ Df (7.5)
−∞
y Z ∞
|g(t)| e−σt dt, para todo s = σ + jω ∈ Dg (7.6)
−∞

Ası́, por (7.4), (7.5) y (7.6), la región de convergencia de L{af (t) + bg(t)} son todos los
s = σ + jω tales que s ∈ Df ∩ Dg .

Teorema 7.2 (Desplazamiento en tiempo). Si la transformada de Laplace de la función


f (t) es F (s) para todo s ∈ D ⊂ C, entonces
L
f (t − t0 ) ←→ e−t0 s F (s), s ∈ D,

para todo t0 ∈ R.

Demostración. Por la definición de la transformada de Laplace se tiene que


Z ∞
L{f (t − t0 )} = f (t − t0 ) e−st dt.
−∞

Haciendo el cambio de variable r = t − t0 , la integral anterior adquiere la forma:


Z ∞
L{f (t − t0 )} = f (r) e−s(r+t0 ) dr
−∞
Z ∞
−t0 s
= e f (r) e−sr dr
−∞
−t0 s
= e L{f (t)}
−t0 s
= e F (s).
222 7.2. PROPIEDADES DE LA TRANSFORMADA DE LAPLACE

Ahora, la región de convergencia de L{f (t − t0 )} son todos los s = σ + jω tales que


Z ∞
|f (t − t0 )| e−σt dt < ∞.
−∞

Haciendo el cambio de variable r = t − t0 , la ecuación anterior adquiere la forma:


Z ∞ Z ∞
−t0 σ −rt
e |f (r)| e dt = |f (r)| e−rt dt < ∞;
−∞ −∞

en otras palabras, la región de convergencia de L{f (t − t0 )} es D.

Teorema 7.3 (Desplazamiento en el dominio de s). Si la transformada de Laplace de la


función f (t) es F (s) para todo s ∈ D ⊂ C, entonces
L
es0 t f (t) ←→ F (s − s0 ), (s − s0 ) ∈ D,

para todo s0 ∈ C.

Demostración. Se tiene que


Z ∞ Z ∞
s0 t s0 t −st
L{e f (t)} = e f (t) e dt = f (t) e−(s−s0 )t dt = F (s − s0 ).
−∞ −∞

Ahora, la región de convergencia de L{es0 t f (t)} son todos los s = σ + jω tales que
Z ∞
es0 t f (t) e−σt dt < ∞.
−∞

Tomando s0 = σ0 + jω0 , la ecuación anterior adquiere la forma


Z ∞ Z ∞
jω0 t −(σ−σ0 )t
|f (t)| e e dt = |f (t)| e−(σ−σ0 )t dt < ∞,
−∞ −∞

de lo cual se deduce que la región de convergencia de L{es0 t f (t)} son todos los s = σ + jω
tales que (s − s0 ) ∈ D.

Teorema 7.4 (Escalamiento en tiempo). Si la transformada de Laplace de la función f (t)


es F (s) para todo s ∈ D ⊂ C, entonces

L 1
f (at) ←→ F (s/a), (s/a) ∈ D,
|a|

para todo a ∈ R.

Demostración. Por la definición de la transformada de Laplace,


Z ∞
L{f (at)} = f (at) e−st dt. (7.7)
−∞

Haciendo el cambio de variable r = at, la integral de (7.7) adquiere la forma:


Z ∞
1 s
L{f (at)} = f (r) e−( a )r dr.
|a| −∞
CAPÍTULO 7. TRANSFORMADA DE LAPLACE 223

Cambiando r por t en la integral se tiene:


Z ∞
1 s 1 1
L{f (at)} = f (t) e−( a )t dt = F (s/a) = F (s/a).
|a| −∞ a |a|

Ahora, la región de convergencia de L{f (at)} son todos los s = σ + jω tales que
Z ∞
|f (at)| e−σt dt < ∞.
−∞

Haciendo el cambio de variable r = at, la ecuación anterior adquiere la forma:


Z ∞
1 σ
f (t) e−( a )r dt < ∞,
|a| −∞

de lo cual se deduce que la región de convergencia de L{f (at)} son todos los s = σ + jω
tales que (s/a) ∈ D.

Teorema 7.5 (Conjugación). Si la transformada de Laplace de la función f (t) es F (s)


para todo s ∈ D ⊂ C, entonces
L
f (t) ←→ F (s), s ∈ D.

Demostración. Como es = es , se tiene que


n o Z ∞ Z ∞ Z ∞
−st −st
L f (t) = f (t) e dt = f (t) e dt = f (t) e−st dt = F (s).
−∞ −∞ −∞
n o
Ahora, la región de convergencia de L f (t) son todos los s = σ + jω tales que
Z ∞ Z ∞
−σt
f (t) e dt = |f (t)| e−σt dt < ∞,
−∞ −∞
n o
en otras palabras, la región de convergencia de L f (t) son todos los s = σ + jω tales
que s ∈ D.

Teorema 7.6 (Convolución). Si las transformadas de Laplace de las funciones f (t) y g(t)
son respectivamente F (s), para s ∈ Df ⊂ C y G(s), para s ∈ Dg ⊂ C, entonces

L
f ∗ g (t) ←→ F (s) G(s), s ∈ Df ∩ Dg .

Demostración. Se tiene que


Z ∞
L{f ∗ g (t)} = f ∗ g (t) e−st dt
−∞
Z ∞ Z ∞ 
= f (τ )g(t − τ ) dτ e−st dt
−∞ −∞
Z ∞ Z ∞ 
= f (τ ) g(t − τ ) e−st dt dτ.
−∞ −∞
224 7.2. PROPIEDADES DE LA TRANSFORMADA DE LAPLACE

Haciendo el cambio de variable r = t − τ , la ecuación anterior adquiere la forma:


Z ∞ Z ∞ 
−s(r+τ )
L{f ∗ g (t)} = f (τ ) g(r) e dr dτ
−∞ −∞
Z ∞  Z ∞ 
= f (τ ) e−sτ dτ g(σ) e−sr dr = F (s) G(s).
−∞ −∞

Ahora, la región de convergencia de L{f ∗ g (t)} son todos los s = σ + jω tales que
Z ∞ Z ∞ Z ∞
|f ∗ g (t)| e−σt dt = f (τ )g(t − τ ) dτ e−σt dt < ∞.
−∞ −∞ −∞

Haciendo el cambio de variable r = t − τ , la ecuación anterior adquiere la forma:


Z ∞  Z ∞ 
−στ −σr
|f (τ )| e dτ |g(σ)| e dr < ∞,
−∞ −∞

en otras palabras, la región de convergencia de L{f ∗ g (t)} son todos los s = σ + jω tales
que s ∈ Df ∩ Dg .

Teorema 7.7 (Diferenciación en el dominio del tiempo). Si la transformada de Laplace


de una función f (t) derivable es F (s) para todo s ∈ D ⊂ C, entonces

d L
f (t) ←→ s F (s),
dt
cuya región de convergencia contiene a D.

Demostración. Asumamos que f (t) es de orden exponencial, esto es, existen constantes
K > 0 y a ∈ R tales que
|f (t)| ≤ K eat , para todo t ∈ R.
Además, si f (t) es de orden exponencial, entonces (se deja como ejercicio para el lector la
prueba de esta ecuación):
 α
lı́m f (t)e−st −α = 0, para todo s ∈ D. (7.8)
α→∞

Se tiene   Z ∞
d d
L f (t) = f (t) e−st dt.
dt −∞ dt
Ası́, usando la ecuación (7.8) y la integración por partes con u = e−st y dv = f ′ (t), se
obtiene   Z ∞
d  
−st α
L f (t) = lı́m f (t)e −α
+s f (t) e−st dt = sF (s).
dt α→∞ −∞
Además, de esta última  se deduce que si s ∈ D, entonces s pertenece a la región
 ecuación
d
de convergencia de L f (t) .
dt
d
Observación 7.2. Generalmente, la región de convergencia de L dt f (t) coincide con la
 d de L {f (t)}, pero existen funciones f (t) tales que la región de
región de convergencia
convergencia de L dt f (t) contiene a la región de convergencia de L {f (t)}.
CAPÍTULO 7. TRANSFORMADA DE LAPLACE 225

Teorema 7.8 (Diferenciación en el dominio de s). Si la transformada de Laplace de la


función f (t) es F (s) para todo s ∈ D ⊂ C, entonces

L d
t f (t) ←→ − F (s), s ∈ D.
ds
Demostración. Se tiene que Z ∞
F (s) = f (t) e−st dt.
−∞
Derivando con respecto a s en ambos lados de la ecuación anterior, se obtiene:
Z ∞ 
d d −st
F (s) = f (t) e dt
ds ds −∞
Z ∞
d  
= f (t) e−st dt
ds
Z−∞∞
= [−tf (t)] e−st dt = −L {t f (t)} ,
−∞

d
de donde se deduce que L {t f (t)} = − F (s). Además, como F (s) es analı́tica en todo
ds
d
s ∈ D, entonces F (s) también es analı́tica en todo s ∈ D, en otras palabras, D es la
ds
d
región de convergencia de F (s).
ds

Teorema 7.9 (Integración). Si la transformada de Laplace de la función f (t) es F (s) para


todo s ∈ D ⊂ C, entonces Z t
L 1
f (τ ) dτ ←→ F (s),
−∞ s
cuya región de convergencia es igual al conjunto D o está contenida en él.

Demostración. Para la prueba usaremos la transformada de Laplace del escalón unitario, a


saber:
1
L{u(t)} = , Re s > 0,
s
la cual la hallaremos más adelante. Recordemos que
Z ∞ Z t
f (t) ∗ u(t) = f (t)u(t − τ )dτ = f (τ ) dτ, (7.9)
−∞ −∞

para toda función f (t) de dominio continuo. De esta forma, usando la propiedad de con-
volución en la ecuación (7.9)

L{f (t) ∗ u(t)} = L{f (t)} L{u(t)}, s ∈ {s : Re s > 0} ∩ D.

Ahora, usando la expresión de la transformada de Laplace del escalón unitario, se obtiene


Z t   
1 1
L f (τ ) dτ = F (s) = F (s), s ∈ {s : Re s > 0} ∩ D.
−∞ s s
226 7.2. PROPIEDADES DE LA TRANSFORMADA DE LAPLACE

Teorema 7.10 (Teorema del Valor Inicial). Sea f (t) una función continua a trozos en R
tal que f (t) = 0 para t < 0. Si la transformada de Laplace de la función f (t) es F (s) para
todo s ∈ D ⊂ C, entonces
f (0+ ) = lı́m s F (s).
s→∞

Demostración. Como f (t) = 0 para t < 0, entonces


  Z ∞
d + d
L f (t) = s F (s) − f (0 ) = f (t) e−st dt. (7.10)
dt 0 dt

Tomando lı́mite en ambos lados de (7.10), cuando s → ∞, se obtiene


Z ∞  " Z α #
+ d −st d −st
lı́m [s F (s) − f (0 )] = lı́m f (t) e dt = lı́m lı́m f (t) e dt .
s→∞ s→∞ 0 dt s→∞ α→∞ ε dt
ε→0

Intercambiando el orden de los lı́mites, se tiene


Z α h i
+ d
lı́m [s F (s) − f (0 )] = lı́m f (t) lı́m e−st dt
s→∞ α→∞ ε
ε→0
dt s→∞

y como
lı́m e−st = 0,
s→∞

la expresión de arriba se reduce a

lı́m [s F (s) − f (0+ )] = 0


s→∞

o, equivalentemente,
f (0+ ) = lı́m s F (s).
s→∞

Teorema 7.11 (Teorema del Valor Final). Sea f (t) una función continua a trozos en R
tal que f (t) = 0 para t < 0. Si la transformada de Laplace de la función f (t) es F (s) para
todo s ∈ D ⊂ C, entonces
lı́m f (t) = lı́m s F (s).
t→∞ s→0

Demostración. Tomando lı́mite en ambos lados de (7.10), cuando s → 0, se obtiene


Z ∞  " Z α #
d d
lı́m [s F (s) − f (0+ )] = lı́m f (t) e−st dt = lı́m lı́m f (t) e−st dt .
s→0 s→0 0 dt s→0 α→∞ ε dt
ε→0

Intercambiando el orden de los lı́mites, se tiene


Z α h i
+ d
lı́m [s F (s) − f (0 )] = lı́m f (t) lı́m e−st dt
s→0 α→∞ ε
ε→0
dt s→0

y como
lı́m e−st = 1,
s→0
CAPÍTULO 7. TRANSFORMADA DE LAPLACE 227

la expresión de arriba se reduce a


Z α Z α
d
lı́m [s F (s) − f (0+ )] = lı́m f (t) dt = lı́m f (t) dt = f (∞) − f (0− ) = f (∞)
s→0 α→∞ ε dt α→∞ ε
ε→0 ε→0

o, equivalentemente,
lı́m f (t) = lı́m s F (s).
t→∞ s→∞

Las propiedades de la transformada de Laplace se resumen en la Tabla 7.1.

Tabla 7.1. Propiedades de la Transformada de Laplace

Sean f (t) y g(t) dos funciones de dominio continuo con transformada de Laplace F (s),
para s ∈ Df , y G(s), para s ∈ Dg , respectivamente.

Propiedad Descripción Matemática


Linealidad L {a1 f (t) + a2 g(t)} = a1 F (s) + a2 G(s), s ∈ Df ∩ Dg

Desplazamiento en tiempo L {f (t − t0 )} = e−st0 F (s), s ∈ Df



Desplazamiento en s L es0 t f (t) = F (s − s0 ), {s ∈ C : (s − s0 ) ∈ Df }

1
Escalamiento en tiempo L {f (at)} = F (s/a), s ∈ {s ∈ C : (s/a) ∈ Df }
|a|
n o
Conjugación L f (t) = F (s), s ∈ Df

Convolución L {f ∗ g (t)} = F (s)G(s), s ∈ Df ∩ Dg


 
d
Diferenciación en tiempo L f (t) = s F (s), s ∈ Df
dt
d
Diferenciación en s L {t f (t)} = − F (s), s ∈ Df
ds
 t 
Z  1
Integración L f (λ) dλ = F (s), s ∈ D ⊇ Df
  s
−∞

Teorema del valor inicial lı́m sF (s) = f (0+ )


s→∞

Teorema del valor final lı́m f (t) = lı́m sF (s)


t→∞ s→0
228 7.3. ALGUNOS PARES DE TRANSFORMADAS

7.3 Algunos Pares de Transformadas


En esta sección se calcula la transformada de Laplace de funciones comunes.

1.
L
δ(t) ←→ 1, s∈C
Se tiene que
Z ∞ Z ∞
L{δ(t)} = δ(t) e−σt e−jωt dt
δ(t) e−st dt =
Z−∞

−∞
Z ∞
−σt
= δ(t) e cos(ωt) dt − j δ(t) e−σt sen (ωt) dt
−∞ −∞
−σ·0 −σ·0
= e cos(ω · 0) − je sen (ω · 0) = 1.

La región de convergencia de L{δ(t)} son todos los s = σ + jω tales que


Z ∞
δ(t)(t) e−σt dt < ∞,
−∞

pero Z ∞
δ(t)(t) e−σt dt = 1;
−∞

por lo tanto, la región de convergencia de L{δ(t)} es todo C.

2.
L 1
u(t) ←→ , Re s > 0
s
Se tiene que
Z ∞ Z ∞   ∞
−st −st e−st 1
L{u(t)} = u(t) e dt = e dt = − = .
−∞ 0+ s 0+ s

Ahora, como
Z ∞ Z ∞
−σt
|u(t)| e dt = e−σt dt < ∞ ⇔ σ > 0,
−∞ 0+

entonces la región de convergencia de L{u(t)} son todos los s = σ + jω ∈ C tales


que Re s = σ > 0.

3.
L 1
−u(−t) ←→ , Re s < 0
s
Hallemos la transformada de Laplace L{−u(−t)} mediante dos procedimientos:
(a) por definición, y (b) usando las propiedades de la transformada de Laplace.
(a) Se tiene que
Z ∞ Z 0−   0−
−st −st e−st 1
L{−u(−t)} = [−u(−t)] e dt = − e dt = = .
−∞ ∞ s ∞ s
CAPÍTULO 7. TRANSFORMADA DE LAPLACE 229

Como Z ∞ Z 0−
−σt
| − u(−t)| e dt = e−σt dt < ∞ ⇔ σ < 0,
−∞ ∞
entonces la región de convergencia de L{−u(−t)} son todos los s = σ + jω ∈ C
tales que Re s = σ < 0.
(b) Usando las propiedades de linealidad y escalamiento en tiempo, se obtiene
 
1 1
L{−u(−t)} = −L{u(−t)} = − − L{u(t)}(−s) = , Re (−s) > 0,
| − 1| s
o, equivalentemente,
1
L{−u(−t)} = , Re s < 0.
s
Observación 7.3. Note que las transformadas de Laplace de u(t) y −u(−t) son alge-
braicamente iguales, pero tienen regiones de convergencia diferentes, más aún, son
complementarias.

4.
1 L
e−αt u(t) ←→, Re s > −Re α
s+α
Usando la propiedad de desplazamiento en el dominio de s, se obtiene
1
L{e−αt u(t)} = L{u(t)}(s − (−α)) = , Re (s + α) > 0,
s+α
o, equivalentemente,
1
L{e−αt u(t)} = , Re s > −α.
s+α
La transformada de Laplace de e−αt u(t) también se puede hallar por definición, lo
cual se deja como ejercicio para el lector.

5.
L 1
t u(t) ←→ , Re s > 0
s2
Se tiene que Z Z
∞ ∞
−st
L{t u(t)} = [t u(t)] e dt = te−st dt.
−∞ 0+
Usando integración por partes, se obtiene
 ∞ Z
te−st 1 ∞ −st
L{t u(t)} = − + e dt
s 0+ s 0+
 −st  ∞
1 e 1
= 0+ − = 2.
s s 0+ s
Como
Z ∞ Z ∞
−σt
|t u(t)| e dt = t e−σt dt
−∞ 0+
Z ∞
1
= e−σt dt < ∞ ⇔ σ > 0,
σ 0+
230 7.3. ALGUNOS PARES DE TRANSFORMADAS

entonces la región de convergencia de L{t u(t)} son todos los s = σ + jω ∈ C tales


que Re s = σ > 0.
La transformada de Laplace de t u(t) también se puede hallar usando la propiedad
de diferenciación en s, lo cual se deja como ejercicio para el lector.

6.
L 1
sen t u(t) ←→ , Re s > 0
s2 +1
Se tiene que
Z ∞
L{sen t u(t)} = [sen t u(t)] e−st dt
−∞
Z ∞  jt 
e − e−jt
= e−st dt
0 + 2j
Z ∞ Z ∞ 
1 (j−s)t −(j+s)t
= e dt − e dt
2j 0+ +
"" #∞ "0 # ∞#
1 e(j−s)t e−(j+s)t
= − −
2j j−s j+s
0+ +
   0
1 1 1 1 2j 1
= − = = 2 .
2j s − j s+j 2j s2 + 1 s +1

Como Z ∞ Z ∞
−σt
|sen t u(t)| e dt ≤ e−σt dt < ∞ ⇔ σ > 0,
−∞ 0+

entonces la región de convergencia de L{sen t u(t)} son todos los s = σ + jω ∈ C


tales que Re s = σ > 0.

7.
L s
cos t u(t) ←→ , Re s > 0
s2 + 1
Usando las propiedades de linealidad y desplazamiento en el dominio de s, se ob-
tiene
 jt  
e + e−jt
L{cos t u(t)} = L u(t)
2
1 
= L{ejt u(t)} + L{e−jt u(t)}
2
1
= [L{u(t)}(s + j) + L{u(t)}(s − j)]
2 
1 1 1
= +
2 s+j s−j
s
= 2
, {s ∈ C : Re (s + j) > 0} ∩ {s ∈ C : Re (s − j) > 0}
s +1
o, equivalentemente,
s
L{cos t u(t)} = , Re s > 0.
s2 +1
CAPÍTULO 7. TRANSFORMADA DE LAPLACE 231

8.
L 2jT senh(T s)
pT (t) ←→ , Re s 6= 0
Ts
Se tiene que
Z ∞
L{pT (t)} = pT (t) e−st dt
−∞
Z T−
= e−st dt
−T +
  T−
e−st
= −
s −T +
esT− e−sT
=
s
2jT sen (sT )
= , s 6= 0.
sT
Como
Z ∞ Z T−
eσT − e−σT
|pT (t)| e−σt dt = e−σt dt = <∞ ⇔ σ 6= 0,
−∞ −T + σ

entonces la región de convergencia de la transformada de Laplace L{pT (t)} son


todos los s ∈ C tales que Re s 6= 0.

En la Tabla 7.2 se muestra un resumen de los pares de transformadas de Laplace de


funciones comunes.
232 7.3. ALGUNOS PARES DE TRANSFORMADAS

Tabla 7.2. Algunos Pares de Transformadas de Laplace

Función Transformada de Laplace


δ(t) 1, s∈C

δ(t − a) e−as , s∈C


1
u(t) , Re s > 0
s
1
−u(−t) , Re s < 0
s
1 − e−as
u(t) − u(t − a) , Re s > 0
s
n!
tn u(t) n+1
, n = 1, 2, . . . , Re s > 0
s
n!
−tn u(−t) n+1
, n = 1, 2, . . . , Re s < 0
s
1
e−at u(t) , Re s > −a
s+a
1
−e−at u(−t) , Re s < −a
s+a
n!
tn e−at u(t) , Re s > −a
(s + a)n+1
n!
−tn e−at u(−t) , Re s < −a
(s + a)n+1
s
cos(ω0 t)u(t) , Re s > 0
s + ω02
2
ω0
sen (ω0 t)u(t) , Re s > 0
s + ω02
2

s2 + 2ω02
cos2 (ω0 t)u(t) , Re s > 0
s(s2 + 4ω02 )
2ω02
sen 2 (ω0 t)u(t) , Re s > 0
s(s2 + 4ω02 )
s+a
e−at cos(ω0 t)u(t) , Re s > −a
(s + a)2 + ω02
ω0
e−at sen (ω0 t)u(t) , Re s > −a
(s + a)2 + ω02
s2 − ω02
t cos(ω0 t)u(t) , Re s > 0
(s2 + ω02 )2
2ω0 s
tsen (ω0 t)u(t) , Re s > 0
(s + ω02 )2
2

2jT senh(T s)
pT (t) , Re s 6= 0
Ts
CAPÍTULO 7. TRANSFORMADA DE LAPLACE 233

7.4 Cálculo de la Transformada Inversa de Laplace


Se utilizarán dos métodos para calcular la transformada inversa de Laplace: Integración
de Contornos e Inversión por Tablas.

7.4.1 Integración de Contornos


El método de Integración de Contornos utiliza el Teorema de los Residuos para calcular
la transformada inversa de Laplace. Expliquemos el fundamento de este método mediante
un ejemplo. La siguiente proposición se utiliza en el ejemplo.

Proposición 7.2. Sea F (s) la transformada de Laplace de f (t), con región de convergencia
dada por Re s > −a, donde a > 0. Si |F (s)| < b|s|−m para b > 0, un entero m > 0, y
R0 > 0 tal que |s| > R0 , entonces f (t) = 0, para t < 0.

Demostración. Se deja como ejercicio para el lector. Ayuda: utilice un contorno cerrado
simple C apropiado y el hecho que |F (s)| < 1/|s|.

Ejemplo 7.1. Determine la transformada inversa de Laplace de

1
F (s) = , Re s > −2.
s+2
Solución. Consideremos el contorno cerrado simple C formado por el contorno C1 , el arco
de la circunferencia
z(t) = R ejt , π/2 ≤ t ≤ 3π/2,
y el contorno C2 dado por el segmento de recta

z(t) = j t, −R ≤ t ≤ R,

con R > 0. En la Figura 7.1 se aprecian los contornos C1 y C2 .

ω Re s > −2

C1
C2

−2
σ

Figura 7.1. Contornos C1 y C2


234 7.4. CÁLCULO DE LA TRANSFORMADA INVERSA DE LAPLACE

Se tiene que
Z Z Z
1 st 1 st 1
e F (s) ds = e F (s) ds + est F (s) ds
2πj C 2πj C1 2πj C2
Z Z jR st
1 est 1 e
= ds + ds. (7.11)
2πj C1 s + 2 2πj −jR s + 2

Se deja como ejercicio para el lector verificar que

Z
est
lı́m ds = 0, para t ≥ 0.
R→∞ C1 s+2

Luego, tomando lı́mite en el lado derecho de (7.11), cuando R → ∞, se obtiene

Z j∞ Z  st 
1 est 1 est e
ds = ds = Res = e−2t ,
2πj −j∞ s+2 2πj C s+2 s=−2 s + 2

de lo cual se deduce que


f (t) = e−2t , para t ≥ 0.

Ahora, como |F (s)| = 1/|s + 2| < |s|−1 para |s| > 2, entonces por la Proposición 7.2,
f (t) = 0 para t < 0.
Por lo tanto, todo lo anterior nos indica que la transformada inversa de Laplace de F (s)
es
f (t) = e−2t u(t),

cuya gráfica se muestra en la siguiente figura.

e−2t u(t)
1

El ejemplo anterior es un caso particular, pero el procedimiento de resolución empleado


en el cálculo de la transformada inversa de Laplace, puede usarse en casos más generales.
Más aún, este mismo procedimiento constituye la base para la demostración de la propo-
sición que damos a continuación. Tal proposición plantea un procedimiento para hallar la
transformada inversa de Laplace, cuando F (s) es una función racional propia o, a lo sumo,
una función racional multiplicada por una función exponencial.
CAPÍTULO 7. TRANSFORMADA DE LAPLACE 235

Proposición 7.3. Sea F (s) la transformada de Laplace de f (t) con región de convergencia
dada por r1 < Re s < r2 , donde r1 , r2 ∈ R. Sean pIk los polos ubicados a la izquierda de la
región de convergencia, para k = 1, . . . , nI , y sean pD
k los polos ubicados a la derecha de la
región de convergencia, para k = 1, . . . , nD (ver, por ejemplo, la Figura 7.2). Entonces, la
transformada inversa de Laplace está dada por
nI D
n
X   X  
st
f (t) = Res e F (s) u(t) − Res est F (s) u(−t).
s=pIk s=pD
k
k=1 k=1

b
pI1 r1 < Re s < r2

b
pD
1

r1 r2 σ
b
pI3

b
pD
2
b
pI2

Figura 7.2. Ubicación de los polos con respecto a la región de convergencia

Ejemplo 7.2. Determinar la transformada inversa de Laplace de


5s − 1
F (s) = , −1 < Re s < 2.
s3 − 3s − 2
Solución. Los polos de F (s) son p1 = −1 y p2 = 2, además, p1 está ubicado a la izquierda
de la región de convergencia y p2 está ubicada a la derecha. Ası́, la transformada inversa
de Laplace de F (s) está dada por:
   
f (t) = Res est F (s) u(t) − Res est F (s) u(−t). (7.12)
s=−1 s=2

Ahora,
 
  st est (5s − 1)
Res e F (s) = Res
s=−1 s=−1 s3 − 3s − 2
 
d est (5s − 1)
= = (2t − 1) e−t (7.13)
dz (s − 2) s=−1
y
 st 
  e (5s − 1)
Res est F (s) = Res 3
s=2 s=2 s − 3s − 2
 st 
e (5s − 1)
= = e2t . (7.14)
(s + 1)2 s=2
236 7.4. CÁLCULO DE LA TRANSFORMADA INVERSA DE LAPLACE

Sustituyendo convenientemente (7.13) y (7.14) en (7.12), se tiene que la transformada


inversa de Laplace de F (s) es

f (t) = (2t − 1) e−t u(t) − e2t u(−t).

7.4.2 Inversión por Tablas


El método de inversión por tablas consiste en expresar a F (s) como la suma

F (s) = F1 (s) + F2 (s) + · · · + FR (s), s ∈ D, (7.15)

donde F1 (s), F2 (s), . . . , FR (s) son funciones tales que se les conoce su transformada inversa
de Laplace f1 (t), f2 (t), . . . , fR (t). Entonces, la transformada inversa de Laplace de F (s)
está dada por
f (t) = f1 (t) + f2 (t) + · · · + fR (t).

Con mucha frecuencia en aplicaciones importantes de Ingenierı́a se consigue con trans-


formadas de Laplace F (s) racionales. A este tipo de transformadas se le prestará una ma-
yor atención. Particularmente, es de mucho interés calcular la transformada inversa de
Laplace de
b0 + b1 s + · · · + bm sm
F (s) = ,
a0 + a1 s + · · · + an sn
donde an 6= 0 y m < n; es decir, cuando F (s) es una función racional propia. Cuando F (s)
es racional propia, la expansión (7.15) se denomina expansión en fracciones parciales, que
se estudió en el Capı́tulo 4. Expliquemos con un ejemplo el método inversión por tablas.

Ejemplo 7.3. Determine la transformada inversa de Laplace de

144s2 + 144s + 144


F (s) = , 2 < Re s < 3.
(s − 3)2 (s − 2)2 (s + 1)

Solución. La expansión en fracciones parciales de F (s) es de la forma

A1 A2,1 A2,2 A3,1 A3,2


F (s) = + + 2
+ + ,
s + 1 s − 2 (s − 2) s − 3 (s − 3)2

donde

A1 = Res [F (s)] = 1,
s=−1
 
A2,2 = (s − 2)2 F (s) s=2 = 336,
 
d
A2,1 = [(s − 2)2 F (s)] = 800,
ds s=2
 
A3,2 = (s − 3)2 F (s) s=3 = 468,
 
d 2
A3,1 = [(s − 3) F (s)] = −801.
ds s=3
CAPÍTULO 7. TRANSFORMADA DE LAPLACE 237

Ası́,
     
−1 1 −1 1 −1 1
f (t) = A1 L + A2,1 L + A2,2 L
s+1 s−2 (s − 2)2
   
1 1
+A3,1 L−1 + A3,2 L−1 .
s−3 (s − 3)2

Ahora, considerando la región de convergencia, 2 < Re s < 3, se halla la transformada


inversa de Laplace de cada uno de los sumandos de la expansión en fracciones parciales
de F (s):
   
1 1
L−1 = e−t u(t), L−1 = e2t u(t),
s+1 s−2
   
−1 1 2t −1 1
L = te u(t), L = −e3t u(−t),
(s − 2)2 s−3
 
−1 1
L = −te3t u(−t).
(s − 3)2

Por lo tanto, la transformada inversa de Laplace de F (s) es:


 
f (t) = e−t + 800 e2t + 336 te2t u(t) + 801 e3t − 468 te3t u(−t).

7.5 Transformada de Laplace con M ATLAB


La herramienta de matemática simbólica de M ATLAB cuenta con el comando laplace
que calcula la transformada unilateral de Laplace de una función f (t). Observe el siguiente
ejemplo donde se calcula la transformada unilateral de Laplace de las siguientes funciones:

f1 (t) = e−2t u(t), f2 (t) = (2t − 1) e−t u(t) − e−2t u(−t).



>> syms t
>> f 1 ( t ) = exp(−2∗ t )∗ h e a v i s i d e ( t ) ;
>> F1 = l a p l a c e ( f 1 )
F1 =
1/( s + 2)
>> f 2 ( t ) = (2∗ t −1)∗exp(− t )∗ h e a v i s i d e ( t )−exp (2∗ t )∗ h e a v i s i d e (− t ) ;
>> F2 = l a p l a c e ( f 2 )
F2 =
2/( s + 1)ˆ2 − 1/( s + 1)
✂ ✁
Las transformadas unilateral de Laplace obtenidas son:
1 2 1
F1 (s) =
s+2
, F2 (s) = 2 − s + 1.
(s + 1)

Note que el comando laplace suministra la expresión matemática de la transformada uni-


lateral de Laplace, pero no nos dice nada acerca de su región de convergencia; además,
para utilizar laplace se debe asumir que f (t) es una función causal. Cuando f (t) no es
causal, entonces laplace halla la transformada unilateral de Laplace de la parte causal de
238 7.5. TRANSFORMADA DE LAPLACE CON MATLAB

f (t). En el Programa 7.1 se muestra la función TransLaplace, la cual calcula la transfor-


mada bilateral de Laplace de una función f (t) cualquiera. La función TransLaplace se
basa en el hecho que toda función f (t) se puede expresar como:

f (t) = fn (t) + fc (t),

donde fn (t) es una función no causal y fc (t) es una función causal. Recuerde que una
función g(t) se dice causal si g(t) = 0 para t < 0, y ella es no causal, si g(t) = 0 para t > 0.

Programa 7.1. Función TransLaplace.m

f u n c t i o n [ F ] = T r a n s L a p l a c e ( fn , f c )
%T r a n s L a p l a c e H a l l a l a t r a n s f o r m a d a b i l a t e r a l de L a p l a c e
% de una f u n c i ó n f ( t ) = f n ( t )+ f c ( t )
syms t s ;
Fc ( s ) = l a p l a c e ( f c ) ;
Fn ( s ) = l a p l a c e ( f n (− t ) ) ;
F ( s ) = Fn(− s ) + Fc ( s ) ;
end

Calculemos la transformada bilateral de Laplace de las funciones f1 (t) y f2 (t) dadas


arriba. Observe que f1 (t) es una función causal y f2 (t) = f2n (t) + f2c (t), donde

f2n (t) = −e−2t u(−t) y f2c (t) = (2t − 1) e−t u(t).

Seguidamente se emplea la función TransLaplace para hallar las transformada bilateral


de Laplace de f1 (t) y la de f2 (t).

>> syms t
>> f 1 c ( t )=exp(−2∗ t )∗ h e a v i s i d e ( t ) ; f1n ( t ) = sym ( 0 ) ;
>> F1 = T r a n s L a p l a c e ( f1n , f 1 c )
F1 ( s ) =
1/( s + 2)
>> f 2 c ( t )=(2∗ t −1)∗exp(− t )∗ h e a v i s i d e ( t ) ;
>> f2n ( t)=−exp (2∗ t )∗ h e a v i s i d e (− t ) ;
>> F2 = T r a n s L a p l a c e ( f2n , f 2 c )
F2 ( s ) =
2/( s + 1)ˆ2 − 1/( s + 1) + 1/( s − 2)
✂ ✁
Note que la expresión f1n(t) = sym(0) indica que f1n (t) es la función 0. De esta forma,
las expresiones matemáticas de las transformadas bilaterales de Laplace obtenidas son:
1 2 1 1
F1 (s) =
s+2
, F2 (s) = 2 − s + 1 + s − 2.
(s + 1)
Por otra parte, M ATLAB también cuenta con el comando ilaplace que halla la trans-
formada inversa de Laplace. Observe el siguiente ejemplo donde se halla la transformada
inversa de Laplace de las transformadas F1 (s) y F2 (s) dadas arriba.

>> syms s
>> F1 ( s ) = 1/( s + 2 ) ; f 1 ( t ) = i l a p l a c e ( F1 )
f1 ( t ) =
exp(−2∗ t )
>> F2 ( s ) = 2/( s + 1)ˆ2 − 1/( s + 1) + 1/( s − 2 ) ;
>> f 2 ( t ) = i l a p l a c e ( F2 )
f2 ( t ) =
exp (2∗ t ) − exp(− t ) + 2∗ t ∗ exp(− t )
✂ ✁
CAPÍTULO 7. TRANSFORMADA DE LAPLACE 239

Las transformadas inversas de Laplace obtenidas son:


f1 (t) = e−2t , f2 (t) = e2t + (2 t − 1) e−t .
Note que ilaplace halla la transformada inversa de Laplace de F (s), asumiendo que F (s)
es la transformada unilateral de Laplace de f (t).

7.6 Problemas Resueltos


Problema 7.1. Determinar la transformada de Laplace del pulso rectangular p1 (t).
Solución. Por definición, la transformada de Laplace de p1 (t) está dada por:
Z ∞ Z 1−
es − e−s 2 senh s
F (s) = p1 (t)e−st dt = e−st dt = = .
−∞ −1+ s s
Ahora bien, la región de convergencia de F (s) son todos los números complejos s = σ + jω
tales que
Z ∞ Z 1−
−σt
|f (t)| e dt = e−σt dt < ∞.
−∞ −1+
Es claro que la desigualdad anterior se satisface para todo σ ∈ R. En consecuencia, la
transformada de Laplace de p1 (t) es:
2 senh s
F (s) = , para s ∈ C.
s
2 senh s
Recuerde que la función se puede redefinir en s = 0 de manera que allı́ sea
s
analı́tica. 

Problema 7.2. Determinar la transformada de Laplace del pulso triangular q1 (t).


Solución. Por definición, la transformada de Laplace de q1 (t) está dada por:
Z ∞ Z 0 Z 1
−st −st
F (s) = q1 (t)e dt = (t + 1)e dt + (1 − t)e−st dt
−∞ −1 0
2
s− es +1 s+ e−s
−1 4 senh (s/2)
= − + = .
s2 s2 s2
Ahora bien, la región de convergencia de F (s) son todos los números complejos s = σ + jω
tales que
Z ∞ Z 0 Z 1
|f (t)| e−σt dt = (t + 1)e−σt dt + (1 − t)e−σt dt
−∞ −1 0
Z 1
≤ e−σt dt < ∞.
−1

Es claro que la desigualdad anterior se satisface para todo σ ∈ R. En consecuencia, la


transformada de Laplace de q1 (t) es:
4 senh2 (s/2)
F (s) = , para s ∈ C.
s2

240 7.6. PROBLEMAS RESUELTOS

Problema 7.3. Determinar la transformada de Laplace de f (t) = e−3t (u(t) − u(t − 4)).

Solución. La función f (t) se puede expresar como

f (t) = f1 (t) − f2 (t − 4),

donde
f1 (t) = e−3t u(t) y f2 (t) = e−3(t+4) u(t).
Ası́, aplicando la propiedad de linealidad tenemos:

F (s) = L {f (t)} = L {f1 (t)} − L {f2 (t − 4)} , para s ∈ D1 ∩ D2 , (7.16)

donde D1 , D2 ⊂ C son las regiones de convergencia de L {f1 (t)} y L {f2 (t − 4)}, respecti-
vamente. Calculemos las transformadas L {f1 (t)} y L {f2 (t − 4)}.

• Aplicando la propiedad de desplazamiento en s, tenemos que la transformada de Laplace


de f1 (t) está dada por:
 1
L {f1 (t)} = L e−3t u(t) = , Re s > −3.
s+3

• Aplicando convenientemente las propiedades de desplazamiento en tiempo y desplaza-


miento en s, tenemos que la transformada de Laplace de f2 (t − t) está dada por:
n o 
L {f2 (t − 4)} = e−4s L e−3(t+4) u(t) = e−4s e−12 L e−3t u(t)
e−4(s+3)
= , Re s > −3.
s+3

De esta forma, sustituyendo convenientemente las expresiones de L {f1 (t)} y L {f2 (t − 4)}
en (7.16), se tiene que la transformada de Laplace de f (t) es:

1 e−4(s+3) 1 − e−4(s+3)
F (s) = − = , Re s > −3.
s+3 s+3 s+3
Seguidamente se muestra el diagrama de polos (rojo) y ceros (azul) de F (s).

ω
.. Re s > −3
. b

−3 σ
b

.. b

Diagrama de polos y ceros


CAPÍTULO 7. TRANSFORMADA DE LAPLACE 241

Problema 7.4. Determinar la transformada de Laplace de f (t) = e2t sen (4t)u(t).

Solución. Se tiene que


 
2t e4tj − e−4tj
f (t) = e u(t)
2j
1 (4j+2)t 1
= e u(t) − e−(4j−2)t u(t).
2j 2j

Ası́, aplicando convenientemente las propiedades de linealidad y desplazamiento en s,


obtenemos:
 
1 (4j+2)t 1 −(4j−2)t
F (s) = L e u(t) − e u(t)
2j 2j

1 n (4j+2)t o 1 n o
= L e u(t) − L e−(4j−2)t u(t)
2j 2j

1 1
= L {u(t)} (s − (4j + 2)) − L {u(t)} (s + (4j − 2))
2j 2j

1 1
= − .
2j(s − (4j + 2)) 2j(s + (4j − 2))
| {z } | {z }
Re s>2 Re s>2

Por lo tanto, la transformada de Laplace de f (t) es:

4
F (s) = , Re s > 2.
(s − (4j + 2))(s + (4j − 2))

Seguidamente se muestra el diagrama de polos y ceros de F (s).

ω Re s > 2
2 + 4j b

2 σ

2 − 4j b

Diagrama de polos y ceros


242 7.6. PROBLEMAS RESUELTOS

Problema 7.5. Determinar la transformada de Laplace de la función f (t) cuya gráfica se


muestra en la siguiente figura.

f (t)

−1 1 t

−1

Solución. Se tiene que la función f (t) se puede expresar equivalentemente como:

f (t) = (u(t + 1) − u(t)) − δ(t + 1) − (u(t) − u(t − 1)) + δ(t − 1)


= u(t + 1) − 2u(t) + u(t − 1) − δ(t + 1) + δ(t − 1).

Luego, aplicando la propiedad de linealidad obtenemos:

F (s) = L {f (t)} = L {u(t + 1) − 2u(t) + u(t − 1) − δ(t + 1) + δ(t − 1)}

= L {u(t + 1)} − 2L {u(t)} + L {u(t − 1)} − L {δ(t + 1)} + L {δ(t − 1)} .

Ahora, aplicando la propiedad de desplazamiento en tiempo tenemos:

F (s) = es L {u(t)} − 2L {u(t)} + e−s L {u(t)} − es L {δ(t)} + e−s L {δ(t)}


es 2 e−s
= − + es + |{z}
− |{z} e−s .
s
|{z} s
|{z} s
|{z} C C
Re s>0 Re s>0 Re s>0

Por lo tanto, la transformada de Laplace de f (t) es:

es + e−s − 2
F (s) = − (es − e−s )
s
2 cosh s − 2
= − 2 senh s, Re s > 0.
s


Problema 7.6. Sea f (t) una función de dominio continuo tal que f (t) = 0 para t < 0, cuya
transformada de Laplace es:

s e−2s
F (s) = , Re s > 2.
(s + 1)(s − 2)

Determinar la transformada de Laplace de la función


 
2 1 d
g(t) = t f t − + t f (t + 1).
2 dt
CAPÍTULO 7. TRANSFORMADA DE LAPLACE 243

Solución. Aplicando convenientemente las propiedades de linealidad y diferenciación en


s, podemos escribir:
   
2 1 d
G(s) = L {g(t)} = L t f t − + t f (t + 1)
2 dt
    
2 1 d
= L t f t− + L t f (t + 1)
2 dt
    
d2 1 d d
= L f t− + L f (t + 1) . (7.17)
ds2 2 ds dt
  d
Ahora, calculemos las transformadas L f t − 21 y L dt f (t + 1) . Aplicando la propie-
dad de desplazamiento en tiempo y considerando la expresión de F (s) dada en el enunci-
ado del problema, tenemos:
  
1 s e−5s/2
L f t− = e−s/2 F (s) = , Re s > 2 (7.18)
2 (s + 1)(s − 2)

Aplicando convenientemente las propiedades de diferenciación en tiempo y desplazamiento


en tiempo, y considerando la expresión de F (s) dada en el enunciado del problema, obte-
nemos:
 
d
L f (t + 1) = s L {f (t + 1)} = s es F (s)
dt
s2 e−s
= , Re s > 2 (7.19)
(s + 1)(s − 2)

De esta forma, usando (7.17), (7.18) y (7.19), la transformada de Laplace de g(t) es:
" #  
d2 s e−5s/2 d s2 e−s
G(s) = +
ds2 (s + 1)(s − 2) ds (s + 1)(s − 2)
5s 
e− 2 25 s5 − 30 s4 − 87 s3 + 100 s2 + 108 s − 96
= −
4 (−s2 + s + 2)3

s e−s −s3 + s2 + s − 4
+ , Re s > 2.
(−s2 + s + 2)2

Problema 7.7. Determinar la transformada inversa de Laplace de

s5 − 2s − 1
F (s) = , 0 < Re s < 1.
s2 (s − 1)3

Solución. Se tiene que F (s) es una función racional no propia. Por ello, es necesario
expresar a F (s) de la siguiente forma equivalente:

F (s) = 1 + F1 (s), 0 < Re s < 1,


244 7.6. PROBLEMAS RESUELTOS

donde
3s4 − 3s3 + s2 − 2s − 1
F1 (s) = , 0 < Re s < 1.
s2 (s − 1)3
Luego, la transformada inversa de Laplace de F (s) es:

f (t) = L−1 {F (s)} = L−1 {1} + L−1 {F1 (s)}


= δ(t) + L−1 {F1 (s)} . (7.20)

Debemos determinar la transformada inversa de Laplace de F1 (s). Se tiene que los polos
a la izquierda y la derecha de la región de convergencia de F1 (s) son z0 = 0 y z1 = 1,
respectivamente. Por la Proposición 7.3, la transformada inversa de Laplace de F1 (s) está
dada por:    
f1 (t) = Res est F1 (s) u(t) − Res est F1 (s) u(−t).
s=0 s=1

Ahora,
 st 
 st
 e (3s4 − 3s3 + s2 − 2s − 1)
Res e F1 (s) = Res
s=0 s=0 s2 (s − 1)3
 st 
d e (3s4 − 3s3 + s2 − 2s − 1)
= = t + 5,
ds (s − 1)3 s=0
 st 
 st  e (3s4 − 3s3 + s2 − 2s − 1)
Res e F1 (s) = Res
s=1 s=1 s2 (s − 1)3
 
1 d2 est (3s4 − 3s3 + s2 − 2s − 1) t 2

= = −e t − 7t + 2 .
2 ds2 s2 s=1

Ası́, la transformada inversa de Laplace de F1 (s) es:



f1 (t) = (t + 5) u(t) + et t2 − 7t + 2 u(−t).

Luego, sustituyendo f1 (t) en (7.20) tenemos que la transformada inversa de Laplace de


F (s) es: 
f (t) = δ(t) + (t + 5) u(t) + et t2 − 7t + 2 u(−t).


Problema 7.8. Determinar la transformada inversa de Laplace de

2s3 + 3s2 + 6s + 4
F (s) = , −1 < Re s < 0.
(s − 2j)(s + 2j)(s + 1 − j)(s + 1 + j)

Solución. Se tiene que los polos a la izquierda de la región de convergencia son: z1 =


−1 + j y z2 = −1 − j, y los polos a la derecha de la región de convergencia son: z3 = 2j
y z4 = −2j. Por la Proposición 7.3, la transformada inversa de Laplace de F1 (s) está dada
por:
 
 st   st 
f (t) = Res e F1 (s) + Res e F1 (s) u(t)
s=−1+j s=−1−j
 
 st   st 
− Res e F1 (s) + Res e F1 (s) u(−t).
s=2j s=−2j
CAPÍTULO 7. TRANSFORMADA DE LAPLACE 245

Ahora, se tiene que

  1   1
Res est F (s) = e−(1−j)t , Res est F (s) = e−(1+j)t
s=−1+j 2 s=−1−j 2
  1   1
Res est F1 (s) = e2jt , Res est F1 (s) = e−2jt .
s=2j 2 s=−2j 2

Luego, la transformada inversa de Laplace de F (s) es:


   2jt 
ejt + e−jt
−t e + e−2jt
f (t) = e u(t) − u(−t)
2 2
= e−t cos(t)u(t) − cos(2t)u(−t).

Problema 7.9. Sea f (t) una función de dominio continuo cuya transformada de Laplace
F (s), −1 < Re s < 0, satisface:

• s1 = 2 y s2 = 3, son los únicos ceros de F (s);

• p1 = −1, p2 = 0 y p3 = 1, son los únicos puntos singulares de F (s), además, p1 y p3


son polos simples y p2 es un polo de orden 2;

1
• F (4) = .
24
Determinar la transformada inversa de F (s).

Solución. Por las condiciones dadas en el enunciado del problema, una expresión ma-
temática de F (s) puede ser (existen otras considerando ceros con multiplicidad mayor
que 1):
k(s − 2)(s − 3)
F (s) = ,
(s + 1)s2 (s − 1)
donde k ∈ C. Ahora bien, como F (4) = 1/24, entonces podemos escribir:

k(4 − 2)(4 − 3) 1
2
= ⇔ k = 5.
(4 + 1)4 (4 − 1) 24

Ası́,
5(s − 2)(s − 3)
F (s) = , −1 < Re s < 0.
(s + 1)s2 (s − 1)
Se tiene que el polo a la izquierda de la región de convergencia es: p1 = −1 y los polos a
la derecha de la región de convergencia son: p2 = 0 y p3 = 1. Por la Proposición 7.3, la
transformada inversa de Laplace de F (s) está dada por:
      
f (t) = Res est F (s) u(t) − Res est F (s) + Res est F (s) u(−t).
s=−1 s=0 s=1
246 7.7. PROBLEMAS PROPUESTOS

Ahora,
 
  5est (s − 2)(s − 3)
Res est F (s) = = −30 e−t ,
s=−1 s2 (s − 1) s=−1
 st 
  d 5e (s − 2)(s − 3)
Res est F (s) = = 25 − 30t,
s=0 ds (s + 1)(s − 1) s=0
 st 
  5e (s − 2)(s − 3)
Res est F (s) = = 5 et .
s=1 (s + 1)s2 s=1

Luego, la transformada inversa de Laplace de F (s) es:

f (t) = −30 e−t u(t) − (25 − 30t + 5 et )u(−t).

7.7 Problemas Propuestos


7.1. Calcular la transformada de Laplace de las siguientes funciones de dominio conti-
nuo.

a) f (t) = δ(t − 1) + δ(t) + e−t u(−t)

b) f (t) = eat (u(t) − u(t − 5)), para a < 0

c) f (t) = e4t sen (4t) u(t)

d) f (t) = e−3t u(t) − e5t u(−t)

e) f (t) = |t|

f) f (t) = (1 − |t|)

g) f (t) = (1 − |t|)u(t)

h) f (t) = tn e−t u(−t), n ∈ Z+

i) f (t) = cos(at)u(t), a > 0

j) f (t) = senh(at)u(−t), a > 0

7.2. Obtener la transformada unilateral de Laplace de las siguientes funciones de do-


minio continuo:

a) f1 (t) = tp2 (t − 1) c) f3 (t) = p1 (t/2)


b) f2 (t) = f1 (t) + 12 δ(t) d) f4 (t) = q1/2 (1 − t)
CAPÍTULO 7. TRANSFORMADA DE LAPLACE 247

7.3. La transformada de Laplace de una función causal f (t) (f (t) = 0 para t < 0) es:

s e−2s
F (s) = , Re s > −1.
s2 + 2s + 1
Determinar la transformada de Laplace de las siguientes funciones:
 
t d
a) g(t) = 3f e) g(t) = t f (t)
3 dt
f) g(t) = δ(t − a) f (t), a ∈ R
b) g(t) = t f (t)
d
c) g(t) = t f (t − 1) g) g(t) = (t − 1) f (t − 1) + f (t)
dt
Z t
d h) g(t) = f (τ ) τ
d) g(t) = f (t)
dt 0

7.4. Determinar los valores inicial y final de las funciones causales f (t), cuya transfor-
mada de Laplace se indica a continuación, sin calcular la transformada inversa de
Laplace. Si no existe valor final, indicar el motivo.
1
a) F (s) = , Re s > −2
s+2
1
b) F (s) = , Re s > −2
(s + 2)25
6
c) F (s) = , Re s > 0
s(s2 + 25)
s+2
d) F (s) = , Re s > −3
s+3
s2 + s + 3
e) F (s) = , Re s > 2
s4 − 9s2 + 4s + 12
s
f) F (s) = 2 , Re s > 3
s − 2s − 3
7.5. Determinar la transformada inversa de Laplace de cada una de las siguientes
transformadas, mediante el procedimiento por residuos.
1
a) F (s) = , Re s < −1
s2 +1
1
b) F (s) = , Re s > −a, con a < 0
s2 + a2
s−2
c) F (s) = , −1 < Re s < 3
(s + 1)(s − 3)
s2 + 2s − 2
d) F (s) = , Re s > 1
s(s − 1)
s3 + 4s2 + s − 1
e) F (s) = , −1 < Re s < 0
s3 (s − 1)(s + 1)2
248 7.7. PROBLEMAS PROPUESTOS

2s3 + 3s2 + 6s + 4
f) F (s) = , −1 < Re s < 0
(s2 + 4)(s2 + 2s + 2)
7.6. Sea f (t) una función de dominio continuo cuya transformada de Laplace F (s)
satisface:

• D = {s ∈ C : −1 < Re s < 1} es la región de convergencia de F (s);


• s1 = 2 y s2 = 3, son los únicos ceros de F (s);
• p1 = −2, p2 = −1 y p3 = 1, son los únicos puntos singulares de F (s), además,
todos son polos simples;
• F (0) = α, donde α > 0.

Utilizar el procedimiento por residuos para determinar la transformada inversa de


F (s).
8
Transformada z
La transformada z es una herramienta tan poderosa para el análisis de sistemas de
tiempo discreto, como lo es la transformada de Laplace para sistemas de tiempo continuo.
En este capı́tulo se presenta la transformada z, su definición y propiedades. Se calculan
las transformadas z de funciones comunes de dominio discreto. Finalmente, se halla la
transformada z inversa mediante tres métodos: integración compleja, expansión en serie de
potencias e inversión por tablas.

8.1 Definición
La transformada z es la contraparte en el dominio discreto de la transformada de La-
place. La transformada z opera sobre funciones de dominio discreto, a diferencia de la
transformada de Laplace que opera sobre una función de dominio continuo.

Definición 8.1 (Transformada z bilateral). Sea f (n) una función de dominio discreto
definida de Z en C. La transformada z bilateral de f (n) se define como

X
F (z) = Z[f (n)] = f (n)z −n ,
n=−∞

para todo z ∈ C tal que |F (z)| < ∞.

Cambiando los lı́mites de la sumatoria en la definición de la transformada z bilateral,


se obtienen las transformadas z unilaterales.

Definición 8.2. Sea f (n) una función de dominio discreto definida de Z en C. Se define
la transformada z unilateral derecha de f (n) como

X
ZD {f (n)} = f (n)z −n ,
n=0

para todo s ∈ C tal que |ZD [f (n)](z)| < ∞.


La transformada z unilateral izquierda de f (n) se define como
0
X
ZI {f (n)} = f (n)z −n ,
n=−∞

para todo s ∈ C tal que |ZI [f (n)](z)| < ∞.

249
250 8.1. DEFINICIÓN

Observación 8.1. De la definición de la transformada z se infiere que f (n) son los coefi-
cientes del desarrollo
P∞ en serie de Laurent de F (z) centrado en z0 = 0. Además, F (z) es la
suma de la serie n=−∞ f (n)z . −n

A lo largo del capı́tulo nos referiremos con transformada z a la transformada z bilateral,


en caso contrario lo indicaremos explı́citamente. En la siguiente proposición se establece
que si la transformada z existe, entonces ella es una función analı́tica.

Proposición 8.1. Si F (z) es la transformada z de la función f (n), entonces F (z) es


analı́tica en todo número complejo z tal que |F (z)| < ∞.

Demostración. Consideremos la transformada z bilateral. Por definición de la transformada


z se tiene que

X
F (z) = f (n)z −n ,
n=−∞
P
además, |F (z)| < ∞; en otras palabras, la serie de potencia ∞ n=−∞ f (n)z
−n converge a

F (z) para cada z tal que |F (z)| < ∞. Por lo tanto, F (z) es una función analı́tica para z tal
que |F (z)| < ∞.

8.1.1 Región de Convergencia

Definición 8.3. La región de convergencia de la transformada z de f (n), es el conjunto


de números complejos z donde F (z) existe, esto es, son todos los z ∈ C tales que
|F (z)| < ∞. En otras palabras, la región de convergencia de F (z) es un anillo centrado
en el origen donde la serie
X∞
f (n)z −n
n=−∞
es convergente.

En el siguiente ejemplo se calculan las transformadas z unilateral derecha y bilateral


de una función f (n), con sus correspondientes regiones de convergencia.

Ejemplo 8.1. Calcular las transformadas z bilateral y unilateral derecha de la función

f (n) = an u(n + 1), a ∈ R, a 6= 0.

Solución. Se tiene que la transformada z bilateral de f (n) es:



X ∞
X ∞
X ∞
X
F (z) = f (n)z −n = an u(n + 1)z −n = an z −n = (az −1 )n
n=−∞ n=−∞ n=−1 n=−1

X z 1
= a−1 z + (az −1 )n = + , |az −1 | < 1
a 1 − az −1
n=0

o, equivalentemente,
z2
F (z) = , |z| > |a|.
a(z − a)
CAPÍTULO 8. TRANSFORMADA Z 251

Ahora, la transformada z unilateral derecha de f (n) es



X ∞
X
ZD {f (n)} = an u(n + 1) z −n = (az −1 )n
n=0 n=0
1 z
= = , |z| > |a|.
1 − az −1 z−a

En la siguiente figura se aprecia la región de convergencia de las transformadas z halladas.


y |z| > |a|

|a|

8.1.2 Transformada z Inversa


La transformada z inversa es la contraparte en el dominio discreto de la transformada
inversa de Laplace en el dominio continuo. En sı́, la transformada z inversa es el proceso
de obtener f (n) a partir de la transformada z, F (z).

Definición 8.4. Sea F (z), para r1 < |z| < r2 , la transformada z de f (n). La transfor-
mada z inversa es el proceso de obtener f (n) a través de F (z) y se define como:
Z
1
f (n) = f (n) z n−1 dz, para cada n ∈ Z, (8.1)
2πj C

donde C es un contorno cerrado simple contenido en la región de convergencia de F (z)


y que confina la circunferencia |z| = r1 .

Para el cálculo de la integral de la ecuación (8.1) se utiliza, generalmente, el Teorema


de los Residuos o la Extensión de la Fórmula Integral de Cauchy. Nos centraremos en
tres métodos alternativos para calcular la transformada z inversa: Integración Compleja,
Inversión por Tablas y Expansión en Serie de Potencias, los cuales se describirán en la Sec-
ción 8.4.
En el resto del capı́tulo usaremos la siguiente notación. El sı́mbolo

Z
f (n) ←→ F (z), z∈D

que se lee par de transformadas, denota que F (z) es la transformada z de f (n) con región
de convergencia el conjunto D ⊂ C, y que f (n) es la transformada z inversa de F (z).
252 8.2. PROPIEDADES DE LA TRANSFORMADA Z

8.2 Propiedades de la Transformada z


A continuación damos las propiedades de la transformada z.

Teorema 8.1 (Linealidad). Si las transformadas z de las funciones f (n) y g(n) son res-
pectivamente F (z), para todo z ∈ Df ⊂ C y G(z), para todo z ∈ Dg ⊂ C, entonces

Z
af (n) + bg(n) ←→ aF (z) + bG(z), z ∈ Df ∩ Dg ,

donde a y b son constantes reales.

Demostración. Por la definición de la transformada z se tiene que



X
Z{af (n) + bg(n)} = (af (n) + bg(n))z −n
n=−∞
X∞ ∞
X
= a f (n)z −n + b g(n)z −n
n=−∞ n=−∞
= aZ{f (n)} + bZ{g(n)} = aF (z) + bG(z),

además, como Df y Df son las regiones de convergencia de la series



X ∞
X
f (n)z −n y g(n)z −n ,
n=−∞ n=−∞

respectivamente, entonces se tiene que el conjunto Df ∩ Dg es la región de convergencia


de Z{af (n) + bg(n)}.

Teorema 8.2 (Desplazamiento en tiempo). Si la transformada z de la función f (z) es


F (z) para todo z ∈ D ⊂ C, entonces
Z
f (n − n0 ) ←→ z −n0 F (z), z ∈ D,

para todo n0 ∈ Z.

Demostración. Por la definición de la transformada z se tiene que



X
Z{f (n − n0 )} = f (n − n0 )z −n .
n=−∞

Haciendo el cambio de variable k = n − n0 , la serie anterior adquiere la forma:



X
Z{f (n − n0 )} = f (k) z −n0 z −k
k=−∞

X
= z −n0 f (n)z −n = z −n0 F (z), z ∈ D.
n=−∞
CAPÍTULO 8. TRANSFORMADA Z 253

Teorema 8.3 (Inversión en el tiempo). Si la transformada z de la función f (z) es F (z)


para todo z ∈ D ⊂ C, entonces
Z
f (−n) ←→ F (z −1 ), z −1 ∈ D.

Demostración. Por la definición de la transformada z se tiene que



X
Z{f (−n)} = f (−n)z −n .
n=−∞

Haciendo el cambio de variable k = −n, la serie anterior adquiere la forma:


−∞
X −∞
X
Z{f (−n)} = f (k) z k = f (k) (z −1 )−k .
k=∞ k=∞

Cambiando k por n en la serie y considerando la conmutatividad de la suma, se tiene:



X
Z{f (−n)} = f (n) (z −1 )−n = F (z −1 ), z −1 ∈ D.
n=−∞

Teorema 8.4 (Escalado en el dominio de z). Si la transformada z de la función f (z) es


F (z) para todo z ∈ D ⊂ C, entonces:
Z
z0n f (n) ←→ F (z/z0 ) , z ∈ D,
Z 
ejω0 n f (n) ←→ F e−jω0 z , z ∈ D,
Z 
an f (n) ←→ F a−1 z , z ∈ D,
para z0 ∈ C, ω0 ∈ R y a ∈ R.

Demostración. Como ejω0 n f (n) y an f (n) son casos particulares de z0n f (n) (tome respec-
tivamente z0 = ejω0 y z0 = a), entonces solo demostraremos que Z{z0n f (n)} = F (z/z0 ),
z ∈ D. Se tiene que

X ∞
X
Z{z0n f (n)} = z0n f (n)z −n = f (n) (z/z0 )−n = F (z/z0 ), z ∈ D.
n=−∞ n=−∞

Teorema 8.5 (Conjugación). Si la transformada z de la función f (z) es F (z) para todo


z ∈ D ⊂ C, entonces
Z
f (n) ←→ F (z), z ∈ D.

Demostración. Se tiene que



X ∞
X ∞
X
Z{f (n)} = f (n)z −n
= f (n) z −n = f (n) (z)−n = F (z), z ∈ D.
n=−∞ n=−∞ n=−∞
254 8.2. PROPIEDADES DE LA TRANSFORMADA Z

Teorema 8.6 (Convolución). Si las transformadas z de las funciones f (n) y g(n) son
respectivamente F (z), para todo z ∈ Df ⊂ C y G(z), para todo z ∈ Dg ⊂ C, entonces

Z
f ∗ g (n) ←→ F (z) G(z), z ∈ Df ∩ Dg .

Demostración. Se tiene que


∞ ∞
" ∞
#
X X X
Z{f ∗ g (n)} = f ∗ g (n)z −n = f (k) g(n − k) z −n
n=−∞ n=−∞ k=−∞

Haciendo el cambio de variable ℓ = n − k, la ecuación anterior adquiere la forma:



" ∞ #
X X
Z{f ∗ g (n)} = f (k) g(ℓ) z −(ℓ+k) ,
ℓ=−∞ k=−∞

de lo cual se deduce que



! ∞
!
X X
−k −ℓ
Z{f ∗ g (n)} = f (k) z g(ℓ) z = F (z) G(z), z ∈ Df ∩ Dg .
k=−∞ ℓ=−∞

Teorema 8.7 (Primera diferencia). Si la transformada z de la función f (z) es F (z) para


todo z ∈ D ⊂ C, entonces
 
Z z−1
f (n) − f (n − 1) ←→ F (z), z ∈ D ∩ {z ∈ C : |z| > 0}.
z

Demostración. Se tiene que



X
Z{f (n) − f (n − 1)} = [f (n) − f (n − 1)]z −n
n=−∞
X∞ ∞
X
−n
= f (n)z − f (n − 1)z −n
n=−∞ n=−∞

X
= F (z) − f (n − 1)z −n .
n=−∞

Haciendo el cambio de variable k = n − 1, de la ecuación anterior se deduce que



X
Z{f (n) − f (n − 1)} = F (z) − f (k)z −(n+1)
k=−∞

X
= F (z) − z −1 f (k)z −n
k=−∞
−1
= F (z) − z F (z)
 
z−1
= F (z), z ∈ D ∩ {z ∈ C : |z| > 0}.
z
CAPÍTULO 8. TRANSFORMADA Z 255

Teorema 8.8 (Acumulación). Si la transformada z de la función f (z) es F (z) para todo


z ∈ D ⊂ C, entonces
n  
X Z z
f (k) ←→ F (z), z ∈ D ∩ {z ∈ C : |z| > 1}.
z−1
k=−∞

Demostración. Para la prueba usaremos la transformada z del escalón unitario discreto, a


saber:
z
Z{u(n)} = , |z| > 1.
z−1
la cual la hallaremos más adelante. Ahora, se tiene que

X n
X
f (n) ∗ u(n) = f (k)u(n − k) = f (k), (8.2)
k=−∞ k=−∞

para toda función f (n) de dominio discreto. . De esta forma, usando la propiedad de Se deja al lector
la prueba de la
convolución en la ecuación (8.2), se tiene ecuación (8.2)

Z{f (n) ∗ u(n)} = Z{f (n)} Z{u(n)}, z ∈ D ∩ {z ∈ C : |z| > 1}. (8.3)

Ahora, usando la expresión de la transformada z del escalón unitario discreto, entonces


por (8.2) y (8.3), se obtiene
( n )  
X z
Z f (k) = F (z), z ∈ D ∩ {z ∈ C : |z| > 1}.
z−1
k=−∞

Teorema 8.9 (Diferenciación en el dominio de z). Si la transformada z de la función


f (z) es F (z) para todo z ∈ D ⊂ C, entonces

Z d
n f (n) ←→ −z F (z), z ∈ D.
dz
Demostración. Se tiene que, para todo z ∈ D,
" ∞ #
d d X
−z F (z) = −z f (n)z −n
dz dz n=−∞

X d  −n 
= −z f (n) z
n=−∞
dz

X ∞
X
= z (n f (n)) z −(n+1) = (n f (n))z −n ,
n=−∞ n=−∞

d
de donde se deduce que Z{n f (n)} = −z F (z), z ∈ D.
dz
256 8.2. PROPIEDADES DE LA TRANSFORMADA Z

Teorema 8.10 (Teorema del Valor Inicial). Sea f (n) una función tal que f (n) = 0 para
n < 0. Si la transformada z de f (n) es F (z) para todo z ∈ C tal que |z| > a con a > 0,
entonces
f (0) = lı́m F (z).
z→∞

Demostración. Se tiene que



X
F (z) = f (z)z −n
n=−∞
= f (0) + f (1)z −1 + f (2)z −2 + · · · + f (n)z −n + · · ·

Tomando lı́mite en ambos lados de la ecuación anterior, cuando z → ∞, se obtiene

f (0) = lı́m F (z).


z→∞

Teorema 8.11 (Teorema del Valor Final). Sea f (n) una función tal que f (n) = 0 para
n < 0. Si la transformada z de f (n) es F (z) para todo z ∈ C tal que |z| > a con a > 0,
entonces  
z−1
lı́m f (n) = lı́m F (z).
n→∞ z→1 z

Demostración. Aplicando la propiedad de primera diferencia se tiene


 
z−1
Z{f (n) − f (n − 1)} = F (z), z ∈ {|z| > a} ∩ {|z| > 1}. (8.4)
z

Como f (n) = 0 para n < 0, entonces se obtiene:



X
Z{f (n) − f (n − 1)} = [f (n) − f (n − 1)] z −n
n=0
N
X
= lı́m [f (n) − f (n − 1)] z −n . (8.5)
N →∞
n=0

De (8.4) y (8.5), se deduce:


N  
X
−n z−1
lı́m [f (n) − f (n − 1)] z = F (z), z ∈ {|z| > a} ∩ {|z| > 1}.
N →∞ z
n=0

De esta forma, tomando lı́mite en ambos lados de la ecuación anterior, cuando z → 1, se


tiene:  
z−1
lı́m f (N ) = lı́m F (z).
N →∞ z→1 z

Las propiedades de la transformada z se resumen en la Tabla 8.1.


CAPÍTULO 8. TRANSFORMADA Z 257

Tabla 8.1. Propiedades de la Transformada z

Sean f (n), f1 (n) y f2 (n), funciones de dominio discreto con transformadas z dadas por:
F (z) para z ∈ D, F1 (z) para z ∈ D1 y F2 (z) para z ∈ D2 , respectivamente.

Propiedad Descripción Matemática


Linealidad Z {a1 f (n) + a2 f2 (n)} = a1 F (z) + a2 F2 (z), z ∈ D1 ∩ D2

(
D, si n0 ≤ 0,
Desplazamiento en tiempo Z {f (n − n0 )} = z −n0 F (z), z∈
D − {0}, si n0 > 0

Escalado en el dominio de z Z {an f (n)} = F (a−1 z), a−1 z ∈ D

Inversión en el tiempo Z {f (−n)} = F (z −1 ), z −1 ∈ D

n o
Conjugación Z f (n) = F (z), z∈D

Convolución Z {f1 ∗ f2 (n)} = F1 (z)F2 (z), z ∈ D ⊇ D1 ∩ D2

z−1

Primera diferencia Z {f (n) − f (n − 1)} = z F (z), z ∈ D ∩ {|z| > 0}

( n
)
X  
z
Acumulación Z f (k) = z−1 F (z), z ∈ D ∩ {|z| > 1}
k=−∞

d
Diferenciación en z Z {n f (n)} = −z F (z), z∈D
dz

Teorema del valor inicial f (0) = lı́m F (z)


z→∞

 
z−1
Teorema del valor final lı́m f (n) = lı́m F (z)
n→∞ z→1 z
258 8.3. ALGUNOS PARES DE TRANSFORMADAS

8.3 Algunos Pares de Transformadas


En esta sección se calcula la transformada z de funciones comunes.
1.
Z
δ(n) ←→ 1, z∈C
Se tiene que

X
Z{δ(n)} = δ(n)z −n = δ(0)z 0 = 1, z ∈ C.
n=−∞

2.
Z z
u(n) ←→ , |z| > 1
z−1
Se tiene que

X ∞
X
−n
Z{u(n)} = u(n)z = (z −1 )n
n=−∞ n=0
1 z
= −1
= , |z| > 1.
1−z z−1
La región de convergencia de Z{u(n)} se muestra en la Figura 8.1. Aquı́ también
se aprecia la ubicación de los polos y ceros de Z{u(n)}. Esta gráfica se denomina
diagrama de polos y ceros.
y
|z| > 1

1
b b

0 1 x

Figura 8.1. Diagrama de polos y ceros de Z{u(n)}

3.
Z z
−u(−n − 1) ←→ , |z| < 1
z−1
Se tiene que

X −1
X
−n
Z{−u(−n − 1)} = [−u(−n − 1)]z =− z −n .
n=−∞ n=−∞

Haciendo el cambio de variable k = −n, se obtiene


1
" ∞
#
X X
Z{−u(−n − 1)} = − zk = − zk − 1
k=∞ n=0
 
1
= − −1 , |z| < 1
1−z
CAPÍTULO 8. TRANSFORMADA Z 259

o, equivalentemente,
z
Z{−u(−n − 1)} = , |z| < 1.
z−1
En la Figura 8.2 se muestra el diagrama de polos y ceros de Z{−u(−n − 1)}.
y
|z| < 1

1
b b

0 1 x

Figura 8.2. Diagrama de polos y ceros de Z{−u(−n − 1)}

4.
Z z
αn u(n) ←→ , |z| > |α|
z−α
Se tiene que

X ∞
X
Z{αn u(n)} = [αn u(n)]z −n = (αz −1 )n
n=−∞ n=0
1
= , |αz −1 | < 1
1 − αz −1
o, equivalentemente,
z
Z{αn u(n)} = , |z| > |α|.
z−α
5.
Z z
−αn u(−n − 1) ←→ , |z| < |α|
z−α
Se tiene que

X −1
X
Z{−αn u(−n − 1)} = [−αn u(−n − 1)]z −n = − (α−1 z)−n .
n=−∞ n=−∞

Haciendo el cambio de variable k = −n, se obtiene


1
"∞ #
X X
Z{−αn u(−n − 1)} = − (α−1 z)k = − (α−1 z)k − 1
k=∞ k=0
 
1
= − −1 , |α−1 z| < 1
1 − α−1 z
o, equivalentemente,
z
Z{−αn u(−n − 1)} = , |z| < |α|.
z−α
260 8.3. ALGUNOS PARES DE TRANSFORMADAS

6.
Z αz
n αn u(n) ←→ , |z| > |α|
(z − α)2

Se tiene que
z
Z{αn u(n)} = , |z| > |α|.
z−α
Luego, aplicando la propiedad de diferenciación en z, se obtiene

d
Z{n αn u(n)} = −z [Z{αn u(n)}]
dz  
d z
= −z , |z| > |α|
dz z − α

o, equivalentemente,

αz
Z{n αn u(n)} = , |z| > |α|.
(z − α)2

7.
Z αz
−n αn u(−n − 1) ←→ , |z| < |α|
(z − α)2

Se tiene que
z
Z{u(−n − 1)} = − , |z| < 1. (8.6)
z−1
Aplicando las propiedades de linealidad y diferenciación en z, se tiene

d
Z{−n αn u(−n − 1)} = z Z{αn u(−n − 1)} (8.7)
dz

Ahora, aplicando la propiedad escalado en el dominio de z, entonces por (8.6) se


tiene

Z{αn u(−n − 1)} = Z{u(−n − 1)}(α−1 z), |α−1 z| < 1


z
= − , |z| < |α|. (8.8)
z−α

Luego, por (8.7) y (8.8), se obtiene


 
n d z
Z{−n α u(−n − 1)} = −z , |z| < |α|
dz z − α

o, equivalentemente,

αz
Z{−n αn u(−n − 1)} = , |z| < |α|.
(z − α)2
CAPÍTULO 8. TRANSFORMADA Z 261

8.
Z z
e−aT n u(n) ←→ , |z| > e−aT
z − e−aT

Se tiene que


X
Z{e−aT n u(n)} = e−aT n u(n)z −n
n=−∞

X n 1
= e−aT z −1 = , e−aT z −1 < 1
1− e−aT z −1
n=0

o, equivalentemente,

z
Z{e−aT n } = , |z| > e−aT .
z − e−aT

9.
Z z(z − cos(ω))
cos(ωn)u(n) ←→ , |z| > 1
z2 − 2z cos(ω) + 1

Se tiene que


X
Z{cos(ωn)u(n)} = cos(ωn)u(n)z −n
n=−∞
∞  jωn 
X e + e−jωn
= z −n
n=0
2
"∞ ∞
#
1 X jω −1 n X −jω −1 n
= e z + e z
2
n=0 n=0
 
1 1 1
= + ,
2 1 − ejω z −1 1 − e−jω z −1

para z ∈ {|ejω z −1 | < 1} ∩ {|e−jω z −1 | < 1} o, equivalentemente,

z(z − cos(ω))
Z{cos(ωn)u(n)} = , |z| > 1.
z2 − 2z cos(ω) + 1

En la Tabla 8.2 se muestra un resumen de los pares de transformadas z de funciones


comunes.
262 8.3. ALGUNOS PARES DE TRANSFORMADAS

Tabla 8.2. Algunos Pares de Transformadas z

Función Transformada z
δ(n) 1, z∈C

z
u(n) , |z| > 1
z−1
z
nu(n) , |z| > 1
(z − 1)2
z(z + 1)
n2 u(n) , |z| > 1
(z − 1)3
z
−u(−n − 1) , |z| < 1
z−1
z
αn u(n) , |z| > |α|
z−α
z
−αn u(−n − 1) , |z| < |α|
z−α
αz
n αn u(n) , |z| > |α|
(z − α)2
αz
−n αn u(−n − 1) , |z| < |α|
(z − α)2
z
e−aT n u(n) , |z| > e−aT
z − e−aT
z(z − cos(ω))
cos(ωn)u(n) , |z| > 1
z 2 − 2z cos(ω) + 1
z sen (ω)
sen (ωn)u(n) , |z| > 1
z 2 − 2z cos(ω) + 1
z(z − α cos(ω))
αn cos(ωn)u(n) , |z| > |α|
z2 − 2zα cos(ω) + α2
zα sen (ω)
αn sen (ωn)u(n) , |z| > 1
z2 − 2zα cos(ω) + α2
CAPÍTULO 8. TRANSFORMADA Z 263

8.4 Cálculo de la Transformada z Inversa


Sea f (n) una función de dominio discreto cuya transformada z es F (z), r1 < |z| < r2 ,
donde r1 y r2 son números reales positivos. Se utilizarán tres métodos para calcular la
transformada z inversa, a saber: Integración Compleja, Expansión en Serie de Potencias e
Inversión con Tablas.

8.4.1 Integración Compleja


El método de Integración Compleja utiliza la definición de la transformada z inversa.
En otras palabras, f (n) se calcula como
Z
1
f (n) = f (n) z n−1 dz, para cada n ∈ Z,
2πj C

donde C es un contorno cerrado simple contenido en la región de convergencia de F (z) y


que confina la circunferencia |z| = r1 .Para el cálculo de la integral se utiliza, generalmente, Sin pérdida de
generalidad,
el Teorema de los Residuos. Expliquemos el método integración compleja mediante un C puede ser la
ejemplo. circunferencia
|z| = r, con
r1 < r < r2
Ejemplo 8.2. Calcular la transformada z inversa de
z
F (z) = , |z| > |a|.
z−a

Solución. Sea C la circunferencia |z| = r > |a|. Ahora, considerando la expresión de F (z)
se tiene
Z
1
f (n) = F (z) z n−1 dz
2πj C
Z
1 zn
= dz, para cada n ∈ Z. (8.9)
2πj C z − a

Supongamos que n ≥ 0. Aplicando el Teorema de los Residuos en la integral de (8.9), se


obtiene:  n 
z
f (n) = Res = an .
z=a z − a

Cuando n < 0, entonces aplicando nuevamente el Teorema de los Residuos en la integral


de (8.9), se consigue
   n 
zn z
f (n) = Res + Res = −an + an = 0.
z=0 z − a z=a z − a

Por lo tanto, la transformada z inversa de F (z) es

f (n) = an u(n).


264 8.4. CÁLCULO DE LA TRANSFORMADA Z INVERSA

8.4.2 Expansión en Serie de Potencias


El procedimiento expansión en serie de potencias consiste en encontrar el desarrollo de
Laurent de F (z) centrado en z0 = 0, que es válido en el anillo r1 < |z| < r2 , y luego definir
a f (n) como los coeficientes de esta serie de potencias. En el siguiente ejemplo se explica
esta metodologı́a.
Ejemplo 8.3. Determinar la transformada z inversa de
1
F (z) = , |z| > 1.
(1 + z −1 )(1 − z −1 )2


Solución. Escribimos a F (z) en potencias positivas:

z3
F (z) = .
(z + 1)(z − 1)2
Recuerde que Luego, expresamos a F (z)/z en fracciones parciales:
el método ex-
pansión en frac- F (z) 1 1 3 1 1 1
ciones parciales, = + + .
solo se aplica a z 4 (z + 1) 4 (z − 1) 2 (z − 1)2
funciones racio-
nales propias. De esta forma, obtenemos:
1 z 3 z 1 z
F (z) = + + , |z| > 1. (8.10)
4 (z + 1) 4 (z − 1) 2 (z − 1)2
Hallamos los desarrollos de Laurent centrados en z0 = 0 y válidos en |z| > 1, de cada uno
de los sumandos de (8.10). Se tiene que
∞ ∞
z z X
n −n
X
= = (−1) z = (−1)n u(n) z −n , |z| > 1, (8.11)
(z + 1) z(1 + z −1 ) n=−∞
n=0

∞ ∞
z z X
−n
X
= = z = u(n) z −n , |z| > 1; (8.12)
(z − 1) z(1 − z −1 ) n=0 n=−∞

ahora, derivando en ambos lados de (8.12) se obtiene


∞ ∞
−1 X
−(n+1) −1
X
= (−n)z = −z nu(n) z −n , |z| > 1,
(z − 1)2 n=0 n=−∞

de donde se deduce

z X
= nu(n) z −n , |z| > 1. (8.13)
(z − 1)2 n=−∞
Sustituyendo convenientemente (8.11), (8.12) y (8.13) en (8.10), se obtiene:
∞ ∞ ∞
1 X 3 X 1 X
F (z) = (−1)n u(n) z −n + u(n) z −n + nu(n) z −n
4 n=−∞ 4 n=−∞ 2 n=−∞
∞  
X 1 3 1
= (−1) u(n) + u(n) + nu(n) z −n , |z| > 1.
n

n=−∞
4 4 2
CAPÍTULO 8. TRANSFORMADA Z 265

Por lo tanto, la transformada z inversa de F (z) es


1 3 1
f (n) = (−1)n u(n) + u(n) + n u(n).
4 4 2


8.4.3 Inversión por Tablas


El método inversión por tablas consiste en expresar a F (z) como una suma de la forma

F (z) = F1 (z) + F2 (z) + · · · + FR (z), (8.14)

donde F1 (z), F2 (z), . . . , FR (z), son funciones tales que se les conoce su transformada z
inversa f1 (n), f2 (n), . . . , fR (n). De esta forma, la transformada z inversa de F (z) está
dada por:
f (n) = f1 (n) + f2 (n) + · · · + fR (n).
Observación 8.2. Cuando F (z) es una función racional propia, se emplea la expansión en
fracciones parciales para obtener la descomposición (8.14).

Ejemplo 8.4. Determinar la transformada z inversa de


1
F (z) = , |z| > 1.
(1 + z −1 )(1 − z −1 )2

Solución. Escribimos a F (z) en potencias positivas:

z3
F (z) = .
(z + 1)(z − 1)2

Ahora, aplicando la expansión en fracciones parciales a

F (z) z2
= ,
z (z + 1)(z − 1)2

se tiene que
F (z) A1 A2,1 A2,2
= + + ,
z (z + 1) (z − 1) (z − 1)2
donde
   
F (z) z2 1
A1 = (z + 1) = 2
= ,
z z=−1 (z − 1) z=−1 4
   2 
d F (z) d z 3
A2,1 = (z − 1)2 = = ,
dz z z=1 dz z + 1 z=1 4
   2 
2 F (z) z 1
A2,2 = (z − 1) = = .
z z=1 z + 1 z=1 2

Ası́,
1 z 3 z 1 z
F (z) = + + , |z| > 1.
4 (z + 1) 4 (z − 1) 2 (z − 1)2
266 8.5. TRANSFORMADA Z CON MATLAB

Como
Z z
(−1)n u(n) ←→ , |z| > 1,
z+1
z
Z
u(n) ←→ , |z| > 1,
z−1
Z z
n u(n) ←→ , |z| > 1,
(z − 1)2

entonces la transformada z inversa de F (z) es

1 3 1
f (n) = (−1)n u(n) + u(n) + n u(n).
4 4 2


8.5 Transformada z con M ATLAB


La herramienta de matemática simbólica de M ATLAB cuenta con el comando ztrans
que calcula la transformada z unilateral derecha de una función f (n). Observe el siguiente
ejemplo donde se calcula la transformada z unilateral derecha de las siguientes funciones:

f1 (n) = 2−n u(n), f2 (t) = n u(n).



>> syms n z
>> f 1 ( n ) = 2ˆ(−n ) ; F1 ( z ) = z t r a n s ( f 1 )
F1 ( z ) =
z / ( z − 1/2)
>> f 2 ( n ) = n ; F2 ( z ) = z t r a n s ( f 2 )
F2 ( z ) =
z / ( z − 1)ˆ2
✂ ✁
Las transformadas z unilaterales derechas obtenidas son:
z z
F1 (z) = , F2 (z) = .
z − 12 (z − 1)2

Note que el comando ztrans suministra la expresión matemática de la transformada z


unilateral derecha, pero no nos dice nada acerca de su región de convergencia; además,
para utilizar ztrans se debe asumir que f (n) es una función causal. Cuando f (n) no es
causal, entonces ztrans halla la transformada z de la parte causal de f (n) . Observe el
siguiente ejemplo.

>> syms n z
>> f 3 ( n ) = n∗ h e a v i s i d e ( n)+3ˆn∗ h e a v i s i d e (−n ) ;
>> F3 ( z ) = z t r a n s ( f 3 )
F3 ( z ) =
z / ( z − 1)ˆ2
✂ ✁
En este ejemplo se calcula la transformada z de f3 (n) = n u(n) + 3n u(−n), obteniendo
como transformada z unilateral derecha a F3 (z) = z/(z−1)2 , es decir, ztrans no consideró
la parte no causal de f (n). En el Programa 8.1 se muestra la función TransZ, la cual calcula
la transformada z bilateral de una función f (n) cualquiera.
CAPÍTULO 8. TRANSFORMADA Z 267

Programa 8.1. Función TransZ.m

f u n c t i o n [ F ] = TransZ ( f l , f r )
%TransZ H a l l a l a t r a n s f o r m a d a z b i l a t e r a l de una
% f u n c i ó n f ( n )
syms n z ;
F l ( z ) = z t r a n s ( f l (−n ) ) ;
Fr ( z ) = z t r a n s ( f r ( n ) ) ;
F ( z ) = F l ( z ˆ( −1)) + Fr ( z ) ;
end

La función TransZ se basa en el hecho que toda función f (n) se puede expresar como:

f (n) = fl (n) + fr (n),

donde fl (t) es una función no causal y fr (n) es una función causal. A continuación se
calcula la transformada z bilateral de la función f3 (n) dada arriba.

>> syms n z
>> f 3 l ( n ) = 3ˆn∗ h e a v i s i d e (−n ) ; f 3 r ( n ) = n∗ h e a v i s i d e ( n ) ;
>> F3 ( z ) = TransZ ( f 3 l , f 3 r )
F3 ( z ) =
z / ( z − 1)ˆ2 + 1/(3/ z − 1) + 1/2
✂ ✁

La expresión matemática de la transformada z bilateral de f3 (n) obtenida es:

z 1 1 z z 1
F3 = + + = 2 − z − 3 + 2.
(z − 1)2 3
z −1 2 (z − 1)

Por otra parte, M ATLAB también cuenta con el comando iztrans que halla la transfor-
mada z inversa. Observe el siguiente ejemplo donde se halla la expresión matemática de
la transformada z inversa de las transformadas F1 (z), F2 (z) y F3 (z) dadas arriba.

>> syms z
>> F1 ( z ) = z / ( z − 1 / 2 ) ; f 1 ( n ) = i z t r a n s ( F1 )
f1 (n) =
(1/2)ˆ n
>> F2 ( z ) = z / ( z − 1 ) ˆ 2 ; f 2 ( n ) = i z t r a n s ( F2 )
f2 (n) =
n
>> F3 ( z ) = z / ( z − 1)ˆ2 + 1/(3/ z − 1) + 1/2;
>> f 3 ( n ) = i z t r a n s ( F3 )
f3 (n) =
n − 3ˆn + k r o n e c k e r D e l t a ( n , 0)/2
✂ ✁

Tenga presente que M ATLAB utiliza el sı́mbolo kroneckerDelta(n, 0) para denotar δ(n).
De esta forma, las transformadas z inversas obtenidas son:

1
f1 (n) = (1/2)n , f2 (n) = n, f3 (n) = n − 3n + δ(n).
2

Note que iztrans halla la transformada z inversa de F (z), asumiendo que F (z) es la
transformada z unilateral derecha de f (n).
268 8.6. PROBLEMAS RESUELTOS

8.6 Problemas Resueltos


Problema 8.1. Determinar la transformada z de f (n) = (1 + n)u(n).

Solución. Por definición, la transformada z de f (n) está dada por:



X ∞
X
F (z) = (1 + n)u(n) z −n = (1 + n) z −n
n=−∞ n=0
X∞ ∞
X
−n
= z + n z −n
n=0 n=0
X∞ ∞
X

−1 n
n
= z + n z −1 . (8.15)
n=0 n=0

Ahora bien, se tiene que



z 1 X n
= −1
= z −1 , |z| > 1,
z−1 1−z
n=0
  "∞ #
z d z d X −1 n
= (−z) = (−z) z
(z − 1)2 dz z − 1 dz
n=0
" ∞
# ∞
X X
−1 n
−1
 n
= (−z) (−z ) n z = n z −1 , |z| > 1.
n=0 n=0

Usando convenientemente los desarrollos en series de potencias anteriores en la ecuación


(8.15), obtenemos que la transformada z de f (n) es:

z z z2
F (z) = + = , |z| > 1.
z − 1 (z − 1)2 (z − 1)2

Seguidamente se muestra el diagrama de polos (rojo) y ceros (azul) de F (z).

y
|z| > 1

b b

1 x

Diagrama de polos y ceros


CAPÍTULO 8. TRANSFORMADA Z 269

Problema 8.2. Determinar la transformada z de f (n) = n 2n cos(2n) u(−n).

Solución. Por definición, la transformada z de f (n) está dada por:



X 0
X
n −n
F (z) = n 2 cos(2n) u(−n) z = n 2n cos(2n) z −n
n=−∞ n=−∞
0  
X e2nj + e−2nj
= n 2n z −n
n=−∞
2
0 0
1 X n 2nj −n 1 X
= n2 e z + n 2n e−2nj z −n .
2 n=−∞ 2 n=−∞

Haciendo un cambio de ı́ndices k = −n y luego colocando n en lugar de k, la expresión


anterior adquiere la forma:
∞ ∞
1X 1X
F (z) = (−n) 2−n e−2nj z n + (−n) 2−n e2nj z n
2 2
n=0 n=0
∞ ∞
1 X n 1X n
= − n 2−1 e−2j z − n 2−1 e2j z . (8.16)
2 n=0 2 n=0

Sea el siguiente desarrollo de Laurent:



w X
2
= n wn , |w| < 1. (8.17)
(1 − w)
n=0

Tomando w = 2−1 e−2j z en el desarrollo de Laurent (8.17), obtenemos:



2−1 e−2j z X
−1 −2j n

= n 2 e z , |z| < 2.
(1 − 2−1 e−2j z)2
n=0

Ahora, tomando w = 2−1 e−2j z en el desarrollo de Laurent (8.17), obtenemos:



2−1 e2j z X n
−1 2j 2
= n 2−1 e2j z , |z| < 2.
(1 − 2 e z)
n=0

Usando convenientemente los desarrollos en serie de potencias anteriores en la ecuación


(8.16), obtenemos que la transformada z de f (n) es:

1 2−1 e−2j z 1 2−1 e2j z


F (z) = − −
2 (1 − 2−1 e−2j z)2 2 (1 − 2−1 e2j z)2

z e2j z e2j
= − − , |z| < 2.
(z − 2 e2j )2 (z e2j − 2)2


270 8.6. PROBLEMAS RESUELTOS

Problema 8.3. Determinar la transformada z de


(
4n − 2n , n ≤ −1,
f (n) = n n
(1/4) − (1/2) , n ≥ 0.

Solución. La función f (n) se puede expresar equivalentemente como:


f (n) = (4n − 2n )u(−1 − n) + (4−n − 2−n )u(n).
Ası́, aplicando la propiedad de linealidad de la transformada z, podemos escribir:

X
F (z) = (4n − 2n )u(−1 − n) + (4−n − 2−n )u(n)z −n
n=−∞
X∞ ∞
X
= (4n − 2n )u(−1 − n)z −n + (4−n − 2−n )u(n)z −n
n=−∞ n=−∞
X∞ X∞
= (4n − 2n )u(−1 − n)z −n + (4−n − 2−n )u(n)z −n
n=−∞ n=−∞
−1
X ∞
X
= (4n − 2n )z −n + (4−n − 2−n )z −n
n=−∞ n=0
−1
X −1
X ∞
X ∞
X
n −n n −n −n −n
= 4 z − 2 z + 4 z − 2−n z −n
n=−∞ n=−∞ n=0 n=0
X∞ ∞
X ∞
X X∞
−1
= (4 z)n − (2−1 z)n + (4−1 z −1 )n − (2−1 z −1 )n .
n=1 n=1 n=0 n=0

Luego, calculando a que converge cada una de las series de potencias anteriores, obtene-
mos:
       
1 1 1 1
F (z) = −1 + −1 + −
1 − 4−1 z 1 − 2−1 z 1 − 4−1 z −1 1 − 2−1 z −1
| {z } | {z } | {z } | {z }
|z|<4 |z|<2 |z|>1/4 |z|>1/2

De esta forma, la transformada z de f (n) es:



3 z −3z 2 + 4z − 3 1
F (z) = , < |z| < 2.
4(z − 4)(z − 2)(z − 14 )(z − 21 ) 2
Seguidamente se muestra el diagrama de polos (rojo) y ceros (azul) de F (z).
y
1
2
< |z| < 2

b b b b b
1/2 2 x
b

Diagrama de polos y ceros


CAPÍTULO 8. TRANSFORMADA Z 271

Problema 8.4. Determinar la transformada z inversa de


1+z 1
F (z) = , |z| > .
z − 12 2

Solución. Inicialmente, hallemos el desarrollo de Laurent de F (z) centrado en z0 = 0 y


válido en |z| > 12 . La función F (z) se puede expresar como:
1 z 1
F (z) = 1 + , |z| > .
z− 2 z − 21 2
1 z
Hallemos los desarrollos de Laurent de 1 y , centrados en z0 = 0 y válidos en
z− 2 z − 21
|z| > 12 . Se tiene que:
∞ ∞
1 1 X X 1
1 = z −1 = z −1
2−n −n
z = 2−n u(n)z −(n+1) , |z| > ,
z− 2
1 − 2−1 z −1 n=0 n=−∞
2
∞ ∞
z 1 X
−n −n
X 1
1 = = 2 z = 2−n u(n)z −n , |z| > .
z− 2
1 − 2−1 z −1 n=−∞
2
n=0

Ası́, el desarrollo de Laurent de F (z) centrado en z0 = 0 y válido en |z| > 12 , es:



X ∞
X
−n −(n+1)
F (z) = 2 u(n)z + 2−n u(n)z −n
n=−∞ n=−∞
X∞ ∞
X
= 2(1−n) u(n − 1)z −n + 2−n u(n)z −n
n=−∞ n=−∞

X 1
= 2−n (2 u(n − 1) + u(n))z −n , |z| > .
n=−∞
2

De esta forma, la transformada z inversa de F (z) es:

f (n) = 2−n (2 u(n − 1) + u(n)).

Problema 8.5. Determinar la transformada z inversa de



z z − 21 1
F (z) = 1 ,
 < |z| < 4.
(z − 4) z − 4 4

Solución. Inicialmente, hallemos el desarrollo de Laurent de F (z) centrado en z0 = 0 y


válido en |z| > 21 . La expansión en fracciones parciales de F (z)/z está dada por:
14 1
F (z) 15 15 
= + ,
z (z − 4) z − 41

ası́, F (z) se puede expresar como:


14 z 1 z 1
F (z) = + , < |z| < 4.
15 (z − 4) 15 z − 41 4
272 8.6. PROBLEMAS RESUELTOS

z z
Hallemos los desarrollos de Laurent de y , centrados en z0 = 0 y válidos en
z − 4 z − 14
1
4 < |z| < 4. Se tiene que:
  ∞
z −4−1 X
= z =− 4−(n+1) z (n+1)
z−4 1 − 4−1 z n=0
−1
X ∞
X
= − 4n z −n = − 4n u(−1 − n)z −n , |z| < 4,
n=−∞ n=−∞

∞ ∞
z 1 X
−n −n
X 1
1 = = 4 z = 4−n u(n)z −n , |z| > .
z− 4
1 − 4−1 z −1 n=−∞
4
n=0

1
De esta forma, el desarrollo de Laurent de F (z) centrado en z0 = 0 y válido en 4 < |z| < 4,
es:
∞ ∞
14 X n 1 X −n
F (z) = − 4 u(−1 − n)z −n + 4 u(n)z −n
15 n=−∞ 15 n=−∞
∞  
X 1 −n 14 n 1
= 4 u(n) − 4 u(−1 − n) z −n , < |z| < 4.
n=−∞
15 15 4

Por lo tanto, la transformada z inversa de F (z) es:

1 −n 14 n
f (n) = 4 u(n) − 4 u(−1 − n).
15 15


Problema 8.6. Determinar la transformada z inversa de

z 3 + 4 z 2 − 11 1 1
F (z) = 3 7 2 3 6 1 , < |z| < .
z +6z −2z+3 3 2

Solución. La función F (z) se puede expresar como F (z) = 1 + F1 (z), donde


17 2
6 z + 32 z − 13 1 1
F1 (z) =  6 , < |z| < .
z − 31 z − 12 (z + 2) 3 2

Ası́, la transformada z inversa de F (z) está dada por

f (n) = Z −1 {1 + F1 (z)} = Z −1 {1} + Z −1 {F1 (z)} = δ(n) + Z −1 {F1 (z)} . (8.18)

Determinemos Z −1 {F1 (z)}. Para ello, hallemos primero el desarrollo de Laurent de F1 (z)
centrado en z0 = 0 y válido en 13 < |z| < 21 . La expansión en fracciones parciales de F1 (z)
está dada por:
73
21 − 17
10 
73
15 1 1
F1 (z) = 1 +
 + , < |z| < .
z− 3 z − 12 (z + 2) 3 2
CAPÍTULO 8. TRANSFORMADA Z 273

Ahora, se tiene que


∞ ∞
1 z −1 X
−n −(n+1)
X 1
= = 3 z = 3(1−n) u(n)z −n , |z| > ,
z − 31 1 − 3−1 z −1 n=0 n=−∞
3

∞ ∞
1 −2 X X 1
 = =− 2(n+1) z n = − 2(1−n) u(−n)z −n , |z| < ,
z − 21 1 − 2z n=0 n=−∞
2
∞ ∞
1 2−1 X
−(n+1) n
X
= = 2 z = 2(n−1) u(−n)z −n , |z| < 2.
(z + 2) 1 + 2−1 z n=0 n=−∞

Ası́, usando los series de potencias anteriores obtenemos que el desarrollo de Laurent de
F1 (z) centrado en z0 = 0 y válido en 13 < |z| < 12 es:
∞ ∞
73 X (1−n) 17 X (1−n)
F1 (z) = 3 u(n)z −n + 2 u(−n)z −n
21 n=−∞ 10 n=−∞

73 X (n−1)
+ 2 u(−n)z −n .
15 n=−∞

Luego, sustituyendo este desarrollo de Laurent en (8.18), tenemos que el desarrollo de


Laurent de F (z) centrado en z0 = 0 y válido en 13 < |z| < 12 es:
∞  
X 73 (1−n) 17 (1−n) 73 (n−1)
F (z) = δ(n) + 3 u(n) + 2 u(−n) + 2 u(−n) z −n .
n=−∞
21 10 15

Por lo tanto, la transformada z inversa de F (z) es:


73 (1−n) 17 73
f (n) = δ(n) + 3 u(n) + 2(1−n) u(−n) + 2(n−1) u(−n).
21 10 15


8.7 Problemas Propuestos


8.1. Determinar la transformada z y su región de convergencia de las siguientes funciones
de dominio discreto.

a) f (n) = (−1)n 2−n u(n)


b) f (n) = (an + a−n )u(n), para |a| > 1
c) f (n) = n 3n sen (n) u(n)
d) f (n) = (−3)n u(−n − 1)
(
1, −5 ≤ n ≤ 5;
e) f (n) =
0, |n| > 5
(
(1/3)n , n ≥ 0;
f) f (n) = −n
(1/2) , n < 0
274 8.7. PROBLEMAS PROPUESTOS

(
(1/4)n − 2n , n ≥ 0;
g) f (n) =
0, n<0
(
(1/5)n sen (πn/2), n ≤ 0;
h) f (n) =
0, n>0
i) f (n) = eαn senh(βn)u(n), α > β > 0.

8.2. La transformada z de una función causal f (n) (f (n) = 0 para n < 0) es:
1 1
F (z) = , |z| > .
(z − 21 )3 2

Determinar la transformada z y su región de convergencia de las siguientes funciones.

a) g(n) = 2n f (n − 2)
b) g(n) = nf (n)
c) g(n) = f (−n − 1) − f (−n)
d) g(n) = (n + 1)(1/3)n f (n − 2)

8.3. Calcular la transformada inversa de las siguientes transformadas z mediante el desa-


rrollo en serie de potencias.
z+1 1
a) F (z) = 1 , |z| < 3
z+3
z
b) F (z) = , |z| > 1
(z − 1)(z + 1)
z 3 − 13 z 2
c) F (z) = , 1 < |z| < 2
(z − 1)2 (z + 2)
z 3 + 4 z 2 − 11
6
d) F (z) = , |z| < 1/3
z 3 + 67 z 2 − 32 z + 31
8.4. Calcular la transformada z inversa de
 
1 −1
F (z) = Log 1 − z , |z| > 1.
3

mediante los siguientes métodos:

a) Utilizando el desarrollo en serie de potencias



X wk
Log (1 − w) = − , |w| < 1.
k
k=1

b) Derivando F (z) y utilizando las propiedades de la transformada z.


Respuesta a los Problemas
Propuestos
Capı́tulo 1 1.5 a) −165 + i 721/32
b) (−3353 + i 2879)/256
1.1 a) 6 + i 4 c) (6 + i 43)/5
b) (11 + i 10)/17 d) (−323 − i 36)/2500
c) (3 − i 5)/2
d) 5 + i 14 1.6 a) 5329/32

e) −352 + i 936 3125 32768/32768
b) √
f) 4 − i 15/2 c) 1885/5
g) −2/5 d) 13/100
h) −4 1.7 4
1.2 a) z1 = i − 1, z2 = 2 − i 1.8 a) π4 + 2nπ, n = 0, ±1, ±2, . . .
b) z1 = 1 + i, z2 = −3 − i b) π3 + 2nπ, n = 0, ±1, ±2, . . .
1.3 a) i c) − π6 + 2nπ, n = 0, ±1, ±2, . . .
b) (1 + i)/2 1.9 a) Cerrado, no acotado
c) i b) Conexo, no acotado
d) −i, ó 2 + i c) Abierto, conexo, no acotado
e) (2 + i)/5 ó − i d) Cerrado, conexo, acotado
1
1.4 a) z0 = 2 5 ,  e) Acotado
√ √ √√ 
f) Abierto, conexo, acotado
1 5 1 2 5+5 i
z1 = 2 5 4 − 4 + 4 , g) Abierto, conexo, no acotado
√ √ √ √  h) Abierto, conexo, no acotado
1 5 1 2 5− 5 i
z2 = −2 5 4 + 4 − 4 , i) Cerrado, no acotado
√ √ √ √  j) Cerrado, conexo, acotado
1 5 1 2 5− 5 i
z3 = −2 5
4 + 4 + 4 ,
 √ √ √√ 
1 1 5 2 5+5 i Capı́tulo 2
z4 = −2 5 − +4 4 4
√√ √ √ 2.1 a) {z ∈ C : |z| 6= 1}
b) z0 = − 22+2 + 2−2 2 i , b) {z ∈ C : z 6= ±i}
√√ √ √
2+2 2− 2 i c) {z ∈ C : Re z 6= 0} √
z1 = − , d) {z ∈ C : z 6= i, (−1 ± 2)i}
√2 √ √2√
2− 2 2+2 i e) {z ∈ C : z 6= −i}√
z2 = − 2 − ,
√ √ √√ 2
f) {z ∈ C : z 6= i ± 2}
z3 = 2− 2
2
+ 2+2 i
2 g) {z ∈ C : z 6= 0}
√ √ √
c) z0 = − 22 3 + 22 i , h) {z ∈ C : z 6= 0, ±i}
√ √ √
z1 = 22 3 − 22 i , 2.2 a) u(x, y) = x2 −y 2
√ x4 +2 y 2 x2 +y 4 ,
z2 =−√ 2 i, v(x, y) = −2xy
x4 +2 y 2 x2 +y 4
z3 = 2 i, √
√  2 x2 −3 x+2 y 2 −5
z4 = 2 23 + 2i , b) u(x, y) = 4 x2 −20 x+4 y 2 +25 ,
√ √  −7 y
v(x, y) =
z5 = − 2 23 + 2i 4 x2 −20 x+4 y 2 +25
3 2
2
c) u(x, y) = x − 3x(y + 1) + 2,
d) z0 = 2 3 ,  √  v(x, y) = y(3x2 − y 2 − 3)
2
z1 = 2 3 − 21 + 3i
, 2x2 +3x+y+2y 2 −2
 2
√ 
d) u(x, y) = 4 x2 −4 x+4 y 2 +4 y+2 ,
2
z2 = −2 3 21 + 3i
2 v(x, y) = −x−5y−2
4 x2 −4 x+4 y 2 +4 y+2

275
276 RESPUESTA A LOS PROBLEMAS PROPUESTOS

2.3 a) z = −i √ 2.17 a) (2n ± 31 )πi, n = 0, ±1, ±2, . . .


b) z = (1 ± 5)/2 b) (2n + 12 )πi, n = 0, ±1, ±2, . . .
c) z = 4/3
c) (2n + 12 )πi − ln 3
2 , n = 0, ±1, ±2, . . .
2.4 a) −(3/5) − 4i/5
d) i
12 − 13 i
b) √
c) 2(1 + i)/2 e) ln 3 + (2n + 1)πi, n = 0, ±1, ±2, . . .
d) (7 − 4i)/13
2.18 a) e−π/2
e) 1/2
f) 1/3 b) eπ+i2 ln 2
g) (1/2) − 2i/3 c) e3π
2
/2
h) 1
i) −π/2 2.19 D = C − {z ∈ C : Re z ≤ 0, Im z = −1}
j) −π/2 2.20 D = C − {z ∈ C : Re z = 0, |Im z| ≥ 1}
2.5 1 + i 1
2.21 a) f (z) = e 2 Log (e +1)z

′ z
√ z ,
2.6 a) C − {−3i} f (z) = e /(2 e + 1)
b) C b) f (z) = cos(Log (z)),
c) C − {z : Im z = 0} f ′ (z) = −sen (Log (z)) /z
2.7 C c) f (z) = Log (ez + 1),
f ′ (z) = ez /(ez + 1)
2.9 a) D = {z = x + i y : y = x}, f ′ (z) = 2x 2
d) f (z) = e(Log (z)) ,
b) D = {z = x + i y : x = −3/2}, 2
f ′ (z) = 2 eLog(z) Log (z) /z
f ′ (z) = 2x + 2
e) f (z) = ecos z Log (i) ,
c) D = {− 21 , − 21 + i}, f ′ (z) = 2y + i f ′ (z) = −Log (i) eLog (i) cos(z) sen (z)
2.11 a) D = C, f ′ (z) = 2(z + 1) f) f (z) = esen z Log (z) ,  
sen(z)
b) D = C − {0}, f ′ (z) = 1 − 1 f ′ (z) = eLog(z) sin(z) cos(z) Log (z) + z
z2
8
c) D = C − {3}, f ′ (z) = (z−3)2
z
2.22 f ′ (z) = log(i) ez ee log(i) ,
3 2 f ′ (πi/2) = 3πe3π/2 /2
d) D = C − {0, ±i}, f ′ (z) = − 4zz2 (z
+3 z +1
2 +1)2
(
 √ 1 1  v ≤ −1/2
e) D = C − ± 2 2 ± 2 i , 2.27 f (D) :
z3 u+v ≥1
f ′ (z) = (z44+1) 2

f) D = C \ {z√∈ C : Im z = 0 y Re z ≤ 2.28 a) f (D) = M1√− M2 , donde


|w| ≥ 2/2,
M1 : (
−4 ó z = ± 22 (1 − i)},
u−v ≥0
1
f ′ (z) = (z2 +i)(z+4) − 2zLog (z+4)
(z 2 +i)2
M2 :
u+v ≤0
z
g) D = C, f ′ (z) = ez+e b) f (D) = M1 − M2 , donde
h) D = C, f ′ (z) = ez cos(ez ) M1 : |w
( − 1 + 2i| ≥ 2,
|w| ≤ 1
2.13 D = {z = x + iy : xy < 1} M2 :
|w + 1 + i| ≤ 1
2.14 a) D = R2 ,
c) f (D) = M1 − M √2 , donde
v(x, y) = y 2 − x2 − 3x + c
M1 : |w + 3| ≥ 8,
b) D = R2 − {(1, 0)},
( √
−y |w − 1| ≤ 2
v(x, y) = x2 −2x+y 2 +1 + c
M2 : √
|w + 1| ≤ 2
c) D = R2 − {(0, 0)},
2 2 d) f (D) = M1 − M2 , donde

u(x, y) = x(xx2+y
+y 2
+1)
+c |w + 1 − 2i| ≤ 2,
M1 : (
d) D = R2 − {(−1, −2)}, |w| ≥ 1
M2 :
u(x, y) = (x+1)x+1
2 +(y+2)2 + c |w − 1 − i| ≥ 1
RESPUESTA A LOS PROBLEMAS PROPUESTOS 277

Capı́tulo 3 3.3 a) z0 = 0, polo de orden 2


z1 = 2 + i, polo simple
3.1 a) |z| < 1 g) |z| < e z2 = 2 − i, polo simple
b) |z| < ∞ h) |z| < e−1 b) z0 = −2, polo de orden 2
c) z = 0 i) |z| < |a| −1 z1 = 3, polo simple
d) |z| < 2 j) |z − 1| > 2 c) z0 = −1, polo simple
z1 = 1,

punto singular removible
e) |z + i| < 1 k) |z + 2| < 2 z2 2
= 2 (1 + i), polo simple

f) |z| < 1 l) |z| < 1 z3 = − 22 (1 + i), polo simple

P (−1)n (z−2i)n
d) z0 = −1, polo simple
3.2 a) (2i)n+1 , |z − 2i| < 2 z1 = 1, polo simple
n=0 √
2
P∞
(−1)n (2i)n z2 = 2 (1 + i), polo simple
|z − 2i| > 2 √
(z−2i)n+1 , z3 = − 22 (1 + i), polo simple
n=0

P e) z0 = 0, polo de orden 2
b) − (z − 2)n , |z − 2| < 1
n=0 f) z0 = 0, punto singular esencial

P −(n+1)
(z − 2) , |z − 2| > 1 g) z0 = 0, punto singular removible
n=0
P∞ h) z0 = 0, polo de orden 2
c) (−1)n z 2n , |z| < 1
n=0
P∞ Capı́tulo 4
(−1)n z −2(n+1) , |z| > 1
n=0 4.1 a) − πi
2
P∞
n+1 2n−2 b) πi
d) (−1) nz , |z| < 1
n=1
P∞ 4.2 0
(−1)n (n + 1)z −2(n+2) , |z| > 1
n=1 4.3 2 − πi
P∞    √ 
1 1 n
e) 3n+1 − 2n+1 (z + 1) , |z + 1| < 2 4.4 8 32+4 i
n=0
∞ ∞
P (z+1)n P 2n 4.5 a) C no debe contener ni confinar el
3n+1 + (z+1)n+1 , 2 < |z+1| < 3
n=0 n=0 punto z = −2
P∞
3n −2n b) C no debe intersectar con el eje real
(z+1)n+1 , |z + 1| > 3
n=0 negativo ni el cero

P (−1)n 1 1
f) 4n+1 (z − 1)n + 8(z−1) + 2(z−1)2 ,
c) C no debe contener ni confinar los ce-
n=0 ros de la función f (z) = 1 + ez
0 < |z − 1| < 4

P (−1)n 4n 1 1
d) C no debe contener ni confinar los ce-
(z−1)n+1 + 8(z−1) + 2(z−1)2 , ros de la función f (z) = 1 − ez
n=0
|z − 1| > 4 1
4.6 a) F (z) = − z−i +c

P n
(−1)
g) (2n)! z 2(n−1) , |z| > 0 1
b) F (z) = − z−i +1
n=0
∞ ∞ 4.7 a) 0
P 8(−1)n P 12(−1)n
h) + (2n)! (z−2)2n−1 +
n=0
(2n)! (z−2)2n
n=0
b) 2 e−2 πi
∞ ∞ sen (1)(e−sen (1)) cos(1)(cos(1)+e)
P 6(−1)n P (−1)n c) + −
(2n)! (z−2)2n−2 + (2n)! (z−2)2n−3 ,
4 4
n=0 n=0 sen (1)(cos(1)+e)
|z − 2| > 0 16
21
∞  n 2i
 d) 20! 2 πi
P e
−2i
−(−1) e (2i) (z+1)n−3
n
i) 2i n! , e) 2πi sen (a)/a
n=0 
|z + 1| > 0 f) ea 1 + a2

P ∞
P
3
+ 3 4.8 a) 2π
j) n! (z−1)n n! (z−1)n−1 +
n=0 n=0
P∞ b) π/3
1
n! (z−1)n−2 , |z − 1| > 0 c) 4π
n=0
278 RESPUESTA A LOS PROBLEMAS PROPUESTOS


d) 6 5 d)
f4 (t)
4.9 a) Res [f (z)] = 1/3, Res [f (z)] = 5/3
z=−1 z=2
1
b) Res [f (z)] = 1, Res [f (z)] = −1
z=0 z=1
c) Res [f (z)] = 1/2, Res [f (z)] = 54 (1− 3i ), t
z=1 z=3i a
5
Res [f (z)] = 4 (1 + 3i )
z=−3i
d) Res [f (z)] = 1/4, Res [f (z)] = 83/4
z=0 z=−4
e) Res [f (z)] = −1/8, Res [f (z)] = 0 e)
z=−3 z=0 f5 (t)
f) Res [f (z)] = 1 1
z=0

Res [f (z)] = 2 ··· ···
g) 4 (1 + i)
z=eπi/4
 √  √  −3π
-π −π π
π 3π t
1+ 3i 3+ 3
h) Res [f (z)] = −sen 2 12
2 2 2 2
z=eπi/3
i) Res [f (z)] = −π
z=π
4.10 a) i) 0, ii) 2πi f)
b) i) π3 (−2 − 4i3 ), ii) 2πi
f6 (t)
6π √
c) i) 0, ii) (2 6−1)(

6+1)
, iii) 0 1
d) i) 0, ii) 0
√ √
e) i) 2πi, ii) 6πi −( 2
2
+ 1) ( 2
2
− 1) t
π
f) i) 0, ii) 25 (−14 + 2i)
19πi
g) i) 108 , ii) − 19πi
108

g)
Capı́tulo 5 f7 (t)
5.1 a) 4
f1 (t)
3
4
2
3
1
2 1/2
1 1 1 3 2 t
2 2

1 2 3 t

b)
f2 (t) h)
f8 (t)
1
4

1 2 3 t 3

c) 2
f3 (t)
1
1

-5 - 9 -4 -3 -2 - 3 -1 1 3 2 3 t
a t 2 2 2
RESPUESTA A LOS PROBLEMAS PROPUESTOS 279

5.2 a) 0 b)
g(n)
b) 12 2 b

c) 1/2 1 b

d) −9
1 2
n
5.3 a) h(t) = pa (t − (a + b))
c)
b) No existe g(n)
b
1 2 1 2
c) h(t) = 2t u(t) − 2 (t − 1) u(t − 1)
b b
1
d) h(t) = −(t + 1)u(t + 1) + 2t u(t)
−(t − 1)u(t − 1)
n
 -20 -16 -12 -4
e) h(t) = u(−t) + 2e−t − e−2t u(t) -1

f) h(t) = (1 − (t + 1)e−t ) u(t) b


-2 b


g) h(t) = 12 3 − e−2t u(t)
h) h(t) = e−2(t−1) u(t − 1) + d)
g(n)
e−2t (et − 1) u(t) 2 b

5.4 a) h(t) = −(t + 2)u(t + 2) + 1


2(t + 1) u(t + 1) −
2(t − 1)u(t − 1) + n
1 2 3
(t − 2) u(t − 2) -1 b

b) h(t) = −(t + 1)u(t + 2) + -2 b

2(t + 1) u(t + 1) − 2 u(t)−


2(t − 1)u(t − 1) +
(t − 1) u(t − 2) e)
g(n)
c) h(t) = 2(t + 2)u(t + 2) − 2
2(t + 1) u(t + 1) − 2t u(t)+ b b b b b b b b b
1
2(t − 1)u(t − 1) + p1 (t − 2) ··· ···
 b b b b


 0, t ≤ −1, -6 -5 -4 -3 -2 -1 1 2 3 4 5 6
n


 2
− (t+1) (t−2) ,

−1 < t ≤ 0,


 6 f)
 2 g(n)
d) h(t) = (t−1) (t+2)−3t(t−2)
6 , 0 < t ≤ 1, 2



 (t−2) 2


 2 , 1 < t < 2, b b b b
1 b b b b

 ··· ···


0, t≥2 b b b b b

-6 -5 -4 -3 -2 -1 1 2 3 4 5 6
n
5.5 a) 5 4
g(n) 5.6 a) h(n) =
P
u(n − k) −
P
u(n + k)
b
2 k=1 k=0

1 b b) h(n) = (4(1/2)n − 2(1/3)n ) u(n) +


(1/2)n−1 u(n − 1)
n c) h(n) = (n + 3) (u(n + 2) − u(n − 1)) +
1 2 3 4 5
-1 b (3 − n) (u(n − 1) − u(n − 3))
d) No existe
b b
-2 3
P
e) h(n) = (n − k)u(n − k)
k=−2
280 RESPUESTA A LOS PROBLEMAS PROPUESTOS

f) h(n) = (n − 1) (u(n + 1) − u(n − 4)) + p1 ((ω − 1)/2)


l) G12 (ω) = + πδ(ω)
(n − 2) (u(n) − u(n − 5)) + jω
2  
P
k u(k − n) m) G13 (ω) = 2j p1 ω+π−1 2 − p1 ω−π−1
2

k=−2 n) G14 (ω) = e−jω p1 ω−1 2
2
P k  
(−1) 1
g) h(n) = [δ(n − k)− 1
k=−2
2k+2 6.5 a) f (t) = 2π πδ(t) − 2 −
jt
δ(n − (k + 1)) + δ(n − (k + 2)) 1 −2

−δ(n − (k + 3))] b) f (t) = 4π t (1 − cos(2t))
 
5
4 P 1 4 sen t
h) h(n) =
P (−1)ℓ
δ(n − (k + ℓ)) c) f (t) = 2π 1 +
ℓ+1 t
k=0 ℓ=0
1
d) f (t) =
πt
Capı́tulo 6 √
6.6 a) A(ω) = 2/ 1 + 4ω 2 , para ω ∈ R
1
6.1 a) F (ω) = φ(ω) = arctan(2ω), para ω ∈ R
1
2 − jω
b) A(ω) = 2|sen ω|/|ω|, para ω 6= 0
2 ejω sen ω ∞
P  
b) F (ω) = φ(ω) = (ω−kπ)pπ/2 ω − (2k+1)π 2 ,
ω k=0
16 sen 2 (ω/8) para ω > 0
c) F (ω) =
ω2 16 sen 2 (ω/8)
c) A(ω) = , para ω 6= 0
8 ω2
d) F (ω) = + 8j πδ ′ (ω) φ(ω) = 0

1 1
e) F (ω) = πj δ ′ (ω) − ω2 d) A(ω) = 2 , para ω 6= 0
ω
f) F (ω) = 2π e−jωt0 φ(ω) = π, para ω 6= 0
g) F (ω) = 2π cos(πω) e) A(ω) =  2|sen ω|/|ω|, para ω 6= 0
0 < ω ≤ π2 ,

 jω
 −2ω,
e 2 2 − 2 e 2 + jω 
h) F (ω) = φ(ω) = 2π − 2ω, π2 < ω ≤ π
2 ω2 

3π − 2ω, π < ω ≤ 2π
2a
i) F (ω) = 2 (periódica, con periodo 2π, para ω >
a + ω2
0)
j) F (ω) = 2π e−a|ω| 1
1 f) A(ω) = 2 , para ω 6= 0
k) F (ω) = + 2πδ(ω) ω
ω φ(ω) = π, para ω 6= 0
2 sen ω 8j e−jω sen 2 (ω/4) 8
l) F (ω) = + g) A(ω) = , para ω 6= 0
ω ω2 |ω|
φ(ω) = − 2 u(−ω) + π2 u(ω)
π
6.3 a) G1 (ω) = p1 ((−ω − 1)/2)
b) G2 (ω) = j [δ(ω + 1) − δ(ω − 3)]
Capı́tulo 7
c) G3 (ω) = ejω p1 ((ω − 1)/2)
1 (s + 1)e−s + s
d) G4 (ω) = 5 e−3jω/5 p1 (−(ω + 5)/10) 7.1 a) F (s) = , Re s < −1
s+1
e) G5 (ω) = −2 ejω p1 ((ω − 1)/2) +
1 − e−5(s−a)
j(δ(ω + 1) − δ(ω − 3)) b) F (s) = , Re s > a
s−a
f) G6 (ω) = jω p1 ((ω − 1)/2) 4
c) F (s) = 2 , Re s > 4
g) G7 (ω) = −p1 ((ω − 1)/2) + s − 8s + 32
δ(ω + 1) + 3δ(ω − 3) 2(s − 1)
d) F (s) = , −3 < Re s < 5
h) G8 (ω) = 1
e−j(ω+2)/2 p1 (ω/4) (s + 3)(s − 3)
2
i) G9 (ω) = p1 ((ω + 1)/2) e) No existe
j) G10 (ω) = j(δ(ω + 3) − δ(ω − 1)) f) No existe
s−1
k) G11 (ω) = j(ej δ(ω + 3) − e−3j δ(ω − 1)) g) F (s) = 2 , Re s > 0
s
RESPUESTA A LOS PROBLEMAS PROPUESTOS 281

n! 1
e) f (t) = − 24 37

+ 5t e−2t u(t) −
h) F (s) = − , Re s < −1 6
(s + 1)n+1 1 1 2 13
 5 t
s 4 2 t − t − 4 + 9 e u(−t)
i) F (s) = 2 , Re s > 0 −t
s + a2 f) f (t) = e cos(t) u(t) − cos(2t) u(−t)
a   
j) F (s) = 2 , Re s < −a 7.6 f (t) = α9 et u(−t) − 20e−2t − 18e−t u(t)
a − s2
1 − (1 + 3s)e−3s
7.2 a) F1 (s) = , Re s > 0 Capı́tulo 8
s2
1 − (1 + 3s)e−3s 1 z
b) F2 (s) = + , Re s > 0 8.1 a) F (z) = , |z| > 21
s2 2 z + 21
  2 
1 − e−2s 2z z − a 2a +1
c) F3 (s) = , Re s > 0
s b) F (z) = , |z| > |a|
2 (z − a)(z − a1 )
2e−3s/2 es/2 − 1
d) F4 (s) = , Re s > 0 3 senh(1) z (z 2 − 9)
s2 c) F (z) = , |z| > 1
(z 2 − 6 cos(1) z + 9)2
18e−6s
7.3 a) G(s) = , Re s > − 13 3
(s + 31 )2 d) F (z) = , |z| < 3
z+3
4e−2s (s + 2) z 11 − 1
b) G(s) = , Re s > −1 e) F (z) = 5 , |z| > 1
(s + 1)3 z (z − 1)
2e−3s (3s + 5) − 35 z
c) G(s) = , Re s > −1 f) F (z) = , 1
< |z| < 2
(s + 1)3 (z − 13 )(z − 2) 3

2s e−2s
d) G(s) = , Re s > −1 − 47 z
(s + 1)2 g) F (z) = , |z| > 2
(z − 14 )(z − 2)
e−2s (4s2 + 6s − 2)
e) G(s) = , Re s > −1 −5 z
(s + 1)3 h) F (z) = , |z| < 15
( 25z 2 + 1
0, a < 0, eα senh(β) z
f) G(s) = −as
s∈C i) F (z) = 2 ,
f (a)e , a ≥ 0, z − 2eα cosh(β)z + e2α
 |z| > eα+β
2e−3s s2 es + s(2 + es ) + 4
g) G(s) = , 16
(s + 1)3 8.2 a) G(z) = , |z| > 1
Re s > −1 z 2 (z − 1)
2e−2s 2z
h) G(s) = , Re s > 0 b) G(z) = , |z| > 12
s(s + 1)3 (z − 21 )3
7.4 a) f (0+ ) = 1, lı́m f (t) = 0 4z 2 (z − 1)
t→∞ c) G(z) = , |z| < 2
(z − 2)2
b) f (0+ ) = 0, lı́m f (t) = 0
t→∞ 10z − 1 1
d) G(z) = , |z| >
+
c) f (0 ) = 0, lı́m f (t) = 6
25
162 z 2(z − 61 ) 6
t→∞
+
d) f (0 ) = ∞, lı́m f (t) = 0 8.3 a) f (n) = δ(n) + 2(−1)n 3n u(n)
t→∞
1
b) f (n) = (1 − (−1)n ) u(n)
e) f (0+ ) = 0, lı́m f (t) = 0 2
t→∞ 13 2
c) f (n) = 27 u(n) + 9 n u(n) −
+
f) f (0 ) = 1, lı́m f (t) = 0 14
t→∞
27 2n u(−n − 1)
7.5 a) f (t) = sen (−t) u(−t) d) f (n) = δ(n) + 37
(−1)n 2n u(−n) +
70
b) f (t) = sen (at) u(t) 17 73
  5 2−n u(−n) − 7 3−n u(−n)
c) f (t) = 41 3e−t u(t) − e3t u(−t)
3−n
d) f (t) = δ(t) + (2 + et ) u(t) 8.4 f (n) = − u(n − 1)
n
A
Introducción a M ATLAB
M ATLAB es un sistema interactivo para el cálculo numérico. A finales de 1970, el ana-
lista numérico Cleve Moler escribió la versión inicial de M ATLAB en Fortran como un ma-
terial didáctico. Se hizo popular para la enseñanza y la investigación y se convirtió en un
paquete de software comercial escrito en C. Desde hace muchos años, M ATLAB ha sido am-
pliamente utilizado en las universidades y la industria. Presentamos brevemente la sintaxis
y los procedimientos básicos de programación en M ATLAB . Para un estudio más extenso de
M ATLAB se recomienda la referencia [4].

A.1 Escritorio de M ATLAB


Por ser M ATLAB un sistema iterativo, al escribir comandos en el prompt (>>) en la
ventana de comandos (Command Window) del escritorio de trabajo de M ATLAB (ver Fi-

gura A.1), los cálculos se realizan a cabo cuando se pulsa la tecla Intro o Retorno .

Figura A.1. Escritorio de trabajo de M ATLAB

En su nivel más simple, M ATLAB se puede utilizar como una calculadora de bolsillo:

>> (1+ s q r t ( 2 ) ) / 2
ans =
1.2071
>> 2ˆ(−87)
ans =
6.4623e−27
✂ ✁

El primer ejemplo calcula (1 + 2)/2 y el segundo 2−87 . Observe que el segundo resultado
se muestra en notación exponencial: esto representa 6.4623 × 10−27 . La variable ans se
crea automáticamente (o se reescribe, si ya existe) cuando una expresión no se asigna a

282
APÉNDICE A. INTRODUCCIÓN A MATLAB 283

una variable, y se puede hacer referencia de ella más adelante, al igual que cualquier otra
variable.
En la ventana de comandos de M ATLAB también se pueden escribir distintas opera-
ciones: suma de números, vectores o matrices, multiplicación de números, vectores o
matrices, entre otras. Al escribir en M ATLAB se debe tener presente lo siguiente:

• M ATLAB distingue entre mayúsculas y minúsculas, por ejemplo, A y a son variables


distintas.
• Es posible escribir varias instrucciones en la misma lı́nea, separándolas por una coma
o por punto y coma.
• Se pueden recuperar comandos anteriores, usando las teclas de flechas: arriba ↑ y
abajo ↓ .
• Con las flechas izquierda ← y derecha → nos podemos desplazar sobre la lı́nea de
comando y modificarla.

En M ATLAB también podemos crear secuencias de comandos relacionados. Para ello


se utiliza un M-archivo (M-file). Más detalles de la creación y uso de los M-archivos se
encuentra en la Sección A.4.
M ATLAB almacena números y calcula con una precisión relativa de aproximadamente
16 dı́gitos decimales. Por defecto se muestran los números en un formato de 5-dı́gitos
fijos. Si bien este formato es conciso, éste no siempre es el más útil. El comando format se
puede utilizar para establecer un formato de punto flotante de 5 dı́gitos (también conocido
como notación cientı́fica o exponencial):

>> 1/3
ans =
0.3333
>> format s h o r t e
>> ans
ans =
3.3333e−01
✂ ✁
El formato por defecto puede ser reintegrado escribiendo format short o simplemente
format. El comando format tiene muchas opciones, que se pueden ver escribiendo help
format. Consulte la Tabla A.1 para algunos ejemplos.

Tabla A.1. 10π representado en varios formatos de salida


format short 31.4159
format long 31.415926535897931
format short e 3.1416e+01
format long e 3.141592653589793e+01
format short g 31.416
format long g 31.4159265358979
format hex 403f6a7a2955385e
format bank 31.42
format rat 3550/113

Todas las salidas de M ATLAB que se muestran en estas notas se han generado con
format compact, que suprime las lı́neas en blanco. En general, help foi muestra infor- Pruebe con help
help
mación sobre el comando foi.
284 A.2. OBJETOS Y SINTAXIS

A.2 Objetos y Sintaxis


El objeto básico de trabajo de M ATLAB es una matriz bidimensional cuyos elementos son
números reales o complejos. Los escalares y vectores son matrices particulares, por ejem-
plo, un número se considera una matriz 1 × 1 y un vector de n componentes, dependiendo
de la definición dada por el usuario, se puede considerar una matriz n × 1 (matriz columna
de n filas) o una matriz 1 × n (matriz fila de n columnas). También se pueden construir
cadenas de caracteres, booleanas, enteras y de polinomios. Como M ATLAB considera a to-
dos estas estructuras de datos una matriz, define sus operaciones aritméticas a través del
algebra de las matrices. En la Sección A.3 daremos más detalles de la manipulación de los
objetos de M ATLAB .

Constantes

Algunas constantes numéricas muy particulares y de gran importancia en el cálculo


cientı́fico también tienen una representación especı́fica. A continuación damos una lista de
algunas constantes predefinidas:
• pi es el número π,
• e es el número e,
• i es la unidad imaginaria,
• eps es la precisión de la máquina (mayor número real en doble precisión para el que
1+eps/2 es indistinguible de 1; aproximadamente se tiene que eps ≈ 2.220×10−16 ),
• inf es el “infinito máquina” (overflow: cualquier número que supere al mayor número
real representable en doble precisión),
• nan es el sı́mbolo NaN (Not a Number) para una operación inválida (por ejemplo, 0/0
es nan).

Operadores Aritméticos, Lógicos y de Comparación

Además de la aritmética real, M ATLAB también puede trabajar con aritmética compleja.
Un número complejo en M ATLAB está representado por a+bi, donde a y b son números
reales. La aritmética compleja tiene la misma sintaxis que la aritmética real. Las siguientes
tablas muestran una lista de los sı́mbolos que denotan los operadores aritméticos, lógicos
Ejecute y de comparación usados en la sintaxis de M ATLAB .
(5-3i)+i en el
promtp y observe
el resultado.
Tabla A.2. Operaciones aritméticas

Operación Sı́mbolo
Suma +
Resta -
Multiplicación *
División /
Potenciación ^
Producto punto de vectores o matrices (producto de Hadamard) .*
División punto de vectores o matrices ./
Elevar a una potencia un vector o matriz .^
APÉNDICE A. INTRODUCCIÓN A MATLAB 285

Tabla A.3. Operadores lógicos

Operador Sı́mbolo
y &
o |
no ∼

Tabla A.4. Operadores de comparación

Operador Sı́mbolo
igual que ==
no igual que ∼=
menor que <
mayor que >
menor o igual que <=
mayor o igual que >=

Variables

En M ATLAB no hace falta declarar las variables tal como se hace en C++ o Visual Basic,
entre otros lenguajes de programación. El tipo de variable y su tamaño cambian de forma
dinámica de acuerdo con los valores que le son asignados. Por ello, una misma variable
puede ser utilizada, por ejemplo, para almacenar un número, después puede ser una matriz
de números enteros con 10 filas y 4 columnas, y luego puede ser una cadena de caracteres.
Las variables se crean automáticamente al asignarles un valor. También existe la posi-
bilidad de eliminar una variable de la memoria si ya no se utiliza. Para asignar un valor a
una variable se utiliza el sı́mbolo “=”, observe el siguiente ejemplo.pPrimero creamos una
variable a asignándole el valor de 5, luego realizamos la operación log(a3.3 + 4).

>> a=5
a =
5
>> s q r t ( l o g ( a ˆ 3 . 3 + 4 ) )
ans =
2.3088
✂ ✁

El resultado de la operación sqrt(log(aˆ3.3 + 4)) se guarda en la variable ans. Ahora,


si colocamos b=sqrt(log(aˆ3.3 + 4)), M ATLAB crea la variable b con el valor obtenido
en la operación, como se puede apreciar seguidamente.

>> b=s q r t ( l o g ( a ˆ 3 . 3 + 4 ) )
b =
2.3088
✂ ✁

Para conocer en cualquier instante el valor almacenado en una variable basta con escri-
bir su nombre. También M ATLAB cuenta con algunos comandos que permiten listar o eli-
minar las variables definidas. Tales comandos se muestran en la Tabla A.5.
286 A.2. OBJETOS Y SINTAXIS

Tabla A.5. Comandos para listar y eliminar variables

Comando Descripción
who lista las variables actuales
whos lista en forma detallada las variables actuales
clear elimina todas las variables que existan en ese momento
clear a b c elimina las variables a, b y c (las variables no se separan con
comas)

Seguidamente se muestra el uso de los comandos de la Tabla A.5, con las variables a y
b creadas en los ejemplos anteriores.

>> who
Your v a r i a b l e s a r e :
a b
>> whos
Name Size Bytes Class Attributes
a 1x1 8 dou ble
b 1x1 8 dou ble
>> c l e a r b
>> whos
Name Size Bytes Class Attributes
a 1x1 8 dou ble
✂ ✁

Funciones Elementales

M ATLAB está provisto de un gran número de funciones elementales. Algunas de estas


funciones se pueden apreciar en la Tabla A.6. Los argumentos de las funciones elementales
pueden ser (siempre que tenga sentido) reales o complejos y el resultado se devuelve en el
mismo tipo del argumento.
En los siguientes ejemplos mostramos como se utilizan las funciones elementales en
M ATLAB . La sintaxis de M ATLAB es muy sencilla. Observe el siguiente ejemplo:

>> c o s ( p i )
ans =
−1
>> c o s (1+ p i ∗ i )
ans =
6.2632 − 9.7179 i
✂ ✁

Con el comando cos(pi) se calcula cos(π) = −1 y con cos(1+pi*i) se calcula cos(1 + πi)
que es número complejo. Ahora calculemos exp(i*pi).

>> exp ( i ∗ p i )
ans =
−1.0000 + 0.0000 i
✂ ✁

¿Qué observa?
APÉNDICE A. INTRODUCCIÓN A MATLAB 287

Tabla A.6. Funciones elementales


Comando Nombre de la función
exp(x) exponencial
log(x) logaritmo natural
log10(x) logaritmo en base 10
sqrt(x) raı́z cuadrada
abs(x) valor absoluto o módulo
conj(z) conjugado
real(z) parte real
imag(x) parte imaginaria
rat(x) aproximación racional
mod(x,y) resto de dividir x entre y
floor(x) entero n tal que (n-1) <= x <= n
ceil(x) entero n tal que n <= x < (n+1)
round(x) redondeo (hacia el entero más cercano)
sin(x) seno (radianes)
cos(x) coseno (radianes)
tan(x) tangente (radianes)
cotg(x) cotangente (radianes)
asin(x) arcoseno
acos(x) arcocoseno
atan(x) arcotangente
sinh(x) seno hiperbólico
cosh(x) coseno hiperbólico
tanh(x) tangente hiperbólica
asinh(x) arcoseno hiperbólico
acosh(x) arcocoseno hiperbólico
atanh(x) arcotangente hiperbólica

Utilidades

En la Tabla A.7 se muestran algunos comandos utilitarios disponibles.

Tabla A.7. Comandos utilitarios


Comando Descripción
ls lista los archivos del directorio actual
dir lista los archivos del directorio (formato distinto a
ls)
pwd devuelve el path (dirección) del directorio actual
cd permite cambiar el directorio actual
clc limpia la ventana de trabajo
date muestra la fecha actual
cputime tiempo de CPU en segundos
288 A.3. MATRICES

A.3 Matrices
Como mencionamos al comienzo de este anexo, M ATLAB tiene la propiedad que sus
objetos básicos son matrices, lo cual permite una manera muy fácil e intuitiva de trabajar
con vectores y matrices.

Generación de Matrices

Muchas matrices elementales pueden ser construidas directamente con funciones de


M ATLAB . La matriz de ceros, la matriz de unos y la matriz identidad, se generan utilizando
las funciones zeros, ones y eye, respectivamente. Por ejemplo, ones(m,n) produce una
matriz m × n de unos, mientras que ones(n) produce una matriz n × n de unos. Ejemplos:

>> z e r o s ( 2 )
ans =
0 0
0 0
>> ones ( 3 , 2 )
ans =
1 1
1 1
1 1
>> eye ( 3 , 3 )
ans =
1 0 0
0 1 0
0 0 1
✂ ✁
Otras dos funciones muy importantes para crear matrices son rand y randn, que ge-
neran matrices de números aleatorios con la misma sintaxis que eye. La función rand
produce matrices con elementos distribuidos uniformemente en el intervalo (0, 1). La fun-
ción randn produce matrices cuyos elementos siguen una distribución normal con media
0 y varianza 1 o simplemente distribución normal estándar. Cuando ambas funciones se
utilizan sin argumentos, producen un único número aleatorio.

>> rand
ans =
0.8147
>> rand ( 4 )
ans =
0.9058 0.0975 0.9649 0.4854
0.1270 0.2785 0.1576 0.8003
0.9134 0.5469 0.9706 0.1419
0.6324 0.9575 0.9572 0.4218
✂ ✁
En M ATLAB también se pueden construir matrices escribiendo uno a uno sus elementos,
por filas, entre corchetes rectos ([ ]). Por ejemplo, una matriz 3 × 3 con los 9 primeros
números primos puede construirse con el siguiente comando:

>> A = [2 3 5
7 11 13
17 19 23]
A =
2 3 5
7 11 13
17 19 23
✂ ✁
APÉNDICE A. INTRODUCCIÓN A MATLAB 289

El final de una fila puede ser especificado por un punto y coma en lugar de un retorno de
carro, por lo que un comando más compacto con el mismo efecto es

>> A = [2 3 5 ; 7 11 13; 17 19 23]
✂ ✁
Dentro de una fila los elementos pueden estar separados por espacios o por comas. En
el primer caso, si los números se especifican con un signo más o menos, se debe tener
cuidado de no dejar un espacio después del signo, de lo contrario M ATLAB interpretará el
signo como una adición u operador de resta. Como ilustración veamos esto con vectores:

>> v = [0 9 −6 5 −7]
v =
0 9 −6 5 −7
>> u = [0 ,9 , −6 ,5 , −7]
u =
0 9 −6 5 −7
>> w = [0 9 − 6 5 −7]
w=
0 3 5 −7
✂ ✁
Las matrices también pueden construirse por bloques. Dada una matriz B=[1,4;3,5],
se puede crear

>> A = [ eye ( 2 ) ones ( 2 )
z e r o s ( 2 ) B]
A =
1 0 1 1
0 1 1 1
0 0 1 4
0 0 3 5
✂ ✁
Esto también se puede utilizar para generar nuevos vectores, utilizando otros previamente
creados:

>> u1 = [1 2 3 4]
u1 =
1 2 3 4
>> u2 = [ u1 , 5 , 0 , 0 , 7 , 8 , 9 ]
u2 =
1 2 3 4 5 0 0 7 8 9
✂ ✁

Subı́ndices y la Notación de Dos Puntos


Para permitir el acceso y la asignación de submatrices, M ATLAB posee una poderosa
notación basada en el carácter dos puntos (:). Los dos puntos se utilizan para definir
vectores que pueden actuar como subı́ndices. Para enteros i y j, i:j denota el vector
fila de números enteros de i hasta j (en pasos de 1). Un paso no unitario (o paso) s se
especifica como i:s:j. Esta notación es válida incluso para números i, j y s no enteros.
Veamos unos ejemplos:

>> 1:6
ans =
1 2 3 4 5 6
>> 5: −1:1
ans =
5 4 3 2 1
>> 0 : 0 . 2 5 : 1 . 2 5
290 A.3. MATRICES

ans =
0 0.2500 0.5000 0.7500 1.0000 1.2500
✂ ✁
Un elemento simple de una matriz A se accede con A(i,j), donde i≥1 y j≥1 (M ATLAB
no soporta subı́ndices negativos o cero). La submatriz que comprende la intersección de
las filas p hasta q y las columnas r hasta s, se denota por A(p:q,r:s). Ahora, dos puntos
solos indicando la fila o columna, comprende todos los elementos de esa fila o columna, ası́
A(:,j) es la j-ésima columna de A y A(i,:) es la i-ésima fila. He aquı́ algunos ejemplos:

>> A = [1 2 3;4 5 6;7 8 9]
A =
1 2 3
4 5 6
7 8 9
>> A( 2 , 2 )
ans =
5
>> A ( 2 : 3 , 2 : 3 )
ans =
5 6
8 9
>> A ( : , 2 )
ans =
2
5
8
✂ ✁
Un caso particular es A(:), que denota un vector que contiene todos los elementos de
A tomados de la primera columna hasta la última:

>> B=A ( : )
B =
1
4
7
2
5
8
3
6
9
✂ ✁
Cuando A(:) se coloca en el lado izquierdo de una instrucción de asignación, éste llena la
matriz A preservando su forma. Usando este hecho la matriz 3 × 3 de los primeros 9 primos
se puede construir como:

>> A=z e r o s ( 3 ) ; A(:)= primes ( 2 3 ) ; A=A ’
A =
2 3 5
7 11 13
17 19 23
✂ ✁
¿Qué uso tiene
el punto y coma?
Con el comando A’ se obtiene la matriz traspuesta de A, de la cual daremos detalles más
Utilice help adelante.
primes para sa- En ciertas circunstancias, cuando el lado derecho es un número escalar, el número de
ber acerca del
comando primes
elementos en una asignación de subı́ndices puede ser de diferente medida en los dos lados
APÉNDICE A. INTRODUCCIÓN A MATLAB 291

de la asignación. En este caso el escalar es expandido para que coincida con el número de
elementos de la izquierda:

>> A = ones ( 4 ) ;
>> A ( 1 : 2 , 1 : 2 ) = 0
A =
0 0 1 1
0 0 1 1
1 1 1 1
1 1 1 1
✂ ✁
La notación [] denota una matriz vacı́a. Asignando [] a una fila o columna es una
manera de eliminar esa fila o columna de una matriz:

>> A = [1 2 3;4 5 6;7 8 9 ] ;
>> A ( 1 , : ) = [ ]
A =
4 5 6
7 8 9
✂ ✁

Operaciones de Matrices

Para las operaciones aritméticas de matrices se utilizan los operadores aritméticos ya


definidos, siempre que esté bien definida la operación. Cuando los operadores aritméticos
van precedidos de un punto significa que la operación se efectúa “elemento a elemento”.
En la Tabla A.8 se resumen las operaciones aritméticas de matrices. Aquı́, A y B son matrices
de elementos aij y bij , respectivamente, y k es un escalar.

Tabla A.8. Operaciones aritméticas de matrices

Operación Descripción
A±B matriz de elementos aij ± bij
A*B producto matricial de A y B
Aˆk matriz A elevada a la potencia k
A±k matriz de elementos aij ±k
k*A matriz de elementos k*aij
A/k = (1/k)*A matriz de elementos aij /k
A.ˆk matriz de elementos aijˆk
k.ˆA matriz de elementos kˆaij
k./A matriz de elementos k/aij
A.*B matriz de elementos aij *bij
A./B matriz de elementos aij /bij
A.ˆB matriz de elementos aijˆbij

La suma y resta están definidas para matrices de igual dimensión. El producto A*B es
el resultado del producto matricial, definido solamente cuando el número de columnas de
A es igual al número de filas de B. Ejemplos:

>> A = [1 2;3 4 ] ; B = ones ( 2 ) ;
>> A+B
ans =
2 3
4 5
>> A∗B
ans =
292 A.3. MATRICES

3 3
7 7
✂ ✁
La operación Aˆk, está definida para matrices cuadradas, en cambio las operaciones
A±k y k*A están definidas para matrices de cualquier dimensión. Con la matriz A del
ejemplo anterior obtenemos:

>> Aˆ3
ans =
37 54
81 118
>> 5∗A
ans =
5 10
15 20
>> A−4
ans =
−3 −2
−1 0
✂ ✁
Los operaciones A.*B, A./B, y A.ˆk, se efectúan elemento a elemento. If A y B son
matrices de igual dimensión, entonces los elementos de la matriz C = A.*B están da-
dos por C(i,j) = A(i,j)*B(i,j), y los elementos de la matriz C = A./B son C(i,j)
= A(i,j)/B(i,j). Para una matriz A cualquiera, los elementos de la matriz C = A.ˆk
están dados por C(i,j) = A(i,j)ˆk. Con las matrices A y B de los ejemplos anteriores
obtenemos:

>> A . ∗ B
ans =
1 2
3 4
>> B . / A
ans =
1.0000 0.5000
0.3333 0.2500
>> A. ˆ 3
ans =
1 8
27 64
✂ ✁
Otra operación importante de las matrices es la trasposición. La traspuesta conjugada
de la matriz A se obtiene con A’. Si A es real, esto no es más que la matriz traspuesta de A. La
transpuesta sin conjugación se obtiene con A.’. Las funciones alternativas ctranspose(A)
Utilice help para y transpose(A) a veces son más convenientes. Para el caso especial de los vectores co-
conocer más de
las funciones
lumna x e y, x’*y es el producto escalar o producto punto, que también se puede obtener
ctranspose y usando la función dot(x,y). El producto vectorial o producto cruz de dos vectores 3 × 1 o
transpose 1 × 3 (tal como se utiliza en mecánica) es producido por cross. Ejemplo:

>> x = [1 −1 0 ] ’ ; y = [3 2 1 ] ’ ;
>> x ’ ∗ y
ans =
1
>> dot ( x , y )
ans =
1
>> c r o s s ( x , y )
ans =
APÉNDICE A. INTRODUCCIÓN A MATLAB 293

−1
−1
5
✂ ✁
La mayorı́a de las funciones predefinidas en M ATLAB están hechas de forma que admi-
ten matrices como argumentos. Esto se cumple particularmente para las funciones ma-
temáticas elementales y cuando se aplican a una matriz se obtiene una nueva matriz, esto
es, si A = (aij ), entonces funcion(A) es otra matriz cuyos elementos son funcion(aij ),
donde funcion es cualquier función matemática elemental. Veamos un ejemplo. Para la
matriz A = [2 0;1 4] se tiene: Función de una
✄ matriz en el sen-
>> s q r t (A) tido de Algebra
ans = Lineal, es un
1.4142 0 tópico avanzado
que no se con-
1.0000 2.0000
sidera en estas
>> exp (A) notas
ans =
7.3891 1.0000
2.7183 54.5982
>> s i n (A)
ans =
0.9093 0
0.8415 −0.7568
✂ ✁

Manipulación de Matrices

M ATLAB tiene disponible comandos que permiten la manipulación de matrices, algunos


de ellos se dan en la Tabla A.9.

Tabla A.9. Comandos para la manipulación de matrices

Comando Descripción
reshape cambia la dimensión de la matriz
diag lista los archivos del directorio (formato distinto a ls)
tril devuelve el path (dirección) del directorio actual
triu permite cambiar el directorio actual

La función reshape cambia la dimensión de la matriz, estos es, reshape(A,m,n) pro-


duce una matriz m × n cuyos elementos se toman uno a uno de las columnas de A. Por
ejemplo:

>> A = [1 3 5;2 4 6 ] , B = r e s h a p e (A, 3 , 2 )
A =
1 3 5
2 4 6
B =
1 4
2 5
3 6
✂ ✁
La función diag trabaja con las diagonales de una matriz y puede tomar un vector o
una matriz como argumento. Para un vector v, diag(v) es la matriz diagonal con v su
diagonal principal: En una matriz
diagonal todo los
elementos fuera
de su diagonal
principal son ce-
ros
294 A.3. MATRICES


>> d i a g ( [ 1 2 3])
ans =
1 0 0
0 2 0
0 0 3
✂ ✁
Más generalmente, diag(v,k) coloca a v en la k-ésima diagonal, donde k > 0 especifica
diagonales por encima de la diagonal principal y k < 0 diagonales por debajo (k = 0
indica la diagonal principal):

>> d i a g ( [ 1 1 ] , 1)
ans =
0 1 0
0 0 1
0 0 0
>> d i a g ( [ 1 1 ] , −2)
ans =
0 0 0 0
0 0 0 0
1 0 0 0
0 1 0 0
✂ ✁
Observe que
diag([1 1], 1)
coloca el vector
Para una matriz A, diag(A) es el vector columna conformado por los elementos de la dia-
[1 1] en la dia- gonal principal de A. Entonces, para producir una matriz diagonal cuya diagonal principal
gonal 1 de una sea igual a la diagonal principal de la matriz A, se escribe diag(diag(A)). Análogamente
matriz 3 × 3 de
ceros al caso del vector, diag(A,k) produce un vector columna con los elementos de la k-ésima
diagonal de A. Por ejemplo, si A = [1 2 3;4 5 6;7 8 9], entonces:

>> d i a g (A)
ans =
1
5
9
>> d i a g (A, −1)
ans =
4
8
✂ ✁
Con las funciones tril y triu se pueden extraer las partes triangulares de una matriz.
La función tril(A) genera la parte triangular inferior de A (los elementos sobre y por de-
bajo de la diagonal principal) y triu(A) genera la parte triangular superior (los elementos
sobre y por encima de la diagonal principal). Más generalmente, tril(A,k) genera los
elementos sobre y por debajo de la k-ésima diagonal de A, mientras que triu(A,k) genera
los elementos sobre y por encima de la k-ésima diagonal de A. Considerando nuevamente
Observe que A = [1 2 3;4 5 6;7 8 9], se tiene:
triu(A) y ✄
tril(A) son ma- >> t r i u (A)
trices triangular ans =
superior y tri- 1 2 3
angular inferior, 0 5 6
respectivamente
0 0 9
>> t r i l (A)
ans =
1 0 0
4 5 0
7 8 9
APÉNDICE A. INTRODUCCIÓN A MATLAB 295

>> t r i l (A , 1 )
ans =
1 2 0
4 5 6
7 8 9
✂ ✁

A.4 Elementos de Programación


En esta sección se dan algunas herramientas de programación necesarias para la im-
plementación del algoritmos.

Instrucciones de Control de Flujo

M ATLAB posee cuatro instrucciones de control de flujo: la instrucción if, el bucle for,
el bucle while y la instrucción switch.

Instrucción if

La estructura simple de la instrucción if es:


i f e x p r e s i ó n
instrucciones
end

donde las instrucciones se ejecutan si expresión es verdadera. Por ejemplo, el siguiente


código intercambia el valor de a y b si a es más grande que b:
if a > b
temp = a ;
a = b;
b = temp ;
end

En caso de ser necesario la ejecución de instrucciones cuando expresión es falsa, a la


instrucción if se le puede agregar las instrucciones else y elseif. He aquı́ un caso general
con tres condiciones:
i f e x p r e s i ó n 1
instrucciones
e l s e i f e x p r e s i ó n 2
instrucciones
e l s e i f e x p r e s i ó n 3
instrucciones
else
instrucciones
end

Las instrucciones de cada grupo se ejecutan si es verdadera la condición que las encabeza;
si ninguna de ellas es verdadera, se ejecutan las instrucciones que están encabezadas por
la instrucción else.

Bucle for

La estructura del bucle for es:


296 A.4. ELEMENTOS DE PROGRAMACIÓN

f o r v a r i a b l e = e x p r e s i ó n
instrucciones
end

Normalmente, expresión es un vector de la forma i:s:j. Las instrucciones se ejecutan


con el valor de variable igual a cada elemento de expresión. Por ejemplo, la suma de
30 términos de la serie geométrica (1/2)i se puede calcular como:

>> s = 0 ;
>> f o r i =0:1:29 , s = s +(1/2)ˆ i ; end , s
s =
2.0000
✂ ✁
Varios bucles for pueden anidarse, en cuyo caso es recomendable utilizar sangrı́a para
mejorar la legibilidad. El siguiente código produce una matriz simétrica A 5 × 5 con ele-
Una matriz A mentos A(i,j) = i*j para j≥i:
es simétrica si
AT = A n = 5 ; A = eye ( n ) ;
for j = 2:n
f o r i = 1 : j −1
A( i , j ) = i ∗ j ;
A( j , i ) = A( i , j ) ;
end
end

Bucle while

La estructura del bucle while es:


w h i l e e x p r e s i ó n
instrucciones
end

Las instrucciones se ejecutan mientras expresión es verdadera. El siguiente ejemplo apro-


xima el número positivo más pequeño en punto flotante:

>> num = 1 ; w h i l e num > 0 , nmin = num; num = num/ 2 ; end , nmin
nmin =
4.9407e−324
✂ ✁
Ejecute los co-
mandos realmin
y realmax. ¿Qué
A veces es necesario salirse de un bucle while de manera abrupta, es decir, detener
observa? repentinamente la ejecución de las instrucciones dentro del bucle. Esto se hace empleando
el comando break (comando que permite finalizar forzosamente un bucle). Normalmente,
break se usa conjuntamente con una instrucción condicional dentro de un bucle infinito.
Un bucle infinito puede construirse utilizando, while 1, ..., end, que es útil cuando no
es conveniente poner expresión en la parte superior del bucle. Con estas consideraciones
podemos reescribir el ejemplo previo:
num = 1 ;
while 1
nmin = num;
num = num/ 2 ;
i f num==0
break
end
end
APÉNDICE A. INTRODUCCIÓN A MATLAB 297

La instrucción break también puede usarse para salir de un bucle for. En un bucle
anidado un break sale al bucle del siguiente nivel superior.

Instrucción switch
La última instrucción de control de flujo que mostramos es switch, cuya estructura
está dada por:
s w i t c h e x p r e s i ó n
case caso 1
instrucciones
. . .
case caso n
instrucciones
otherwise
instrucciones
end

La instrucción switch compara el valor de expresión con cada uno de los casos, si el
valor de expresión coincide con alguno de los casos, se ejecutan las instrucciones que el
caso coincidente encabeza. Si no hay coincidencia con ninguno de los casos, entonces se
ejecutan las instrucciones que están encabezadas por otherwise. Vea el siguiente ejemplo
donde se evalúa la norma-p de un vector v (esto es, norm(v,p)) para tres valores de p. Utilice help para
conocer más del
switch p comando norm
case 1
norma = sum( abs ( v ) ) ;
case 2
norma = s q r t ( dot ( v , v ) ) ;
case i n f
norma = max( abs ( v ) ) ;
otherwise
e r r o r ( ’ p debe s e r 1 , 2 o i n f . ’ )
end

Para un vector u, sum(u) arroja la suma de todos los elementos de u, max(u) arroja el
máximo valor de los elementos de u y abs(u) es un vector cuyos elementos son |u(i)|. El
comando error muestra un mensaje para indicar que se ha producido un error, tal mensaje
es la cadena de caracteres dentro de los apóstrofes.

Scripts y Funciones
No todo se puede realizar en la ventana de comandos de M ATLAB , generalmente en
la mayorı́a de las aplicaciones se necesitan hacer cálculos que requieren de instrucciones
muy elaboradas que, inevitablemente, deben ser programadas. Ası́, para llevar a cabo
una tarea, en vez de escribir las instrucciones una por una en la ventana de comandos
de M ATLAB , se pueden escribir una detrás de otra en un M-archivo, que es un archivo
de texto con una extensión *.m. Estos son los equivalentes de los programas, funciones,
subrutinas y procedimientos en otros lenguajes de programación. Distinguiremos dos tipos
de M-archivos: Scripts y Funciones.

Scripts
Un script es un conjunto de instrucciones (de cualquier lenguaje) guardadas en un
archivo de texto que son ejecutadas normalmente mediante un intérprete, además, no
298 A.4. ELEMENTOS DE PROGRAMACIÓN

tienen argumentos de entrada ni de salida y operan sobre las variables del espacio de
trabajo (workspace). Es útil para automatizar pequeñas tareas. También puede hacer las
veces de un programa principal para ejecutar una aplicación. Un simple ejemplo de un
M-archivo script, numero.m, se muestra en el siguiente programa.

Programa A.1. Script numero.m

%NUMERO
% V e r i f i c a s i un número r e a l i n t r o d u c i d o por
% t e c l a d o e s cero , más i n f i n i t o , menos i n f i n i t o
% o d i s t i n t o de c e r o y f i n i t o

x = i n p u t ( ’ I n t r o d u z c a un número r e a l : ’ ) ;
switch x
case i n f
d i s p ( ’ Más i n f i n i t o ’ )
c a s e −i n f
d i s p ( ’ Menos i n f i n i t o ’ )
case 0
d i s p ( ’ Cero ’ )
otherwise
d i s p ( ’ Número d i s t i n t o de c e r o y f i n i t o ’ )
end

Este corto script le pide al usuario que introduzca por teclado un número real, luego
muestra un mensaje indicando que tipo de número es: +∞, −∞, 0 o distinto de 0 y finito.
Las primeras cuatro lı́neas comienzan con el sı́mbolo % lo cual indica que es un comentario.
Cada vez que M ATLAB encuentra un % ignora el resto de la lı́nea. Esto permite introducir
texto que hace mejor la comprensión del código.
En el script numero.m también se observa el comando disp(’string’) que imprime el
string de caracteres en el escritorio de M ATLAB . Asumiendo que este script existe como un
archivo numero.m, escribir numero es equivalente a ejecutar todas lı́neas en la ventana de
comandos de M ATLAB . Más adelante trataremos la edición y el manejo de los M-archivos.

Funciones

En muchas aplicaciones es necesario definir funciones, es decir, procedimientos que


permiten introducir valores y devuelven, después de ejecutar las instrucciones programa-
das, los valores de salida. En M ATLAB es posible definir funciones M ATLAB , las cuales son
generalmente archivos muy parecidos a los scripts, pero se diferencian en que una función
tiene una “interfase” de comunicación con el exterior mediante argumentos de entrada y
de salida. Las funciones escritas en M-archivos permiten ampliar el lenguaje de M ATLAB ,
escribiendo nuestras propias funciones que aceptan y devuelven argumentos. Se pueden
utilizar exactamente de la misma manera que las funciones de M ATLAB existentes tales
como sin, cos, eye, etc. Las funciones de M ATLAB tienen la siguiente estructura:
f u n c t i o n [ s a l i d a ] = name( e n t r a d a )
%name Breve d e s c r i p c i ó n .
% f u n c t i o n [ s a l i d a ] = name( e n t r a d a )
% D e s c r i p c i ó n

instrucciones
end
APÉNDICE A. INTRODUCCIÓN A MATLAB 299

Aquı́ se ilustran algunas caracterı́sticas. La primera lı́nea comienza con la palabra clave
function seguida del argumento de salida, salida, y el sı́mbolo =. A la derecha del
sı́mbolo = viene el nombre de la función, name, seguido por el argumento de entrada,
entrada, dentro de paréntesis. (En general, puede haber cualquier número de argumentos
de entrada y de salida.) El nombre de la función debe ser el mismo que el nombre del M-
archivo donde ella es almacenada.
La segunda lı́nea de un archivo de función se llama lı́nea H1 (ayuda 1). Debe ser una
lı́nea de comentario con una forma especial: comienza con un carácter %, seguido sin
espacio por el nombre de la función, seguido de uno o más espacios, y luego una breve
descripción. La descripción debe comenzar con una letra mayúscula, termina con un punto
y se deben omitir las palabras “the” y “a”. Todas las lı́neas de comentario que están encima
de las lı́neas no comentadas (normalmente una lı́nea en blanco, para facilitar la lectura del
código fuente) se muestran cuando se escribe help name. De esta forma, estas lı́neas
deben describir la función y sus argumentos. Veamos algunos ejemplos de funciones.

Programa A.2. Función minelement

f u n c t i o n y = minelement (A)
%minelement Elemento más pequeño de una m a t r i z .
% minelement (A) e s e l mı́nimo de l o s elementos
% de A en v a l o r a b s o l u t o .

y = min ( min ( abs (A ) ) ) ;


end

El Programa A.2 muestra una simple función, minelement, que halla el menor elemento
en valor absoluto de una matriz. Para esta función tenemos que

>> h e l p minelement
minelement Elemento más pequeño de una m a t r i z .
minelement (A) e s e l mı́nimo de l o s elementos
de A en v a l o r a b s o l u t o .
✂ ✁
Además, minelement se llama igual que cualquier otra función de M ATLAB :

>> minelement ( −1:1:10)
ans =
0
>> minelement ( magic ( 6 ) )
ans =
1
✂ ✁

Programa A.3. Función fpoli

function [ f , df ] = f p o l i (x , a )
%f p o l i Polinomio y su d e r i v a d a .
% [ f , d f ] = f p o l i ( x , a ) e v a l ú a l a f u n c i ó n f ( x ) = x . ∗ ( a−x )
% y su d e r i v a d a d f en l a m a t r i z x , donde a e s un
% parámetro e s c a l a r .

f = x . ∗ ( a−x ) ;
d f = a−2∗x ;
end
300 A.4. ELEMENTOS DE PROGRAMACIÓN

La función fpoli mostrada en el Programa A.3 evalúa la función escalar f (x) = x(a−x)
y su derivada f ′ (x) = a − 2x con respecto a x. Los dos argumentos de salida f y df están
encerrados entre corchetes. Cuando se llama a una función con múltiples entradas o ar-
gumentos de salida no es necesario pedir todos los argumentos de salida, sino aquellos
argumentos que se deseen mostrar comenzando de izquierda a derecha, sin omitir ningún
argumento intermedio. Si se requieren más de un argumento de salida, éstos deben apa-
recer entre corchetes. He aquı́ algunos ejemplos del uso de fpoli:

>> f = f p o l i ( −3 ,0.1)
f =
−9.3000
>> [ f , d f ] = f p o l i ( −3 ,0.1)
f =
−9.3000
df =
6.1000
✂ ✁
Note que en la función fpoli aparece la operación matricial .∗, que se usa en la instrucción
f = x.*(a-x). De esta forma, si un vector o una matriz es suministrada a través de x, la
función se evalúa en cada elemento de forma simultánea:

>> [ f , d f ] = f p o l i ( 1 : 4 , 2 )
f =
1 0 −3 −8
df =
0 −2 −4 −6
✂ ✁

Programa A.4. Función estcal

f u n c t i o n [ x ord , x prom , x med , x d e s ] = e s t c a l ( x )


%e s t c a l A n á l i s i s e s t a dı́ s t i c o de c a l i f i c a c i o n e s .
% Dado un v e c t o r de c a l i f i c a c i o n e s x ,
% [ x ord , x prom , x med , x d e s ] = e s t c a l ( x ) c a l c u l a
% una l i s t a ordenada de l a s c a l i f i c a c i o n e s , y e l
% promedio , l a mediana y l a d e s v i a c i ó n e s t á n d a r
% de l a s c a l i f i c a c i o n e s .

x ord = s o r t (x ) ;
i f nargou t > 1
x prom = mean( x ) ;
end
i f nargou t > 2
x med = median ( x ) ;
end
i f nargou t > 3
x des = std ( x ) ;
end
end

Una función más complicada se muestra en el Programa A.4, la cual realiza un pequeño
análisis estadı́stico a un vector de clasificaciones. Para un vector v, el comando sort(v)
ordena en forma creciente los elementos de v, y los comandos mean(v), median(v) y
std(v), calculan el promedio, la mediana y la desviación estándar de v, respectivamente.
Seguidamente se ilustra el uso de la función estcal.
APÉNDICE A. INTRODUCCIÓN A MATLAB 301


>> c a l i f i c a c i o n e s = [4 12 15 20 18 2 1 3 4 1 4 ] ;
>> x o r d = e s t c a l ( c a l i f i c a c i o n e s )
x ord =
1 2 3 4 4 12 14 15 18 20
>> [ x ord , x prom , x med ] = e s t c a l ( c a l i f i c a c i o n e s )
x ord =
1 2 3 4 4 12 14 15 18 20
x prom =
9.3000
x med =
8
✂ ✁

En el Programa A.4 también se observa el comando nargout, que retorna el número


de argumentos de salida. Si el comando nargout se escribe en el interior del cuerpo
de una función, el devuelve el número de argumentos de salida que se emplearon para
llamar a la función. Otra comando análogo a nargout es nargin que retorna el número
de argumentos de entrada; además, si se escribe en el interior del cuerpo de una función,
nargin devuelve el número de argumentos de entrada que se emplearon para llamar a la
función.

Creación y Edición de M-Archivos

Para crear y editar M-archivos se tienen dos opciones. Se puede usar cualquier editor
de archivos ASCII (si se trata de un procesador de textos es necesario asegurarse de que
guarda los archivos en formato ASCII estándar, no en el propio formato del procesador de
textos). O bien, se puede utilizar el editor de M ATLAB (Editor/Debugger), que se muestra
en la Figura A.2. Éste se invoca tecleando edit en la lı́nea de comandos o escogiendo File-
New o File-Open del menú. El editor de M ATLAB tiene varias caracterı́sticas que ayudan
a la edición de los M-archivos, incluyendo sangrı́a automática de bucles e instrucciones
if, resaltado de sintaxis en color, etc. Estas y otras caracterı́sticas se pueden desactivar o
personalizar a través de File-Preferences del menú del editor.

Figura A.2. Editor de M ATLAB


302 A.4. ELEMENTOS DE PROGRAMACIÓN

Lectura y Escritura de archivos


Utilizaremos tres maneras de pasar datos a y de M ATLAB , a saber:

⊲ Introducción de datos a través del teclado o el ratón

⊲ Escritura en un archivo

⊲ Uso de los comandos save o load

Introducción de datos a través del teclado


La instrucción input le permite a M ATLAB aceptar datos de entrada a través del teclado.
Con la siguiente instrucción se introduce un número:
x = input ( ’ s t r i n g ’ )

La cadena string se muestra como un mensaje en el escritorio de M ATLAB . También se


pueden introducir cadenas desde el teclado, por ejemplo:
z = input ( ’ s t r i n g ’ , ’ s ’ )

El sı́mbolo ’s’ indica que la entrada del teclado es una cadena. Se puede introducir una
cadena con input, sin ’s’ si la cadena se teclea encerrada entre apóstrofes; por ejemplo,
z = i n p u t ( ’ I n t r o d u z c a su a p e l l i d o ( e n c e r r a d o en a p ó s t r o f e s ) : ’ )

Escritura en un archivo
Daremos un breve vistazo a los formatos de salida de M ATLAB . La función disp muestra
el valor de una variable, de acuerdo con el formato actual, sin necesidad de imprimir
primero el nombre de la variable y el sı́mbolo =. Si el argumento es una cadena, disp
muestra la cadena. Ejemplo:

>> d i s p ( ’ Se muestra l a m a t r i z magic 3x3 ’ ) , d i s p ( magic ( 3 ) )
Se muestra l a m a t r i z magic 3x3
8 1 6
3 5 7
4 9 2
✂ ✁
El comando fprintf permite imprimir mensajes y números con un formato especı́fico;
por ejemplo:

>> f p r i n t f ( ’ E l t e r c e r número primo e s : %d\n ’ , 5 )
E l t e r c e r número primo e s : 5
✂ ✁
Aquı́ se observa el sı́mbolo \n que es el operador de lı́nea nueva que avanza en una lı́nea
la posición de la pantalla. Este operador se puede colocar en cualquier lugar de la cadena.
El sı́mbolo %d indica que el número que se imprime es entero. Ahora, en el ejemplo

>> f p r i n t f ( ’ E l v a l o r de p i e s : %6.3 f \n ’ , p i )
E l v a l o r de p i e s : 3.142
✂ ✁
el sı́mbolo %6.3f especifica un formato con ancho de campo de 6 y 3 dı́gitos decimales. Si
f se reemplaza por e, entonces el número se muestra en notación exponencial o cientı́fica:

>> f p r i n t f ( ’ E l v a l o r de p i e s : %6.3e\n ’ , p i )
E l v a l o r de p i e s : 3.142 e+00
✂ ✁
APÉNDICE A. INTRODUCCIÓN A MATLAB 303

Al elegir el ancho del campo recuerde que para un número negativo el signo menos ocupa
una posición:

>> f p r i n t f ( ’ %5.2 f \n%5.2 f \n ’ , s i n ( p i /6) , − s i n ( p i / 6 ) )
0.50
−0.50
✂ ✁
Para imprimir % se usa \% y para imprimir \ se usa \\, en la cadena de formato. Otro
especificador de formato útil es %g, que utiliza a %e o %f, según sea el caso:

>> f p r i n t f ( ’%g %g\n ’ , exp ( 1 ) , exp ( 2 0 ) )
2.71828 4.85165 e+08
✂ ✁
Si se suministran los elementos de una matriz para imprimir, entonces las especifica-
ciones del formato de fprintf, se aplican a los elementos de la matriz, comenzando por
los elementos de la primera columna, luego se toman los de la segunda columna, y ası́
sucesivamente. Esta caracterı́stica se puede utilizar para evitar un bucle. Ejemplo:

>> A = [40 60 80 100 120];
>> f p r i n t f ( ’%g mi/h = %g km/h\n ’ , [A ; 8∗A/ 5 ] )
40 mi/h = 64 km/h
60 mi/h = 96 km/h
80 mi/h = 128 km/h
100 mi/h = 160 km/h
120 mi/h = 192 km/h
✂ ✁
Para imprimir una variable de cadena se utiliza el operador %s. Ejemplo:

>> t e x t o = s p r i n t f ( ’ C á l c u l o Numérico , Año %d ’ , 2013);
>> f p r i n t f ( ’ A s i g n a t u r a : %s \n ’ , t e x t o )
A s i g n a t u r a : C á l c u l o Numérico , Año 2013
✂ ✁
La función sprintf es análoga a fprintf pero devuelve su salida como una cadena. Esta
función es útil para la producción de etiquetas de gráficos.
Pasemos ahora a describir como se escriben datos en un archivo de texto con un formato
especı́fico. Inicialmente se debe abrir o crear el archivo; para ello se utiliza la función fopen
como se indica a continuación:
f i d = fopen (name , permiso )

donde name es una cadena que contiene el nombre del archivo que se abrirá, fid es un
escalar entero denominado identificador del archivo y permiso indica el modo de apertura
del archivo. Los modos de apertura de archivos son:
’r’ abre el archivo para leer
’w’ abre el archivo para escribir; descarta el contenido existente
’a’ abre o crea un archivo para escribir; agrega la data al final del archivo
’r+’ abre (no crea) el archivo para leer y escribir
’w+’ abre o crea el archivo para leer y escribir; descarta el contenido existente
’a+’ abre o crea el archivo para leer y escribir; agrega la data en al final del
archivo
’W’ abre el archivo para escribir sin limpieza automática
’A’ abre el archivo para agregar sin limpieza automática
Después de abrir o crear el archivo, los datos se escriben utilizando la función fprintf,
que toma como primer argumento el identificador de archivo, fid. Finalmente se cierra el
archivo con la función fclose(fid). Ası́, el código
304 A.4. ELEMENTOS DE PROGRAMACIÓN

A = [40 60 80 100 120];


ind = fopen ( ’ m i s a l i d a ’ , ’w ’ ) ;
f p r i n t f ( ind , ’%g mi/h = %g km/h\n ’ , [A ; 8∗A / 5 ] ) ;
f c l o s e ( ind ) ;

crea el archivo misalida que contiene:


40 mi/h = 64 km/h
60 mi/h = 96 km/h
80 mi/h = 128 km/h
100 mi/h = 160 km/h
120 mi/h = 192 km/h

Uso de los comandos save y load


La función save(name) guarda todas las variables actuales en un archivo tipo binario
de extensión .mat de nombre name. En la función save se puede especificar las variables
que desean guardarse: el comando
s a v e (name , v a r i a b l e s )

guarda solo las variables que se indiquen en variables. También se puede especificar el
formato de escritura: .MAT o ASCII. Por defecto save(name,variables) utiliza el formato
.MAT; en cambio,
s a v e (name , v a r i a b l e s , ’− a s c i i ’ )

Use help para utiliza el formato ASCII. El siguiente código


conocer más de
save
a = [1 2 3 4 ] ;
b = [9 8 7 6 ] ’ ;
save ( ’ misdatos ’ , ’ a ’ , ’ b ’ ) ;

crea el archivo binario misdatos.mat que contiene los vectores a y b. Si en el código


anterior se cambia la instrucción save(’misdatos’,’a’,’b’); por
s a v e ( ’ m i s d a t o s . t x x ’ , ’ a ’ , ’ b ’ , ’− a s c i i ’ ) ;

el código resultante crea un archivo llamado misdatos.txx en formato ASCII de 8 dı́gitos.


Si abre el archivo misdatos.txx, se verá ası́:
1.0000000e+00 2.0000000e+00 3.0000000e+00 4.0000000e+00
9.0000000e+00
8.0000000e+00
7.0000000e+00
6.0000000e+00

La función load(name) carga todas las variables de un archivo .MAT con nombre name.
El comando
l o a d (name , v a r i a b l e s )

carga solo las variables indicadas en variable. Veamos el uso de load con el siguiente
ejemplo:

>> v = primes ( 3 7 ) ; A = r e s h a p e ( primes ( 3 7 ) , 4 , 3 ) ’ ;
>> s a v e ( ’ m a t r i z p r i m o s ’ , ’ A ’ , ’ v ’ ) ;
>> whos
Name Size Bytes Class Attributes
A 3x4 96 dou ble
v 1x12 96 dou ble
APÉNDICE A. INTRODUCCIÓN A MATLAB 305

>> clear
>> whos
>> load ( ’ matrizprimos ’ )
>> whos
Name Size Bytes Class Attributes
A 3x4 96 dou ble
v 1x12 96 dou ble
>> c l e a r
>> l o a d ( ’ m a t r i z p r i m o s ’ , ’ v ’ )
>> whos
Name Size Bytes Class Attributes
v 1x12 96 dou ble
✂ ✁

A.5 Gráficos
M ATLAB tiene potentes y versátiles capacidades gráficas. Muchos tipos de figuras se
pueden generar con relativa facilidad y su aspecto es altamente personalizable. En esta
sección cubriremos el uso básico de las herramientas básicas de M ATLAB para la represen-
tación gráfica de datos de dos dimensiones.
Nos centraremos en la generación de gráficos a través de la lı́nea de comandos o de
M-archivos, pero existen figuras que también pueden ser modificadas interactivamente
utilizando el editor de gráficos (Plot Editor). Para usar el editor de gráficos ver help
plotedit.

Gráficos simples en 2D
La función plot se puede usar para generar gráficos uniendo puntos x−y. Por ejemplo,
escribiendo

>> x = [ 0 . 5 1 . 5 3 . 1 3 . 2 4 . 6 5 . 5 8 . 2 ] ;
>> y = [ 1 . 1 3 . 3 6 . 1 7 . 2 8 . 8 9 . 1 2 . 6 ] ;
>> p l o t ( x , y )
✂ ✁
se produce la imagen de la izquierda de la Figura A.3, donde los puntos (x(i),y(i)) están
unidos en secuencia.
10 10

9 9

8 8

7 7

6 6

5 5

4 4

3 3

2 2

1 1
0 1 2 3 4 5 6 7 8 9 0 1 2 3 4 5 6 7 8 9

Figura A.3. Simple gráfico x − y. Izquierda: por defecto. Derecha: personalizado

Más generalmente, se puede reemplazar plot(x,y) con plot(x,y,’string ’), donde


string combina hasta tres elementos que controlan el color, marcador y estilo de lı́nea.
Por ejemplo, plot (x, y, ’r*--’) especifica que un asterisco rojo se va a colocar en cada
306 A.5. GRÁFICOS

punto (x(i),y(i)) y que los puntos se van a unir por una lı́nea discontinua de color rojo.
La Tabla A.10 muestra una lista de las opciones disponibles. La imagen de la derecha de
la Figura A.3 fue generada con plot(x,y,’bd:’), que produce una lı́nea de puntos azul y
un marcador en forma de rombo también azul.

Tabla A.10. Opciones para el comando plot

Marcador
o Cı́rculo
* Asterisco
Color
. Punto
r Rojo
+ Más
g Verde Estilo de Lı́nea
x Cruz
b Azul - Lı́nea sólida
s Cuadrado
c Cyan -- Lı́nea de trazos
d Rombo
m Magenta : Lı́nea de puntos
ˆ Triángulo ascendente
y Amarillo -. Lı́nea de trazo-puntos
v Triángulo descendente
k Negro
> Triángulo a la derecha
w Blanco
< Triángulo a la izquierda
p Estrella de cinco puntas
h Estrella de seis puntas

Más de un conjunto de datos pueden aparecer en un gráfico; por ejemplo,


p l o t ( x , y , ’ g− ’ , b , c , ’ r−− ’ )

superpone los gráficos x − y y b − c, con lı́nea sólida verde y lı́nea roja discontinua, respec-
tivamente.
Se puede ejercer aun más control sobre el estilo de las lı́neas y de los marcadores
agregando más argumentos a plot. Las propiedades LineWidth (por defecto 0.5 pun-
tos) y MarkerSize (por defecto 6 puntos) se pueden definir en puntos. Por ejemplo,
los comandos plot(x,y,’LineWidth’,4) y plot(x,y,’p’,’MarkerSize’,12) producen
gráficos con un ancho de lı́nea de 4 puntos y un marcador tipo estrella de cinco puntas
de tamaño 12 puntos, respectivamente. Para los marcadores que tienen un interior bien
definido, las propiedades MarkerEdgeColor y MarkerFaceColor se pueden ajustar a uno
de los colores de la Tabla A.10; por ejemplo,
p l o t ( x , y , ’ o ’ , ’ MarkerEdgeColor ’ , ’m ’ )

hace que los bordes del cı́rculo sean magenta.


10 10

9 9

8 8

7 7

6 6

5 5

4 4

3 3

2 2

1 1
0 1 2 3 4 5 6 7 8 9 0 1 2 3 4 5 6 7 8 9

Figura A.4. Dos gráficos x − y con estilos personalizados


APÉNDICE A. INTRODUCCIÓN A MATLAB 307

El gráfico de la izquierda de la Figura A.4 fue producido con


p l o t ( x , y , ’ r−−ˆ ’ , ’ LineWidth ’ , 2 , ’ MarkerFaceColor ’ , ’ y ’ , ’ MarkerSize ’ , 9 )

y el gráfico de la derecha con


p l o t ( x , y , ’−−ms ’ , ’ MarkerSize ’ , 2 0 , ’ MarkerFaceColor ’ , ’ g ’ )

Para ciertas aplicaciones es necesario utilizar escala logarı́tmica. Esto se puede lograr
con la función loglog. El siguiente código genera la gráfica en escala logarı́tmica de
|1 + h + h2 /2 − exp(h)| versus h, para h = 10−4 , 10−3 , 10−2 , 10−1 , 1, la cual se aprecia en
la Figura A.5. Está gráfica representa el error al aproximar exp(h) con los tres primeros
términos de su desarrollo de Taylor. Tal error es del orden h3 .
h = 10.ˆ( −4:1:0);
e r r o r = abs ((1+h+h . ˆ 2 / 2 ) − exp ( h ) ) ;
l o g l o g (h , e r r o r , ’− ’ , h , h . ˆ 3 , ’−− ’ )
xlabel ( ’h ’ )
y l a b e l ({ ’ V a l o r a b s o l u t o ’ , ’ d e l e r r o r ’ })
t i t l e ( ’ E r r o r de aproximaci ón de l a s e r i e c ú b i c a de T a y l o r de exp ( h ) ’ )
box o f f

Se utilizaron title(’string ’), xlabel(’string ’) y ylabel(’string ’). Estas fun-


ciones muestran string encima del gráfico, en el eje x y en el eje y, respectivamente.
También se empleó el comando box off, que quita el cuadro del gráfico, dejando sólo los Use help para
conocer más de-
ejes x e y. talles de title,
xlabel y ylabel
Error de aproximación de la serie cúbica de Taylor de exp(h)
0
10

−2
10

−4
10

−6
Valor absoluto

10
del error

−8
10

−10
10

−12
10

−14
10
−4 −3 −2 −1 0
10 10 10 10 10
h

Figura A.5. Ejemplo de escala logarı́tmica

Por supuesto, loglog no acepta valores negativos como argumento; en este caso M A -
TLAB muestra una advertencia y grafica solo los valores positivos. Otras funciones relacio-
nadas son semilogx y semilogy, que colocan escala logarı́tmica solamente en el eje x o en
el eje y, respectivamente.
Si uno genera un gráfico usando plot e inmediatamente genera otro gráfico con un
nuevo llamado de plot, la nueva imagen reemplaza el gráfico anterior. Escribiendo hold
on causa que gráficos subsiguientes se superpongan al actual, mientras que hold off espe-
cifica que cada gráfico nuevo sustituye el actual. El estado por defecto corresponde a hold
off.
El comando clf borra la figura actual, mientras que close cierra la ventana de figura.
Es posible tener varias ventanas de figura en la pantalla. La forma más sencilla de crear
308 A.5. GRÁFICOS

una nueva ventana de figura es escribir figure. La n-ésima ventana de figura (donde se
muestra n en la barra de tı́tulo) se puede crear escribiendo figure(n). El comando close
all cierra todas las ventanas de figuras.
Cuando la imagen se muestra en la ventana de figura, muchos aspectos de la misma se
pueden cambiar. Para ello, se pueden utilizar los elementos de la barra de herramientas de
la ventana de figura o en su menú desplegable Tools.

Ejes y Anotaciones
Varios aspectos de los ejes de un gráfico pueden controlarse con el comando axis.
Algunas de estas opciones se muestran en la Tabla A.11.

Tabla A.11. Algunos comandos para controlar los ejes

axis([xmin xmax ymin ymax]) Especifica los lı́mites de los ejes x e y


axis auto Regresa al tipo de ejes por defecto
axis equal Iguala las unidades de los datos x e y
axis off Remueve los ejes
axis square Hace que los ejes sean cuadrados
xlim([xmin xmax]) Especifica los lı́mites del eje x
ylim([ymin ymax]) Especifica los lı́mites del eje y

Los ejes se pueden remover del gráfico con el comando axis off. La función axis
equal establece una sola unidad de proporción para que los incrementos de marcas en los
ejes x e y sean del mismo tamaño. La opción axis square hace que la caja actual sea
cuadrada. Para ilustrar, la imagen de la izquierda de la Figura A.6 fue producida por
p l o t ( f f t ( eye ( 1 7 ) ) ) , a x i s equ al , a x i s s q u a r e

Use help fft y la imagen de la derecha fue producida por


para mayor in-
formación p l o t ( f f t ( eye ( 1 7 ) ) ) , a x i s equ al , a x i s o f f

0.8

0.6

0.4

0.2

−0.2

−0.4

−0.6

−0.8

−1
−1 −0.8 −0.6 −0.4 −0.2 0 0.2 0.4 0.6 0.8 1

Figura A.6. Uso de axis

La instrucción axis[xmin xmax ymin ymax] ajusta los lı́mites del gráfico en el eje x
de xmin a xmax y en eje y de ymin a ymax. Para volver a la escala de los ejes por defecto,
que M ATLAB elige automáticamente en base a los datos que están siendo representados,
APÉNDICE A. INTRODUCCIÓN A MATLAB 309

se escribe axis auto. Los lı́mites de los ejes x e y se pueden ajustar individualmente con
xlim([xmin xmax]) y ylim([ymin ymax]).
1 2
En el siguiente ejemplo se grafica la función f (x) = 2 + en el intervalo [−1, 2]:
x (x − 1)2
x = l i n s p a c e ( −1 ,2 ,500);
plot (x , 1 . / x .ˆ2 + 2 . / ( x −1).ˆ2);
g r i d on

El comando linspace(x1,x2,n) genera un vector fila de n puntos igualmente espa-


ciados desde x1 hasta x2. Con grid on se añaden las lı́neas de cuadrı́cula a los ejes. El
resultado obtenido se muestra a la izquierda de la Figura A.7. Debido a las singularidades
en x = 0 y x = 1, el gráfico es poco informativo. Ahora, ejecutando el comando adicional
ylim ([−1 50])

se produce la gráfica de la derecha de la Figura A.7, en el cual se percibe mejor el compor-


tamiento de la función.
5
x 10
5 50

4.5 45

4 40

3.5 35

3 30

2.5 25

2 20

15
1.5

10
1

5
0.5

0
0
−1 −0.5 0 0.5 1 1.5 2 −1 −0.5 0 0.5 1 1.5 2

Figura A.7. Uso de ylim (derecha) para cambiar los lı́mites automáticos del eje y (iz-
quierda)

Epicicloide: a=12, b=5


25

20

15

10

5
y(t)

−5

−10

−15

−20

−25
−25 −20 −15 −10 −5 0 5 10 15 20 25
x(t)

Figura A.8. Gráfica del epicicloide

En la Figura A.8 se muestra la gráfica del epicicloide


(
x(t) = (a + b) cos(t) − b cos((a/b + 1)t)
0 ≤ t ≤ 10π,
y(t) = (a + b)sen (t) − bsen ((a/b + 1)t)
310 A.5. GRÁFICOS

para a = 12 y b = 5. Las instrucciones utilizadas para generar esta gráfica son:


a = 12; b = 5 ;
t = 0:0.05:10∗ pi ;
x = ( a+b )∗ c o s ( t ) − b∗ c o s ( ( a /b+1)∗ t ) ;
y = ( a+b )∗ s i n ( t ) − b∗ s i n ( ( a /b+1)∗ t ) ;
plot (x , y)
a x i s equal
a x i s ([−25 25 −25 25])
g r i d on
t i t l e ( ’ E p i c i c l o i d e : a=12, b=5 ’ )
xlabel ( ’ x( t ) ’ ) , ylabel ( ’ y( t ) ’ )

Ejecute el código Los lı́mites de los ejes fueron elegidos para agregar más espacio alrededor del epicicloide.
sin la instrucción
axis([-25 25 A continuación se construye la gráfica los polinomios de Chebyshev de grados 1 a 4
-25 25]). ¿Qué y usamos la función legend para agregar una leyenda. El resultado se muestra en la
observa?
Figura A.9.
x = −1:.01:1;
p1 = x ;
p2 = 2∗x . ˆ 2 − 1 ;
p3 = 4∗x . ˆ 3 − 3∗ x ;
p4 = 8∗x . ˆ 4 − 8∗ x . ˆ 2 + 1 ;
p l o t ( x , p1 , ’ r : ’ , x , p2 , ’ g−− ’ , x , p3 , ’ b−. ’ , x , p4 , ’m− ’ )
a x i s square
ylim ([ −1.5 1 . 5 ] )
box o f f
legend ( ’ \ i t n=l ’ , ’ \ i t n=2 ’ , ’ \ i t n=3 ’ , ’ \ i t n=4 ’ , ’ L o c a t i o n ’ , ’ S o u t h E a s t ’ )
x l a b e l ( ’ x ’ , ’ F o n t S i z e ’ , 1 2 , ’ FontAngle ’ , ’ i t a l i c ’ )
y l a b e l ( ’ P n ’ , ’ F o n t S i z e ’ , 1 2 , ’ FontAngle ’ , ’ i t a l i c ’ , ’ R o t a t i o n ’ , 0 )
t i t l e ( ’ P o l i n o m i o s de Chebyshev ’ , ’ F o n t S i z e ’ ,14)
t e x t ( − . 3 5 , 1 . 3 , ’ P {n+1}(x ) = 2xP n ( x ) + P {n−1}(x ) ’ , ’ F o n t S i z e ’ , 1 2 , . . .
’ FontAngle ’ , ’ i t a l i c ’ )

Polinomios de Chebyshev
1.5

P (x) = 2xP (x) − P (x)


n+1 n n−1

0.5

P
n
0

−0.5

−1
n=l
n=2
n=3
n=4
−1.5
−1 −0.8 −0.6 −0.4 −0.2 0 0.2 0.4 0.6 0.8 1
x

Figura A.9. Ejemplo de los polinomios de Chebyshev, usando legend


APÉNDICE A. INTRODUCCIÓN A MATLAB 311

Por lo general, legend(’string1 ’,’string2 ’,...,’stringn ’) creará un cuadrado


con la leyenda que contiene a stringi justo al lado de información del color, marcador y
estilo de lı́nea del gráfico correspondiente. Por defecto, el cuadrado aparece en la esquina
superior derecha (noreste) del área de los ejes. La ubicación de la caja se puede especificar
con la sintaxis legend(’,...,’Location’,ubicación ), donde ubicación es una cadena
que puede incluir los valores que se indican en la Tabla A.12.

Tabla A.12. Opciones para la ubicación de la leyenda

’North’ dentro de la caja del gráfico en la parte superior


’South’ dentro de la caja del gráfico en la parte inferior
’East’ dentro de la caja del gráfico a la derecha
’West’ dentro de la caja del gráfico a la izquierda
’NorthEast’ dentro de la caja del gráfico arriba a la derecha
’NorthWest’ dentro de la caja del gráfico arriba a la izquierda
’SouthEast’ dentro de la caja del gráfico abajo a la derecha
’SouthWest’ dentro de la caja del gráfico abajo a la izquierda
’NorthOutside’ fuera de la caja del gráfico en la parte superior
’SouthOutside’ fuera de la caja del gráfico en la parte inferior
’EastOutside’ fuera de la caja del gráfico a la derecha
’WestOutside’ fuera de la caja del gráfico a la izquierda
’NorthEastOutside’ fuera de la caja del gráfico arriba a la derecha
’NorthWestOutside’ fuera de la caja del gráfico arriba a la izquierda
’SouthEastOutside’ fuera de la caja del gráfico abajo a la derecha
’SouthWestOutside’ fuera de la caja del gráfico abajo a la izquierda
’Best’ menos conflictos con los datos dentro del gráfico

En este ejemplo también se usa el comando text(x,y,’string ’), que coloca a string
en la posición cuyas coordenadas están dadas por x e y. Observe que las cadenas en
los comandos ylabel y text usan la notación del sistema TEX para especificar las letras TEX es un sis-
tema de tipo-
griegas, los sı́mbolos matemáticos y los superı́ndices y subı́ndices. La Tabla A.13 lista grafı́a escrito por
algunos comandos TEX que pueden emplearse. Donald E. Knuth
en el año 1985

Tabla A.13. Algunos comandos TEX que se admiten en cadenas de texto

Sı́mbolos seleccionados
Letras Griegas ≈ \approx
Minúsculas ◦ \circ
α \alpha ≥ \geq
β \beta Im \Im
γ \gamma ∈ \in
.. .. ∞ \infty Fuentes
. . R
\int Normal \rm
ω \omega
≤ \leq Negrita \bf
Mayúsculas
6= \neq Cursiva \it
Γ \Gamma
⊕ \oplus
∆ \Delta
∂ \partial
Θ \Theta
± \pm
.. ..
. . ℜ \Re
Ω \Omega ∼
√ \sim
\surd
312 A.5. GRÁFICOS

Una nota final acerca de este último ejemplo es que hemos utilizado las propiedades
FontSize y FontAngle para ajustar el tamaño y el ángulo del texto producido por los
comandos title, xlabel, ylabel y text (el valor predeterminado de FontSize es 10 y
de FontAngle es normal). Sin embargo, la leyenda no acepta estos argumentos, ası́ que
utilizamos notación TEX para producir fuentes cursivas en la leyenda.

Guardar Figuras

Con regularidad es necesario introducir figuras en reportes, informes, etc., con un for-
mato gráfico especı́fico (BMP, TIFF, JPEG, EPS, PDF, etc.). Una alternativa es utilizar el
comando print para guardar la figura como un archivo. Por ejemplo,
p r i n t −deps2 m i f i g u r a . eps

guarda la figura actual en el archivo mifigura.eps de tipo PostScript encapsulado de nivel


2 en blanco y negro, que posteriormente se puede imprimir en una impresora PostScript o
incluirlo en un documento. Algunas opciones del comando print pueden apreciarse en la
Use help print Tabla A.14.
para ver más
opciones de
print Tabla A.14. Algunas opciones de formato gráfico de print

Formato Gráfico Opción


BMP monocromático -dbmpmono
BMP 24-bit -dbmp
EPS blanco y negro -deps
EPS a color -depsc
EPS nivel 2 blanco y negro -deps2
EPS nivel 2 a color -depsc2
JPEG 24-bit -djpeg
PDF archivo pdf a color -dpdf
PNG 24-bit -dpng
TIFF 24-bit -dtiff

El comando print también tiene una forma funcional, a saber:


p r i n t ( ’−deps2 ’ , ’ m i f i g u r a . eps ’ )

Para ilustrar la utilidad de esta forma funcional, el próximo ejemplo genera una secuencia
de cinco figuras y guarda las mismas en los archivos figura1.eps, ..., figura5.eps:
x = l i n s p a c e (0 ,2∗ pi , 5 0 ) ;
f o r i =1:5
p l o t ( x , c o s ( i ∗x ) )
name = s p r i n t f ( ’ f i g u r a%d . eps ’ , i ) ;
p r i n t ( ’−deps2 ’ ,name)
end

El segundo argumento de print es una cadena que contiene el nombre del archivo y su
extensión, la cual la creamos con el comando sprintf. Ası́ cuando i=1, por ejemplo, la
instrucción print es equivalente a print(’-deps2’,’figura1.eps’).
Es importante tener en cuenta que los gráficos que se ven bien en la pantalla pueden
no ser tan buenos para la impresión. En particular, los valores por defecto como el tamaño
de la fuente, el ancho de las lı́neas, entre otros, pueden hacer que sea difı́cil leer el texto
o no permitan apreciar el estilo de las lı́neas en las figuras impresas. Para producir figuras
APÉNDICE A. INTRODUCCIÓN A MATLAB 313

visualmente atractivas, con impresiones legibles, por lo general es necesario aumentar el


valor de los parámetros que controlan estos efectos. Esto se puede hacer de dos maneras:

1. Añadiendo modificadores como FontSize,12 a los comandos pertinentes.

2. Estableciendo los valores predeterminados de las propiedades, antes de la creación


de la figura de M ATLAB . Esto se realiza con los comandos get y set. Use help para
conocer más de
get y set.
El comando saveas guarda una figura en un archivo en una forma que puede ser
recargada en M ATLAB . Por ejemplo,
saveas ( gcf , ’ mifigura ’ , ’ f i g ’ )

guarda la figura actual como un archivo-FIG binario, que puede ser recargado en M ATLAB
con open(’mifigura.fig’).
También es posible guardar, imprimir o exportar figuras desde el menú desplegable
File de la ventana de figura de M ATLAB .
Bibliografı́a
[1] Churchil, R. y Ward, J. (2004). Variable Compleja y Aplicaciones 7ma. ed. McGraw-
Hill, Madrid.

[2] David, A. (1997). Variable Compleja con Aplicaciones 2da. ed. Addison-Wesley Ibero-
americana, Wilmington, Delaware.

[3] Derrick, W. (1987). Variable Compleja con Aplicaciones. Grupo Editorial Iberoamérica,
México.

[4] Higham, D.J. and Higham, N.J. (2005). Matlab Guide. SIAM, Philadelphia.

[5] James, G. (2002). Matemáticas Avanzadas para Ingenierı́a 2da. ed. Prentice Hall,
México.

[6] Kamen, E.W. y Heck, B.S. (2008). Fundamentos de Señales y Sistemas Usando la Web
y Matlab 3era. ed. Prentice Hall, México.

[7] Lindner, D.K. (1999). Introduction to Signals and Systems. McGraw-Hill, Boston.

[8] Markushévich, A.I. (1967). Teorı́a de las Funciones Analı́ticas, vol. I. Editorial Mir,
Moscú.

[9] Soliman, S.S. y Srinath, M.D. (1999). Señales y Sistemas Continuos y Discretos 2da.
ed. Prentice Hall, México.

314
Índice

A Fase, 200
Argumento, 9 definición, 200
principal, 10 Frontera de un conjunto, 15
Aritmética compleja con M ATLAB , 17 Función armónica, 36
armónica conjugada, 36
C Función de variable compleja, 24–66
Cero de una función, 36
analı́tica, 34
Circunferencia unitaria, 13
continua, 28
Conjunto abierto, 14
definición, 24
Conjunto acotado, 16
derivada de, 31
Conjunto cerrado, 15
dominio de, 24
Conjunto conexo, 16
entera, 35
Conjunto no acotado, 16
funciones componentes, 26
Continuidad, 28–30
lı́mite de, 26
teoremas de, 29
Contorno, 106, 107 monovaluada, 24
cerrado simple, 107 multivaluada, 24
curva, 106 punto singular de, 35
curva suave, 107 racional, 25
Convolución en el dominio continuo, 151 rango de, 24
definición, 151 Funciones de dominio continuo,
propiedades, 151 140–155
Convolución en el dominio discreto, 159 convolución, 151
definición, 159 definición, 140
propiedades, 159 escalón unitario, 143
función de muestro o Sa , 147
D función rampa, 147
Derivada generalizada, 149 función seno cardinal o sinc , 148
Desarollo de MacLaurin, 82 función signo, 145
Diferenciación, 31–34 impulso unitario, 141
ecuaciones de Cauchy-Riemann, 32 propiedades, 142
reglas de, 31 pulso exponencial, 146
Disco unitario, 13 pulso rectangular, 144
Dominio, 16 pulso triangular, 145
Funciones de dominio discreto, 155–161
E
Euler, Leonhard, 12 convolución, 159
Expansión en fracciones parciales, 121 definición, 155
escalón unitario discreto, 156
F función rampa discreta, 156
Fórmula integral de Cauchy, 115 función signo discreto, 157
extensión, 116 impulso unitario discreto, 155

315
316 ÍNDICE

Funciones elementales, 39–50 principio de independencia de la


exponencial, 40 trayectoria, 113
exponente complejo, 46–47 teorema de Cauchy-Goursat, 110
exponencial de base c, 47 extendeión del, 111
funciones hiperbólicas inversas, Introducción al cálculo operacional con
49–50 Matlab, 162
funciones trigonométricas inversas,
48–49 M
inversa de la tangente, 48 Magnitud, 200
inversa del coseno, 48 definición, 200
inversa del seno, 48 Mapeo, 50–60
hiperbólicas, 43–44 bilineal, 59–60
cosecante hiperbólica, 44 inversión, 56–59
coseno hiperbólico, 43 inyectivo, 51
cotangente hiperbólica, 44 lineal, 51–56
secante hiperbólica, 44 w = bz, 53
seno hiperbólico, 43 w = bz + c, 55
tangente hiperbólica, 44 w = z + c, 51
logaritmo, 44–46 Matlab, 282–313
corte ramal, 46 aritmética compleja con, 17
rama principal del, 45 constantes, 284
ramas del, 45 definición de variables, 285
valor principal del, 45 editor de gráficos, 305
trigonométricas, 41–42 escritorio, 282
cosecante, 42 funciones, 298–301
coseno, 41 funciones, lı́mite, derivada y mapeos
cotangente, 42 con, 61
secante, 42 gráficos, 305–311
seno, 41 FontAngle, 312
tangente, 42 FontSize, 312
Funciones, Lı́mite, Derivada y Mapeos LineWidth, 306
con Matlab, 61 MarkerEdgeColor, 306
MarkerFaceColor, 306
I MarkerSize, 306
Integración y residuos con Matlab, 124 axis, 308
Integral, 105–116 clf, 307
contorno, 107 close, 307
cerrado simple, 107 figure, 308
curva, 106 hold off, 307
suave, 107 hold on, 307
de lı́nea, 106–109 linspace, 309
propiedades, 108 loglog, 307
definida, 105 plot, 305
propiedades, 106 semilogx, 307
fórmula integral de Cauchy, 115 semilogy, 307
extensión de la, 116 title, 307
indefinida, 113 xlabel, 307
primitiva, 114 ylabel, 307
ÍNDICE 317

guardar figuras, 312–313 285


print, 312 tabla de operadores lógicos, 285
saveas, 313 tabla de utilizades, 287
instrucciones de control de flujo, tablas de opciones de plot, 306
295–297 transformada z con, 266
for, 295 transformada de Fourier con, 205
if, 295 transformada de Laplace con, 237
switch, 297 Matlab:integración y residuos con, 124
while, 296
introducción al cálculo operacional N
con, 162 Número complejo, 2–20
introducción de datos, 302 argumento, 9
input, 302 principal, 10
lectura y escritura de archivos, conjugado, 6
302–305 coordenadas polares, 9
disp, 302 definición, 2
fclose, 303 desigualdad triangular, 7
fopen, 303 fórmula de Euler, 12
fprintf, 302 inverso aditivo, 4
load, 304 inverso multiplicativo, 4
save, 304 operaciones algebraicas, 3
sprintf, 303 división, 3
m-archivos, 298, 301 multiplicación, 3
creación y edición, 301 resta, 3
matrices, 288–295 suma, 3
manipulación de , 293 parte imaginaria, 3
notación de dos puntos, 289 parte real, 3
operaciones, 291 plano complejo, 5
scripts, 297 potencia n-ésima, 13
series de potencias y singularidades raı́z n-ésima, 13
con, 94 representación geométrica, 5
tabla de comandos TEX, 311 unidad imaginaria, 2
tabla de comandos para controlar los valor absoluto, 6
ejes, 308
P
tabla de comandos para la
Parte principal, 89
manipulación de matrices, 293
Polinomio complejo, 25
tabla de comandos para lsitar y
Punto de acumulación, 15
eliminar variables, 286
tabla de formatos de salida, 283 R
tabla de funciones elementales, 287 Residuo, 117
tabla de opciones de print, 312 cálculo del, 118
tabla de opciones para ubicar la en un polo, 118
leyenda, 311 en un punto singular esencial, 118
tabla de operaciones aritméticas, en un punto singular removible, 118
284 teorema de los residuos, 120
tabla de operaciones aritméticas de
matrices, 291 S
tabla de operadores de comparación, Serie, 77–88
318 ÍNDICE

de Laurent, 85 unilateral derecha, 249


de números complejos, 78 unilateral izquierda, 249
convergente, 78 Transformada z con Matlab, 266
de potencias, 79 Transformada z inversa, 251
de Taylor, 81 definición, 251
desarollo de MacLaurin, 82 expansión en serie de potencias, 264
propiedades adicionales de, 87 integración compleja, 263
Teorema de Cauchy-Hadamard, 80 inversión por tablas, 265
Teorema de Laurent, 86 Transformada de Fourier, 185–204
Teorema de Taylor, 82 definición, 185
Serie de Laurent, 85 propiedades de la, 187
Serie de números complejos, 77 tabla de pares de transformadas, 199
Series de potencias y singularidades con tabla de propiedades, 193
Matlab, 94 Transformada de Fourier con Matlab,
Singularidades aisladas, 88–94 205
polo de orden m, 90 Transformada de Laplace, 218–246
punto singular aislado, 88 bilateral, 218
punto singular esencial, 93 propiedades de la, 220
punto singular removible, 94 región de convergencia, 219
Sucesión, 77 tabla de pares de transformadas, 232
de números complejos, 77 tabla de propiedades, 227
convergente, 77 unilateral derecha, 218
T unilateral izquierda, 218
Teorema de Cauchy-Goursat, 110 Transformada de Laplace con Matlab,
Teorema de Cauchy-Hadamard, 80 237
Teorema de Laurent, 86 Transformada inversa de Fourier, 186
Teorema de los Residuos, 120 definición, 186
Teorema de Taylor, 82 Transformada inversa de Laplace, 220
Transformada z, 249–273 definición, 220
bilateral, 249 integración de contornos, 233
propiedades de la, 252 inversión por tablas, 236
región de convergencia, 250
tabla de pares de transformadas, 262 V
tabla de propiedades, 257 Vecindad, 13

También podría gustarte